You are on page 1of 945

PROBLEM SOLUTIONS CHAPTER 1.

Solution 1.1. (a) Charge on one electron: -1.601910


-19
C. This means that charge on 10
13
electrons
is: -1.601910
-6
C. Net charge on sphere is: 1.601910
-6
C (POSITIVE).
Solution 1.2. (a) 1 atom -4.64610
-18
C. By proportionality, 64g NA atoms.
3.1g ? atoms 3.1g
3.1NA
64
atoms.
Total Charge = 4.646 10
18
C
atom

3.1 6.023 10
23
64
atoms =1.3554 10
5
C
(b) Total charge per atom is -4.64610
-18
C. Total charge per electron is 1.601910
-19
C. Therefore,
there are 29 electrons per atom of copper.
(c) 0.91 A 0.91 C/s. i =
Q
t
t =
Q
i
=
1.36 10
5
0.91
=1.49 10
5
sec .
(d) We know there are
3.1NA
64
= 2.9174 10
22
atoms in the penny. Removing 1 electron from
0.05
3.1NA
64
atoms means removing 0.05
3.1NA
64
electrons. Therefore,
Net charge = 0.05
3.1NA
64
1.6019 10
19
= 234C
Solution 1.3 (a) 7.573 10
17
1.6019 10
19
( )
= 0.1213C
(b) Current =
0.1213
10
3
= 121.3A flowing from right to left.
(c) Again, use proportionality:
10 A =
x 1.6019 10
19
60sec
x =
10 60
1.6019 10
19
= 3.75 10
21
(d) i t ( ) =
dq
dt
= 1 e
5t
A. This is an exponential evolution with an initial value of 0, a final value of 1,
and a time-constant of 1/5 (signal reaches ~63% of its final value in one time-constant).
0.2 time in sec
i(t)
1
(e) Current is the slope of the charge waveform. Therefore, by inspection:
Solution 1.4 (a) 6.02310
23
(-1.601910
-19
) = 9.6510
4
C.
(b) Current flows from right to left (opposite electrons), and:
I =
9.65 10
4
10
3
= 9.65 10
7
A
(c) Using proportionality:
5A =
x 1.6019 10
19
60sec
x =
5 60
1.6019 10
19
=1.87 10
21
(d) i t ( ) =
dq
dt
= 1+ 0.5cos t ( ) i 1sec ( ) = 1 1.57 = 0.57A. Current flows from left to right.
Solution 1.5 (a) i t ( ) = 1 4e
2t
+ 3e
3t
t 0. Then
q t ( ) = i t ( )dt
0
t

= 1 4e
2
+ 3e
3
( )
d
0
t

= ]

t
4 e
2
d
0
t

+ 3 e
3
d
0
t

= t 4 0.5e
2
[ ]
0
t
+ 3 0.333e
3
[ ]
0
t
= t + 2e
2t
e
3t
1
(b) By inspection:
(c) q t ( ) = 120cos 120t ( ) . Hence
i t ( ) =
dq
dt
= 120 120sin 120t ( ) =14400sin 120t ( ) A
Solution 1.6. (a) i(t) = 1 cos(t) A. Hence
q(t) = i()d
0
t

= 1 cos() ( )d
0
t

= t
1

sin()



0
t
= t
1

sin(t) C
(b) Charge is integral of current. Graphically, the charge at time t is the area under the current curve up to
time t: (note the quadratic nature between 2 and 4 seconds)
Solution 1.7
Again, Q is the running area under the current curve. Between 0 and 3 seconds, current decreases linearly
until zero. So, Q
tot
= 7.5 C. From 0 to 6: Q
tot
= 7.5 + Q
3_6
= 7.5 -1/10.5 + -1/10.5 + -11 = 5.5 C,
where the curve from 3 to 6 was divided into two triangular sections and one rectangular one.
Solution 1.8 Charge is the area under the current curve. Thus, Q = 0.1*4 0.1*2 = 0.2 C.
Solution 1.9 Calculate the change in energy for the electron: E = Q V = 3.21810
-15
.
Equate this to kinetic energy:
3.218 10
15
=
1
2
mv
2
v = 8.4 10
7
m / s
where the mass of an electron, 9.106610
-31
has been substituted.
Solution 1.10 P = VI. Hence I = P/V = 210
3
/120 = 16.6667 A
PROBLEM Solution 1.11 (a) It is necessary to integrate the i(t) curve to obtain q(t). We do this
interval by interval:
(i) 0 t 1 ms, q(t) = 0 + d
0
t

= t C
(ii) 1 ms t 2 ms, q(t) = 1 2 d
1
t

= 3 2t C
(iii) 2 ms t 3 ms, q(t) = 1+ d
2
t

= 3 + t C
(iv) 3 ms t 5 ms, q(t) = 0 + 8 2 ( )d
3
t

= 8t t
2
15 C
(v) 5 ms t, q(t) = 0 C
0 1 2 3 4 5 6
-1
-0.8
-0.6
-0.4
-0.2
0
0.2
0.4
0.6
0.8
1
C
h
a
r
g
e

i
n

m
i
c
r
o

C
TextEnd
Time in ms
(b) Voltage is the ratio of the power and current curve. In this case, the division can be done graphically
by inspection. Note that the ratio of a quadratic function and a linear function is a linear function:
0 1 2 3 4 5 6
-1
-0.5
0
0.5
1
1.5
2
V
o
l
t
a
g
e

i
n

V
TextEnd
Time in ms
Solution 1.12 (a) V
A
= P/I = 20/4 = 5 V
(b) P
B
= VI = 27 = 14 W
(c) V
C
= P/I = -3W/3A = -1V
(d) I
D
= P/V = -27W/3V = -9A
(e) I
E
= P/V = 2/1 = 2A
(f) P
F
= VI = -45 = -20W
In all of the above, note that the direction of the current flow relative to the polarity of the voltage across a
device determines whether power is delivered or absorbed. Power is absorbed when current flows from the
positive terminal of the device to the negative one.
Solution 1.13 (a) By inspection: Circuit Element (CE) 1 absorbs 5W, and CE 2 absorbs 6W.
(b) Compute power absorbed by all elements including independent sources:
I
3A
: -15
CE1: -5
V
3V
: -12
CE2: +6
V
5V
: 10
I
2A
: 16
----------
Sum: 0 (Verifies conservation of power.)
Solution 1.14 (a) Compute power absorbed:
I
5A
: -85
CE1: 98
V
3V
: 33
CE2: 16
V
7V
: -42
I
2A
: -20
-------------
Sum: 0
(b) Add all terms:
I-source: P
absorbed
= 3 1 e
t
( )
= 3 + 3e
t
watts
V-source: P
absorbed
= 2 3e
t
1
( )
= 6e
t
+ 2 watts
CE1: P
absorbed
= 3e
t
3 1 e
t
( )
= 9e
t
9e
2t
watts
CE2: P
absorbed
= 3e
t
1
( )
3e
t
1
( )
= 9e
2t
6e
t
+1 watts
Simple algebraic manipulation of the the sum of all the above terms reveals that the result is zero.
Solution 1.15 (a) When I
L
= 1, P = V
L
I
L
= (16-4)1 = 12 W. When I
L
= 2, P = V
L
I
L
= (16-16)1 = 0.
(b) P = (16-4I
L
2
)I
L
. Differentiate this w.r.t. I
L
and set to zero: 16 12I
L
2
= 0. Therefore, I
L
= 1.155A.
Solution 1.16 (a) When I
L
= 2, P = (16-4)2 = 24W. When I
L
= 3, P = (16 - 9)3 = 21 W.
(b) Maximum occurs in the interval from 0 to 4: P = (16 - I
L
2
) I
L
Differentiate w.r.t. I
L
and set to zero: 16 3I
L
2
= 0.
Therefore, I
L
= 2.31 A.
Solution 1.17 (a) Power is the product of the current and voltage. We can compute the product
graphically:
0 0.5 1 1.5 2 2.5 3
0
2
4
6
8
10
P
o
w
e
r

i
n

W
a
t
t
s
TextEnd
Time in s
(b)
W t ( ) = p t ( )dt
0
t

= 10 10e
7
( )
d
0
t

= 10]
0
t

10
7
e
7
[ ]
0
t
=10t +
10
7
e
7t

10
7
This can be used as an aid to plot the work function:
0 0.5 1 1.5 2
0
1
2
3
4
5
6
7
8
9
Time in s
E
n
e
r
g
y

i
n

J
TextEnd
Solution 1.18 (a) Since, i t ( ) = 115 23t mA,
q 7 ( ) = i ( )d
0
7

= 115t
23t
2
2




0
7
10
3
= 0.2415 C
(b) Energy is the integral of power:
E = p t ( )dt
0
7

= v t ( ) i t ( )dt
0
7

= 25 i t ( )dt
0
7

= 25 0.2415 = 6.0375 C
Solution 1.19 (a) t = 100
o
F, Rate of temp. increase is 2.5 Wh/
o
F per gallon:
Energy = 2.5Wh/
o
F/gallon100
o
F30gallons = 7500 Wh = 2.710
7
J.
(b) Heater generates P = 12010 = 1200 W. We want 7500 Wh. Therefore, the total number of hours
needed is 7500Wh/1200W = 6.25 h.
Solution 1.20 First compute the change in temperature required, in
o
F:
t = 80-25 = 55
o
C = 559/5
o
F= 99
o
F
Next, compute the energy spent every hour, which means on 40 gallons of water:
E = 2.5 Wh/
o
F/gallon99
o
F40gallons = 9900 Wh
Since the heater is not 100 % efficient, we spend more energy than is actually needed to heat the water:
E_spent = 9900 Wh/0.9 = 11000 Wh
So, far, this was the energy spent every hour. Over six hours, the total energy spent is:
E
6h
= 110006 = 66,000 Wh
Finally, the total energy spent per month is E
m
= 66,00030 = 1980 kWh
and the bill is 1980 kWh0.14$/kWh = $277.2
Solution 1.21
Energy = 120 W 6 h = 720 Wh = 0.72 kWh
Therefore, cost per day = 0.72 kWh 8 = 5.76 cents, and cost per month is 5.7631 = $1.785.
Solution 1.22
We need to compute the difference between the inner diameter of the tube and the outer one in order to get
the cross-sectional area:
area = R
out
2
R
in
2
= 0.003
2
0.0018
2
=1.81 10
3
m
2
Then, R = 1.710
-5
(12/1.81) = 11.3 m.
Solution 1.23 L = 20 m, W = 0.015m, H = 0.001 m. Thus, A = WH, and R = 5.1
copper
L/A
= 0.116 .
Solution 1.24. (a) 500 ft, 20 gauge wire: 10.35 /1000 ft from table 1.3. This implies that
R = 5.175 .
(b) 55 ft, 20 gauge, nickel wire:
R = 5.1
10.35
1000
55 = 2.9
(c) R
tot
= 2.9 + 5.175 = 8.08 .
Solution 1.25. R T ( ) = R 20 ( ) 1 + T 20 ( ) [ ]. Substituting at T = 10 yields:
21 = R 20 ( ) 1+ 0.0039 30 ( ) [ ] or R 20 ( ) = 23.78
Evaluating at T = +10 yields,
R 10 ( ) = 23.78 + 23.78 0.0039 10 ( ) or R 10 ( ) = 22.85
Solution 1.26. For tungsten, we know that = 0.0045. Therefore:
R 150 ( ) = R 20 ( ) 1 + T 20 ( ) [ ] = 200 1 + 0.0045 150 20 ( ) [ ]
= 317
Rate of change of resistance is (317-200)/(150-20) = 0.9 /
o
C.
Solution 1.27. Plug numbers directly into the same formula as problem 1.26:
0.0022 = 0.002 + 0.0020.0039(T-20)
Rearrange to obtain: T = 45.64
o
C.
Solution 1.28. (a) Power in a wire: P = I
2
R. Rearranging, we can express the current as
I =
P
R
.
Substitute given P and R to obtain I = 0.707 mA.
(b) Use the same formula for current obtained above to get 50 A.
Solution 1.29. Use formula for power: P = V
2
/R. Rearranging, R = V
2
/P = 96 .
Solution 1.30 (a) I = V/R = 12 A, out of the positive terminal of the battery.
(b) Up through the resistor.
(c) Absorbed power by resistor: P = V
2
/R = 14.4 W. Same power is delivered by source.
(d) From table 1.2 and 1.3, 1000 feet of 18 AWG aluminum wire has resistance:
R1000ft = 1.6*6.51
R1000ft = 1.0416e+01
By proportionality, 1000 0.1 = L 10.416 . Hence,
L = 100/10.416
L = 9.6006e+00 meters.
Solution 1.31 (a) V = 10 V.
(b) P = V
2
/R, which means that R = V
2
/P = 100/25 = 4 .
(c) I = V/R = 10/4 = 2.5 A. Current flow is downwards through resistor.
(d) Up through resistor.
(e) P = V
2
/R
10
= 100/10 = 10 W. Hence, I
10
= V/R
10
= 1 A. Without applying material from a future
chapter, a legitimate way to obtain I
source
is to apply conservation of power first and then compute I
source
from the power formula. Hence, P
source
=10 + 25 = 35 watts. Using material from a later chapter, in
particular KCL, we may conclude that, I
src
= 2.5 + 1 = 3.5 A. Thus, P
source
= VI
source
= 103.5 = 35 W.
This approach indicates that power is conserved.
Solution 1.32 (a) From 0 to 1 s, i(t) = 10
-3
t. Thus, i
2
R = 10
6
t
2
R is the power absorbed during this
interval. Integrating this expression for the power from 0 to 1 s gives us the total energy used:
10
6
t
3
R
3
0
1
= 5000
10
6
3
= 0.001667 J.
Finally, we need to multiply this by 2 to account for the interval from 1 to 2 seconds. Thus, the total energy
spent is 3.33 mJ.
(b) The same charge that got transported in one direction during the interval from 0 to 1 is being
transported back in the interval from 1 to 2 (by symmetry). Therefore, total charge transfer is zero.
Solution 1.33. (a) 60 W + 120 W = 180 W.
(b) P = IV I = P/V = 180/12 = 15 A.
(c) P = Energy/Time Time = 1.2 MJ/180 W = 6.6710
4
sec = 1.85 h.
Solution 1.34. P = I
2
R. Therefore, 325 = 25(5+4+2R). Solving for R, yields R = 2 .
Solution 1.35. (a) Use definition of power and substitute given power:
V
2
= P R = 98 2 = 14 V
Similarly, I
3
=
P
R
=
12
3
= 2 A, V
4
= P R = 16 4 = 8 V, I
5
=
768.8
5
= 12.4 A, and
V
6
= 486 6 = 54 V.
(b) P
tot
= P
dissipated
= 98 + 12 + 16 + 768.8 + 486 = 1380.8 W.
(c) V
in
= V
2
+ V
6
= 68 V. I
in
+ I
3
= I
5
+ I
4
and I
4
=
V
4
4
. Thus, I
in
= I
5
+ I
4
I
3
= 12.4 A.
Solution 1.36. (a) Sources are the top, right-most, and bottom left. The reason is that current flows out
of the positive terminal of the device.
(b) The 32/16 element is a 2 resistor. The 54.5/18.167 element is a 3 resistor. The 13/2.167 element
is a 6 resistor. The 93/2.833 element is a 32.827 resistor. The 24/5 element is a 4.8 resistor.
Solution 1.37. Power: 12 = I
x
2
R, which means that R = 12/I
x
2
. Now, analyze the loop: 16 = I
x
(R+4).
Substitute the value of R into this expression: 16 = I
x
12
I
x
2
+ 4I
x
. Hence: I
x
2
4I
x
+ 3 = 0 .
This equation has two solutions: one is at I
x
= 1 A or R = 12 . The other is at I
x
= 3 A or R = 4/3 .
Solution 1.38. (a) Conservation of power:
16I
x
= 4I
x
2
+12 + 10 6
Hence
0 = I
x
2
4I
x
+ 4 = I
x
2 ( )
2
Thus, I
x
= 2 A.
(b) 32I
x
= 4I
x
2
+ 28 I
x
2
8I
x
+ 7 = I
x
7 ( ) I
x
1 ( ) = 0 . Hence, I
x
= 7A or I
x
= 1A.
Solution 1.39. (a)
(i) AA: I = 36/12 = 3A
BB: I = 24/12 = 2A
CC: I = 14.4/12 = 1.2A
(ii) Sum = 6.2A
(iii) P = VI = 6.212 = 74.4W. This is equal to the sum of the powers absorbed by the bulbs.
(iv) R = V/I
AA: R = 12/3 = 4
BB: R = 12/2 = 6
CC: R = 12/1.2 = 10
(b) Each AA bulb draws 3 A. Thus, up to five bulbs can be connected without blowing the fuse (53=15).
So, 6 or more would blow the fuse.
(c) Similar analysis suggests that 13 or more bulbs would blow the fuse. Intuitively, the bulbs draw less
current, so more of them can be used.
Solution 1.40. (a)
p(t) = i
2
t ( )R = 20cos 2t ( ) [ ]
2
10 = 4000
1 + cos 4t ( )
2
= 2000 1 + cos 4t ( ) ( ) W
(b)
W t ( ) = p t ( )dt
0
t

= 2000t + 2000 cos 4t ( )dt


0
t

= 2000t +
2000sin 4t ( )
4
J
0 0.5 1 1.5 2 2.5 3
0
1000
2000
3000
4000
5000
6000
E
n
e
r
g
y

i
n

J
TextEnd
Time in s
Solution. 1.41. When the switch is closed, a constant current of 5/10000 = 0.5 mA flows through the
circuit. When the switch is open, no current flows. So, 50% of the time, a 0.5 mA current flows, and the
other 50% no current flows. The average current is therefore 0.25 mA.
Solution. 1.42 When the switch is at A, the current is 5/5000 = 1 mA. When the switch is at B, the
current is 5/10000 = 0.5 mA. Now, the switch is at position A 20% of the time (1ms out of a 5ms period,
after which the events repeat). So, the average current is 0.2*1 mA + 0.8*0.5 mA = 0.6 mA.
Solution. 1.43 The current in the load resistor is 2 A. So, the power is 2
2
R
L
= 8 W.
Solution 1.44. V
in
= I
in
R
1
I
out
= V
in
/R
2
= I
in
R
1
/R
2
.
Solution 1.45 (a) I
1
= V
in
/R
1
. Hence, V
out
= V
in
R
2
/R
1
.
(b)
V
out
V
in
=
R
2
R
1
=
100 10
R
1
= 5 R
1
= 200
(c)
Power gain =

2
V
in
2
R
1
2
R
2
V
in
2
R
1
=

2
R
2
R
1
= 500
Solution 1.46 (a) V
1
= 200 mA 5 = 1 V implies V
2
= 0.88 = 6.4 V. Hence
V
out
= 56.4 = 32 V and I
out
= 32/64 = 0.5 A.
(b) Current Gain = 0.5/0.2 = 2.5.
(c) Power values for the 5, 8, and 64 resistors are, respectively, P
5
= 0.2 W, P
8
= 5.12 W, P
64
= 16 W.
Solution 1.47 (a) I
1
= 5A I
2
= 35/3 = 5 A I
out
= 25 A, V
out
= 50 V.
(b) Voltage Gain = 5.
(c) P
in
= 552 = 50 W, P
1
= 553 = 75W, P
2
= 2550 = 1250W.
Solution 1.48 I
1
= V
in
/10 = 0.1 A, V
R
= 10(V
in
/10)R = R; V
out
= 5R10 = 50R =50RV
in
V
out
/V
in
= 50R. If we want V
out
/V
in
to be 150, R has to be 3 .
SOLUTIONS CHAPTER 2
SOLUTION 2.1. Using KCL at the center node of each circuit:
(a) I
3
= I
2
I
1
= 1 2 = 3A
(b) I
3
= I
1
+ I
2
I
4
= 2 1 0.5 = 0.5A
SOLUTION 2.2. KCL at the bottom node gives I
1
= 7 8 = 15A , and at the right node
I
4
=6 8 = 14 . From these, KCL at the top node gives I
3
= I
4
5 = 19A, and finally at the central
node gives I
2
= 6 + I
3
7 = 20 A,
SOLUTION 2.3. Use a gaussian surface on the top triangle. Performing KCL around this surface yields
1A 2A + 3A + 4 A 5A = I = 1A.
SOLUTION 2.4. Use a gaussian surface around the bottom rectangle. KCL yields
I
1
= 2A +10A + 3A = 15A.
SOLUTION 2.5. Using KVL, V
1
= 55V 15V + 105V 100V 30V = 15V.
SOLUTION 2.6. Using KVL, V
x
= 5V 1V 1V 1V +1V 1V = 2V.
SOLUTION 2.7. Using KVL once again.
v
1
= 7 + 6 + 5 = 18V
v
2
= 6 + 7 8 = 5V
v
3
= 5 6 = 11V
v
4
= 8 7 =1V
SOLUTION 2.8. KVL is used to find the voltage across each current source, and KCL to find the current
through each voltage source.
I
3V
= 6A 7A = 1A
I
4V
= I
3V
+ 8A = 7A
I
5V
=8A 6A = 14A
V
7 A
= 4V + 3V = 7V
V
8A
= 4V + 5V = 1V
V
6 A
= V
8 A
3V = 2V
Chap 2 Probs P2 - 2 R. A. DeCarlo, P. M. Lin
SOLUTION 2.9. Using the same method as before, the current and voltages are found through and across
each sources.
I
5V
= 9 +8 7 = 10 P = 50W
I
4V
= 6 I
5V
= 16 P =64W
I
2V
= 6 7 = 1A P = 2W
I
3V
= I
2V
9 = 8A P =24W
V
8A
= 4 5 = 1V P =8W
V
9 A
= 3 + V
8 A
= 2V P = 18W
V
7 A
= 2 V
9 A
= 0 P = 0W
V
6 A
= 5 V
7A
= 5 P = 30W
Summing all the power give 0W, hence conservation of power.
SOLUTION 2.10. Doing KVL around the right loop does not balance out. Changing 8V to 5V would fix
this.
SOLUTION 2.11. Using KVL to determine the voltages, and KCL to determine the currents:
V
y
= 8V
V
x
= V
y
4 = 4V
I
a
= 4 A
I
y
= 4 14 + 2I
a
= 2A
I
x
= I
a
I
y
= 6A
SOLUTION 2.12. First V
in
= I
2
8 = 24V. Then I
1
= V
in
/ 3 = 8A and I
3
= 12A I
1
I
2
=1A.
Therefore R
L
= V
in
/ I
3
= 24 P = I
3
V
in
= 24W
SOLUTION 2.13. (a) First, from current division, get
I
1
=
1/ 3
1/ 3 +1/ 6 + 1/ R
L






12 aI
1
( ) I
1
=
12 / 3
1 + a ( ) / 3 +1/ 6 +1/ R
L






.
(b) Using the previous equation and solving for 1/ R
L
= 12/ 3I
1
( ) 1/ 6 1+ a ( ) / 3 = 0.5S or R
L
= 2.
The power P = I
3
2
R
L
=
1/ R
L
1/ 3 +1/ 6 +1/ R
L






12 aI
1
( )






R
L
= 18W
SOLUTION 2.14. For the power delivered by the source to be 60W, the voltage across it should be
V = P / 2A = 30V. Therefore the current through the 20 must be I
20
= 30 / 20 = 1.5A, and by KCL the
current through I
RL
= 2 I
20
= 0.5A. From this, R
L
= V / I
RL
= 60.
Chap 2 Probs P2 - 3 R. A. DeCarlo, P. M. Lin
SOLUTION 2.15. Writing KVL around the loop 25V 4I 15V 5I I = 0 I =1A, and
P
5
= I
2
R
5
= 5W
SOLUTION 2.16. The total power supplied by the source is P = 50V 0.5A = 25W. The power absorbed
by the resistor is P
60
= 0.5A ( )
2
60 = 15W. Therefore by conservation of power, the power absorbed by
X is 10W.
SOLUTION 2.17. (a) As this loop is open, no current flows through it, so I
R
is 0A. The output voltage is
V
OUT
= 2V + 3V 2V = 1V by KVL.
(b) Writing out the KVL equation around the loop 3 2 I
R
R 2 I
R
2R I
R
R = 0 1 = I
R
4R.
Therefore I
R
=1/ 4R and V
OUT
= I
R
R = 1/ 4V .
SOLUTION 2.18. Writing out KVL around the loop 60 30I 30 20 + 60 40I = 0 I = 1A. From
ohm's law R = V / I = 30.
SOLUTION 2.19. (a) Using Ohms law I
in
= V
2
/ 20 +12 ( ) = 0.75A, and V
1
=12 I
in
= 9V . To find R, write
KCL and get V
R
= 30 V
2
= 6V . Therefore using Ohms law again, R = V
R
/ I
IN
= 8.
(b) Writing KVL around the loop, 30 = aV
1
+ I
in
R + I
in
20 + V
1
, and substituting I
in
= V
1
/ 12,
V
1
= 30 / R + 32 ( ) /12 + a [ ] is obtained. Next substitute back V
1
=12I
in
and solve for
R =
30
12I
IN
a






12 32 = 40
SOLUTION 2.20. (a) i. Using R = V
xy
/ I
bat
the value of each resistors starting with the top one are 2.7,
0.6, and 0.25. Using the same relationship, the resistance for the motor is 1.25.
ii. Using P = V
xy
2
/ R the power dissipated by each resistor is 16.875W, 3.75W, 1.5625W, and for the
motor 7.8125W.
iii. The relative efficiency is = 7.8125/ (12 2.5) 100 = 26 %
(b) i. Performing voltage division across each resistor
V
AB
= 0
V
BC
=12 R
BC
/ (R
BC
+ R
CD
+ R
motor
) = 3.43V
V
CD
= 12 R
CD
/ (R
BC
+ R
CD
+ R
motor
) =1.43V
V
motor
= 12 R
motor
/ (R
BC
+ R
CD
+ R
motor
) = 7.14V
Chap 2 Probs P2 - 4 R. A. DeCarlo, P. M. Lin
ii. I
bat
= 12 / (R
BC
+ R
CD
+ R
motor
) = 5.71A
iii. The relative efficiency is = (V
motor
2
/ R
motor
)/ (12 5.71) 100 = 59.5 %
(c) i. Repeating the steps from (b), the voltages across the first two resistance are 0, then across the other
and the motor 2V, and 10V
ii. I
bat
= 12 / (R
CD
+ R
motor
) = 8A
iii. And the relative efficiency is = (V
motor
2
/ R
motor
)/ (12 8) 100 = 83.3 %
(d) What is the largest equivalent resistance of the motor that will draw 30A? R = 12/ 30A = 0.4.
SOLUTION 2.21. (a) Observe that i = I
O
, thus v = ki
3
= kI
0
3
.
(b) Using KVL and previous equation, v
x
= (R
1
+ R
2
)I
O
+ V
O
+ kI
O
3
.
(c) The power is = I
O
v
x
= (R
1
+ R
2
)I
O
2
+ V
O
I
O
+ kI
O
4
SOLUTION 2.22. I
100
=
0.04
100
= 0.02 A. Therefore V
300
= 0.02 (100 + 200) = 6 V. By KCL,
I
150
= 0.02 +
6
300
= 0.04 A. R
eq
, seen by the source, is 300 . Therefore V
s
= 0.04 300 = 12 V.
SOLUTION 2.23. Using KCL I
R
= 5 3 = 2A, and KVL V
R
= 10 + 6 = 16V . Thus
R = V
R
/ I
R
=16 / 2 = 8.
SOLUTION 2.24. Using KCL, KVL, along with Ohms law,
I
5
= 6 7 = 1A
I
4V
= 8 I
5
= 9A
I
2
= 8 + 7 =15A
V
6 A
= 4 + 5I
5
= 1V
V
8A
= 4 + 2I
2
= 26V
V
7 A
= V
8 A
5I
5
= 31V
Now, the power delivered or absorbed by each element is calculated:
P
6A
= I
6A
V
6 A
= 6W
P
7A
= I
7A
V
7 A
= 217W
P
8A
= I
8A
V
8A
= 208W
P
4V
= I
4V
4 = 36W
P
5
= I
5
2
5 = 5W
P
2
= I
2
2
2 = 450W
Chap 2 Probs P2 - 5 R. A. DeCarlo, P. M. Lin
Note that for passive elements, when the power is positive it is absorbed, while for independent sources it is
generated when the power is positive.
SOLUTION 2.25. Note that I
1
= 6A. Thus by KCL
I
3
= 6 0.5I
1
= 3A
I
2
= 2 + 0.2I
1
= 3.2A
I
4
= 8 0.3I
1
= 6.2A
And finally using KVL
V
2
= 8A 1+ 4I
4
+ 3I
3
= 41.8V
V
1
= 2I
2
3I
3
= 2.6V
SOLUTION 2.26. (a) Using KCL,
I
4
= 5 4 =1A
I
3
= I
4
2 = 1A
I
2
= 3 2 = 1A
I
1
= I
2
5 = 6A
(b) Using KVL and Ohms law,
V I V
V I V
V I V
V I V
4 4
10 10
12 12
18 3
4 4
3 3
2 2
1 1
= =
= =
= =
= =
(c)
W V V V P
W V V P
W V V V P
W V V A P
A
A
A
A
60 ) ( 5
24 ) ( 4
40 ) ( 2
90 ) ( 3
1 3 4 5
4 3 4
3 2 1 2
1 2 3
= + =
= =
= =
= =
Chap 2 Probs P2 - 6 R. A. DeCarlo, P. M. Lin
SOLUTION 2.27. Write KVL around the outside loop, 40 = 500I
x
+ (400 + 200)i . And write KCL
equation i = I
x
2I
x
. Solving yields I
x
= 0.4A. The dependent source delivers 2I
x
(600i) = 192W, and
the independent 40I
x
= 16W. Finally the resistors absorb 500I
x
2
+ 400i
2
+ 200i
2
=176Wverifying the
conservation of energy since the source generate 192W-16W=176W.
SOLUTION 2.28. By voltage division V
2
=
[((90 ||180) + 60)||40]
[((90||180) + 60)||40] +160

60
60 + (90||180)
V
s
= 1/14 V
s
.
Therefore V
s
= 14V
2
= 280V .
SOLUTION 2.29. By voltage division
v
x
= 9V
18 + 3
(18 + 3) + 6
= 7V
SOLUTION 2.30. By voltage division we get the following two equations in order to solve for the two
unknowns.
V
2
= V
1

R
1
R
1
+ R
2
V
1
=100V
R
1
+ R
2
R
1
+ R
2
+ 60
Solving yields R
1
= 40, and R
2
= 100.
SOLUTION 2.31. Dividing 1400 in four gives 350. If we only need 1/4 and 2/4, the resistor string can be
made of three resistances: 350, 350, and 700.
SOLUTION 2.32. Using voltage division, at t=0 v
R
= 15
2R
3R
= 10V , and t = 5 s v
R
= 10V , and at t = 10
the voltage goes back to 0V.
Chap 2 Probs P2 - 7 R. A. DeCarlo, P. M. Lin
0 5 10 15
0
1
2
3
4
5
6
7
8
9
10
time (sec)
V
o
l
t
s
SOLUTION 2.33. By voltage division
V
b
=
R
b
R
a
+ R
b
V
in
and V
d
=
R
d
R
c
+ R
d
V
in
By KVL, if V
out
= 0, then
0 = V
out
= V
b
V
d
=
R
b
R
a
+ R
b

R
d
R
c
+ R
d






V
in
For arbitrary V
in
, this requires that
R
b
R
a
+ R
b
=
R
d
R
c
+ R
d
or equivalently that R
b
R
c
= R
a
R
d
.
SOLUTION 2.34. First G
eq
= 1m +1.5m+ 2m+ 3m = 7.5mS . By current division
I
2
=100mA
1.5m
G
eq
= 20mA, P = 100mA I
2
/1.5mS =1.33W.
SOLUTION 2.35. By current division, for I
1
to be 2A then 160 + R = 300 ||600 for an even split. Thus
R = 40.
SOLUTION 2.36. By current division, i
1
= 0.4A
1/10
1/10 + 1/ 40
= 0.32 A. Therefore using KVL
v
d
=10i
1
0.25i
1
= 3.12V .
SOLUTION 2.37. (a) R
eq
= (8k | | 2k) + (9k ||1k) = 2.5k
Chap 2 Probs P2 - 8 R. A. DeCarlo, P. M. Lin
(b) R
eq
= 2k ||[(2k | | 2k) + (2k | | 2k)] = 1k
SOLUTION 2.38. (a) R
eq
= 2 + 15 +10 + 10 + 40 + 30 + 20 + 8 =135.
(b) Four of the resistors are shorted, thus R
eq
= 2 + 15 +10 + 8 = 35.
(c) Lumping the series resistance together R
eq
= 8 +[50 ||(50||25)] + 2 = 22.5
SOLUTION 2.39. (a) R
eq
= [2R + (4R | | 4R)]||[2R+ (4R| | 4R)] = 2R
(b) R
eq
= 2R| | 2R + (4R | | 4R | | 4R| | 4 R) = 2R
SOLUTION 2.40. (a) First R
eq
= 150 + [375||(250 + 500)] = 400. Next I
in
= 14V / R
eq
= 35mA. The
power delivered by the source is then = 14I
in
= 0.49W .
(b) R
eq
= 150 + [375||(250 + 500)||1k] = 350, and I
in
= 14 / R
eq
= 40mA. The power delivered by the
source is = 14I
in
= 0.56W .
As the equivalent resistance decreases, more of it gets dissipated by it.
SOLUTION 2.41. R
eq1
> R
eq2
. Without going into a detailed analysis using methods of Chapter 3, we
present the following intuitive argument. First note that the points a and b represent points on an
unbalanced bridge circuit meaning that the voltage between a and b would not be zero. Also note that when
two resistors are placed in parallel, the equivalent resistance becomes smaller than either resistance. The
addition of the resistor R in circuit 2 essentially creates an internal parallel resistance resulting in an R
eq2
lower than R
eq1
.
SOLUTION 2.42. R
eq1
= R
eq2
. As was the case in the previous problem, this is a balanced bridge circuit.
Hence no voltage appears between a and b making the additional resistor irrelevent.
SOLUTION 2.43. (a) R
in
= [(20||20) +10]||(1/ 0.12)||(1/ 0.08) = 4.
(b) R
in
= 6R ||[( R || R ||0.5R) + 0.75R + (2R| | 2R)] = 1.5R
SOLUTION 2.44. (a) The infinite resistance are essentially open circuits, thus
R
eq
= 1 + 2 + 3 + 4 + 2 + 4 + 3 + 2 +1 = 22
(b) 0 resistances are short circuits. Labeling one branch x and the other y, it can be seen that the circuit is a
set of 3 resistor strings in parallel to each other between x and y, then added in series to the two 1 Ohm
resistor. Thus R
eq
= [(2 + 3)||(4 + 2 + 4)||(2 + 3)]+1 +1 = 4.
Chap 2 Probs P2 - 9 R. A. DeCarlo, P. M. Lin
(c) Writing out R
eq
= 1 +[R
x
||(2 + 3 + 4 + 2 + 4)] + 3 + 2 +1 = 7 +[R
x
||15] , and solving for R
x
= 3.75.
(d) No, it requires methods to be covered in the next chapter.
SOLUTION 2.45. Using the formulas for parallel resistances, the circuit of figure 2.45 reduces to
(a) R
AC
= (2 +1) / /6 ( ) + 8 [ ] / /10 = 5
(b) R
AB
cannot be calculated by series parallel formulas, but R
BC
can be done.
R
BC
= (8 +10) / /6 ( ) + 2 [ ] / /1 = 0.86667
SOLUTION 2.46. (a) R
eq
= 300 + (R ||5.6k), thus R = 800.
(b) R
eq
= R+ (R ||1.2K) , the following quadratic equation must be solved R
2
+1.4k R 1.2M = 0 . This
yields R = 600.
(c) R
eq
= 500 + 300 + (800||400 || R). Solving for R yields 800.
SOLUTION 2.47.
(a) Using the fact that the resistance seen into terminal a-b is the same as that seen in terminal c-d, we can
obtain the following relationship.
eq eq
R R R R || + = . This produces a quadratic equation whose solution is
R
eq
= 1.618R.
(b) Using the previous argument ) 5 ( || 10 5
eq eq
R R + + = . Solving for = 18 . 11
eq
R .
SOLUTION 2.48. By current division I
x
=
1/ 18k
1/18k +1/ 9k






1
6k +(9k ||18k)
1
6k + (9k ||18k)
+
1
4k










36m= 3mA
Chap 2 Probs P2 - 10 R. A. DeCarlo, P. M. Lin
SOLUTION 2.49. The 500 resistor has no effect on the current entering the circuit to its right.
0.15 =
30
R
+
30
600
=
30
R
+ 0.05
Hence R = 30/0.1 = 300 .
SOLUTION 2.50. (a) First, express the total current as I =
120
0.5 + (20 | | 30| | 40 || R
L2
)
. Next, find R
L2
that
will cause I to be 15A. Thus R
L2
= 40 or less will cause the fuse to blow as this will cause the current to
be 15A or more.
(b) Repeating the previous procedure, R
L2
= 20.
(c) R
L2
= 120.
SOLUTION 2.51. At time 0, all switches are open and V
out
=
260
260 + 40
220 =190.7V .
Then at t = 5s, switch one closes and
V
out
=
260||260
(260 ||260) + 40
220 = 168.2V .
At t = 10s,
V
out
=
130 ||260||260
(130 ||260||260) + 40
220 =136.2V .
Finally at t =15 s,
V
out
=
65||130 ||260||260
(65||130 ||260||260) + 40
220 = 98.5V .
0 2 4 6 8 10 12 14 16 18 20
80
100
120
140
160
180
200
Time (sec)
V
o
l
t
s
Chap 2 Probs P2 - 11 R. A. DeCarlo, P. M. Lin
SOLUTION 2.52. (a) Lumping the two sources together and the resistors into an equivalent resistor gives
i
1
(t) =
9cos(2t) 3cos(2t)
7k + 9k + 8k + (2k | | 3k | | 6k)
= 0.24cos(2t)mA.
(b) By current division i
2
(t) =
1/ 6k
1/ 2k +1/ 3k +1/ 6k





i
1
(t) = 40cos(2t) A.
SOLUTION 2.53. (a) Starting with,
R
eq1
= 5||(10 + 10) = 4
R
eq 2
= 10||(6 + R
eq1
) = 5
R
eq 3
= 5 + R
eq2
= 10
(b) Using the values R just obtained,
V
a
= 100
R
eq 2
R
eq3
= 50V
V
b
= V
a

R
eq1
6 + R
eq1
= 20V
V
c
= V
b

10
10 +10
= 10V
(c) Finally,
I
in
=
100
R
eq3
=10A
I
d
=
V
a
6 + R
eq1
= 5A
I
e
=
V
b
10 +10
= 1A
.
SOLUTION 2.54. (a) Circuit a: Using voltage division,
v
out
(t) = v
in
(t)
300||(20 + 30 + 50)
[300 ||(20 +30 + 50)] + 5






30
30 + 20 + 50




= 33.75sin(377t)V, and Ohms law
i
out
(t) = v
out
(t)/ 30 = 1.125sin(377t )A. The instantaneous power is then
P(t) = i
out
(t) v
out
(t) = 37.969sin
2
(377t)W .
Circuit b: By current division
i
out
(t) = i
in
(t)
1/ (20 + 30 + 50)
1/ (20 + 30 + 50) +1/ 300 +1/ (50 +100)





= 60sin(377t)A, and from Ohms law
v
out
(t) = 50 i
out
(t) = 3000sin(377t)V . The instantaneous power is P(t) = 180sin
2
(377t)kW.
(b) No, since the current source forces the amount of current in the circuit.
Chap 2 Probs P2 - 12 R. A. DeCarlo, P. M. Lin
SOLUTION 2.55. (a) Noting that i
2
= v
1
/10 = 6A, then we can write KCL at the top left node,
i
source
= i
2
+ v
1
/ 6 + (v
1
5i
2
)/ 5 = 22A. Thus P = 60 22 = 1.32kW.
(b) First, determine the current through each resistor:
i
2
= 60 /10 = 6A
i
2.5
=
60
2.5 + (5||5)





= 12A
i
5
= 1/ 2 i
2.5
= 6A
Then calculate the power absorbed by each resistor:
P
10
= 10i
2
2
= 360W
P
2.5
= 2.5i
2.5
2
= 360W
P
5
= 5i
5
2
= 180W
SOLUTION 2.56. From Ohms law I
1
= 100m/ 200 = 0.5mA. By current division
I
RL
=
20k
20k + 200




150I
1
= 75.257mA, and P
RL
= 200I
RL
2
= 1.103W .
SOLUTION 2.57. First, using voltage division, V
x
= V
s
2
2 +1




= (2 / 3)V
s
. Then using KCL and the
previous equation, I
s
= (V
s
/ 3) V
x
= (1/ 3)V
s
. Finally using Ohms law R
eq
= V
s
/ I
s
=3.
SOLUTION 2.58. Observing the following relationship, V
1
= V
in
, the following nodal equation can written:
I
in
= V
in
/ 3 + V
in
/ 6 2V
in
= 1.5V
in
.
SOLUTION 2.59. Step 1. From voltage division
V
1
=
18
18 + 4 + 2
V
s
= 0.75V
s
and V
in
=
22
24
V
s
=
11
12
V
s
Hence
P
in
=
V
in
2
22
=
1111
22 144
V
s
2
=
11
288
V
s
2
Step 2. For the load, by current division
Chap 2 Probs P2 - 13 R. A. DeCarlo, P. M. Lin
I
2
=
6
6 + 2
AV
1
=
3
4
A
3
4
V
s




=
9A
16
V
s
Therefore
P
2
= 2 I
2
2
= 2
81A
2
256
V
s
2
=
81A
2
128
V
s
2
Step 3. P
2
= 10 P
in
implies that
81A
2
128
V
s
2
=10
11
288
V
s
2
Hence
A =
128 110
81 288
= 0.7769
SOLUTION 2.60. By voltage division V
1
=
6
6 + 2
V
in
= (3/ 4)V
in
. By current division, and substituting the
previous equation I
2
=
3
3 + 6
4V
1
= V
in
. Using voltage division and Ohms law, and substituting the previous
equation,
V
out
= 4.5I
2
10
10 + 5
= 3V
in
= 30V
I
out
= 4.5I
2
/ (10 + 5) = 0.3V
in
= 3A
Finally, from the previous equations | V
out
/ V
in
|= 3 .
SOLUTION 2.61. Writing out KCL when the switch is closed, i
bat
= 150A +
V
bat
0.04i
bat
240




. Solving
gives i
bat
= 150.02A and V
out
6V . When the switch is open V
out
= V
bat
240
240 + 0.04




12V. Therefore,
the reason for the radio stopping is insufficient supply voltage.
SOLUTION 2.62. (a) Using the following relationship P = V
2
/ R, the resistance of each headlight on low
beam is R = V
2
/ P = 4.11.
(b) Using the same relationship R = 2.22.
(c) By voltage division, V
out
=14.7
240
240 + 0.04
= 14.698V .
(d) Using voltage division, V
out
= 14.7
240 ||4.11||4.11
(240||4.11||4.11) + 0.04
= 14.417V
Chap 2 Probs P2 - 14 R. A. DeCarlo, P. M. Lin
(e) Using voltage division, V
out
=14.7
240||2.22||2.22
(240 ||2.22||2.22) + 0.04
= 14.186V
SOLUTION 2.63. By voltage division
11.96 =
15
15 + R
0
12 =
180
15 + R
0
Therefore
R
0
=
180 15 11.96
11.96
= 0.050167
SOLUTION 2.64. (a) Using KVL V
t
= 102 0.05 80 = 98 V.
(b) Using KVL V
t
= 102 + 0.05 50 =104.5 V.
(c) P = V
t
50 = 5.225. kW
SOLUTION 2.65. Minimum load means the minimum load resistance that the system can handle.
MaxPwr = 0.8*50e6
MaxPwr = 40000000
Vs = 750e3;
Iline = MaxPwr/Vs
Iline = 5.3333e+01
Rmin = Vs/Iline
Rmin = 1.4062e+04, i.e., Rmin = 14.062 k.
SOLUTION 2.66. (a) Using the following general form for a non-ideal voltage source: v
out
= R
s
i
out
+ V
s
,
one sees that for zero current v
out
= V
s
= 40V . The slope of the line is
40
1000
= R
s
= 0.04, thus
R
s
= 0.04.
(b) This curve represents a resistors I-v characteristic, thus the slope
60
1
1
3
= R = 45.
(c) The general form for a non-ideal current source is i
out
=
1
R
s
V
out
+ I
s
. When the voltage is zero,
i
out
= I
s
= 5A. From the slope of the line,
4000
5
, R
s
= 4000 / 5 = 800 .
Chap 2 Probs P2 - 15 R. A. DeCarlo, P. M. Lin
SOLUTION 2.67. Using the following formula:
T
n
nI
C
n






= 1, solve for T, and get 0.625 hrs, or 37.5 min.
SOLUTION 2.68. Using the same equation as before and solving for C
n
= nI
T
n




1/
, with n=10, and
T=55/60 hrs, the capacity obtained is 20 Ah.
SOLUTION 2.69. C
20
= 50 Ah
(a) In eq. 9, solving for I with n=20, and T=10, I=4.2A
(b) Calculate the capacity for n=10 and T=10, this yields 42 Ah.
SOLUTION 2.70. (a) Using a sequence of voltage division,
V
1
= 50mV
48
50
= 48mV
V
2
= 50V
1
195
200
= 2.34V
V
load
= 2.5V
2
= 5.85V
And the power is R
RL
= V
load
2
/ R
L
= 2.278W .
(b) Following is the graph, and the script used to generate it.
0 10 20 30 40 50 60 70
0
1
2
3
4
5
P
o
w
e
r

i
n

W
a
t
t
s
0 10 20 30 40 50 60 70
0
200
400
600
800
Resistance in Ohms
C
u
r
r
e
n
t

i
n

m
A
%Script for Question 70 in chapter 2
RL=8:1:64;
V2=2.34;
IL=2.5*V2 ./ RL;
%Note the use of the ".*" which means that the division
%is performed for each value of RL.
Chap 2 Probs P2 - 16 R. A. DeCarlo, P. M. Lin
PL=RL .* (IL .^ 2);
%Plot the Power versus RL
subplot(2,1,1);
plot(RL,PL);
ylabel('Power in Watts');
%Plot the Current versus IL
subplot(2,1,2);
plot(RL,1000.*IL);
xlabel('Resistance in Ohms');
ylabel('Current in mA');
%The use of subplot lets you subdivide the graphing
%window in two halfs.
SOLUTION 2.71. (a) Using the following script:
%Script for problem 2.71
R1=15; R2=4; R3=9; R4=2; R5=8;
R6=18;
Ra= R4+R5;
Ga= 1/Ra;
Gb= Ga+1/R1;
Rb= 1/Gb;
Rc= 1/(1/R6+1/R3)+Rb;
Gc= 1/Rc;
Geq= Gc+1/4;
Req= 1/Geq;
Irc= 20*Gc/Geq;
Vrb= Irc*Rb;
Vout= Irc*(Ga/Gb)*8;
Req
Vout
So (a) R
eq
= 3, and (b) V
out
= 24V
SOLUTION 2.72. Using the following script:
%Script for problem 2.72
R1=1e3; R2=2.2e3; R3=2e3; R4=5e3; R5=3e3; R6=R5;
R7=3.2e3; R8=1.2e3; R9=1.6e3;
Chap 2 Probs P2 - 17 R. A. DeCarlo, P. M. Lin
Ga=1/R7+1/(R8+R9);
Ra=1/Ga;
Gb=Ga+1/R6;
Rb=1/Gb;
Gc=1/R4+1/(R5+Rb);
Rc=1/Gc;
Gd=1/R2+1/(R3+Rc);
Rd=1/Gd;
Geq=1/R1+Gd;
Req=1/Geq
%Going through the same step to find Vout
Id=200e-3*(Gd/(Geq));
Ic=Id*((1/(R3+Rc))/Gd);
Ib=Ic*((1/(R5+Rb))/Gc);
Ia=Ib*Ga/Gb;
Iout=Ia*((1/(R8+R9))/Ga)
Vout=Iout*R9
The following values are obtained:
Req =
5.9121e+02
Iout =
5.5431e-03
Vout =
8.8689e+00

R1
a b c d
R9 R7 R6 R4 R2
SOLUTION 2.73. Using the following script:
%Script for problem 2.73
R1=20; R2=40; R3=60; R4=30; R5=10; R6=135;
R7=150; R8=300; R9=130; R10=200; R11=50;
Ga=1/R10+1/R11;
Ra=1/Ga;
Rb=Ra+R9+(1/(1/R7+1/R8));
Chap 2 Probs P2 - 18 R. A. DeCarlo, P. M. Lin
Gb=1/Rb;
Gc=Gb+1/R6;
Rc=1/Gc;
Rd=Rc+R5+(1/(1/R3+1/R4));
Gd=1/Rd;
Ge=Gd+1/R2;
Re=1/Ge;
Rin=R1+Re
Ie=10/Rin;
Id=Ie*Gd/Ge;
I1=Id*(1/R6)/Gc
Ib=Id*Gb/Gc;
Vout=Ib*Ra
The following values are obtained:
R
in
= 50
V
out
= 0.667V
I
1
= 33.3mA

R1
e d
R2
R11
R10
R9
R8
R7
R6
R5
R3
R4
a c b
SOLUTION 2.74. An identical procedure to the one followed in the previous problem will yield the
following values:
R
in
= 50.53
I
out
= 133.8mA
PROBLEM SOLUTIONS CHAPTER 3.
Solution 3.1. Select the bottom node as the reference node, and write a node equation at the positive
terminal of the V
1
resistor:
V
1
V
0
3R
+
V
1
6R
+
V
1
4V
0
6R
0
2V
1
2V
0
+ V
1
+V
1
4V
0
0
4V
1
6V
0
V
1
1.5V
0
Solution 3.2 Write a node equation at the top node:
0.6
V
x
100

2V
x
100

V
x
50
0
V
x
2V
x
2V
x
60
5V
x
60
V
x
12V
Solution 3.3
0.6
V
x
100
+
25V
x
100

V
x
50

V
x
0.2V
x
40
0

3V
x
100
+
25V
x
100

8V
x
400
0.6
V
x

0.6 400
80
V
x
3V
Solution 3.4 (a)
It is evident from the figure that V
c
= 20. We need to write two equations in V
a
and V
b
and put them in
matrix form. In this case, we can write the matrix equation by inspection. Note that the resistors are
identified by conductance values.
15m 5m
5m 35m



1
]
1
V
a
V
b



1
]
1

0.5
0.5



1
]
1
(b) Solve the matrix equation by inverting the left-most matrix:
V
a
V
b



1
]
1

1
525 25
35m 5m
5m 15m



1
]
1
0.5
0.5



1
]
1

1
0.5
35 5
5 15



1
]
1
0.5
0.5



1
]
1

40
20



1
]
1
(c) V
x
V
ab
V
a
V
b
20V , V
da
= -V
a
= -40, V
db
= -V
b
= -20.
(d) P
i
= 0.540 = 20W, P
v
= 20(20-20) = 0. P
diss
= 404010m + 20205m + 20205m = 20 W.
Power delivered equals dissipated power.
Solution 3.6 Write two nodal equations:
V
s1
V
1
3000
I
s3
+
V
1
V
2
6000
V
2
30000

V
s2
V
2
12000
+
V
1
V
2
6000
Rewrite equations as:
2V
s1
2V
1
6000I
s3
+V
1
V
2
2V
2
5V
s2
5V
2
+10V
1
10V
2
Cast into a matrix equation
3 1
10 17



1
]
1
V
1
V
2



1
]
1

6000I
s3
2V
s1
5V
s2



1
]
1
Solving the matrix equation yields:
V
1
V
2



1
]
1

181.46
124.39



1
]
1
Power absorbed by the 6k resistor is (V
1
-V
2
)
2
/R = 0.5429W .
Similarly, P
s1
= (V
s1
-V
1
)/3000V
s1
= 4.7W, P
s2
(V
s2
V
2
) /12000 V
s2
0.32W
P
s3
I
s3
(V
s2
V
1
) 1.21W
Solution 3.7 (a) Again, the matrix equation can be written by inspection:
G
1
+ G
2
+ G
4
G
4
G
4
G
3
+ G
4
+ G
s



1
]
1
V
B
V
C



1
]
1

50G
1
50G
3



1
]
1
(b) Substituting the values of conductances and inverting the above matrix equation yields:
V
B
34.0132V
V
C
= 33.6842V
(c) Power delivered is 80.7566W . Using the Principle of Conservation of Power:
P
del
P
1
+ P
2
+ P
3
+ P
4
+ P
5
or,
P
del
50
V
A
V
B
20
+
V
A
V
C
20



1
]
1
80.7566W
(d) In this part, we take the above matrix equation and solve it for each value of G
s
. If we do this, we can
get a feel for the behavior of V
B
and V
C
w.r.t. changes in G
s
. The following plot is the voltage difference
between the two nodes as a function of G
s
, and hence as a function of temperature.
As can be seen, in this figure, the voltage difference between B and C does not change linearly with R
s
.
Since this resistance itself changes linearly with temperature, this means that V
B
-V
C
does not change
proportionally to temperature.
Solution 3.8 The answer is:
G
1
+ G
2
G
1
G
2
0
G
1
G
1
+ G
3
+ G
4
+ G
7
G
4
G
7
G
2
G
4
G
4
+ G
5
+ G
6
+ G
2
G
6
0 G
7
G
6
G
6
+ G
8
+ G
7







1
]
1
1
1
1
1
V
1
V
2
V
3
V
4







1
]
1
1
1
1
1

I
s1
0
0
I
s2







1
]
1
1
1
1
1
Solution 3.9 Write the matrix equation by inspection:
4 /100 1/100 1/100
1/100 4 /100 1/100
1/100 1/100 4 /100





1
]
1
1
1
V
1
V
2
V
3





1
]
1
1
1

V
s1
100
V
s1
100
I
s2







1
]
1
1
1
1
1
Solving the equation in MATLAB, we get: V
1
= 7, V
2
= 7, V
3
= 11, and P = 0.6.
P V
s1

V
S1
V
1
100
+
V
S1
V
2
100


_
,
10
3
10
+
3
10


_
,
0.6W
Solution 3.10 (a)
Nodal equation for A:
V
A
V
s1
10
+
V
A
V
B
10
+
V
A
V
C
10
0
(b) At node B:
V
B
10
+
V
B
V
A
10
I
s2
0
(c) At node C:
I
s2
I
s3
+
V
C
V
A
10
0
(d) Manipulate algebraically to cast as the following matrix equation:
3 1 1
1 2 0
1 0 1





1
]
1
1
1
V
A
V
B
V
C





1
]
1
1
1

V
s1
10I
s2
10 I
s3
I
s2
( )





1
]
1
1
1
V
A
V
B
V
C





1
]
1
1
1

13
1
3
13
1
3
43
1
3






1
]
1
1
1
1
(e) P = (-10-13.333)/10(-10) = 23.33W.
Solution 3.12
We are required to write the equations in matrix form. First, write a node equation at V
A
and V
out
:
V
A
5 +
V
A
5
+
V
A
V
out
10
0
V
out
V
A
10
7.5V
A
+
V
out
10
0
Now group the coefficients for V
A
and V
out
, and write the matrix equation:
1+1/ 5 +1/10 1/10
1/10 + 7.5 2/10



1
]
1
V
A
V
out



1
]
1

5
0



1
]
1
V
A
V
out



1
]
1

1
37



1
]
1
where the matrix inversion was performed in MATLAB. The ratio of the output voltage to the input voltage
is -37/5.
Solution 3.13 (a) Nodes A and B are already labeled:
V
A
9 ( )0.1 + V
A
0.5V
B
( )0.2 + V
A
V
B
( )0.3 0
V
B
V
A
( )0.3 + V
B
9 ( )0.5 + 0.4V
B
0
This can be rearranged Into:
0.1+ 0.2 + 0.3 ( )V
A
0.3V
B
0.2 0.5 ( )V
B
0.9
0.3V
A
+ 0.3 + 0.5 + 0.4 ( )V
B
4.5
The matrix equation can now be easily obtained:
0.6 (0.5)(0.2) 0.3
0.3 1.2



1
]
1
V
A
V
B



1
]
1

0.9
4.5



1
]
1
V
A
V
B



1
]
1

4.8
4.95



1
]
1
(b) I
in
= -(V
A
-V
s
)0.1 - (V
B
-V
s
)0.5 = 2.445A
(c) P
s
= V
S
I
in
= 22.005W, P
dep
0.5 V
out
(0.5V
out
V
A
) 0.2 1.151W .
(d) P = V
2
/R = (4.95)(4.95)0.4 = 9.801W.
Solution 3.14 (a) We write node equations at V
A
and V
B
:
I
s1
+
V
A
20k
+ gm
1
V
A
+
V
A
V
B
10k
0
V
B
V
A
10k
gm
2
V
A
V
B
( ) +
V
B
2.5k
+ I
s2
gm
1
V
A
0
Rearranging, we have:
1
20k
+ gm
1
+
1
10k


_
,
V
A

V
B
10k
I
s1
1
10k
gm
2
gm
1


_
,
V
A
+
1
10k
+ gm
2
+
1
2.5k


_
,
V
B
I
s2
(b)

1/20k + gm
1
+1/10k 1/10k
1/10k gm
2
gm
1
1/10k + gm
2
+1/2.5k



1
]
1
V
A
V
B



1
]
1

I
s1
I
s2



1
]
1
(c) The above matrix is inserted into MATLAB, with all the values substituted, to obtain:
V
A
V
B



1
]
1

9.722
5.972



1
]
1
(d) V
o
= V
A
-V
B
= 3.75V
(e) P
1
= V
A
I
s1
= 0.0292W, P
gm1
= -V
o
gm
1
V
A
= -0.008W, P
gm2
= V
B
gm
2
V
o
= 0.0112W, P
2
= -V
B
I
s2
= -
0.0119W.
Solution 3.15 I
1
= 0.4. Write nodal equations at A and B
V
A
100
+
V
A
V
B
20
+ 0.03 V
A
V
B
( ) 0.4
V
B
V
A
20
+
V
B
40
+
V
B
80
V
B
40
40
0
Rearranging and casting into matrix form:
1/100 +1/20 + 0.03 1/20 0.03
1/20 1/20 +1/40 1/40



1
]
1
V
A
V
B



1
]
1

0.4
0



1
]
1
V
A
V
B



1
]
1

40
40



1
]
1
It is obvious then, that V
x
, the voltage between A and B, is zero.
Solution 3.16
1) V
A
3000i
x
3000
V
A
V
B
9000

V
A
V
B
3
Equation at node B:
V
B
V
A
9000
+
V
B
6000
+
V
B
V
D
18000
0 is equivalent to:
2) 2V
A
+ 6V
B
V
D
0
Equation at node D:
V
D
V
B
18000
+
V
D
9000
+ I
S
0 which can be rewritten as:
3) 3V
D
V
B
360 Solving the system formed by equations (1), (2) and (3) we obtain:
V
A
9V , V
B
18V , V
D
126V .
Solution 3.17 (a) Choose E as the reference node
V
A
= 2i
x
At node B
6 = (V
B
-V
C
)/3 + (V
B
-V
A
)/2
Or 6 = 5/6V
B
1/3V
C
V
A
/2
At node C, V
C
= 2i
y
At node D, VD = -12V
iy = (VD-VA)/2 = VD/2 ix
From here on, the solution involves algebraic manipulations to solve the system of equations. MATLAB or
hand analysis can be performed to obtain:
V
A
= 48V, V
B
= 12V, V
C
= -60V.
(b) P
6A
= 6(12 = 72W
I
12V
= 30-8 = 22A ( P12V = 264W.
P
2i
x
2i
x
(i
z
i
Y
) 2 24 (+18 + 30) 2304W
P
2i
y
2i
y
(i
x
+
v
c
v
D
6
) 2 (30) (24 8) 1920W
(c) P
3
= i
x
(V
B
V
C
) 1728W = 1728W
P
6
= 688 = 384W
P
2 y
= 2i
y
i
y
= 1800W
P
2
= (V
B
-2i
x
) (V
B
-2i
x
)/2 = 648W.
(c) V
D
= -12V, V
A
= 2i
x
, V
C
= 2i
z
.
Substitute the above V
A
and V
C
into the node equation for node B:
i
z
= (V
B
-2i
x
)/2 = V
B
/2 i
x
and
i
x
= (V
B
-2i
z
)/3 = V
B
/3 2/3i
z
Substitute i
z
into i
x
to obtain: i
x
= 0, V
A
= 0. Then, V
B
can be deduced to be 12.
Finally, V
C
= 12V.
Now, compute the powers.
P
6A
= 72W , P
3
= 0, P
6
= (V
C
-V
D
)
2
/6 = 96W, P
2 y
= 72W, P
2 z
= 72W.
P
2ix
0W, P
2iy
12 4 48W
Solution 3.18 The three node equations at A, C, and D are:
0.8 0.3 0.015V
A
+ 0.02V
A
0.02V
C
V
c
440
0.8 + 2.5 0.005V
D
+ 0.025V
C
0.025V
D
As can be seen, these really reduce to only two equations in two unknowns. These can be solved rather
easily either by hand or by MATLAB to obtain: V
A
= 220V, V
D
310V . Note that all of these voltages
are already referenced to node B (i.e. V
A
= V
AB
, etc).
Solution 3.19 (a) Supernode is BC (50 V source).
(b) Only one node equation needs to be written:
P
12V
12 (
V
A
V
D
2
+
V
C
V
D
6
) 12 (6 + 4) 120W,
V
B
90

V
A
90
+
V
B
10
+
V
C
10

V
A
10
+
V
C
90
0
with the constraint that V
C
V
B
50.
(c) The constraint equation can be substituted into the B node equation to obtain
V
B
= 125V. Thus, V
C
= 175V, and i
x
= (V
C
-V
A
)/10 = -12.5A.
(d) (V
A
-V
B
)/90 = 1.94A P
300
= 300(1.94+12.5) = 4332W.
V
C
/90 = 1.94 P
50
= -528W.
P
50
50 (i
x
+
V
C
90
) 528W
Solution 3.20 (a) V
B
= V
A
440 and V
C
= V
A
460.
(b) Supernode is one including A, B, and C.
(c) V
C
40 ( )0.15 + 0.05V
B
+ 0.25V
A
25 + 0.2 V
A
40 ( ) 0
This can easily be rearranged to get V
A
= 200V.
(d) power P
s
V
A
I 200 25 5000W 5KW
Solution 3.22 (a) V
C
= V
s2
= 6V
(b) I
x
= 0.01V
A
(c) Supernode at A,B, encompassing the controlled source. So, we have one equation:
I
s1
0.01V
A
+ 0.0125V
B
+ 0.1V
B
0.1V
C
(d) Substitute the constraint equations, V
A
V
B
= 20I
x
= 0.2V
A
, (equivalently: V
A

V
B
0.8
) into the above
equation: V
A

V
B
0.8
I
s1
0.01
V
B
0.8
+ 0.0125V
B
+ 0.1V
B
0.1V
C
V
B
6.4V
V
A
8V
(e) I
x
= 0.08A.
(f) P
0.0125
V
B
2
/ R 0.512W
(g) P I
s
1
V
A
0.2 8 1.6W
Solution 3.23 (a) V
C
= V
s2
= 50V.
(b) i
x
= V
A
/100.
(c) Supernode A,B:
I
s1
0.01V
A
+ 0.05V
B
+ 0.05V
B
0.05V
s2
+ 0.09V
A
0.09V
s2
V
A
V
B
+ 300i
x
V
B
+ 3V
A
(d) Solving the above two equations yields: V
A
= -90V, V
B
= 180V.
(e) i
x

V
A
100

90
100
0.9A
(f) (V
B
-V
C
) (V
B
-V
C
)/R = 845W.
(g) P
S
1
I
S
1
V
A
2 (90) 180W
Solution 3.24 (a) V
B
V
C
3V
x
3V
B
; V
C
2V
B
(b) Supernode at B and C, encompassing controlled source.
(c)
I
s2
(
V
B
10

V
A
10
) +
V
B
10
+
V
C
10
+ (
V
C
10

V
A
10
); V
C
2V
B
; 10I
s
2
2(V
B
+V
A
)
equivalently: 2V
A
+ 2V
B
10
(d)
(0.1V
A
0.1V
s1
) + (0.1V
A
0.1V
B
) + (0.1V
A
0.1V
C
) 0; 0.3V
A
+ 0.1V
B
0.1V
S
1
; equivalently:
3V
A
V
B
10
(e) Again, any method can be used to simplify and solve the system of two equations. The solution is:
V
A
= -2.5V, V
B
= -2.5V.
Solution 3.25 (a) V
A
= V
s1
= 16V.
(b) Supernode at C and D, encompassing controlled voltage source.
(c) I
s2
(0.75mV
D
0.75mV
B
) + (1mV
C
1mV
A
)
(d) V
C
= 4V
B
+ V
D
(e)
0.75mV
D
+ (0.75mV
B
0.75mV
D
) + (0.25mV
B
0.25mV
A
) 0
or 1mV
B
0.25mV
A
0
(f) We now have three equations in V
B
, V
C
, and V
D
. These can be solved using any method. By inspection,
we can immediately deduce V
B
from V
A
using the last equation: V
B
= 4V.
The remaining two equations can be solved to obtain: V
C
= 20V and V
D
= 4V.
Solution 3.26 (a) The supernode is the combination of A, C, and the controlled voltage source.
(b) Write node equations starting at the supernode:
(G
2
V
A
G
2
V
in
) + (G
3
V
A
) + (G
4
V
A
G
4
V
B
) + (G
6
V
C
) + (G
5
V
C
G
5
V
B
) 0
G
2
+ G
3
+ G
4
( )V
A
+ G
4
G
5
( )V
B
+ G
6
+ G
5
( )V
C
G
2
V
in
and
(2G
6
V
C
) + (G
4
V
B
G
4
V
A
) + (G
5
V
B
G
5
V
C
) + (G
1
V
B
G
1
V
in
) 0
G
4
( )V
A
+ G
1
+ G
4
+ G
5
( )V
B
+ 2G
6
G
5
( )V
C
G
1
V
in
and
V
A
V
C
3V
x
, V
A
V
C
3(V
in
V
A
), 4V
A
V
C
3V
in
In matrix form:
0.8
0.4
4






0.5
0.6
0

0.2
0.1
1
1
]
1
1
1
V
A
V
B
V
C





1
]
1
1
1

6
6
180





1
]
1
1
1
(c) The above system of equations can be solved to obtain: V
A
= 38.75V, V
B
= 40V, V
C
= -25V.
(d) I
in
= (G
7
V
in
)+ (G
2
V
in
G
2
V
A
) + (G
1
V
in
G
1
V
B
) = 5A.
R
eq
= 12 and P = 300W.
P V
in
I
in
60 5 300W
(e) I
out
= V
C
G
6
= -2.5A P = 62.5W.
Solution 3.27 (a) Supernode is A,B encompassing controlled voltage source.
(b)
(V
A
V
s1
) + 0.4V
B
+ (0.2V
B
0.2V
C
) = 0
V
A
+ 0.6V
B
0.2V
C
= V
s1
(c) V
A
V
B
= I
B
= 0.4 V
B
V
A
= 1.4V
B
.
(d) I
s2
= 0.2V
B
+ (0.2V
C
0.2V
B
) = 0.2V
C
.
(e)
In matrix form:
1
1
0






0.6
1.4
0

0.2
0
0.2
1
]
1
1
1

V
A
V
B
V
C





1
]
1
1
1

8
0
2





1
]
1
1
1
The solution is: V
C
10V,V
A
7V,V
B
5V .
(f) i = (V
A
-V
s1
)1S = -1 P
ccvs
(V
A
V
B
) (V
A
V
s
1
) 1S 2W
Pvccs = (V
s1
-V
C
)(0.2V
B
) = -2W.
Solution 3.28 (a) Supernode at A,C, CCVS.
(b) Node equation at supernode:
I
s
+ 0.25mV
A
= G
1
V
A
+ (G
2
V
A
G
2
V
B
) + (G
5
V
C
) + (G
4
V
C
G
4
V
B
)
I
s
= (G
1
+ G
2
0.25m)V
A
+ (G
2
G
4
)V
B
+ (G
4
+G
5
)V
C
Constraint:
V
A
V
C
= 10
4
i
x
= 10
4
G
3
V
B
0 = V
A
10
4
G
3
V
B
V
C
At node B:
G
3
V
B
+ (G
2
V
B
G
2
V
A
) + (G
4
V
B
G
4
V
C
) = 0
G
2
V
A
+ (G
3
+G
2
+G
4
) G
4
V
C
= 0
(c) Matrix equation:
G
1
+ G
2
0.25m G
2
G
4
G
4
+ G
5
1 10
4
G
3
1
G
2
G
3
+ G
2
+ G
4
G
4





1
]
1
1
1
V
A
V
B
V
C





1
]
1
1
1

I
s
0
0





1
]
1
1
1
;
0
1000
0.2






1
1000
1.1

1
1000
0.8
1
]
1
1
1

V
A
V
B
V
C





1
]
1
1
1

2
0
0





1
]
1
1
1
(d) Substitute the values of conductances and solve the above matrix equation in MATLAB to obtain:
V
A
= -38V, V
B
= -20V, V
C
= -18V.
(e) P V I (10
4
G
3
V
B
) I
s
+ V
A
G
1
+ G
2
(V
A
V
B) [ ]
(20) 2m1.9m 3.6m [ ] 0.15W
Solution 3.29
Loop equation: V
in
= 2kI
1
+ 500(I
1
+ 20m)
V
in
= 2500I
1
+ 10 I
1
= 20mA.
P
vin
= 20m60 = 1.2W.
P
I
= 20m(500I
1
+ 50020m) = 0.4W.
P
2k
= I
1
I
1
R = 0.8W.
P
500
= (I
1
+ 20m)
2
R = 0.8W.
total power absorbed by resistors: P
R
= 0.8 + 0.8 = 1.6
total power delivered by sources: P
s
=1.2 + 0.4 W=1.6W
Conservation of power is verified.
Solution 3.30
Loop equation: 100(I
1
0.5) + 200I
1
+ 500 (I
1
+ 20m) = 0
I
1
= 0.05A.
P
0.5A
I V
100
; V
100
100 (0.5 0.05); where V
100
is the voltage on the 100 resistor.
P
0.5A
= 0.5(0.5100 0.05100) = 22.5W.
P
20m
= 20m(I
1
+20m)500 = 0.7W.
Solution 3.31
Loop equation: 3.3 = 50I
1
+ (50m + I
1
)100 + (I
1
30m)40 + (I
1
50m)60
I
1
= 0.01A.
The power delivered by the independent voltage source:
P = I
1
3.3 = 0.033W.
Solution 3.32
Loop equation: 50 = 300I
1
+ (I
1
0.4I
1
)500
50 = (300 + 500 200)I
1
I
1
= 0.0833A.
Power absorbed by the 500 resistor.
P
500
= (I
1
0.4I
1
)
2
500 = 1.25W.
Solution 3.33
Loop equation: 1000(I
1
Is) + 4000I
1
+ 5000(I
1
g
m
V
x
) = 010000I
1
2V
x
50
and
V
x
= 1000(I
s
I
1
) 1000I
1
+ V
X
50 .
Solve the above two equations in I
1
and V
x
to obtain: I
1
= 12.5mA, V
x
= 37.5V.
Thus, R
eq
= V
x
/I
s
= 750_,
P I
vccs
V
vccs
g
m
V
x
5000 (g
m
V
x
I
1
)=0.1875W
Solution 3.34
Loop equation: V
in
= 2I
in
+ 14I
in
10V
1
V
1
= 2I
in
After replacing V
1
in the loop equation we obtain:
V
in
= 4I
in

R
1eg

V
in
I
in
4
Solution 3.35
Loop equation: V
s
= 500I
1
+ 100(I
1
+ 0.5) + 400(I
1
0.001V
x
) +100(I
1
+ 0.005V
y
)
V
x
= 500I
1
,V
y
= 400I
1
- 4000.001V
x
= 400I
1
200I
1
200I
1
After replacing V
x
and V
y
in the loop equation we obtain:
V
s
50 = 1000I
1
I
1
= 0.1A
V
y
= 200I
1
= 20V P
400ohm
= V
y
2
/400 1W
R
eq
= V
s
/I
1
= 150/0.1 = 1500.
Solution 3.36
Select clockwise loop current I
1
in the left loop. Select anti clockwise loop current I
2
in the right loop.
The two mesh equations are:
12 = I
1
+10(I
1
+ I
2
)
and 10(I
2
+ I
1
) + 2I
2
+ 12 = 0
The two simultaneous equations can be solved easily to obtain: I
1
= 0.75A, I
2
= +0.375A.
P
10ohm
= (I
1
+ I
2
)
2
/10 = 0.127W.
Battery 1 supplies more current. (I
1
> I
2
)
Solution 3.37
(a) The equation for the left loop is:
660 = I
1
R + 1.296(I
1
+ I
2
)+ 590 + I
1
R
The equation for the right loop is:
660 = (0.3 R) I
2
+ 1.296 (I
1
+ I
2
) + 590 + (0.3 R) I
2
Simplifying the two equations:
70 = 1.596I
1
+ 1.296I
2
70 = 1.296I
1
+ 1.596I
2
The solution of these two equations is: I
1
= I
2
= 24.2A.
(b) I
1
+ I
2
= 48.4 , voltage across locomotive = 590 + 48.41.296 power = 31592W.
(c) Because the locomotive is 1/3 distance from either station it follows that
R = 1/30.3 = 0.1. The two equations become:
70 = I
1
(2R + 1.296) + 1.296I
2
70 = 1.296I
1
+ (1.296 + 0.6 2R)I
2
The solution of these two equations is: I
1
= 32.64A, I
2
= 16.32A.
Current in locomotive motor I
1
+ I
2
= 48.96A.
Voltage across locomotive 590 + (I
1
+ I
2
) 1.296
It follows that:
P = (I
1
+ I
2
)(590 + 491.296) = 31993W.
Solution 3.38
(a)
(b) The three loop equations are:
660 590 = 0.1I
1
+ 1.296 (I
1
I
2
) + 0.1I
1
0 = 1.296 ( I
2
I
1
) + 0.2I
2
+ 1.296 ( I
2
I
3
)
70 = 1.296( I
3
I
2
) + 0.2I
3
These three equations can be solved using any method to obtain:
I
1
= 46.8A, I
2
= 0, I
3
= 46.8A.
(c) Motor currents are 46.8A each.
(d) Ps = VI = 66046.8 = 30.9kW. Each source supplies 30.9kW.
Solution 3.39 (a) Define three meshes with three mesh currents. The first, I
1
, is a clockwise current
around the first mesh. The second, I
2
, is a clockwise current around the middle loop of the circuit
(through the 10mS, 5ms, and 5ms conductances). The third, I
3
, is a counterclockwise current through the
right-most loop containing the voltage source.
*current names shown above.
(b) I
1
0.5A
(
I
2
10m

I
1
10m
) +
I
2
5m
+ (
I
2
5m
+
I
3
5m
) 0.
(
I
3
25m
+
I
3
5m
) +
I
2
5m
20
These are two equations in two unknown currents. After grouping the terms, it can be verified that:
I
2
= 0.1A, I
3
= 0.
(c) V
x
= 20V
V
ad
= (0.5 I
2
)/10m = 40V
V
bd
= 20V
(d) P
0.5
= V
a
0.5 400.5 = 20W
P
20V
= 0W
P
resistors
= 2 I
2

2
/5m + (0.5 I
2
)
2
/10m = 20W
The conservation of power is verified.
Solution 3.40 (a) We can either write down the equations or evaluate the matrix by inspection:
90 (I
2
4.8m) + 10kI
2
+ 50 = 0
90kI
3
+ 10 (I
3
4.8m) = 50
OR
100k 0
0 100k



1
]
1
I
2
I
3



1
]
1

90k 4.8m 50
10k 4.8m + 50



1
]
1
(b) The solution of the above equation is: I
2
= 3.82mA, I
3
= 0.98mA.
(c) Current source: P = 4.8m[(4.8m I
2
)90k + (4.8m I
3
)10k] = 0.61W.
Voltage source: P = 50(I
3
I
2
) = 142mW.
Solution 3.41 (a) By inspection:
112k 90k 10k
90k 100k 0
10k 0 100k





1
]
1
1
1
I
1
I
2
I
3





1
]
1
1
1

180
60
60





1
]
1
1
1
(b) Using MATLAB:
I
1
= 4.4mA, I
2
= 3.36mA, I
3
= 1.04mA
i
x
I
2
3.36mA
(c) P
180
= 1804.4m = 0.792W, P
60
= 60( I
3
I
2
) = -0.139W.
Solution 3.42 The matrix equation is:
8 6 3
6 8 2
3 2 4





1
]
1
1
1
I
1
I
2
I
3





1
]
1
1
1

14
0
6





1
]
1
1
1
whose solution is: I
1
= 4A, I
2
= 3A, I
3
= 0
A
v = ( I
1
+ I
3
+ I
2
) 2 = 2V
Solution 3.43 (a) First, note that two mesh currents are needed. Two clockwise currents are defined: I
1
in the middle loop, and I
2
in the right-most loop:
Middle loop equation:
100I
1
100I
s1
+ 20I
x
+ 80I
1
80I
2
= 0, where I
x
= I
s1
I
1
and
Right-most loop equation:
80I
2
80I
1
+ 10I
2
+ V
s2
= 0
These can easily be cast into the following matrix equation:
160
80




80
90
1
]
1
I
1
I
2



1
]
1

16
6



1
]
1
(b) The solution of this equation is: I
1
= 0.12A, I
2
= 0.04A.
(c) V
A
= 100(I
s1
I
1
) = 8V and V
B
= 80x(I
1
I
2
) 6.4V .
(d) P
s
1
I
s1
V
A
= 1.6W.
(e) P
0.0125S
(I
1
I
2
)
2
/0.0125 = 0.512W.
Solution 3.44 (a) Create two clockwise mesh currents in the top loop (I
2
) and the bottom-left loop (I
1
) .
The bottom-right loop has an independent current source. Writing the loop equations:
V
s1
= 200 (I
1
I
2
) + 200 (I
1
+ I
s2
)
200 (I
2
I
1
) + 100I
2
+ 300I
x
+ 200 (I
2
+ I
s2
) = 0, where I
x
= I
1
I
2
(b) Solving, we get: I
1
= -0.1A, I
2
= -0.7A, I
x
= 0.6A.
(c) V
B
= (I
1
+ I
s2
)200 = 130V.
(d) P
vs1
= I
1
V
s1
= 25W, P
is2
= (V
B
+ (I
s2
+ I
2
)200)I
s2
= 105W, P
300ix
= (I
2
)(300I
x
) = 126W.
Solution 3.45 (a) Create two clockwise mesh currents in the top loop (I
1
) and the middle loop (I
2
) (all
resistor loop):
Top loop equation:
0.5v
x
= 500 (I
1
- I
2
,) +500I
1
where v
x
= 500I
1
and
Middle loop equation:
600 (I
2
I
s1
) + 500 (I
2
I
1
) + 900 (I
2
+ I
s2
) = 0
(b) Solving, we get: I
1
= 0.015A, I
2
= 0.0375A, v
x
= -7.5V.
(c) P
is1
= I
s1
[0.5v
x
+ (I
s1
I
2
)600] = 109.7W
P
0.5vx
= 0.5v
x
(I
1
I
s1
) = 1.63W
P
is2
= (I
2
+ I
s2
)900I
s2
= 53.2W
Solution 3.46 Write the mesh equations in terms of Rs and then substitute the values from the matrix:
Mesh 1 equation:
v
1
= R
1
(i
1
i
3
) + R
2
(i
1
i
2
)-25i
2
From this equation, and the first row of the matrix equation, we can deduce that
R
1
= 5
and
R
2
+ 25 = 40 R
2
= 15.
Similarly:
Mesh 3 equation: R
1
(i
3
i
1
) + i
3
R
4
+ R
3
(i
3
i
2
) =0
From which we can deduce:
R
3
= 25 and R
4
= 5 .
Solution 3.47
Modified loop 1 equation:
V
s1
= 3MI
1
+ v + 2MI
1
Constraint equation:
I
1
+ I
2
= I
s3
Modified loop 2 equation:
v = 2MI
2
+ V
s2
+ 8MI
2
Or in matrix form:
5M 0 1
1 1 0
0 10M 1





1
]
1
1
1
I
1
I
2
v





1
]
1
1
1

V
s1
I
s3
V
s2





1
]
1
1
1
Solving: I
1
= -1.1 A, I
2
= -0.95 A.
The power P
s3
= I
s3
xv= 7.58 W
Solution 3.50 I
2
= 2A, I
3
= 7A
Loop 1 equation:
V
s
3I
1
+ 3(I
1
I
2
) + 6(I
1
+ I
3
I
2
) + 2v
y
+ 2(I
1
+ I
3
) 14I
1
+ 2v
y
74 14I
1
+ 2V
Y
88
v
y
= 3(I
1
I
2
)= 3I
1
6
Solving the above system, we obtain: I
1
= 5A v
y
= 9, v can be found from the loop 3 equation
v + (I
1
+ I
3
I
2
).6 + 2v
y
+ 2(I
3
+ I
1
) 0 . Solving, we obtain v 10V
Finally, P
vs
= V
s
I
1
= 70W.
Solution 3.51
Mesh 1 equation:
9kI
1
+ 3k (I
1
I
4
) + 6k (I
1
I
3
) + 12k (I
1
I
2
) = 0
where we have used the fact that I
x
= I
1
I
4
(and I
4
= 4mA)
Mesh 3 equation:
6k (I
3
I
1
) v
2
+2kI
3
+ v = 0
Mesh 2 equation:
2.4kI
2
+ 12k (I
2
I
1
) v = 0
Constraint equations:
I
2
I
3
= 0.5mA
I
3
I
4
= 0.5I
x
= 0.5I
1
0.51I
4
0.51I
1
+ 2mA; 0.51I
1
I
3
2mA
The above five equations need to be put into matrix form:
30k 12k 6k 0 0
6k 0 8k 1 1
12k 14.4k 0 1 0
0 1 1 0 0
0.5 0 1 0 0








1
]
1
1
1
1
1
1
I
1
I
2
I
3
v
v
2








1
]
1
1
1
1
1
1

12
0
0
0.5m
2m








1
]
1
1
1
1
1
1
The solution is:
I
1
I
2
I
3
I
1
I
2
I
3
v
v
2








1
]
1
1
1
1
1
1

0.002A
0.0025A
0.003A
12V
24V








1
]
1
1
1
1
1
1
I
x
I
1
I
4
2mA
Power delivered by 0.5mA current source: P
1
0.5mxv 6mW
Power delivered by the dependent current source: P
2
0.5I
x
v
2
1m (24) 24mW
Power delivered by 4mA current source: P
3
4m (3k I
x
V
2
) 120mW
Solution 3.52
Write the following equations:
Mesh 2 equation:
100 (I
2
I
1
) + 150 (I
2
I
4
I
6
) + v = 0
Mesh 4 equation:
3v
x
+ 30 (I
4
I
3
) + 150 (I
4
- I
2
+ I
6
) = 0
Mesh 5 equation:
800I
5
3v
x
+ 10( I
5
I
1
+ I
6
) =0
Mesh 6 equation:
250I
6
+150(I
6
+ I
4
I
2
) +10(I
6
+ I
5
I
1
) 0
Constraint equation:
I
1
I
2
I
3
I
4
I
5
I
6
I
s
2
I
2
+ I
3
Substituting the values of I
1
I
s
1
1.15 and I
3
I
s3
0.95 , and noting that
v
x
= (I
1
I
6
I
5
)10
we can write:
250 150 0 150 1
150 180 30 120 0
0 0 840 40 0
150 150 10 410 0
1 0 0 0 0








1
]
1
1
1
1
1
1
I
2
I
4
I
5
I
6
v








1
]
1
1
1
1
1
1

100I
1
30I
3
30I
1
40I
1
10I
1
I
3
I
S2








1
]
1
1
1
1
1
1
I
2
I
4
I
5
I
6
v








1
]
1
1
1
1
1
1

0.65
0.45
0.05
0.1
35








1
]
1
1
1
1
1
1
Finally, v
x
= 10V and v is as given above.
Solution 3.53 (a)
Replace the voltage source by current sources:
At node 1
i
s1
= (V
2
V
1
) /12k
At node 2
(V
2
V
1
) /12k + (V
2
V
3
) /90k + (V
2
V
4
) /10k = 0
At node 3
i
s2
= (V
3
V
2
) /90k + V
3
/10k
At node 4
i
s2
+ (V
4
V
2
) /10k + V
4
/90k = 0
V
1
= 180
V
4
V
3
= 60
(b) In matrix form:
1/12k 1/12k 0 0 1 0
1/12k 1/12k +1/90k +1/10k 1/90k 1/10k 0 0
0 1/90k 1/90k +1/10k 0 0 1
0 1/10k 0 1/90k +1/10k 0 1
1 0 0 0 0 0
0 0 1 1 0 0










1
]
1
1
1
1
1
1
1
1
V
1
V
2
V
3
V
4
i
s1
i
s2









1
]
1
1
1
1
1
1
1

0
0
0
0
180
60









1
]
1
1
1
1
1
1
1
The solution from MATLAB is
180.0000
127.2000
33.6000
93.6000
-0.0044
0.00232
(c) Power delivered by S
1
is:
P
s1
V
s1
(I
s1
) 0.792W
Power delivered by S
2
is:
P
s2
V
s2
(I
s2
) 0.139W
Solution 3.54 (a) Replace the 100 ohm resistor, the controlled voltage source, and v
s2
by current source.
Then write the node equations:
I
s1
= i
x
+ I
1
+ 0.09 (V
1
V
3
)
I
1
= V
2
/20 + 0.05 (V
2
V
3
)
I
2
= 0.05 (V
3
V
2
) + 0.09 (V
3
V
1
)
V
3
= V
s2
V
1
V
2
= 300i
x
= 300V
1
/100
In matrix form:
0.1
0
0.09
0
2









0
0.1
0.05
0
1

0.09
0.05
0.14
1
0

1
1
0
0
0

0
0
1
0
0
1
]
1
1
1
1
1
1

V
1
V
2
V
3
I
1
I
2








1
]
1
1
1
1
1
1
=
2
0
0
50
0








1
]
1
1
1
1
1
1
(b) Using MATLAB to solve the above system the solution is:
V
1
= -90.0000V
V
2
= 180.0000V
V
3
= 50.0000V
I
1
= 15.5000A
I
2
= -6.1000A
(c) Power delivered by the current source is
P
s1
I
s1
V
1
180W
Power delivered by the voltage source is:
P
s2
V
s2
(I
2
) 305W
Solution 3.55 Modify the circuit so that it looks like the following:
The modified node equations are:
I
s1
= - I
y
+ I
x
I
b
= I
y
+ 0.2 (V
C
V
B
)
I
s2
= 0.2 (V
C
V
B
) + I
x
The equations describing the constitutive relationships of elements in the original network are:
V
A
V
B
= I
b
I
x
0.2V
b
I
b
0.4V
b
I
y
8 V
A
These can be cast into a matrix equation and solved easily to obtain the same result as previously arrived
at.
In matrix form:
0
0
0
1
0
0
1













0
0.2
0.2
1
0.2
0.4
0

0
0.2
0.2
0
0
0
0

1
0
1
0
1
0
0

1
1
0
0
0
0
1

0
1
0
1
0
1
0

1
0
0
0
0
0
0
1
]
1
1
1
1
1
1
1
1
1
1

V
A
V
B
V
C
I
x
I
y
I
b
I
s1












1
]
1
1
1
1
1
1
1
1
1
1
=
0
0
2
0
0
0
8












1
]
1
1
1
1
1
1
1
1
1
1
V
A
V
B
V
C
I
x
I
y
I
b
I
s1












1
]
1
1
1
1
1
1
1
1
1
1
=
7V
5V
10V
1A
1A
2A
0A












1
]
1
1
1
1
1
1
1
1
1
1
Solution 3.56 Modify the circuit as follows:
The modified node equations are:
At node A: I
s
= G
1
V
A
+ G
2
(V
A
V
B
) + I
a
At node B: G
2
(V
A
V
B
) = I
x
+ G
4
(V
B
V
C
)
At node C: 0.25mV
A
+ I
a
= G
4
(V
C
V
B
) + G
5
V
C
The equations describing the constitutive relationships of elements in the original network are:
I
x
= G
3
V
B
V
A
V
C
= 10
4
I
x
These can be cast into a matrix equation that can be solved in MATLAB.
In Matrix form:
0.25m
0.2m
0.25m
0
1









0.2m
1m
0.8m
0.1m
0

0
0.8m
1m
0
1

1
0
1
0
0

0
1
0
1
10
4
1
]
1
1
1
1
1
1

V
A
V
B
V
C
I
a
I
x








1
]
1
1
1
1
1
1
=
2m
0
0
0
0








1
]
1
1
1
1
1
1
The solution is:
V
A
V
B
V
C
I
a
I
x








1
]
1
1
1
1
1
1

38V
20V
18V
0.0075A
0.002A








1
]
1
1
1
1
1
1
We observe that we have obtained the same results as in problem 3.28.
Solution 3.57 Replace dependent source by i
35
(from 3 to 5). Also, replace voltage source by i
10
(from 1
to 0). Now, write the modified noted equations. The reference node is O:V
O
0V :
At node 1: i
10
= (V
6
V
1
) + (V
2
V
1
)
At node 2: 2 = (V
2
V
1
) + (V
2
V
3
)
At node 3: i
35
= (V
4
V
3
) + (V
2
V
3
)
At node 4: 2 = 2 + (V
4
V
3
) + V
4
At node 5: i
35
= (V
5
V
6
) + V
5
At node 6: 2 = (V
5
V
6
) V
6
Constraints:
V
3
V
5
= 15v
x
= 15V
4
V
1
= 5
The following matrix equation is obtained:
2 1 0 0 0 1 1 0
1 2 1 0 0 0 0 0
0 1 2 1 0 0 0 1
0 0 1 2 0 0 0 0
0 0 0 0 2 1 0 1
0 0 0 0 1 2 0 0
0 0 1 15 1 0 0 0
1 0 0 0 0 0 0 0













1
]
1
1
1
1
1
1
1
1
1
1
1
V1
V2
V3
V 4
V5
V6
i10
i35













1
]
1
1
1
1
1
1
1
1
1
1
1

0
2
0
0
0
2
0
5













1
]
1
1
1
1
1
1
1
1
1
1
1
The solution of this equation is obtained from MATLAB:
V
1
5.0000V
V
2
3.3571V
V
3
0.2857V
V
4
-0.1429V
V
5
1.8571V
V
6
-0.0714V
i
10
-6.7143A
i
35
3.7857A
The power delivered by the dependent voltage source connected between nodes 3 and 5:
P
35
15v
x
(i
35
) 15 V
4
(i
35
) 8.115W
The power delivered by the current source connected between nodes 2 and 4:
P
24
(2A) (V
2
V
4
) 7W
The power delivered by the current source connected between nodes 4 and 6:
P
46
(2A) (V
4
V
6
) 0.143W
The power delivered by the voltage source connected between nodes 1 and 0:
P
10
5V (I
12
+ I
16
) 5x (V
1
V
2
) + (V
1
V
6
) [ ] 33.57W
Solution 3.59 Using the appropriate element stamps for each element of the circuit, we obtain the
following system:
0.15 + 0.2 0.15 0.2 0
0.15 0.15 + 0.05 0 1
0.2 0 0.25 + 0.2 1
0 1 1 0







1
]
1
1
1
1
1
V
A
V
B
V
C
I
x







1
]
1
1
1
1
1

8 3
3
25
440







1
]
1
1
1
1
1
Solution 3.60
1/20k +1/10k + g
m1
1/10k
1/10k g
m1
g
m2
g
m2
+1/10k +1/2.5k



1
]
1
V
1
V
2



1
]
1

I
s1
I
s2



1
]
1
The solution is the same as that of problem 3.14.
Solution 3.62 (a) Because R
T
(T) can be approximated by a straight line between

(250,0
o
C) and

(80,50
o
C) it follows that:
R
T
(T)= 3.4T + 250
(b) For T 25
o
C, R
T
= 165
(c) The voltage across the R
T
+ R
L
series combination can be obtained from voltage division:
V
T,L

R
T
+ R
L
R
T
+ 2R
L
+ R
12 4.7857V
This is the same as the voltage across Rx because the meter is at zero deflection. Thus,
R
x
R
x
+ R
12 4.7857. It follows that R
x
165.84.
(d) We first denote the nodes:
A - the node common to R, R
x
and the voltmeter;
B - the node common to R, Rand the voltage source;
C - the node common to R
L
, R
T
and R
L
;
D - the node common to R
x
, R
L
and the voltage source.
The reference node is D:V
D
0. It follows that V
B
12V .
We also have: v
out
V
A
V
C
The node equations are:
1
2
0
At node A:
V
A
V
C
R
m
+
V
A
12
R
+
V
A
R
x
0
Equivalently: V
A
(RR
x
+ R
m
R
x
+ R
m
R) - V
C
RR
x
12R
m
R
x
V
A
4199.86 V
C
41.46 19900.8 (1)
At node C:
V
C
12
R + R
L
+
V
C
R
T
+ R
L
+
V
C
V
A
R
m
0
Equivalently: (V
C
12) 0.004 +
V
C
R
T
+ 2.5
+ (V
C
V
A
) 10
4
0
V
C
(0.004 R
T
+ 0.01 +1+ 0.00025 + R
T
10
4
)
V
A
(R
T
10
4
+ 0.00025) 0.48 (R
T
+ 2.5)
The last equation can be rewritten as:
V
C
(0.0041R
T
+1.01025) - V
A
(R
T
10
4
+ 0.00025) 0.48(R
T
+ 2.5) (2)
From (1) and (2), we obtain:
(0.0041R
T
+1.01025)
19900.8 + V
A
4199.86
41.46

V
A
(R
T
10
4
+ 0.00025) 0.48(R
T
+ 2.5)
Equivalently: V
A
(R
T
0.415 +102.337) 2.448 R
T
+ 486.12
It follows that V
A

2.448 R
T
+ 486.12
0.415 R
T
+102.337
From the equation at node A:
v
out
V
A
V
C
R
m

V
A
12
R
+
V
A
R
x



_
,

=
2.448 R
T
+ 486.12
0.415 R
T
+102.337
(100.3) + 480
At T 0
o
C: R
T
250. It follows that v
out
54.4415V
At T 50
o
c: R
T
80. It follows that v
out
80V
(e): The formula has been derived at part d):
T R
T
v
out
0
o
C 250 54.4415V
5
o
C 233 52.4136V
10
o
C 216 50.2368V
15
o
C 199 47.8938V
20
o
C 182 45.3650V
25
o
C 165 42.6273V
30
o
C 148 39.6537V
Solution 3.63
Place a source V
in
between C and D, and calculate the current drawn from the source as below:
Loop 1 equation:
I
1
R
1
+ (I
1
I
2
)R
2
+ (I
1
I
3
)R
3
0
Equivalently:
I
1
(R
1
+ R
2
+ R
3
) I
2
R
2
I
3
R
3
0
Loop 2 equation:
(I
2
I
1
)R
2
+ I
2
R
4
1 0
Equivalently:
I
1
R
2
+ I
2
(R
2
+ R
4
) 1
Loop 3 equation:
1 + I
3
R
5
+ (I
3
I
1
)R
3
0
Equivalently:
R
3
I
1
+ I
3
(R
3
+ R
5
) 1
We obtain the following system of equations:
30I
1
4I
2
6I
3
0
4I
1
+ 6I
2
1
6I
1
+14I
3
1

'



I
1
0.0096A, I
2
0.1731A, I
3
0.0673A;
I
in
I
2
I
3
0.2404 A
R
eg,CD

V
in
I
in

1
0.2404
4.16
Solution 3.64
The node equation at node A is:
V
A
G
1
+ (V
A
V
B
)G
2
+ (V
A
V
C
)G
3
0
Equivalently:
(G
1
+ G
2
+ G
3
)V
A
V
B
G
2
V
C
G
3
0
The supernode is identified by a Gaussian surface enclosing the controlled voltage source. The supernode
equation is:
G
2
(V
A
+ V
B
) 6 + G
4
V
C
+ G
3
(V
C
V
A
) 0
Equivalently, we have:
V
A
(G
2
+ G
3
) + G
2
V
B
+V
C
(G
3
+ G
4
) 6
One way of obtaining the solution to the problem is:
We multiply the above two equations by 30.
30(G
2
+ G
3
)V
A
+ 30G
2
V
B
+ 30(G
3
+ G
4
)V
C
180
and
30(G
1
+ G
2
+ G
3
)V
A
30G
2
V
B
30G
3
V
C
0
By equating the coefficient of the above two equations with the coefficients of the first and second given
equations, we obtain:
30G
2
30 G
2
0.1S
30G
3
2 G
3
0.067S
30(G
1
+ G
2
+ G
3
) 11 G
1
0.2S
30(G
3
+ G
4
) 32 G
4
0.87S
can be obtained as follows:
V
C
V
B
V
X
(V
C
V
A
)
Equivalently:
V
A
V
B
+ (1 )V
C
0
By comparing with the third given equation 3.
Solution of 3.66
(a)
At node A: (V
A
-V
C
)/2 + (V
A
-V
B
)/2 + (V
A
-V
D
)/2 = 14
At node B: (V
B
-V
A
)/2 + (V
B
-V
C
)/2 + (V
B
-V
D
)/2 = 7
At node C: (V
C
-V
A
)/2 + (V
C
-V
B
)/2 + (V
C
-V
D
)/2 + 2V
C
= 0
At node D: (V
D
-V
A
)/2 + (V
D
-V
B
)/2 + (V
D
-V
C
)/2 + 0.5V
D
= 0
These can be solved in MATLAB to obtain:
22.0000
18.5000
7.5000
12.0000
(b) Mesh analysis would result in the same voltages
The loops and their current loops are:
A,C,14A:I
1
A, C, D:I
2
A, B, D:I
3
B,C, D:I
4
C, D,reference node:I
5
B, D,7 A:I
6

'









I
1
14A, I
6
7A
Loop ACDequation: 2(I
2
I
1
) + 2(I
2
I
3
) + 2(I
2
+ I
4
+ I
5
) 0
Loop BCD equation: 2(I
3
+ I
6
+ I
4
) + 2(I
2
+ I
4
+ I
5
) + 2I
4
0
Loop ABD equation: 2(I
3
+ I
4
+ I
6
) + 2(I
3
I
2
) + 2I
3
0
Loop CD
ref
node equation: 2(I
5
+ I
2
+ I
4
) + 2(I
5
I
6
) + 0.5(I
5
+ I
1
) 0
In matrix form:
6
2
2
2








2
2
6
0

2
6
2
2

2
2
0
4.5
1
]
1
1
1
1
1

I
2
I
3
I
4
I
5







1
]
1
1
1
1
1

28
14
14
7







1
]
1
1
1
1
1
;
I
2
I
3
I
4
I
5







1
]
1
1
1
1
1

6.75
1.75
5.5
1







1
]
1
1
1
1
1
V
A
2(I
1
I
2
) + 0.5(I
1
+ I
5
) 22V
V
B
2(I
4
+ I
3
+ I
6
) + 2(I
6
I
5
) 18.5V
V
C
0.5(I
1
+ I
5
) 7.5V
V
D
2(I
6
I
5
) 12V
(c) Mesh analysis requires more work.
(d) The removal of the top resistor will result in more node equations than loop equations. The addition of
a resistor between node A and reference node will result in more loop equations than node equations.
Thevenin Probs, 7/11/01 - P4.1 - @R.A. Decarlo & P. M. Lin
PROBLEM SOLUTIONS CHAPTER 4
SOLUTION 4.1. First, find V
out
/ V
s
for each circuit. Then solve for R knowing
V
out
P10 t14.142V .
(a) Writing KCL at the inverting terminal, 1/ 1k(v

v
s
) 1/ R(V
out
v

) V
out
/ V
s
R/ 1k , since
the inverting terminal is a virtual short. Solving for R V
out
1k / V
s
2.828k.
(b) Writing KCL at the inverting terminal, V
s
/1.5k (V
out
V
s
)/ R V
out
/ V
s
R / 1.5k +1, solving
for R 1.5k(V
out
/ V
s
1) 2.743k.
(c) From (a) V
out
/ V
s
12k / R, thus R 12k V
s
/ V
out
4.243k.
(d) This is the same circuit as (b) except the output voltage is taken across two resistors. Thus
V
out

P
10
10 + 6 ( ) 22.627V . Using the general form from (b), R 400 V
out
/ V
s
1 ( ) 1.410k
SOLUTION 4.2. (a) First, find the voltage at the non-inverting terminal as v
+
1/ 2 V
s
. Then write KCL
at the inverting terminal, and make use of the virtual short property,
(V
s
/ 2)/10 k (V
out
V
s
/ 2) / 30k V
out
/ V
s
30k(1/ 20k +1/ 60k) 2 .
(b) Relating the output of the amplifier to the output of the circuit, V
out
V
amp
(500 / 800) . Then writing
KCL at the inverting terminal, V
s
/ 400 (V
amp
V
s
) /1.2k V
amp
/ V
s
1.2k / 400 +1 4. Therefore
V
out
/ V
s
(V
amp
/ V
s
) (V
out
/ V
amp
) 2.5.
(c) Note that since no current goes into the non-inverting terminal of the op-amp, the voltage at that node is
V
s
. KCL at the inverting terminal, V
s
/ 4k (V
out
+ V
s
)/ 20k V
out
/ V
s
6.
SOLUTION 4.3. Write KCL for both terminals,
(V

V
i
) / 1k (V
o
V

) / 2k
V

/1k (V
o
V

)/ 3k
Solving and doing the appropriate substitutions, V
o
/ V
i
8 .
SOLUTION 4.4. This is essentially the basic inverting configuration, which is defined as
V
o
/ V
i
2k / 1k 2 .
Thevenin Probs, 7/11/01 - P4.2 - @R.A. Decarlo & P. M. Lin
SOLUTION 4.5. (a) By voltage division V
L
1V
100
200
0.5V . Using Ohms law
I
s
I
L

1
100 +100
5mA.
(b) No current flows in the input terminal of an ideal op-amp, thus I
s
0A and V
L
1V . From Ohms
law I
a
I
L
V
L
/100 10mA.
SOLUTION 4.6. (a) Using voltage division,
V
1
V
s
32||(8 + 24)
[32||(8 + 24)] + 8



_
,

2
3
V
s
V
out
V
1
24
24 + 8


_
,
0.5V
s
(b) By voltage division,
V
1
V
s
32
32 + 40


_
,
0.8V
s
V
out
V
1
24
24 + 8


_
,
0.6V
s
(c) Using voltage division, V
1
V
s
32
32 +8


_
,
0.8V
s
, as no current enters the non-inverting terminal of the
op-amp. Due to the virtual short property, V
out
V
1
24
24 + 8


_
,
0.6V
1
. This is indeed the same results as
(b), which should be expected because of the isolation provided by the ideal buffers.
SOLUTION 4.7. Write KCL at the inverting terminal,
V
s1
/1k V
s2
/ 2k V
out
/ 4k V
out
4V
s1
2V
s2
40mV.
SOLUTION 4.8. (a) The voltage at the non-inverting terminal is V
+
3 / 2V , KCL at the inverting
terminal gives (1.5 2.5) /10k (V
out
1.5)/ 30k V
out
1.5V . The power is
P V
out
2
/ 500 4.5mW.
(b) The voltage at the non-inverting terminal is 3V this time, thus KCL
(3 2.5)/ 10k (V
out
3) / 30k V
out
4.5V . The power is P V
out
2
/ 500 40.5mW.
SOLUTION 4.9. (a) Define the point between the two op-amp as V
int
. Observe that the first op-amp is in
the basic inverting configuration, and the second the non-inverting configuration. By inspection,
Thevenin Probs, 7/11/01 - P4.3 - @R.A. Decarlo & P. M. Lin
V
int
/ V
s
R
1
/ 1k
V
out
/ V
int
(1+ R
2
/1k )
Cascading the two stages, (V
int
/ V
s
)(V
out
/ V
int
) V
out
/ V
s
R
1
/1k(1+ R
2
/1k) . Solving for
R
1
20 1k / (1+ R
2
/1k) 5k. The power absorbed is P (20 0.5)
2
/ 8 12.5W .
(b) Using the same equations as (a), solve for R
2
(20 1k / 2k 1)1k 9k.
(c) Rewriting the equation obtained in (a), R
1
2
+1kR
1
20M 0, and solving the quadratic equation yields
R
1
R
2
4k.
SOLUTION 4.10. This is a cascade of two non-inverting configuration op-amp of the form
V
o
/ V
s
(1+10k /10k ) for each. Therefore 2 2 4.
SOLUTION 4.11. This system is made up of a non-inverting stage with a gain of 1+10k/10k, a voltage
divider of gain 8k/(2k+8k), and a second non-inverting stage of gain 1+10k/10k. The product of all three
yields V
out
/ V
in
(2)(0.8)(2) 3.2.
SOLUTION 4.12. (a) By inspection, the gain of the first stage is 1. Then write KCL for the second stage
V
s1
/ 2R V
s2
/ R V
out
/ 2R V
out
V
s1
2V
s2
10V .
(b) The first stage gain is 0.5, thus V
out
2R(0.5V
s1
) / 2R 2R(V
s2
)/ 0.5R 7.5V, using the same
procedure as in (a).
SOLUTION 4.13. (a) This is a cascade of a summing amplifier with the following transfer characteristic,
V
o
4V
s1
2V
s2
, and an inverting stage of gain 1.5. Thus V
out
1.5(4V
s1
+ 2V
s2
) 2.25V .
(b) Notice that the only difference is the gain of the inverting stage, which is now 2. Therefore
V
out
2(4V
s1
+ 2V
s2
) 3V .
SOLUTION 4.14. This circuit is a cascade of two summing amplifier where the output of the first is an
input of the second stage. The transfer function of the first stage is V
o
2RV
s1
/ 2R 2RV
s2
/ R, which
is substituted in the transfer function of the second stage to obtain
V
out
R[2RV
s1
/ 2R 2RV
s2
/ R]/ R RV
s3
/ R V
s1
+ 2V
s2
V
s3
2V .
SOLUTION 4.15. Writing KCL at the inverting node, V
1
/ R
1
V
2
/ R
2
V
3
/ R
3
V
out
/ R
f
, and
solving for V
out

R
f
R
1
V
1
+
R
f
R
2
V
2
+
R
f
R
3
V
3



1
]
1
.
Thevenin Probs, 7/11/01 - P4.4 - @R.A. Decarlo & P. M. Lin
SOLUTION 4.16. Referring figure P4.15, the value of the resistance must satisfy the following
constraints:
R
1
R
2
R
3
3R
R
f
R
These will yield the inverted average. If polarity is a concern, a second inverting stage should be added
with a unity gain, i.e. both Rs equal.
SOLUTION 4.17. Using the topology of 4.12 the following parameters are chosen,
G
a1
3, G
a2
5, G
b1
2, G
b2
4
For the time being assume G
f
1. Now we calculate (1+ 3 + 5) (2 + 4) 3, this sets G
g
3.
(a) The requirement for G
f
10S sets the scaling factor K 10 / 1 10 . This then yields the
following set of parameters,
G
a1
30S, G
a2
50 S, G
b1
20S, G
b2
40S, G
f
10S, G
g
30S
(b) The requirement for G
f
2S , sets the scaling constant to 2uS. So the following parameters are
obtained:
G
a1
6S, G
a2
10 S, G
b1
4 S, G
b2
8S
Furthermore for G
g
12S , G 6S in order to make the incident conductance equal at both terminal.
(c) Using the starting values from (a), one could choose a scaling constant of 5 S. This will yield the
following resistances:
R
a1
66.67k, R
a2
40k, R
b1
100k, R
b2
50k, R
f
200k, R
g
66.67k
These are all reasonable values for circuit implementation.
SOLUTION 4.18. (a) Choosing the following initial values:
G
a1
3S, G
a2
5S, G
b1
11S, G
b2
4S, G
f
1S
Thevenin Probs, 7/11/01 - P4.5 - @R.A. Decarlo & P. M. Lin
then calculate (1+ 3 + 5) (11+ 4) 6 . Thus G
g
1S, and G 1 + 6 7S . Scaling everything by
1S, yield this final set of parameters, which meet the requirements.
G
a1
3S, G
a2
5S, G
b1
11S, G
b2
4S, G
f
1S, G
g
1S, G 7uS
(b) The set of parameters remains unchanged, except for G which now becomes 6uS in order to
maintain the equal termination conductance requirement due to G
g
0S.
(c) Scale the initial parameters of (a) by 5uS, and get the following set of resistances:
R
a1
66.67k, R
a2
40k, R
b1
18.18k, R
b2
50k, R
f
200k, R
g
200k, R 28.57k
SOLUTION 4.19. (a) Choosing the following initial set of parameters:
R
a1
1/ (4S) 0.25, R
a2
1/ (2S) 0.5, R
b1
1/ (5S) 1/ 5, R
b2
1/ (4S) 0.25, R
f
1
and (1+ 4 + 2) (5 + 4) 2 , thus choose R
g
1/ (1S) 1 and R 1/ (1+ 2) 1/ 3. To meet
the R
f
50k requirement, all the parameters must be scaled by 50k, which gives
R
a1
12.5k, R
a2
25k, R
b1
10k, R
b2
12.5k, R
f
50k, R
g
50k, R 16.67k
(b) Same as (a) with a 100k scaling constant:
R
a1
25k, R
a2
50k, R
b1
20k, R
b2
25k, R
f
100k, R
g
100k, R 33.33k
SOLUTION 4.20. (a) When the op-amp is in its active region v
out
/ v
s
5 . Thus it will operate in its
active region when 3 v
s
3, and will saturate at 15V when v
s
3 , and at 15V when v
s
3. SPICE
yield the following plot:
Thevenin Probs, 7/11/01 - P4.6 - @R.A. Decarlo & P. M. Lin
(b) Using SPICE the following plot is obtained:
Thevenin Probs, 7/11/01 - P4.7 - @R.A. Decarlo & P. M. Lin
SOLUTION 4.21. The first stage is in a summing configuration, thus its output is, assuming it's in the
active region of operation, -15 V which means it is just about to saturate. The second stage is in the
inverting configuration with a gain of -1.5, which means that the overall output will be saturated at 15V.
SOLUTION 4.22. When v
in
80k
v
in
+1.5
100k


_
,
> 0, or v
in
> 6 the output of the comparator saturates at
15 V, when it is v
in
< 6 , it will saturate at 15 V. The following plot is obtained from SPICE.
SOLUTION 4.23. When v
in
10k
v
in
+ 20
110k


_
,
> 0 , or v
in
> 2 the output of the comparator will be
saturated at -15V. Otherwise when it is < 2V the output saturates at 15V. In SPICE:
Thevenin Probs, 7/11/01 - P4.8 - @R.A. Decarlo & P. M. Lin
SOLUTION 4.24. Based on the same reasoning as the previous questions,
The output will be +V
sat
, when v
in
< v
ref
1
R
1
+ R
2
R
2



_
,


R
1
R
2
v
ref
, and V
sat
for
v
in
> v
ref
1
R
1
+ R
2
R
2



_
,


R
1
R
2
v
ref
.
SOLUTION 4.25. Using the previously derived relationship, and the topology of figure P4.24, set
v
ref
1.5V , and R
1
2k and R
2
3k. Set the power supplies to the Op-amp to +/ 10V to satisfy
the V
sat
requirement. Also the input to the inverting and non-inverting terminal are reversed for fig. P4.24.
Verifying in SPICE we obtain the following,
Thevenin Probs, 7/11/01 - P4.9 - @R.A. Decarlo & P. M. Lin
SOLUTION 4.26. The design that fulfills the requirement is the same as for P4.25, with the input to the op
amp reversed. The following is obtained from SPICE,
Thevenin Probs, 7/11/01 - P4.10 - @R.A. Decarlo & P. M. Lin
SOLUTION 4.27. First, for the comparator to give +Vsat for the lower voltages, the inputs to the op amp
in the topology of P4.24 must be interchanged. Then the components are chosen to satisfy the following
relationship, v
switch
v
ref
1
R
1
+ R
2
R
2



_
,

R
1
R
2
v
ref
. Choose v
ref
1.5V , and R
1
= 2k and R
2
= 1k.
Verifying in SPICE,
Thevenin Probs, 7/11/01 - P4.11 - @R.A. Decarlo & P. M. Lin
SOLUTION 4.28. Write KCL at the inverting terminal, noting that the no current flows into it:
(V

v
in
)/ R (v
out
V

)/ R. Use the following relationship v


out
A(V
+
V

) AV

. Solving
using the previous two equations yields v
out
/ v
in

A
A + 2
.
SOLUTION 4.29. (a) By inspection the voltage gain for the ideal case is 1. When A=1000, the gain
becomes 0.998, thus 0.2%.
(b) Repeating the method of P4.28, and setting v
out
/ v
in

AR
f
R
f
+ R
1
+ AR
1
to 1 and solving for
R
f
10.417k.
(c) Solving the previous equation when the gain is 1, R
f
/ R
1
(A+ 1)/ ( A1).
SOLUTION 4.30. (a)The first part was obtained in P4.29. Rearranging the equation yields
v
out
/ v
in

R
f
R
1
1
1 + 1+ R
f
/ R
1 ( )
/ A




1
]
1
1
.
Thevenin Probs, 7/11/01 - P4.12 - @R.A. Decarlo & P. M. Lin
(b) The error is caused by (1+ R
f
/ R
1
) / A in the denominator, and may be defined, in percent, as
100
1
1+ 1+ R
f
/ R
1 ( )
/ A




1
]
1
1
100. Thus for the conditions listed in the problem, it will always be less than
2.05%. With A = 10000 it will be less than 0.21%.
SOLUTION 4.31. (a) Substituting the non-ideal model, and writing KCL at the inverting terminal,
(V

v
in
)/ R
1
+ V

/ R
in
(v
out
V

) / R
f
is obtained. Now observe the following dependencies,
i
out
v
out
/ R
L
, and v
out
AV

(i
out
+ (v
out
V

)/ R
f
)R
out
. Using these three equations, substitute
the second into the third and then solve for v
out
/ v
in
using the last two. This yields
V

v
out
+ v
in
R
f
R
1
1/
R
f
R
1
+
R
f
R
in
+1



_
,

v
out
1
R
out
R
L

R
out
R
f
A
R
out
R
f






_
,




and
v
out
/ v
in

R
f
R
1
1
1+
1 +
R
out
R
f
+
R
out
R
L



_
,

1+
R
f
R
1
+
R
f
R
in



_
,

A
R
out
R
f







_
,



















_
,












A gain of 9.988
(b) For an ideal op-amp the gain is R
f
/ R
1
=-10.
(c) The error is about 0.1175%.
SOLUTION 4.32. The gain is 9.883, and the error 1.16%
SOLUTION 4.33. This derivation was performed in P4.31.
Thevenin Probs, 7/11/01 - P4.13 - @R.A. Decarlo & P. M. Lin
SOLUTION 4.34. Assume that the sliding contact is at the bottom of R
p
. Then, writing KCL at the
inverting terminal yields v
in
/ R
o
(v
out
v
in
)/ R
p
. This implies v
out
/ v
in
1 + R
p
/ R
o
. When the
slider is at the top, it is evident that v
out
v
in
. Therefore 1 v
out
/ v
in
1 +
R
p
R
o
.
SOLUTION 4.35. Writing KCL at inverting input, and making use of voltage division,
v
in
/ R
1
v
out
/ R
f [ ]
where is the fraction of v
out
that appears across R
f
. Hence,
v
out
v
in

R
f
R
1
.
When the slider is at the top 1 and
v
out
v
in

R
f
R
1
. When the slider is at the bottom, the fraction of vout
appearing across R
f
is
R
f
/ / R
0
R
f
/ / R
0
+ R
p

R
f
R
0
R
f
+ R
0

1
R
f
R
0
R
f
+ R
0
+ R
p

R
f
R
0
R
f
R
0
+ R
p
(R
f
+ R
0
)
. Hence
1

R
f
R
0
+ R
p
(R
f
+ R
0
)
R
f
R
0
1 +
R
p
R
0
+
R
p
R
f
. It follows that
v
out
v
in

R
f
R
1

R
f
R
1
1 +
R
p
R
0
+
R
p
R
f



_
,

.
Therefore the range of achievable voltage gain is

R
f
R
1

v
out
v
in

R
f
R
1
1 +
R
p
R
0
+
R
p
R
f



_
,

SOLUTION 4.36. Using the basic non-inverting configuration of figure 4.10 characterized by
v
out
/ v
in
1+
R
f
R
1



_
,

, i.e., 1+
R
f
R
1



_
,

.
SOLUTION 4.37. At first glance, one might use two inverting configurations, figure 4.5, in cascade.
However, such would not have infinite input resistance. To circumvent this problem we add a buffer
amplifier as per figure 4.7 at the front end of a cascade of two inverting configurations. The resulting
overall gain is
R
f1
R
11



_
,

R
f 2
R
12



_
,

. Indeed, such a configuration can achieve theoretically any gain greater
than zero.
Thevenin Probs, 7/11/01 - P4.14 - @R.A. Decarlo & P. M. Lin
SOLUTION 4.38. Using a single inverting amplifier configuration, figure 4.5, preceded by a buffer stage
of figure 4.7. The gain is
R
f
R
1
.
SOLUTION 4.39. By KVL for figure P4.39a, V
o
i
1
R
f
. Thus to achieve V
o
i
1
r
m
in figure P4.39b,
we set R
f
r
m
.
SOLUTION 4.40. Writing KCL at the inverting node of the ideal op amp yields I
L
V
i
/ R
a
, which is
indeed independent of the load resistor which has no effect on the load current.
SOLUTION 4.41. The current through the LED is I
L
10
R
1
10k


_
,
/ 3.8k , so for (a) it is 1.32mA and for
(b) 2.11mA.
SOLUTION 4.42. Applying KCL at the inverting terminal, I
L
v
in
/ R
1
. Again, ideally, R
L
does not
affect I
L
.
SOLUTION 4.43. (a) Defining a temporary voltage V
o
at the output of the op-amp, we can write KCL at
the inverting and non-inverting terminal:
(V

2)/ 1k (V
o
V

) / 2k
V

/100 + (V

V
o
) / 200 I
out
Substituting the first equation into the second and simplifying causes V
o
to drop out and I
out
20mA.
(b) The answer remains the same as the value of the load resistance was not used for finding the load
current.
SOLUTION 4.44. Using the same approach as for the previous question, but with resistor labels instead,
the following equations are obtained from KCL:
V

R
2
V
s
+ R
1
V
o
R
2
+ R
1
I
out
V

R
2
+ R
1
R
1
R
2



_
,

V
o
R
2
Substituting the first into the second yields I
out
V
s
/ R
1
.
Thevenin Probs, 7/11/01 - P4.15 - @R.A. Decarlo & P. M. Lin
SOLUTION 4.45. (a) V
s
5 V, (b) I
out
10mA sets R
1
V
s
/ I
out
500 . (c) From KVL and Ohm's
law, I
s
(V
s
+ R
L
I
out
) / R
1
. We require I
s
< 0.5 mA. This means that in the worst case, R
L
= 500 ,
(V
s
+ R
L
I
out
)
0.5 10
3
R
1

5 + 500 0.01
0.25
40 < .
(d) From KVL and Ohm's,
V
o
R
L
I
out
(I
out
+(R
L
I
out
)/ R
1
)R
2
20 V
Hence
R
2

20 R
L
I
out
(I
out
+ (R
L
I
out
) / R
1
)

20 5
0.01 + 5/ 500
750
Hence one design is to pick R
2
750 and 40 which impliesR
2
30 k.
SOLUTION 4.46. The exact same design as P4.45 can be used with the isolation buffer of figure 4.7
placed at the input of it in order to provide the infinite input resistance needed by P4.46b.
SOLUTION 4.47. The general expression for this summing circuit is
V
out

R
f
R
o
V
o

R
f
R
1
V
1

R
f
R
2
V
2

R
f
R
3
.
(a) Using the expression above | V
out
|| 1 0 0 8| E 9E.
(b) | V
out
|| 0 2 4 0| E 6E.
(c) It has to be a linear combination of 8, 4, 2, 1, thus [1 1 0 1] would yield 13E.
(d) With the same approach, [0 1 1 1].
SOLUTION 4.48. For this implementation we add an extra R-2R branch along with an extra summing
input to the op amp. From the theory developed in Example 4.9 the total resistance seen by the source is
2R. For the total power supplied by the source to be less than 0.02 W, we require R
E
2
2 0.02 ( )
2.5 k.
Thevenin Probs, 7/11/01 - P4.16 - @R.A. Decarlo & P. M. Lin
SOLUTION 4.49. The same steps as in the previous questions are repeated. Because the resistance seen
by the source is unchanged no matter how many branches are added to the R-2R network,
R
E
2
2 0.01 ( )
5 k.
SOLUTION 4.50. (a) If the input is 3v
max
/ 8, then the first comparator will give V
sat
, keeping S
2
down.
The next comparator will output +V
sat
, causing S
1
to go up. After subtraction, the input to the last
comparator is v
max
/ 8 yielding +V
sat
at its output since its input is slightly above the reference input
level. Thus the logic output values are [0 1 1].
(b) Putting in 6v
max
/ 8, will cause +V
sat
and S
2
to go up. The input to the second comparator will be
2v
max
/ 8, which will cause +V
sat
and S
1
to go up. The input to the last comparator will be 0, thus it will
output V
sat
. The corresponding logic output is [1 1 0].
SOLUTION 4.51. Simply add a subtractor and switch to the last comparator, followed by an additional
comparator. The reference level to the new (additional) comparator will be v
max

/16, and its output will be


the new least significant bit.
SOLUTION 4.52. (a) Writing the node equation for figure P4.52c,
V
out
R
L
'
+
(V
out
V
1
)
10k

A(0 V
1
) V
out
R
out
which implies that
V
out
/ V
1

1
10k

A
R
out



_
,

1
R
L
'
+
1
10k
+
1
R
out



_
,

For figure P4.52a, the corresponding node equation is
V
out
R
L

A(0 V
in
) V
out
R
out
which leads to
Thevenin Probs, 7/11/01 - P4.17 - @R.A. Decarlo & P. M. Lin
V
out
/ V
in

A
R
out



_
,

1
R
L
+
1
R
out



_
,

Note that
1
R
L

1
R
L
1
+
1
10k
, which when substituted into the later equation make both of them
approximately the same since the 1/10k term in the numerator of V
out
/ V
1
has a negligible contribution.
(b) Writing the node equation for figure P4.52d, yields
V
out
R
L
'
+
(V
out
V
2
)
10k

A(0 V
1
) V
out
R
out
Hence
V
1
V
2
100|| R
in
(100 || R
in
) +10k



_
,

V
2
101
Solving produces V
out
/ V
2

1
10k

A
101R
out



_
,

1
R
L
1
+
1
10k
+
1
R
out



_
,

. Note that as in (a) the 1/10k term in the numerator is
negligible; after eliminating this negligible term, one sees that
V
out
V
2
is 101 time smaller than
V
out
V
in
.
SOLUTION 4.53. (a) Using the equation just derived, after substituting in the values, the gain is 980.392
(b) From the previous equation, V
out
/ V
1
980.382 ; write KCL at the non-inverting terminal to obtain,
V
in
V
1
10k

V
1
R
in
+
V
1
V
out
10k
; substitute V
out
980.382V
1
; solve for V
1
/ V
in
, and then multiply both gains
to obtain (V
out
/ V
1
)(V
1
/ V
in
) V
out
/ V
in
0.9979 .
(c) They only differ by about 0.01%, thus they are very similar.
SOLUTION 4.54. Writing out the transfer equation, V
out

R
2
R
1
V
s2

R
2
R
1
V
s1
, thus R
2
/ R
1
4 . Using
R
2
100k, R
1
25k. As expected SPICE shows to noticeable difference in outputs when the source
resistances are varied.
SOLUTION 4.55. Due to the ideal nature of the op-amp, the voltage V
R
b
V
s2
V
s1
. By KVL
Thevenin Probs, 7/11/01 - P4.18 - @R.A. Decarlo & P. M. Lin
V
2
V
s2
+ R
a
(V
s2
V
s1
)/ R
b
V
1
V
s1
R
a
(V
s2
V
s1
) / R
b
Next, V
1
V
2
(V
s1
V
s2
)(1+ 2R
a
/ R
b
) .
SOLUTION 4.56. (a) Noticing that the final stage is a summing op-amp in which V
out

R
2
R
1
V
1

R
2
R
1
V
2
.
From the previous question, V
out

R
2
R
1
(V
1
V
2
)
R
2
R
1
(1+ 2R
a
/ R
b
)(V
s1
V
s2
) . Thus

R
2
R
1
(1+ 2R
a
/ R
b
). The gain can be varied by adjusting the single resistance R
b
.
(b) Picking the set of values below will satisfy the requirement:
R
2
100k, R
1
100k, R
a
20k, R
b
10k.
(c) Doing the SPICE simulation using the parameters from (b) yield 5 V at the output for
V
s1
V
s2
2 1 V. Setting R
b
arbitrarily to 20 k, the output now becomes 3 V, which agrees with the
relationships developed earlier.
PROBLEM SOLUTIONS CHAPTER 5.
Solution 5.1. (a) V
s
= 10 V, P = 20 W and P = V
s
I
s
implies I
s
= 2 A.
(b) R
in
= V
s
/I
s
= 10/2 = 5
(c) By the linearity/proportionality property:
V
s
new
V
s
old

I
s
new
I
s
old
which implies
2
10

I
s
new
2
implies
I
s
new
0.4 A.
(d) P
new
V
s
new
I
s
new
2 0.4 0.8 watts. Observe that
P
new
P
old

0.8
20

V
s
new
V
s
old

2
10
It follows that the proportionality property does not hold for power calculations.
Solution 5.2 First note that the ratio I
R
/V
S
is constant. With the given values of voltage and current, this
ratio is:
I
R
/V
S
= 0.25/25 = 0.01
Power dissipated in the resistor is
P = I
R
2
R = 2.5 I
R
2
= 2.5/R = 0.25 I
R
= 0.5
Since I
R
is always 0.01V
S
, it follows that V
S
= 50V.
Solution 5.3 Label the resistances R
1
, R
2
, and so on in the manner shown in Example 5.11. In this
problem, we have R
1
to R
10
(the last being the 2 Ohm resistance at the voltage source). First, assume that
V
1
(the voltage across R
1
) is 1V. Then evaluate the rest of the currents and voltages until you deduce the
resulting V
S
. It should be noted that the equivalent resistance looking into R
3
, R
5
, R
7
, and R
9
is always
2.
V
1
1 I
1

V
1
4
0.25 I
2
0.25 V
2
I
2
2 0.5 V
V
3
V
1
+ V
2
1.5 I
3

V
3
3
0.5 I
4
I
3
+ I
2
0.75 V
4
I
4
4 3
V
5
= V
3
+ V
4
4.5 I
5

V
5
3
1.5 I
6
I
5
+ I
4
2.25 V
6
I
5
4 9
V
7
V
6
+ V
5
13.5 I
7

V
7
3
4.5 I
8
I
7
+ I
6
6.75 V
8
I
8
4 27
V
9
V
8
+ V
7
40.5 I
9

V
9
3
13.5 I
10
I
9
+ I
8
20.25 V
10
I
10
2 40.5
V
S
V
9
+ V
10
40.5 + 40.5 = 81 V
Thus, an 81 V input produces a 1 V output V
out
= (1/81)V
S
= 2 V.
Solution 5.4 Label the resistances R
1
to R
10
progressively from right to left just like in the previous
problem. Then, assume I
out
= 1 and proceed as follows:
I
out
= 1 V
1
I
out
4 4 I
2
=
V
1
4
1
I
3
I
1
+ I
2
2 V
3
I
3
4 8 V
4
V
3
+ V
1
12 I
4

V
4
3
4
I
5
I
4
+ I
3
6 V
5
I
5
4 24 V
6
V
5
+ V
4
36 I
6

V
6
3
12
I
7
I
6
+ I
5
18 V
7
I
7
4 72 V
8
V
7
+ V
6
108 I
8

V
8
3
36
I
9
I
8
+ I
7
54 V
9
I
9
4 216 V
10
V
9
+ V
8
324 I
10

V
10
3
108
I
S
I
10
+ I
9
162
I
out
/I
S
= 1/162 I
out
= (1/162)40.5 = 0.25A.
Solution 5.5 (a) MATLAB code given in problem.
(b) Subsitute to obtain V
1
= 10V.
(c) R
eq
= V
S
/I
S
= 11.6667.
(d) First, define r1 = 1:0.25:10;
then create an outermost loop around the code of part (a) as: for j=1:length(r1)
then, in the statement defining R, do R = [R1(j), R2, R3, R4, R5, R6, R7, R8];
Finally, replace the last statement with Vs(j) = V(n) + V(n-1); end;
The following is the resulting plot:
Solution 5.6 (a)
The following code can be used:
n = 9;
v = zeros(n,1);
i = zeros(n,1);
r = [r1 r2 r3 r4 r5 r6 r7 r8 r9];
i(1) = 1;
v(1) = i(1)*r(1);
i(2) = i(1);
for k=2:2:n-2
v(k) = r(k)*i(k);
v(k+1) = v(k)+v(k-1);
i(k+1) = v(k+1)/r(k+1);
i(k+2) = i(k+1) + i(k);
end;
v(8) = i(8)*r(8);
v(9) = v(8) + v(7);
i(9) = v(9)/r(9);
Is = i(9) + i(8);
It follows that I
s
16.9877A.
(b) By the proportionality property:
I
1
new
I
1
old

I
s
new
I
s
old
I
1
new

1
16.9877
200mA 11.77mA
(c) R
eq
= v(9)/Is = 38.15.
Solution 5.7 V
a
= 12V, i
B
= 60m
By inspection:
V
out_a
= 300/90012 = 4V
V
out_b
= (300||600)60m = 12
_ V
out
= 4 + 12 = 16V.
*SOLUTION 5.8. Part 1: Set the 3 A current source to zero. This generates an open circuit in place of
the current source eliminating the effect of the series 0.1 S resistor. The equivalent circuit is:
By voltage division,
V
L
12V

1
0.25 + 0.2 + 0.05
1
0.25 + 0.2 + 0.05
+
1
0.1
12
2
2 +10
12 2 V
Part 2: Set the 12 V source to zero. This generates a short circuit in place of the voltage source which
shorts out the effect of the 0.5 S resistor. The equivalent circuit is:
Note that the 0.1 S resistor in series with the 3 A source is redundant to the calculation of VL. Hence, by
Ohm's law,
V
L
3A

1
0.25 + 0.2 + 0.05 + 0.1
3
3
0.6
5 V
Therefore by superposition,
V
L
V
L
12V
+ V
L
3A
2 + 5 7 V
Solution 5.9 Replace the dependent source by an independent voltage source V
S
:
In the following analysis, we have to always compute V
a
because It defines the constraint on V
s
. So, when
only the 88V source Is active, VA Is the result of voltage division between the 60||30 resistor and the
120||30 resistor. So,
V
a_1
= 40V
And, since deactivated V
S
, V
out_1
= 0.
Now, due to the 55V source, we have
V
S
Now, the 120 and 60 resistors are in parallel, and the same can be said about the 30 and 30 resistors.
Thus, another voltage divider gives:
V
a_2
= 15V and V
out_2
= 0V.
Finally, when V
S1
is active, the left part of the circuit consists only of resistances, so V
a_3
= 0. V
out
is given
by another divider formula:
V
out_3
= 90/100V
S
Now add all contributions:
V
a
= 40 15 + 0 = 25V
V
out
= 0 + 0 + 0.9V
S
, where V
S
= 2V
a
.
V
out
= 0.9225 = 45V.
Finally, P = V
2
/R = 22.5W.
Solution 5.10
Due to 3A source:
i
out
= 1A by current division between the two paths. So, v
out_1
= 2 V.
Due to 1A source:
i
out
= 2/3A again by current division. So, v
out_2
= 4/3V.
Due to 1vV source:
i
out
= 14/6 = 7/3A (by Ohms Law). So, v
out_3
= 14/3V.
Finally, v
out
= 6/3 + 4/3 + 14/3 = 8V, and the power delivered by the source is 81 = 8 W.
Solution 5.11
Due to 22 V source:
R
eq
= 900||225 = 180. Now, by voltage divider:
V
out_1
= 0.522 = 11 V.
Due to the 20 V source:
R
eq
= 180||225 = 100 . So, again, by voltage division:
V
out_2
= 900/(900+100)20 = 18 V.
Finally, due to current source:
We have three resistances in parallel with a resistance equal to 90 . So, V
out_3
= 0.190 = 9 V.
V
out
= 11 + 18 + 9 = 38V and P = 3838/900 = 1.6 W.
Solution 5.12
Find contribution to V
out
:
First, note that no current flows through R1 - R3 because of the virtual ground property of the op-amp.
Thus, this circuit is identical to the inverting amplifier studied in Chapter 4. So,
0
0
0 _
V
R
R
V
f
out

Similarly, when each of the other sources is activated, the circuit will be an inverting amplifier. So,
V
out _1

R
f
R
1
V
1
, V
out _2

R
f
R
2
V
2
, V
out _3

R
f
R
3
V
3
V
out
R
f
V
0
R
0
+
V
1
R
1
+
V
2
R
2
+
V
3
R
3



_
,

Solution 5.13 Due to the 4 V source, the circuit looks like an inverting amplifier:
So, V
out_1
= -30/10(-4) = 12 (from the results of Chapter 4).
Again, note here that no current flows through the two resistances connected to the + terminal of the op
amp. Since no current flows through them, then no voltage develops across them. So, the + terminal can be
assumed to be connected to ground, and this is why we say that the circuit looks like that of the inverting
amplifier.
Now, due to the 6 V source:
The voltage at the + terminal is established by a resistive divider between the two 10K resistors. So, this
voltage is 3V. Thus, the voltage at the negative terminal is also 3V. We can now use KVL on the inverting
side of the op-amp to get:
V
out_2
= 3 + 30k0.3m = 12V
So, V
out
= 12 + 12 = 24V, and P = (24)(24)/500 = 1.15W.
Solution 5.14
(a) When V
S2
is deactivated. The circuit looks like two inverting amplifiers in cascade.
Thus, by inspection, V
1
= 2V
S1
and V
out_1
= 3V
1
= 6V
S1
= 3V.
(b) Similarly, when V
S1
is zero, V
1
is zero because the first inverting amplifier has zero input. Thus, the
circuit consists just of the second inverting amplifier:
V
out_2
= 3/2V
S2
= 4.5
(c) V
out
= 3 4.5 = 1.5V.
*SOLUTION 5.15. For V
s1
and V
s2
, the situation reduces to the analysis of two inverting amplifiers in
cascade. For V
s3
, the situation is simply a single stage inverting amplifier. Note that because of the
virtual ground at the inverting terminal of the op amp, when V
s1
and V
s2
are zero, they have no
contribution to the input of the second stage.
(a) With V
s2
and V
s3
set to zero,
V
out
s1

R
R


_
,
2R
2R


_
,
V
s1
V
s1
5 V
(b) With V
s1
and V
s3
set to zero,
V
out
s2

R
R


_
,
2R
R


_
,
V
s2
2V
s2
2 (2.5) 5 V
(c) With V
s1
and V
s2
set to zero,
V
out
s3

R
R


_
,
V
s3
V
s3
2 V
(d) By superposition,
V
out
V
out
s1
+ V
out
s2
+ V
out
s2
2 V
Solution 5.16
If the op-amp were ideal, we would get:
V
out
= 4V
S1
2V
S2
= 26
This is clearly beyond the linear range of operation of the op-amp. In other words, the amplifier responds
in a non-linear manner to this level of input. Hence, superposition, which relies on linearity, cannot be
used.
Solution 5.17
We know by linearity that
V
out
= aI
s1
+ bV
s2
Substitute the first measurement to obtain:
5 = 0 + b10 b = 0.5
Now, take the second measurement:
1 = a10 + 0 a = 0.1
So,
V
out
= 0.1I
s1
+ 0.5V
s2
At 20A, 20V: V
out
= 12V
Solution 5.18
Again V
out
= aI
s1
+ bV
s2
Substitute the two measurements to obtain:
5a + 10b = 15
2a + 5b = 10
These two simultaneous equations in a and b can easily be solved to obtain:
a = 5 and b = 4
Therefore, at 1A and 5V,
V
out
= 5 + 20 = 15V
Solution 5.19 (a)
Again,
V
out
= aI
s1
+ bV
s2
Substituting the result of the first measurement into this equation yields:
a4cos(2t) + 0 = 2cos(2t) a = 0.5
Now, substitute the second measurement:
0 + 10b = 55 b = 5.5
Therefore, for the given input current and voltage:
V
out
= cos(2t) 55cos(2t) = 56cos(2t) V
(b) V
out
= 2cos(5t) + 110cos(5t) = 112cos(5t) V
Solution 5.20 (a)
First of all,
I
load
= aV
a
+ bI
b
Now, substitute the two measurements into this equation:
7a + 3b = 1
9a + b = 3
Solving these two equations for the unknowns a and b, we get
a = 0.4 and b = 0.6
(b) I
load
= 0.415 0.69 = 0.6A
Solution 5.21 (a)
We know that the output is going to be a linear combination of the three inputs:
V
out
= aI
s1
+ bV
s2
+ cV
s3
Now, substitute the three measurements into this relationship:
50ma 2b + 5c = 13
0 + 3b + 5c = 2
0 + 2b + 4c = 0
These equations can be written in matrix form and solved as follows:
0.05 2 5
0 3 5
0 2 4





1
]
1
1
1
a
b
c





1
]
1
1
1

13
2
0





1
]
1
1
1

a
b
c





1
]
1
1
1

100
4
2





1
]
1
1
1
(b) Substitute the given values of the input sources to obtain:
V
out
= 1001 440 + 210 = 40V
Solution 5.22
V
out
= AI
s1
+ BV
s2
+ CV
s3
Substituting the measurements into this equation results in a system of three equations and three
unknowns. This system can be written in matrix form by inspection:
30 10
3
2 1
20 10
3
4 2
10 10
3
3 1






1
]
1
1
1
1
A
B
C





1
]
1
1
1

11.5
27
14





1
]
1
1
1

A
B
C





1
]
1
1
1

150
5.5
4





1
]
1
1
1
This can be solved to obtain:
Solution 5.23 (a)
The coefficient matrix is inverted, and both sides of the nodal equation are multiplied by it to obtain:
V
a
V
b



1
]
1

43.0108 0.233
43.0108 1.4337



1
]
1
0.02V
s1
0.00125V
s2
V
s2



1
]
1
Expanding the first row of the above equation gives:
V
a
= 0.8602V
s1
0.0538V
s2
+ 0.233V
s2
This is exactly in the form required, where A = 0.8602 and B = 0.1792
(b) For this part, we expand the second row of the equation:
V
b
= 0.8602V
s1
0.0538V
s2
1.4337V
s2
Again, this is in the desired form, where A = 0.8602 and B = 1.4875
(c) V
ab
= V
a
V
b
= 0 + 1.667 V
s2
Thus, A = 0 and B = 1.667
Solution 5.24 (a)
Again, we invert the coefficient matrix to obtain
I
1
I
2
I
3
v
1
v
2








1
]
1
1
1
1
1
1

0.0022 0.0022 0.0022 0.6296 0.0741


0.0022 0.0022 0.0022 0.7037 0.2593
0.0022 0.0022 0.0022 0.3704 0.0741
0.2222 0.7778 0.2222 137.037 92.5926
0.3333 0.3333 0.6667 11.1111 77.7778








1
]
1
1
1
1
1
1
V
s1
0
0
I
s2
0








1
]
1
1
1
1
1
1
Expanding the first row of this equation:
I
1
= 0.0022V
s1
+ 0.6296I
s2
So, A = 0.0022, B = 0.6296
(b) Similarly, expanding the third row:
I
3
= 0.0022V
s1
+ -0.3704I
s2
So, A = 0.0022, B = -0.3704
(c) By the same procedure, A = -0.3333 and B = -11.111
Solution 5.25
Invert the coefficient matrix and mujltiply both sides of the equation in the problem by this inverse matrix
to obtain
V
1
V
2
V
3
I
a
I
b
I
c
I
d












1
]
1
1
1
1
1
1
1
1
1
1

0 0 0 3 2 1 4
0 0 0 3 1 1 4
0 0 0 12 3 3 16
0 0 0 4 1 1 5
0 1 0 16.6 9.2 5.2 21.8
1 1 0 42.1 16.2 11.7 53.8
1 1 1 26.1 12.2 7.7 32.8












1
]
1
1
1
1
1
1
1
1
1
1
I
s1
0
0
0
V
s2
0
0












1
]
1
1
1
1
1
1
1
1
1
1
The second to last row can be expanded to get I
c
= I
s1
16.2V
s2
A = 1, B = 16.2
Solution 3.26 (a)
First compute the response due to V
1
:
By voltage divider:
V
out_1
= 6/(2+6+1)V
1
= 2/3V
1
Then, due to I
2
:
I
2
flows through the 20 ohm resistor in series with 6||3. Thus,
V
out_2
= 2I
2
Therefore, V
out
= 2/3V
1
2I
2
(b) V
out
= 8cos(10t) 4
*SOLUTION 5.27. (a) By linearity V
out
A V
s1
+ B I
s2
.
To find A, let I
s2
= 0. The circuit becomes a ladder network as follows.
Let V
out
A
= 1 V. Then
I1 = 1/420 + 1/70
I1 = 1.6667e-02
V400 = I1*40 + 1
V400 = 1.6667e+00
I400 = V400/400
I400 = 4.1667e-03
I2 = I1 + I400
I2 = 2.0833e-02
Vs1 = I2*20 + V400
Vs1 = 2.0833e+00
A = 1/Vs1
A = 4.8000e-01
To find B, let V
s1
= 0. The circuit becomes a ladder network as follows.
Again assume that V
out
B
= 1 V. Then
I1 = 1/420 + 1/70
I1 = 1.6667e-02
V400 = I1*40 + 1
V400 = 1.6667e+00
I400 = V400/400
I400 = 4.1667e-03
I20 = V400/20
I20 = 8.3333e-02
Is2 = I20 + I400 + I1
Is2 = 1.0417e-01
B = 1/Is2
B = 9.6000e+00
Hence by linearity V
out
0.48 V
s1
+ 9.6 I
s2
.
(b)
Vout = A*20 + B* 0.5
Vout = 1.4400e+01
(c) Doubling resistances does not change voltage ratios hence A is the same. However, the doubling also
doubles the voltage to current ratio. Hence, B is doubled. It follows that if all resistances are
doubled, then
V
out
0.48 V
s1
+ 19.2 I
s2
Solution 5.28 (a) (b)
We solve parts (a) and (b) at the same time. First, we find the responses to V
s1
:
Equivalent resistance across V
o2
: 60||30 = 20
Now, by voltage divider: V
o2_1
= 20/140V
s1
This voltage now divides between V
o1
and the 10 ohm resistance:
V
o1_1
= 20/30V
o2_1
= 2/31/7V
s1
= 2/21V
s1
Now, compute the responses due to I
s2
:
Equivalent resistance across V
o2
: 60||120 = 40
By voltage division: V
o2_2
= 40/50V
o1_2
Where V
o1_2
= I
s2
(50||20) = 14.286I
s2
Now, add the contributions:
V
o1
= 0.0952V
s1
+ 14.286I
s2
V
o2
= 0.1429V
s1
+ 11.4288I
s2
Solution 5.29 (a)
Find contribution due to v
1
:
The parallel combination results in a 12 ohm resistance in series with the remaining two. Thus, by voltage
division:
v
out1
= 24/48v
1
= 0.5v
1
Now, due to v
2
The equivalent resistance across v in this figure is 36||(24+12) = 18, which means that by voltage divider:
v = 0.5v
2
Similarly, by another voltage division application
v
out2
= 24 / 36 v= 0.660.5v
2
= 0.333v
2
Now, due to i
3
Define Req1 = 24 + 36||18 = 36 . This resistance is in parallel with the 12 ohm resistance to introduce
an equivalent of 9 . The total voltage that develops across this 9 ohm resistance is 9i
s3
. This voltage
divides between v
out
and the 36||18 resistance:
v
out3
= 24/36v
1
= -924/36i
s3
= -6i
s3
Finally, due to i4:
A similar analysis of this resistive network can reveal that v
out4
= 6i
s4
. Thus
v
out
= 0.5v
1
+ 0.333v
2
6i
s3
+ 6i
s4
(b) For this part, note that scaling resistance values does not affect voltage ratios. This can be evident from
the application of any voltage divider formula. On the other hand, scaling resistances does affect current-
to-voltage or voltage-to-current ratios. This is by definition of a resistance! So, in the above equation,
doubling the resistances does not affect the first two terms, but doubles the second two terms.
Solution 5.30 (a)
Define two clockwise mesh currents: I
1
in the bottom left loop and I
2
in the top loop. The bottom right
loop has a current source, so it will not be considered:
V
a
(I
1
-I
2
)3 I
2
(I
1
i
b
) = 0
and
6I
2
I
2
+ (I
2
I
1
)3 = 0
Solving these two equations for the two currents gives:
I
1
= 8 A and I
2
= 3 A
The power delivered by the dependent source is: P i
x
I
ix
where I
ix
is the current leaving the ' +'
terminal of the dependent voltage: P i
x
(I
2
I
1
) I
2
(I
2
I
1
) 15W
v
out
= I
1
+ i
b
= 8 + 26 = 34V
(b) Now, we express v
out
= Av
a
+ Bi
b
Turning off i
b
, we still have two loops, in which we can define the same mesh currents as above to obtain:
v
a
3(I
1
I
2
) I
2
I
1
= 0
I
2
3(I
2
I
1
) 6I
2
= 0
Solving for the two currents, we get I1 = 4/13v
a
which sets up v
out
across the 1 ohm resistor:
V
out1
= 4/13v
a
Now, turn off the voltage source:
Write a node equation at v
out
:
b out
b
v
out out out
i v
i
v v v
out
13
9
0
3 1 6
2
6
?

+ +
So, v
out
= 4/13v
a
+ 9/13i
b
(c) Substitute into the above equation: v
out
= 35V
Solution 5.31 By inspection:
where the leftmost current is 0.25 with a resistance of 32. Similarly, the downward current is 0.75 and its
resistance is 32. This reduces to:
Solution 5.32
The circuit can be transformed as follows:
Write two nodal equations at 1 and 2:
0 5
16 6 . 9
9
2
0
2
9
2
1
2 2 1 2
2 1 1
+ + +

+ +
m
k
v
k
v
m
k
v v
k
v v
m
k
v
m
These two equations can be solved using any method to obtain:
v
1
= 11.2V and v
2
= 2.4V
The power absorbed by the 9.6k resistor is:
P
V
2
2
9.6 10
3
0.6mW
Solution 5.33
Then
Output voltage is 5V
(b) P = 1.25W
(c) For a given resistance, doubling the voltage increases the current by two times. So, the current is
doubled. It follows that V
out
new
2 V
out
old
10V
Solution 5.34
This circuit is easy enough to solve by inspection. V
s
= 28V.
Solution 5.35
Now, write two node equations:
m
v
v
v v
m
v v v
80
125
032 . 0
500 500
12
500 500 500
2
1
1 2
1 2 1
+ +
+
These two equations can be solved to get v
1
= 2.8V, v
2
= -0.4V.
Solution 5.36
Now, we can write the node equations:
0 2 . 0 1 . 0 1
100 50 50 100
100
100
0 95 . 0
20
4 . 0 2 . 0
25
100
25 50 50
2 1 2 2
1 1 2 1
+ + +
+ + + +
v v v v
v v v v
Solving these two equations yields:
v
1
= 25V and v
2
= 20V
*SOLUTION 5.37. After a source transformation on the 30 V independent source and one on the 9Vx
dependent source we obtain the circuit below.
Transforming the two dependent voltage sources and combining yields the following circuit.
Writing a single node equation for Vx yields
7.5
V
x
4
+
V
x
2
+
V
x
2
1.25V
x
Hence, V
x
= 6 V.
Solution 5.38
Replace the dependent source with a temporary independent source. When doing the analysis, always
compute Va in order to keep track of the constraint on the dependent source.
When Vt is not active, v
out
is obtained from a voltage divider between the 60||30 combination and the 15
ohm resistor:
v
out1
= 4/7V
s
Similarly v
a1
= 3/7V
s
Now, compute the response due to the temporary source.
Straightforward voltage division also applies here to get:
v
a2
= -2/7Vt
and v
out2
= -5/7Vt
So, v
out
= 4/7V
s
5/7Vt
va = 3/7V
s
2/7Vt, where Vt = v
a
v
a
= 3/7V
s
2/7v
a
Rearranging,
s a
V v
2 7
3
+

Then,
v
out

4
7
V
s

5
7
v
a
v
out

4
7 + 2
V
s
Solution 5.39 Replace the dependent source by a temporary independent source:
When Vt is shorted, the result is a ladder network. The input resistance looking each of the vertical
branches is R. Label these vertical branches V
1
, V
a
, V
2
from left to right. It follows that
V
1
= R/3RV
s
_ V
a_1
= R/2R R/3RV
s
= 1/6V
s
Also, V
out_1
= 0
Now, short V
S
and turn on Vt.
Again, the result is a ladder network (note the symmetry in the above figure). Thus, we can write by
inspection:
V
a_2
= 1/6Vt
V
out_2
= Vt
Adding the contributions:
V
out
= 0 + Vt = Vt = V
a
where we have substituted the constraint on Vt
and V
a
= 1/6V
s
+ 1/6Vt = 1/6V
s
+ 1/6 V
a
Va = [1/(6 )]V
s
Substitute this Va into the expression for Vout:
s out
V V

6
Solution 5.40
The first step is to replace the dependent source with a temporary independent source. Then, superposition
can proceed as usual.
Now, lets short the temporary source, Vt. Again, this network is a ladder network, like the one in the
previous problem. However, now, the equivalent resistance looking into each of the vertical branches (from
left to right) is different. Now, it is
R
eq
= 20R||(R+4R) = 20R5R/25R = 4R
Now, again, define the voltages across these three vertical branches (from left to right) as V
1
, V
a
, V
2
. It
follows by voltage division that
V
1
= 4/9V
s
V
a_1
= 4/5 4/9 V
s
V
a_1
= 16/45 V
s
It should be noted that V
out
= 0.
Now, short the input source and find the response due to Vt. Again, in this case, the circuit is identical to
the case when V
s
was active (note the symmetry in the above figure). Therefore,
V
a_2
= 16/45 Vt
V
out_2
= Vt
Adding the two contributions:
( )
s a
a s a
V V
V V V

?
+
16
45
1
45
16
45
16
and
s out
a t out
V V
V V V
16 45
16


Solution 5.41
First, replace the controlled source by a temporary independent source:
Now, do a source transformation on the Vt source:
The equivalent resistance seen by the current source is 99. Therefore:
V
out
= -99/100Vt = -0.99Vt
Now, Vt = V
g
and I
L
= -0.99Vt/990k where I
L
is the current through R
2
V
R2
= I
L
R
2
= (100k/990k)(-0.99Vt) = -0.1Vt
By KVL
V
g
= V
R2
+ V
s
= -0.1Vt + V
s
= -0.1V
g
+ V
s
Rearranging
s g
V V
1 1 . 0
1
+

Finally, and substituting:


91 . 8
1 . 0 1
1
99 . 0

+

s
out
s
out
V
V
V
V
Solution 5.42
R a
R b b
x a a
x b b
V
V
in
out
1
1 0
1 0
1 0
1+
+

+
+

At R = 0
20
20
1
0
0
?

b
b
V
V
in
out
At infinite R
1 1
1
1
8
8
a b
a
b
V
V
in
out
?

Finally, at R = 10
4
5 . 0
10
10 1
80 20
1
1
1
1
?
?

+
+
b
a
a
a
Substituting these values for R = 2 yields
14
2 5 . 0 1
2 4 20

+
+

in
out
V
V
SOLUTION 5.43. From the chapter
V
out
V
in

b
0
+ b
1

a
0
+ a
1

b
0
+ b
1

1+ a
1

Equivalently,

V
out
V
in
a
1
+ b
0
+ b
1

V
out
V
in
Plugging in the data yields three equations in three unknowns which in matrix form are:
0 1 0
154 1 1
168 1 2





1
]
1
1
1
a
1
b
0
b
1





1
]
1
1
1

264
154
84





1
]
1
1
1
A=[0 1 0;-154 1 1;-168 1 2]
A =
0 1 0
-154 1 1
-168 1 2
y = [264 154 84]'
y =
264
154
84
Coefs = A\y
Coefs =
2.8571e-01
2.6400e+02
-6.6000e+01
Therefore, a
1
= 0.28571, b
0
= 264, and b
1
= 66. Making a
1
= 1, will yield a different set of answers.
When m = , V
out
/V
in
= b
1
/a
1
= 231.
Thevenin Probs, 7/24/01 - P6.1 - @R.A. Decarlo & P. M. Lin
PROBLEM SOLUTIONS CHAPTER 6
SOLUTION 6.1. (a) V
oc
is found by removing R
L
and doing voltage division.
V V V
OC
28
100 600
600
300 ) 700 || 600 (
) 700 || 600 (
63
+

R
TH
is found by setting the source to zero and by calculating the equivalent resistance seen looking back
between the A and B terminal.
+ 200 600 || ] 100 ) 600 || 300 [(
TH
R
(b) Using
L L
I R P
the power for each resistance may be found by substituting the appropriate R
L
in the following equation.
L
L TH
OC
R
R R
V
P

,
_

2
For 50 , 200 , and 800 , the power obtained is 627.2 mW, 980.0 mW, and 627.2 mW respectively.
The use of Thevenin equivalent does reduce the effort in obtaining the answer.
SOLUTION 6.2. (a) To find R
TH
, open circuit the current source and short-circuit the voltage source. The
resulting resistance seen from terminal A-B is 1 k. Using superposition, the contribution of the current
and voltage source at the open circuit output may be summed as 30 V (2 k/4 k) + 10 mA (2 k/4 k) (2 k).
V
OC
is then 25 V and I
SC
= V
OC
/R
TH
is 25 mA.
(b) Following is a plot of
L
L TH
OC
R
R R
V
P

,
_

2
for R
L
from 100 to 4 k .
Thevenin Probs, 7/24/01 - P6.2 - @R.A. Decarlo & P. M. Lin
SOLUTION 6.3. (a) Turning off the two sources
R
TH
(60 + 60)||40 30 ,
and using superposition
V
OC
6V
40
40 + 60 + 60


_
,
+ 0.1A
60
40 + 60 + 60


_
,
40 3 V.
I
SC
is the obtained as 100 mA.
(b) Using
L
L TH
OC
R
R R
V
P

,
_

2
a load of 90 will absorb 56.25 mW.
(c) It absorbs 75 mW; hence the 30 resistor absorbs more power.
SOLUTION 6.4. As both resistor divider ration are the same (3/6), the voltage at A and B is the same
resulting in a V
OC
of 0 V.
R
TH
(3K | | 6K) + (9K ||18K) 8 k
The relation V
OC
/I
SC
cannot be used in this situation.
SOLUTION 6.5. Using superposition
V
OC
20
20 10
3
20 10
3
+ 5 10
3



_
,

10
5 10
3
20 10
3
+ 5 10
3



_
,

+ 20 20sin(50t) 14 + 400sin(50t) V,
and
Thevenin Probs, 7/24/01 - P6.3 - @R.A. Decarlo & P. M. Lin
R
TH
(20k | | 5k) + 20k 24 k.
I
SC
can then be found using the V
OC
/R
TH
relationship as 0.58+20sin(50t) mA.
SOLUTION 6.6. Once again using superposition
V K mA K
K K K
K
mA
K K K
K
V V
OC
204 2 54 6
5 8 4
4
54
4 8 6
6
72 +

,
_

+ +
+

,
_

+ +
,
and
+ + + K K K K K K R
TH
10 2 4 ] 6 || ) 8 4 [(
SOLUTION 6.7. Using source transformation, (a) is obtained from the original circuit. Then combining
in series the resistors and voltage sources, and retransforming them (b) is obtained. Finally adding the two
currents and transforming back the circuit to its Thevenin form (c) is obtained.
From (c),
W R I P
mA k k I
k R
V V
L L
L
TH
oc
15 . 0
5 ) 6 6 /( 60
6
60
2

+

Thevenin Probs, 7/24/01 - P6.4 - @R.A. Decarlo & P. M. Lin


SOLUTION 6.8. First, each source in series with 2R, can be replaced by an up going current source of
R V
x
2 / in parallel with 2R. Then starting from the left, the two 2R in parallel are combined and then
retransformed to a voltage source of 2 /
o
V in series with 2R once added to the series resistance. Repeating
the previous steps,
R R V V V V R R V R V R V R V
R V R R R V R V R V R V V V
R R V V V R R V R V R V
R V R R R V R V R V V
R R V V R R V R V
R V R R R V R V
o o
o o
o o
o o
o o
o o
+ + + + + ? + + +
+ + ? + + +
+ + + + ? + +
+ ? + +
+ + + ? +
? +
2 / 4 / 8 / 16 / || 2 / 4 / 8 / 16 /
2 / || 2 || 2 || 4 / 8 / 16 / 2 2 / 4 / 8 /
2 / 4 / 8 / || 2 / 4 / 8 /
2 / || 2 || 2 || 4 / 8 / 2 2 / 4 /
2 / 4 / || 2 / 4 /
2 / || 2 || 2 || 4 / 2 2 /
3 2 1 3 2 1
3 2 1 2 1
2 1 2 1
2 1 1
1 1
1
Thus 2 / 4 / 8 / 16 /
3 2 1
V V V V V
o oc
+ + + and R R
TH
2 .
SOLUTION 6.9. First we find R
TH
R
TH
2K +[ 6K | | 3K ( ) + 15K ||10K ( )]| | 24 K 8K
Use nodal analysis to solve for V
OC
. At node a
1mA 1/ 6K(V
a
V
c
) +1/ 3K(V
a
V
d
) + 1/ 24K(V
a
) or
(1/ 6K +1/ 3K +1/ 24K)V
a
1/ 6K(V
c
) 1/ 3K(V
d
) 1m
doing the same for the supernode, and the equation inside the supernode yields the following two
equations:
(1/ 6K +1/ 3K)V
a
(1/ 6K +1/15 K)V
c
(1/ 3K +1/10 K)V
d
0, and
V
c
V
d
30
using these three equations, it is now possible to solve for the three unknowns V
A
, V
C
, and V
D
. Using
MATLAB they are respectively 4.5V, 22.8750V, and -7.1250V. V
OC
being V
A
, 4.5V.
a
c d
SOLUTION 6.10.
R
TH
[(18K | | 9K) + 66K]| | 36K 24K
Thevenin Probs, 7/24/01 - P6.5 - @R.A. Decarlo & P. M. Lin
By superposition, noting that all resistances are in k-ohms,
V
OC

66+(9| | 18)
66+(9| | 18)+36
36 2.5 +
(66 + 36)||9
[(66 + 36)||9] +18

36
36 + 66
18
36
36 + (9||18) + 66
30 52 V
When a 2 k is connected, the current I
L
becomes V
OC
/(R
TH
+2k) = 2mA; thus the power absorbed is 8
mW.
SOLUTION 6.11. (a) Introduce a test voltage source at the output, and write the nodal equations in matrix
form:
(1/100 +1/ 50) 1/ 50 1/100
1/ 100 1/100 (1/ 100 +1/ 200 +1/100)
1/ 50 (1/ 50 +1/100 +1/100) 1/100





1
]
1
1
1
V
test
V
C
V
D





1
]
1
1
1

i
test
0
V
s
/ 100





1
]
1
1
1
Solving we obtain, V
test
100i
test
+ 2V
s
/ 3. From eq. 6.10R
TH
100 , v
oc
2V
s
/ 3.

C D
(b) To obtained the power the following equation is used, P
v
oc
R
TH
+ R
L



_
,

2
R
L
.
Thevenin Probs, 7/24/01 - P6.6 - @R.A. Decarlo & P. M. Lin
0 20 40 60 80 100 120 140 160 180 200
0.3
0.4
0.5
0.6
0.7
0.8
0.9
1
1.1
1.2
Load resistance in Ohms
P
o
w
e
r
/
V
s

i
n

m
W
SOLUTION 6.12.
V
OC
= 0, as no independent sources are present. Writing the following nodal equation where v
x
is the
voltage across both ports,
i
x
((v
x
i
z
) i
x
) + (v
x
i
z
)/ 2, R
TH
can be found as v
x
/ i
x

2.5 +
1.5
,
SOLUTION 6.13.
Defining v
i
and i
i
as the voltage across and current into the input ports, writing the nodal equation at the
input: i
i
+ g
m
v
x
1/ 200K(v
i
v
x
) . We can also get the following equation v
x
200K(g
m
v
x
+ i
i
) . Using
the previous two equations we can solve for R
TH
v
i
/ i
i
200K 1+
200K g
m
1 + 200K g
m
+
1
1 + 200K g
m



_
,

.
This yields a g
m
of 10 S.
SOLUTION 6.14.
I
SC
is null as no independent source are present. To find R
TH
v
i
and i
i
are defined as the voltage across and
current into the input ports. Writing the nodal equation we get:
i
i
V
x
/ 1.8K + (1/ 200)(V
x
3 / 4V
x
) , and v
i
V
x
300 i
i
. Solving R
TH
v
i
/ i
i
600.
SOLUTION 6.15.
First, write out the equation around loop 1:
Thevenin Probs, 7/24/01 - P6.7 - @R.A. Decarlo & P. M. Lin
V
x
i
1
1200 i
2
100 i
3
300 . Then substituting the following relationships,
i
1
I
x
, i
2
2I
x
, and i
3
0.01V
x
+ i
2
0.01V
x
+ 2I
x
, and solving for V
x
/ I
x
R
TH
100. V
OC
0 as no
independent sources are present.

1
2
3
SOLUTION 6.16.
Introduce a test source at the output terminals, and write out the nodal equations in matrix for the top node,
and the supernode comprised of the current controlled voltage source (ccvs). v
b
is the node left of ccvs, and
v
c
the node to the right.
1/ 30 +1/ 20 1/ 20 1/ 30
1/ 20 1/ 30 + 0.4 1/ 20 0.4 2 / 30
1/ 6 1 5/ 6





1
]
1
1
1
v
test
v
b
v
c





1
]
1
1
1

i
test
0
0





1
]
1
1
1
Solving, we obtain v
test
90i
test
, thus R
TH
90 .

b c
SOLUTION 6.17.
(a) Turn off independent source. Introduce a test source and write loop equation:
v
x
6i
x
4i
1
+10i
x
. Note that i
1
i
x
. Now solve for v
x
/ i
x
R
TH
12.
Thevenin Probs, 7/24/01 - P6.8 - @R.A. Decarlo & P. M. Lin
Short the input and write the loop and nodal equations:
3A i
1
+ I
SC
10i
1
4i
1
+ 6I
SC
Solving yields I
SC
1.5A, and V
OC
I
SC
R
TH
18V .
(b) In MATLAB the following plot is generated: P
V
OC
R
TH
+ R
L



_
,

2
R
L
for 1 R
L
24.
0 5 10 15 20 25
1
2
3
4
5
6
7
Resistance in Ohms
P
o
w
e
r

i
n

W
a
t
t
s
Maximum power is absorbed by 12 load.
SOLUTION 6.18.
To find thevenin resistance, introduce a test source and write the following equations:
v
s
20i
1
+ 40i
1
+ 40i
1
100i
1
, and i
s
v
s
/100 + i
1
. Solving for v
s
/ i
s
R
TH
50.
Next, use the following nodal equation; 0.2A i
1
+ I
SC
, and loop equation 20i
1
+ 40i
1
40I
SC
. Solving
using these two equations yields I
SC
0.12A, and consequently V
OC
R
TH
I
SC
6V .
SOLUTION 6.19.
(a) Introduce a test source, v
s
, and get the following two equations: v
s
15i
s
V
x
, and
kV
x
+ V
x
/ 3 + V
x
/ 5 + i
s
0. Solving yields v
s
/ i
s
R
TH
15 +
1
k +8 /15



_
,
, or 95/6 for k = 2/3.
Next, write the following nodal equation kV
x
+ V
x
/ 3 + (V
x
1) / 5 0 and observe that V
OC
1V V
x
.
Thus solving yields V
OC
1
1/ 5
k +
8
15





_
,



5/ 6V for k = 2/3, and I
SC
V
OC
/ R
TH
1/19 A.
Thevenin Probs, 7/24/01 - P6.9 - @R.A. Decarlo & P. M. Lin
(b) Solving the previously obtained equation for k when V
OC
0, yields k = -1/3, and consequently
R
TH
20 .
SOLUTION 6.20.
Introduce a test source, v
t
, and get the following equations: i
t
+ i
x
i
x

V
s
300i
x
300
V
s
300i
t
+ 300i
x
,
and v
t
V
s
300i
x
+100i
t
. Solving yields v
t
/ i
t
R
TH
400.
Observe how no current flows through the 300 resistor in parallel with the dependent source. Thus V
OC
will always be 0 V and is independent of V
S
.
SOLUTION 6.21. (a) For this part, consider the modified circuit below.

Step 1: Applying KCL to node A, we have
1
400
V
A
100 ( ) +
V
A
800
+ 0.1 I
s
0
Multiplying through by 800 yields
3V
A
800I
s
+120
Step 2. Computing V
s
, we have
V
s
V
A
2000i
x
V
A

2000
400
100 V
A
( ) 6V
A
500
Hence
V
s
6
800
3
I
s
+ 40


_
,
500 1600I
s
260 R
th
I
s
+ V
oc
Therefore R
th
1.6 k and V
oc
260 V.
(b) By linearity, V
oc
130 V.
Thevenin Probs, 7/24/01 - P6.10 - @R.A. Decarlo & P. M. Lin
SOLUTION 6.22.
Introduce a test source, v
t
,, and get the following two equations: v
t
(30m V
1
/ 100)400 V
1
12 5V
1
,
and i
t
0.06V
1
+ 30m V
1
/ 100 30m+ 0.05V
1
. Solving obtain v
t
100i
t
+15. Thus R
TH
100,
and I
SC
V
OC
/ R
TH
150mA.
SOLUTION 6.23. Insert I
test
as per text. Hence
1
40
+
1
120


_
,
V
C

16
40
I
test
Solving, we obtain
V
C
30I
test
+12
By KVL,
V
test
V
C
+ 30 2i
1
+ I
test
( ) V
C
+
60
40
16 V
C
( ) + 30I
test
0.5V
C
+ 30I
test
+ 24
Substituting for V
C
yields
V
test
15I
test
+18
Hence, V
oc
18 V and R
th
15 . Thus i
18
75 + 15
0.2 A. Further, P
75
75(0.2)
2
3 W.

C
SOLUTION 6.24.
(a) and (b)
Thevenin Probs, 7/24/01 - P6.11 - @R.A. Decarlo & P. M. Lin

v
s
-
+
-

+
v

2
1
(c) Writing KVL around loop 1, loop 2, and finally relating v to v
x
,
v
s
10(0.02 I
1
) + v 20I
1
v 5I
2
15(I
2
0.02)
3v
x
I
1
I
2
3[10(0.02 I
1
)] 0.6 30I
1
(d) Rewriting these in matrix form,
30 0 1
0 20 1
31 1 0





1
]
1
1
1
I
1
I
2
v





1
]
1
1
1

1
0
0





1
]
1
1
1
v
s
+
0.2
0.3
0.6





1
]
1
1
1
(e), (f), and (g). Using MATLAB,
A=[-30 0 1;0 20 1; 31 -1 0];
b1 = [1 0 0]';
b2 = [-.2 .3 .6]';
I1 = inv(A)*b1
I1 =
-1.5385e-03
-4.7692e-02
9.5385e-01
I2 = inv(A)*b2
I2 =
1.9231e-02
-3.8462e-03
3.7692e-01
Rth=-1/I1(1)
Rth = 650
Note: Isc = I2(1) = 0.01923 A
Voc= -Rth*I2(1)
Thevenin Probs, 7/24/01 - P6.12 - @R.A. Decarlo & P. M. Lin
Voc =
1.2500e+01
(h)
SOLUTION 6.25.
(a)
(b) Write four nodal equations,
i
s
(V
A
V
C
) / 2k
i
s
+ 1m (V
C
V
D
)/ 6k + (V
C
V
E
)/ 3k
V
E
/ 2k (V
C
V
D
) / 6k V
D
/15k
V
E
/ 2k V
E
/10k + (V
E
V
C
)/ 3k
(c)
1/ 2k 1/ 2k 0 0
0 1/ 2k 1/ 6k 1/ 3k
0 1/ 6k (1/ 6k + 1/ 15k) 1/ 2k
0 1/ 3k 0 1/ 15k






1
]
1
1
1
1
V
A
V
C
V
D
V
E







1
]
1
1
1
1
1

1
1
0
0






1
]
1
1
1
1
i
s
+
0
1m
0
0






1
]
1
1
1
1
(d) Solving in MATLAB V
A
5.818ki
s
+ 3.8182 , thus
R
TH
5.818k
V
OC
3.8182V
(e) This only changes V
OC
3818 8m 30.54V .
SOLUTION 6.26. For this circuit, no current flows though the 20 resistor. Therefore V
AB
V
CB
.
Further, from the examples in the chapter, V
CB
4V
s
. Hence, V
oc
V
AB
V
CB
4V
s
. Also, shorting
terminals A and B, yields I
sc

V
CB
20

4V
s
20

V
s
5
. It follows that R
th

V
oc
I
sc
20 . Note that the
Thevenin equivalent to the left of C-B is a voltage source of value 4V
s
. Therefore the Thevenin equivalent
to the left of A-B is 20 in series with 4V
s
.
SOLUTION 6.27. (a) From the previous problem V
CB
4V
s
. Thus by voltage division,
V
OC
V
AB
V
CB
180
180 + 20


_
,
3.6V
s
. Next find I
SC
V
CB
/ 20 V
s
/ 5, and then R
TH
V
OC
/ I
SC
18.
Thevenin Probs, 7/24/01 - P6.13 - @R.A. Decarlo & P. M. Lin
(b) This changes the voltage division at the output, thus V
AB
/ V
s
4
180||162
(180||162) + 20



_
,
3.24 .
SOLUTION 6.28. (a) Writing the following two KCL equations,
v
test
/ 4k (V
o
v
test
) /12k
I
test
(v
test
V
o
) /15k
where V
o
is the voltage at the output of the op-amp. Doing the appropriate substitution get
v
test
15k / 3 I
test
, thus
R
TH
5k
V
OC
0V
Since no independent source exist right of A-B
(b) Applying Ohms law I
s
1/ (10k 5k )V
s
0.2mV
s
SOLUTION 6.29. (a) Adding a test source at terminal A-B, and noting that the voltage at the output of the
op-amp is V
o
5 / 2V
s
. Write KCL at terminal A,
v
test
/ 900 (V
o
v
test
)/ 100 + i
test
v
test
90i
test
2.25V
s
Where one sees by inspection that
R
TH
90
V
OC
2.25V
s
I
SC
V
OC
/ R
TH
0.025V
s
(b) Noticing the virtual short to ground provided by the ideal op-amp, R
TH
20k, and V
OC
0V since no
independent sources are present right of the input terminal.
SOLUTION 6.30. The output voltage of the ideal op amp is 2.5V
s1
2V
s2
which drives a voltage divider
circuit. Hence
V
oc
V
AB
0.9(2.5V
s1
2V
s2
) 2.25V
s1
1.8V
s2
Further,
I
sc

2.5V
s1
2V
s2
100
0.0225V
s1
0.018V
s2
Finally,
R
th

V
oc
I
sc

0.9(2.5V
s1
2V
s2
)
(2.5V
s1
2V
s2
)
100
90
Thevenin Probs, 7/24/01 - P6.14 - @R.A. Decarlo & P. M. Lin
Equivalently if one sets V
s1
and V
s2
to zero, then the ouput terminal of the op amp goes to ground. Hence
R
th
100 / /900 90
SOLUTION 6.31. Define the node at the output of the op-amp as V
o
. Note how the circuit left of this
node is a general summing circuit as per text. Thus, V
o
2V
s2
4V
s1
. Hence we simply replace the op
amp circuit to the left of the 20 resistor by an ideal voltage source of value V
0
. Hence
V
oc

80
80 + 20
V
0
+ 2I 0.8V
0
+ 2
V
0
100


_
,
0.78V
0
1.56V
s2
3.12V
s1
Alternately, one can introduce a test source at the output terminal and write out a set of equations using
KVL,
V
o
2V
s2
4V
s1
20I 80(I i
test
)
v
test
2I + 80(i
test
I)
Solving yields v
test
17.6i
test
+ 0.78(2V
s2
4V
s1
) , and by inspection,
R
TH
17.6
V
OC
0.78(2V
s2
4V
s1
) 56V
s2
3.12V
s1
SOLUTION 6.32. Upon inspection when the op-amp is in active range, for inputs from 3V to 3V,
R
TH
1k
V
OC
0V
However when the input exceeds 3V, the output of the op-amp will saturate at 15V, and
V
s
15V + 6kI
s
. Therefore from eq. 6.10,
R
TH
6k
V
OC
15V
When the input is less than 3V, V
s
15 + 6kI
s
, thus
R
TH
6k
V
OC
15V
SOLUTION 6.33. (a) The op-amp configuration has a gain of 4. So when the input is between 3V and
3V it is operating in its active region, thus
Thevenin Probs, 7/24/01 - P6.15 - @R.A. Decarlo & P. M. Lin
R
TH
4k
V
OC
0V
When the input is greater than 3V the output saturates at 12V and
V
s
20kI
s
12
R
TH
20k
V
OC
12V
When the input is less than 3V the output saturates at 12V,
V
s
20kI
s
+12
R
TH
20k
V
OC
12V
(b) When the input is in the active range,
R
TH
16k
V
OC
4V
s
When it is greater than 3V,
R
TH
0k
V
OC
12V
And when less than 3V,
R
TH
0k
V
OC
12V
The last two obtained using figure 6.28.
The maximum power is when the output is in saturation, P V
OC
( )
2
/ 28k 6mW.
SOLUTION 6.34. From the table the following two equations can be written:
6 2R
TH
+ v
oc
12 8R
TH
+ v
oc
Putting in matrix form and solving,
1 1
8 1



1
]
1
R
TH
v
oc



1
]
1

6
12



1
]
1
1/ 6 1/ 6
4 / 3 1/ 3



1
]
1
6
12



1
]
1

R
TH
v
oc



1
]
1
Thus R
TH
1 k since current was in mA, and v
oc
4 V.
SOLUTION 6.35. (a) From Ohms law, I
AB
V
AB
/ R
L
. Thus 0.2uA and 0.1uA.
(b) Note how using this topology V
AB
v
oc
I
AB
R
TH
, thus
Thevenin Probs, 7/24/01 - P6.16 - @R.A. Decarlo & P. M. Lin
1 0.2u
1 0.1u



1
]
1
v
oc
R
TH



1
]
1

0.4
1



1
]
1
1 2
10M 10M



1
]
1
0.4
1



1
]
1

v
oc
R
TH



1
]
1
Thus, v
oc
1.6V , and R
TH
6M.
SOLUTION 6.36. (a) Although the text describes finding R
th
from a measurement or calculation of both
V
oc
and I
sc
, measurement of I
sc
is often impractical. Hence the procedure outlined in this problem
provides a more practical means of determining the Thevenin equivalent.
Since the internal meter resistance is 10 M,
I
AB
(A)
V
AB
10
+
V
AB
R
L
Hence, the completed table is:
R
L
(M )
v
AB
(V)
I
AB
(A)
2 0.4 0.24
10 1 0.2
(b) The terminal relationship assuming a Thevenin equivalent is given by
V
AB
V
oc
R
th
I
AB
In matrix form with the data from the table
1 0.24
1 0.2



1
]
1
V
oc
R
th



1
]
1

0.4
1



1
]
1
Hence V
oc
= 4 V and R
th
= 15 M. Note that since we used A and V, R
th
is in M.
SOLUTION 6.37. (a) For this scenario, the circuit is essentially a voltage source with a resistance R in
series with the circuit under test, in parallel with a voltmeter measuring the voltage division between the
Thevenin Probs, 7/24/01 - P6.17 - @R.A. Decarlo & P. M. Lin
later two. Therefore if replacing R = 0 by R = R
2
causes the voltage measured by the voltage meter to
drop by half, then by voltage division, E
o
/ 2 E
o
R
TH
R
2
+ R
TH



_
,

, and R
TH
R
2
.
(b) Using the same reasoning and voltage division,
E
o
V
s
R
TH
R
TH
+ R
s
E
o
/ 2 V
s
R
TH
R
TH
+ R
s
+ R
2
Therefore R
TH
+ R
s
R
2
or equivalently R
TH
R
2
R
s
.
(c) Again by voltage division,
E
o
V
s
R
TH
|| R
m
(R
TH
|| R
m
) + R
s
E
o
/ 2 V
s
R
TH
|| R
m
(R
TH
|| R
m
) + R
s
+ R
2
Therefore (R
TH
|| R
m
) + R
s
R
2
or
R
TH
R
m
R
TH
+ R
m
R
2
R
s
. Solving for R
TH

R
2
R
m
R
s
R
m
R
m
+ R
s
R
2
.
SOLUTION TO 6.38. (a) The voltmeter measures the voltage division between the two resistors involved
thus,
E
o
V
oc
E
o
/ 2 V
oc
R
2
R
TH
+ R
2
and R
TH
R
2
.
(b) Now
E
o
V
oc
R
m
R
m
+ R
TH
E
o
/ 2 V
oc
R
m
|| R
2
(R
m
|| R
2
) + R
TH
From the division of the former by the later R
2
R
TH
|| R
m
. And from the former
V
oc
(R
m
+ R
TH
)E
o
/ R
m
(1+ R
TH
/ R
m
)E
o
.
SOLUTION TO 6.39. Using the relation developed in problem 6.37 R
TH

R
2
R
m
R
s
R
m
R
m
+ R
s
R
2
5k
SOLUTION TO 6.40. Using the equation developed in problem 6.38,
Thevenin Probs, 7/24/01 - P6.18 - @R.A. Decarlo & P. M. Lin
R
TH

R
2
R
m
R
m
R
2
4M
V
oc
(1+ R
TH
/ R
m
)E
o
20V
SOLUTION TO 6.41. Writing the line equation in the following general form,
i v
ab
/ R
TH
i
sc
i 2v
ab
4
Thus R
TH
0.5, and i
sc
4A.
SOLUTION TO 6.42. (a) In this range the curve appears to be varying linearly between (0V,0mA) and
(0.2V, 0.1mA) pair, thus writing the line equation i (0.1m/ 0.2)v yields R
TH
2k V
oc
0V .
(b) Writing the line equation of the following form, i v / R
TH
i
sc
, into a matrix equation,
0.2 1
0.7 1



1
]
1
1/ R
TH
i
sc



1
]
1

0.1m
10.1m



1
]
1
and solving gives
R
TH
50
i
sc
3.9mA
v
oc
0.195V
(c) First make a guess as to which region of the curve, N will operate in. Assuming that it will operate in
the A-B region, then by KVL i(t) (v
s
(t) + v
b
v
oc
)/ (R + R
TH
) (50sin(1000t) 0.095) / 550. It can be
seen that this guess is wrong as the range of i(t) is not in the appropriate region. Assuming the 0-A region,
by KVL i(t) (v
s
(t) + v
b
v
oc
)/ (R + R
TH
) (50sin(1000t)m+ 0.1)/ 2500 0.02sin(1000t) + 0.04mA.
Note that the highest current is 0.06mA, thus still in the appropriate region of operation.
(d) Repeating the procedure above and guessing the region A-B, by KVL
i(t) (v
s
(t) + v
b
v
oc
)/ (R + R
TH
) (200sin(1000t)m+ 305m) /100 2sin(1000t) + 3.05mA. The
maximum and minimum current are 5.05 and 1.05 mA respectively, thus the assumption made was correct.
SOLUTION TO 6.43. (a) By the power transfer theorem R
L
R
TH
. For circuit (a) R
L
80 ||240 60,
and circuit (b) R
L
(900||180) + 50 200.
(b) Finding i
sc
for each circuit: (a) i
sc
36 / (80 + 240||60) 240 / 300 225mA and (b)
i
sc
60mA
900 ||180
(900||180) + 50 + 200



_
,
22.5mA. The power is now obtained from P i
sc
/ 2 ( )
2
R
L
. Thus
759mW for (a) and 25.3mW for (b).
Thevenin Probs, 7/24/01 - P6.19 - @R.A. Decarlo & P. M. Lin
SOLUTION TO 6.44. Finding the Thevenin equivalent,
R
TH
30||15 ( ) +10
[ ]
||80 16
V
oc
32V
The value for the load resistance is 16, and the power is P
V
oc
2


_
,
2
/ R
L
16W.
SOLUTION TO 6.45. (a) Find the Thevenin equivalent,
R
TH
12k | | 6k + 8k 12k R
L
V
oc
8k(2mA) + 24
6
6 +12


_
,
24V
The power is P V
oc
/ 2 ( )
2
/ R
L
12mW.
(b) The maximum power will be transferred to the load, when the value of its load is closest to 12k. Thus
the power is P
V
oc
R
TH
+10k



_
,

2
10k 11.9mW.
(c) Same reasoning as (b) the power is P
V
oc
R
TH
+15k



_
,

2
15k 11.9mW.
SOLUTION TO 6.46. (a) The Thevenin equivalent to the left of R
L
has R
th
12 + 20 / /180 30 and
V
oc
1 18 +
180
180 + 20
40 54 V. Therefore, for maximum power transfer
G
L
+ 2G
L
+ 3G
L
6G
L

6
R
L

1
30
Hence R
L
180 .
(b) For this part, let V
L
denote the voltage (top to bottom) across the load. With R
L
180 , then the
parallel combination equals R
th
and hence V
L
27 V. It follows that
P
180

(27)
2
180
4.05 watt.
Since the terminal voltages are the same, the absorbed power is inversely proportional to the resistance.
Hence P
90
8.1 watt, and P
60
12.15 watt.
Thevenin Probs, 7/24/01 - P6.20 - @R.A. Decarlo & P. M. Lin
SOLUTION 6.47. To find the Thevenin equivalent introduce a test voltage source at the output, and write
KCL at the two node in the circuit. By inspection the following matrix expression is obtained:
(1/ 200 +1/ 400) 1/ 400
1/ 400 0.0015 +1/ 1k +1/ 400



1
]
1
V
A
V
test



1
]
1

0.1
I
test



1
]
1
Solving V
test
240I
test
+ 8. And R
TH
240 , V
oc
8 V. By voltage division, the voltage across the load
resistor is 4V, and the power delivered to it is 66.7mW.

VA
V
test
SOLUTION 6.48. Using KCL get i
x
1mA, which independent of what is connected to the output. Thus
i
sc
10i
x
10mA
R
TH
3k
The power is then P 10mA / 2 ( )
2
3k 75mW.
SOLUTION 6.49. Performing a source transformation on (a), and combining the elements will simplify to
one current source going up of 2/3A in parallel with a 10 resistor. This is essentially the Norton
equivalent of the circuit,
i
sc
2/ 3A
R
TH
10
For (b), combine the voltage sources and resistor in series. The circuit obtained is one voltage source of
5V in series with a 45 resistor. This is the Thevenin equivalent,
v
oc
5V
R
TH
45
(a) The value of the load resistor is simply the thevenin resistance obtained above.
(b) Using Ohms law for (a) V
L
i
sc
(10 ||10) 10/ 3V , and voltage division for (b) V
L
v
oc
(1/ 2) 2.5V
(c) Using the following formula, P V
L
2
/ R
L
, (a) absorbs 1.1W and (b) 139mW, thus (a) absorbs more
power.
Thevenin Probs, 7/24/01 - P6.21 - @R.A. Decarlo & P. M. Lin
SOLUTION 6.50. (a) Note that the circuit left of the terminal is already in its Thevenin form. The load
R
L
R | | ( R+ 300)
R
2
+ R(300 2R
L
) 300R
L
0
Solving, R 71.6. By voltage division, the voltage across the load is 5V. The power absorbed is
P V
oc
/ 2 ( )
2
/ R
L
416.7mW.
(b) The following script can be used to plot the power absorbed by the load versus R:
%Script for problem 6.50b
R=0:2:400;
%Calculate Load resistance
RL= 1./((1./R)+1./(R+300));
%Calculate the power
P=(10.*(RL./(RL+60))).^2./RL;
%Plot the power versus R
plot(R,P);
ylabel('Power in Watts');
xlabel('Resistance in Ohms')
SOLUTION 6.51. First, find the Thevenin equivalent by writing out the transfer equation v
ab
200i + 40.
Thus R
TH
200 V
oc
40V . The maximum power will then be P (V
oc
/ 2)
2
/ 200 2W.
SOLUTION 6.52. The assumption that all controlling voltages or currents for dependent sources within
N
i
are assumed to be in N
i
, implies that the nodal equation matrix of figure P6.52a has the partitioned
form:
G
11
G
12
0
G
21
G
22
G
23
0 G
32
G
33





1
]
1
1
1
V
N1
V
m
V
N2





1
]
1
1
1

I
N1
I
m
I
N2





1
]
1
1
1
(*)
where V
N1
is the vector of UNKNOWN and INDEPENDENT node voltages internal to N
1
and V
N2
is
the vector of UNKNOWN and INDEPENDENT node voltages internal to N
2
. The right side of the
Thevenin Probs, 7/24/01 - P6.22 - @R.A. Decarlo & P. M. Lin
equation consists of (effective) currents injected into the appropriate node. However, I
N1
depends only on
sources in N
1
and I
N2
depends only on sources in N
2
.
At this point we must presume that the matrix equation (*) has a unique solution, i.e., the determinant
of the coefficient matrix is non-zero. Hence we can calculate V
N1
, V
m
, and V
N2
uniquely. As such, by
considering the first row of (*), we can assert that V
N1
satisfies
G
11
V
N1
I
N1
G
12
V
m
(**)
Note that we are not claiming that we can solve for V
N1
from this equation.
Replacing N
2
by a voltage source of value V
m
results in the network of figure P6.52b. For this
network, the nodal equations are
G
11
V
N1
I
N1
G
12
V
m
(***)
where G
ij
is the same as in (*). Clearly, this is the same as equation (**). Again we presume there is a
unique solution to this equation, i.e., the determinant of G
11
is non-zero. If so, we can solve for V
N1
uniquely and the result is the same as that obtained by solving (*).
This theorem can be extended to RLCM networks (to be studied in later chapters) or even
nonlinear networks under appropriate conditions.
To emphasize the subtlety of this result and the need for unique solvability in each network,
consider the following circuit.
The resulting nodal equations are:
Thevenin Probs, 7/24/01 - P6.23 - @R.A. Decarlo & P. M. Lin
2 1 0 0
1 0.5 0.5 0
0 0.5 2.5 1
0 0 1 2






1
]
1
1
1
1
V
1
V
2
V
3
V
4







1
]
1
1
1
1
1

2
0
0
4






1
]
1
1
1
1
There exists a unique solution and from MATLAB, we find
V
m
= V
3
= 2 V
To apply the voltage source substitution, we replace N
2
by a voltage source of 2 V and obtain the
following circuit.
The nodal equations here are
2 1
1 0.5



1
]
1
V
1
V
2



1
]
1

2
1



1
]
1
Observe that the coefficient matrix has a zero determinant. Thus there is either no solution or many
solutions, i.e., no unique solution. This demonstrates that unique solvability of the larger network does not
imply the unique solvability of the smaller derived network.
SOLUTION 6.53. The assumption that all controlling voltages or currents for dependent sources within
N
i
are assumed to be in N
i
, implies that the loop equation matrix of figure P6.53a has the partitioned form:
R
11
R
12
0
R
21
R
22
R
23
0 R
32
R
33





1
]
1
1
1
I
N1
I
m
I
N2





1
]
1
1
1

E
N1
E
m
E
N2





1
]
1
1
1
(*)
Thevenin Probs, 7/24/01 - P6.24 - @R.A. Decarlo & P. M. Lin
where I
N1
is the vector of unknown and independent loop currents internal to N
1
and I
N2
is the vector of
unknown and independent loop currents internal to N
2
and I
m
a single independent loop current common
to N
1
and N
2
. The right side of the equation represents the net contribution of voltage sources present in
the appropriate loop. However, E
N1
depends only on N
1
and E
N2
depends only on N
2
.
At this point we must presume that the matrix equation (*) has a unique solution, i.e., the determinant
of the coefficient matrix is non-zero. Hence we can calculate I
N1
, I
m
, and I
N2
uniquely. As such, by
considering the first row of (*), we can assert that I
N1
satisfies
R
11
I
N1
E
N1
R
12
I
m
(**)
Note that we are not claiming that we can solve for I
N1
from this equation.
Replacing N
2
by a current source of value I
m
results in the network of figure P6.53b. For this
network, the loop equations are
R
11
I
N1
E
N1
R
12
I
m
(***)
where R
ij
is the same as in (*). Clearly, this is the same as equation (**). Again we presume there is a
unique solution to this equation, i.e., the determinant of R
11
is non-zero. If so, we can solve for I
N1
uniquely and the result is the same as that obtained by solving (*). For some subtlety in the proof refer to
the solution of 6.52.
This theorem can be extended to RLCM networks (to be studied in later chapters) or even
nonlinear networks under appropriate conditions.
SOLUTION 6.54. (a) The thevenin equivalent to the left of terminal A-B is
R
TH
[(30||60) + 20]||10 8, and using Ohms law along with voltage division
V
OC
15 10
15V
15 + 60


_
,
30
30 + 30


_
,
14V
(b) Doing the same for the circuit right of terminal A-B. R
TH
[(30||15) + 10]||20 10 and
V
OC
7.5 20
7.5V
15 +15


_
,
30
30 + 30


_
,
5V .
(c) Using superposition, V
AB
14
10
18


_
,
+ 5
8
18


_
,
10V .
(d) (e) (v
CB
15) / 30 + v
CB
/ 60 (10 v
CB
)/ 20 Hence, v
CB
10V .
Thevenin Probs, 7/24/01 - P6.25 - @R.A. Decarlo & P. M. Lin
SOLUTION TO P6.55. The proof is based on superposition. Let us consider the figure below where N
1
and N
2
are differently named but identical networks.
We first compute the contribution to i
a
from the independent sources in N
1
with those of N
2
deactivated.
Let this current be i
a
1
. The contribution to i
a
from the independent sources in N
2
with those of N
1
deactivated is i
a
2
. But because N
1
and N
2
are identical, i
a
1
i
a
2
. Hence by superposition
i
a
i
a
1
+ i
a
2
0. By the current source substitution theorem we can replace the lines by current sources of
value 0-amp. This defines an open circuit and the connecting line can be replaced by an open circuit.
From the given network we also note by KVL that V
x
+ V
y
= 0 which implies that V
x
= V
y
. On the other
hand, since the networks are identical, V
x
= V
y
. Thus we conclude that V
x
= V
y
= 0. Thus we can replace
V
x
and V
y
by a voltage source of 0-volt (voltage source substitution theorem) which is the definition of a
short circuit.
SOLUTION 6.56. Label the potential between each line starting from the top as V
x1
, V
x2
on the left, and V
y1
and V
y2
on the right. Now by superposition and linearity notice that
V
x1
V
y1
V
x2
V
y2
because the independent source is negative on the right side. Additionally, from KVL,
V
x1
V
y1
V
x2
V
y2
The only way all these condition can be met, is if all the voltages are 0 V, or short circuited.
Thevenin Probs, 7/24/01 - P6.26 - @R.A. Decarlo & P. M. Lin
SOLUTION 6.57. (a) Using the results from P6.55, no current flows between the two halves. So the right
hand side circuit may be analyzed as if it was stand alone. By voltage division then,
v
a

3 + 2
3 + 2 +1


_
,
18 15V .
(b) From the results of P6.56, all the lines crossing the symmetry line are shorted together. Consequently,
by voltage division, v
a

6 | | 3
(6 ||3) +1



_
,
(18) 12V .
SOLUTION 6.58. Note how this circuit is the same as in P6.57: it is just redrawn with the neighboring
resistors added in parallel or in series. Using superposition, we can solve for v
a
when the sources [v
s1
v
s2
]
are [18 18], and then [18 18]. By linearity adding the two contribution will be equivalent to solving for
[36 0] directly, since adding the source contributions [18 18]+[18 18] =[36 0]. The contributions of
15V12V were obtained in P5.57; thus v
a
3V .
SOLUTION 6.59. Yes since
45 27 [ ] 36 36 [ ] + 9 9 [ ] 2 18 18 [ ] +1/ 2 18 18 [ ] 2(15) + (12) / 2 24V .
SOLUTION 6.60. For this proof we attach an arbitrary network N to each of the networks N
1
and N
2
in
figure P6.60 as shown below.
N may have internal independent sources, but we consider N
1
and N
2
external excitations to N and
we assume no violation of KVL in the attachment. Choose node 3 as a reference node. Then
V
13
V
a
V
c
and V
23
V
b
V
c
for both figures. Hence N
1
and N
2
provide the identical external
Thevenin Probs, 7/24/01 - P6.27 - @R.A. Decarlo & P. M. Lin
excitations to N and hence all currents and voltages in N remain unchanged.
The extension of this result to 4 external nodes is shown in the figure below. The verification is
the same as above. The extension of course to n-terminals is clear.
SOLUTION 6.61. Using the E-shift theorem, remove the 9 V source from each branch and add it to the
4V source, and notice by inspection that V
OC
4 + 9 5 V.
SOLUTION 6.62. (a) Writing a KCL equation for each node in N2:
I
1
I
a
I
2
I
a
I
a
0
I
3
I
a
I
a
0
I
4
I
a
Do the same for N1:
I
1
I
a
I
2
0
I
3
0
I
4
I
a
This shows that the two have identical outcomes.
(b)
Thevenin Probs, 7/24/01 - P6.28 - @R.A. Decarlo & P. M. Lin
SOLUTION 6.63. Using the I-shift theorem, this circuit is essentially a 2A source in parallel with a
series combination of resistors, and a 5A source. Thus i
sc
5 2 3A.
SOLUTION 6.64. (a) This can be done by inspection. An equal source is connected between A-C and C-
B; thus by the I-shift theorem, it is equivalent to the same source just connected between A-B.
(b) In figure 6.64c the VCCS is replaced by a resistor using the Ohms law relationship
100 V
1
/ (0.01V
1
).
(c) (d) By voltage division V
1
V
s
100||900
(100 ||900) + 10



_
,
, and by Ohms law
V
out
V
s
100||900
(100 ||900) +10



_
,

0.01 20k | | 5k ( ) 36V
s
V
out
/ V
s
36
L&C Probs, 11/15/01 P:7-1 R. A. DeCarlo, P. M. Lin
PROBLEM SOLUTIONS CHAPTER 7
SOLUTION 7.1. Given the coil has 48 turns and 12 turns/cm, we know that the length of the coil is 4 cm.
Since the length of the coil is greater than 0.4 times its diameter, the formula given in the question can be
used:
L
4 10
5
0.02 ( )
2
48 ( )
2
18 0.02 ( ) + 40 0.04 ( )
18.81 H
SOLUTION 7.2. Part 1. Applying (7.1)
The voltage v
L
(t) can be computed using the inductor v-i relationship:
v
L
(t) L
di
L
(t)
dt
The calculations for v
L
(t) for t = 0s to 5s are summarized in the following table:
Time Interval d/dt (i
in
(t)) v
L
(t)
0s< t 1s 2 As
-1
1V
1s< t 3s -2 As
-1
-1V
3s< t 4s 2 As
-1
1V
4s< t 5s 0 As
-1
0V
Below is the plot of V
L
vs time.
Part 2. Applying (7.4)
w
L
(t)
1
2
(0.5 i
in
2
(t))
In the time interval 0s < t 1s, i
in
(t) 2t . Thus
w
L
(t)
1
2
0.5i
in
2
(t)
( )
t
2
In the time interval 1s < t 3s, i
in
(t) 4 2t . Thus
L&C Probs, 11/15/01 P:7-2 R. A. DeCarlo, P. M. Lin
w
L
(t)
1
2
0.5i
in
2
(t)
( )

1
4
(4 2t)
2
t
2
4t + 4
In a similar way, w
L
(t) can be computed in the remaining intervals.
The calculations for w
L
(t) for t = 0s to 5s are summarized in the following table:
Time Interval i
in
(t) w
L
(t)
0s< t 1s 2t t
2
1s< t 3s 4-2t t
2
-4t+4
3s< t 4s -8+2t t
2
-8t+16
4s< t 5s 0 0
Below is the plot of w
L
vs t.
SOLUTION 7.3. Applying (7.2)
i
L
(t) i
L
(0) +
1
L
v
L
( )d
0
t

It is assumed that i
L
(0) = 0 A. Using the preceding formula and the fact that v
in
(t) = t in the time interval
0s < t 2s, the current is
i
L
(t)
1
0.5
d t
2
0
t

in the time interval 0s < t 2s


In the time interval 2s < t 3s v
in
(t) 2 . Hence
i
L
(t)
1
0.5
v
L
( )d
0
t

i
L
(2) +
1
0.5
2dt
2
t

4 + 4
2
t
4t 4
In a similar way i
L
(t) can be computed in the remaining intervals.
The calculations for i
L
(t) for t = 0s to 6s are summarized in the following table:
L&C Probs, 11/15/01 P:7-3 R. A. DeCarlo, P. M. Lin
Time Interval v
in
(t), V i
L
(t), A
0s< t 2s t t
2
2s< t 3s 2 4t-4
3s< t 5s -2 20-4t
5s< t 6s 0 0
Below is the plot of i
L
vs t.
SOLUTION 7.4. Part 1. Using (7.1), we have
v
in
(t) 0.2 10
3
d
dt
i
s
(t) [ ] 0.2 10
3
1000cos(1000t) 0.2cos(1000t)mV
For the 2mH inductor
i
out
(t) i
out
(0) +
1
L
10v
in
0
t

( )d
1
2 10
3
2cos
0
t

(1000 )d sin(1000t)mA
Below is a sketch of i
out
vs t.
Part 2. Instantaneous power delivered by the dependent source is given by
p
L
(t) v
L
(t) i
L
(t) 10v
in
(t) i
out
(t) 2cos(1000t) sin(1000t) sin(2000t) W
L&C Probs, 11/15/01 P:7-4 R. A. DeCarlo, P. M. Lin
Part 3. The energy stored in the 2mH inductor is given by
W
L
(t)
1
2
Li
L
2
(t)
1
2
Li
out
2
(t) sin
2
(1000t)nJ
Below is a sketch of W
L
vs t
SOLUTION 7.5. Part 1 For the excitation in Figure P7.5b,
i
1
(t) i
1
(0) +
1
0.5
v
in
0
t

( )d , i
2
(t) i
2
(0) +
1
0.25
v
in
0
t

( )d 2i
1
(t)
It is assumed that

i
1
(0) i
2
(0) 0 A. In the interval 0s < t 1s, v
in
(t) 10V. Hence, in this interval
i
1
(t)
1
0.5
v
in
( )d
0
t

1
0.5
(10)d
0
t

20t
i
2
(t)
1
0.25
v
in
( )d
0
t

1
0.25
(10)d
0
t

40t
In the interval 1s < t 2s, v
in
(t) 5V. Hence, in this interval,
i
1
(t) i
1
(1) +
1
0.5
v
in
( )d
1
t

20 +
1
0.5
(5)d
1
t

10t 10
i
2
(t) i
2
(1) +
1
0.25
v
in
( )d
1
t

40 +
1
0.25
(5)d
1
t

20t 20
In a similar fashion i
1
(t) and i
2
(t) can be computed in other intervals.
The calculations for i
1
(t) and i
2
(t) for t = 0s to 5s are summarized in the following table:
L&C Probs, 11/15/01 P:7-5 R. A. DeCarlo, P. M. Lin
Time Interval v
in
(t), V i
1
(t), A i
2
(t), A
0s< t 1s -10 -20t -40t
1s< t 2s -5 -10-10t -20-20t
2s< t 3s 0 -30 -60
3s< t 4s 5 -60+10t -120+20t
4s< t 5s 10 100+20t 200+40t
Below are the plots of i
1
vs t and i
2
vs t.
Part 2 For the excitation in Figure P7.5c,
i
1
(t) i
1
(0) +
1
0.5
v
in
0
t

( )d , i
2
(t) i
2
(0) +
1
0.25
v
in
0
t

( )d 2i
1
(t)
It is assumed that i
1
(0) = i
2
(0) = 0 A. In the interval 0s < t 1s, v
in
(t) 10t . Hence, in this interval
i
1
(t) i
1
(0) +
1
0.5
v
in
( )d
0
t

1
0.5
(10 )d
0
t

10t
2
and
i
2
(t) i
2
(0) +
1
0.25
v
in
( )d
0
t

1
0.25
(10 )d
0
t

20t
2
In the interval 1s < t 3s, v
in
(t) 10t 20 . Hence, in this interval
i
1
(t) i
1
(1) +
1
0.5
v
in
( )d
1
t

10 +
1
0.5
(10 20)d
1
t

10t
2
40t + 40
i
2
(t) i
2
(1) +
1
0.25
v
in
( )d
1
t

20 +
1
0.25
(10 20)d
1
t

20t
2
80t + 80
The calculations for i
1
(t) and i
2
(t) for t = 0s to 3s are summarized in the following table:
L&C Probs, 11/15/01 P:7-6 R. A. DeCarlo, P. M. Lin
Time Interval v
in
(t), V i
1
(t), A i
2
(t), A
0s< t 1s 10t 10t
2
20t
2
1s< t 3s 10t-20 10t
2
-40t+40 20t
2
-80t+80
Below are the plots of i
1
vs t and i
2
vs t.
SOLUTION 7.6. For the circuit in Figure P7.6, the parallel combination of the 0.75mH and 1.5mH
inductors can be replaced by an inductor with the inductance of (0.751.5)mH. The v-i relationship for
this inductor is
i
in
(t) i
in
(0) +
1
[(0.75) (1.5)] 10
3
v
s
( )d
1
0.5 10
3
v
s
( )d
0
t

A
0
t

The series combination of the 0.8mH and 0.6mH can be replaced by an (0.8mH + 0.6mH) inductor. The
v-i relationship for this inductor gives:
v
out
(t) (0.8 + 0.6) 10
3
d
dt
6i
in
(t) [ ]
0.8 + 0.6
0.5
6
d
dt
v
s
0
t

( )d




1
]
1
1
16.8v
s
(t) V
Below is a plot of v
out
(t) vs t.
L&C Probs, 11/15/01 P:7-7 R. A. DeCarlo, P. M. Lin
Assume i
L
(0) =0 for all inductors.
i
out
(t) 6i
in
(t)
6
0.5 10
3
v
s
0
t

( )d 12 10
3
v
s
0
t

( )d A
In the interval 0s < t 1s, v
s
(t) 1mV. Hence,
i
in
(t)
1
0.5 10
3
v
s
( )d
0
t

2t(A), i
out
(t) 6i
in
(t) 12t(V)
The power p(t) on the same interval is computed as:
p(t) 6i
in
(t) v
out
(t) (12t) (16.810
3
) 0.202t(W )
In the interval 1s < t 3s, v
s
(t) 2mV. Hence,
i
in
(t) i
in
(1) +
1
0.5 10
3
v
s
( )d
1
t

2 + 2 2
1
t
4t 2(A)
i
out
(t) 6i
in
(t) 24t 12(A)
and
p(t) 6i
in
(t) v
out
(t) (24t 12) (16.8 2 10
3
) 0.806t 0.403(W )
In a similar fashion, i
out
(t) and p(t) can be computed for the remaining intervals.
The calculations for i
out
(t) and the instantaneous power delivered by the dependent source, p(t), for t = 0s
to 6s are summarized in the following table:
Time Interval v
s
(t), V v
out
(t), V i
out
(t), A p(t), W
0s< t 1s 110
-3
16.810
-3
12t 0.202t
1s< t 3s 210
-3
33.610
-3
-12+24t 0.806t-0.403
3s< t 5s -210
-3
-33.610
-3
132-24t 0.806t-4.435
5s< t 6s 0 0 12 0
L&C Probs, 11/15/01 P:7-8 R. A. DeCarlo, P. M. Lin
SOLUTION 7.7. For 0 t < 2 s, using the inductor v-i relationship, we have
i
in
(t)
1
0.5
v
in
( )d
0
t

2
0.25
cos(0.25 ) [ ]
0
t

1 cos(0.25t) [ ] A
The associated energy stored as a function of t for this time interval is
W
0.5
(t)
16

2
1 cos(0.25t) [ ]
2
J
The energy for the second inductor remains zero over this interval.
For 2 s t, we have
i
in
(t) i
0.5
(t) + i
0.25
(t) i
0.5
(t) +
1
0.25
v
in
( ) d
2
t

1 cos(0.25t) [ ] +
4
0.25
cos(0.25 ) [ ]
2
t

1 cos(0.25t) [ ]
16

cos(0.25t)

24

cos(0.25t) A
Here the current
i
0.5
(t)
8

1 cos(0.25t) [ ] A
in which case the energy stored over the interval [2,t] is
W
0.5
(2,t)
1
2
0.5 i
0.5
2
(t) i
0.5
2
(2)
( )

1
2
0.5 i
0.5
2
(t)
64

2



_
,
J
Further
i
0.25
(t)
16

cos(0.25t) A
in which case the energy stored over the interval [2,t] is
W
0.25
(2,t)
1
2
0.25 i
0.25
2
(t) i
0.25
2
(2)
( )

1
2
0.25i
0.25
2
(t) J
SOLUTION 7.8. Let the 5mH inductor be L
1
and the 20mH inductor be L
2
.
For 0 t < 3ms,
i
L1
(t) i
L1
(0) +
1
L
1
v
s
0
t

( )d
1
5 10
3
12cos
0
t

(500 )d 4.8sin(500t) mA
L&C Probs, 11/15/01 P:7-9 R. A. DeCarlo, P. M. Lin
i
L2
(t) i
L2
(0) +
1
L
2
v
s
0
t

( )d
1
20 10
3
12cos
0
t

(500 )d 1.2sin(500t) mA
For t 3ms,
i
L1
(t) = 4.8sin(500t) mA as L
1
is still subjected to the same voltage.
on the other hand,
i
L2
(t) i
L2
(3

ms) +
1
L
2
v
L2
3
+
ms
t

( )d i
L2
(3

ms) +
1
L
2
0
3
+
ms
t

d
i
L2
(3

ms)
1.2sin(500 3 10
3
) 1.197mA
For 0 t < 3ms, the energies stored in the inductors are given as follows:
W
L1
(t)
1
2
L
1
i
L1
(t) [ ]
2
57.6sin
2
(500t) nJ,
W
L2
(t)
1
2
L
2
i
L2
(t) [ ]
2
14.4sin
2
(500t) nJ
For t 3ms, the energies stored in the inductors are given as follows:
W
L1
(t)
1
2
L
1
i
L1
(t) [ ]
2
57.6sin
2
(500t) nJ,
W
L2
(t)
1
2
L
2
i
L2
(t) [ ]
2
14.328 nJ
SOLUTION 7.9. Given the dielectric parameters and the dimensions of the capacitor, the capacitance of the
paper capacitor is given by
C
r 0
A
d
3 8.854 10
12

0.04 0.8
10
4
8.5nF
SOLUTION 7.10. . Part 1. Applying the capacitor v-i relationship:
i
C
(t) C
d
dt
v
C
(t) ( ) 1 100 1000 (sin(1000t)) 0.1sin(1000t) A
Part 2. Applying the capacitor v-i relationship:
L&C Probs, 11/15/01 P:7-10 R. A. DeCarlo, P. M. Lin
i
C
(t) C
d
dt
v
C
(t) ( )
10 10
3
cos(1000t) C
d
dt
sin(1000t) ( )
10 10
3
cos(1000t) 1000Ccos(1000t)
Therefore, C=10F.
SOLUTION 7.11. Applying the capacitor v-i relationship for C
1
and C
2
:
i
C1
(t) (2mF)
d
dt
v
in
(t) ( ), i
C2
(t) (6mF)
d
dt
v
in
(t) ( ) 3i
C1
(t),
In the interval 1s < t 0s, v
in
(t) = 5t + 5. Hence, in this time interval
i
C1
(t) 2 10
3

d(v
in
(t))
dt
10
2
A
and
i
C2
(t) 6 10
3

d(v
in
(t))
dt
310
2
A
Using KCL, i
in
(t) can be computed as
i
in
(t) i
C1
(t) + i
C2
(t) 4 10
2
A
In the interval 0s < t 1s, v
in
(t) = 5. Thus
i
C1
(t) i
C2
(t) 0 A
Using KCL, i
in
(t) i
C1
(t) + i
C2
(t) 0 A.
In a similar fashion i
C1
(t), i
C2
(t) and i
in
(t) can be computed for the remaining intervals.
The calculations for i
C1
(t), i
C2
(t) and i
in
(t) for t = -1s to 6s are summarized in the following table:
Time Interval d/dt (v
in
(t)), Vs
-1
i
C1
(t), mA i
C2
(t), mA i
in
(t), mA
-1s< t 0s 5 10 30 40
0s< t 1s 0 0 0 0
1s< t 3s -5 -10 -30 -40
3s< t 4s 20 40 120 160
4s< t 5s 0 0 0 0
5s< t 6s -15 -30 -90 -120
Below are the plots of i
C1
(t), i
C2
(t) and i
in
(t).
L&C Probs, 11/15/01 P:7-11 R. A. DeCarlo, P. M. Lin
SOLUTION 7.12. Applying (7.6)
v
C
(2) v
C
(0) +
1
C
i
C
( )d
0
2

4V +
1
C
d
0
2

5V
v
C
(3) v
C
(2) +
1
C
i
C
( )d
2
3

v
C
(2) +
1
C
2d
2
3

5V+1V 6V
Applying (7.11), the energies stored in the capacitor over the intervals [0,2] and [2,3] are given by
W
C
[0,2]
1
2
C v
C
2
(2) v
C
2
(0)
[ ]
9 J
W
C
[2,3]
1
2
C v
C
2
(3) v
C
2
(2)
[ ]
11 J
SOLUTION 7.13. Part 1 Applying (7.6)
L&C Probs, 11/15/01 P:7-12 R. A. DeCarlo, P. M. Lin
v
C1
(t) v
C1
(0) +
1
C
1
i
in
( )d
0
t


1
0.25
i
in
( )d
0
t

v
C2
(t) v
C2
(0) +
1
C
2
i
in
( )d
0
t


1
0.1
i
in
( )d
0
t

In the interval 0s < t 1s, i


in
(t) 2t . Hence, in this interval
v
C1
(t)
1
0.25
i
in
( )d
1
0.25
(
2
)
o
t

0
t
4 10
6
t
2
(V)
and
v
C2
(t)
1
0.1
i
in
( )d
1
0.1
(
2
)
o
t

0
t
10 10
6
t
2
(V)
In the interval 1s < t 3s, i
in
(t) 2 10
3
(A). Thus
v
C1
(t) v
C1
(1) +
1
0.25
i
in
( )d
1
t

4 +
1
0.25
(2 10
3
)
1
t
8 10
3
t 4(V)
and
v
C2
(t) v
C2
(1) +
1
0.1
i
in
( )d
1
t

10 +
1
0.1
(2 10
3
)
1
t
20 10
3
t 10(V)
In a similar fashion, v
C1
(t) and v
C2
(t) can be computed for the remaining intervals.
The calculations for v
C1
(t) and v
C2
(t) for t = 0s to 8ms are summarized in the following table:
Time Interval i
in
(t), A v
C1
(t), V v
C2
(t), V
0s< t 1ms 2t 410
6
t
2
1010
6
t
2
1ms< t 3ms 210
-3
-4 + 810
3
t -10 + 2010
3
t
3ms< t 5ms -210
-3
44 - 810
3
t 110 - 2010
3
t
5ms< t 8ms 0 4 10
Below are the plots of v
C1
(t) and v
C2
(t).
L&C Probs, 11/15/01 P:7-13 R. A. DeCarlo, P. M. Lin
Part 2 Applying (7.12),
W
C1
(t)
1
2
C
1
v
C1
2
(t) , W
C2
(t)
1
2
C
2
v
C2
2
(t)
The expressions for W
C1
(t) and W
C2
(t) for t = 0ms to 8ms are listed in the following table:
Time Interval W
C1
(t), J W
C2
(t), J
0s< t 1ms 0.125 (4 10
6
t
2
)
2
0.05 (1010
6
t
2
)
2
1ms< t 3ms 0.125 (-4 + 810
3
t)
2
0.05 (-10 + 2010
3
t)
2
3ms< t 5ms 0.125 (44 810
3
t)
2
0.05 (110 2010
3
t)
2
5ms< t 8ms 0.125 (4)
2
0.05 (10)
2
Below are the plots of W
C1
(t) and W
C2
(t).
Part 3 Since the current i
C1
(t) stays constant at 0A after t = 5ms,
v
C1
() v
C1
(5) 4V
v
C2
() v
C2
(5) 10V
SOLUTION 7.14. Part 1 Applying (7.6),
L&C Probs, 11/15/01 P:7-14 R. A. DeCarlo, P. M. Lin
v
in
(t) v
in
(0) +
1
C
i
in
( )d
0
t


1
0.5 10
3
i
in
( )d
0
t

In this part, we use the current excitation signal described in Figure P7.14b. In the interval 0s < t 1s,
i
in
(t) 10(mA). Thus, in this interval
v
in
(t)
1
0.5 10
3
(10 10
3
)d
o
t

20t(V)
In the interval 1s < t 2s, i
in
(t) 5(mA). Hence, in this interval
v
in
(t) v
in
(1) +
1
0.5 10
3
(5 10
3
)d
1
t

20 + (10t +10) 10t 10(V)


In a similar fashion, v
in
(t) can be computed for the remaining intervals. The calculations for v
in
(t) for t = 0s
to 5s are summarized in the following table:
Time Interval i
in
(t), mA v
in
(t), V
0s< t 1s -10 -20t
1s< t 2s -5 -10 -10t
2s< t 3s 0 -30
3s< t 4s 5 -60+10t
4s< t 5s 10 -100+20t
Below is the plot of v
in
(t) vs. time.
Part 2. In this part, we use the current excitation signal described in Figure P7.14c. In the interval 0s < t
1s, i
in
(t) 10(mA) . Thus, in this interval
v
in
(t)
1
0.5 10
3
(1010
3
)d
o
t

10t
2
(V)
In the interval 1s < t 3s, i
in
(t) 10t 20(mA) . Hence, in this interval
v
in
(t) v
in
(1) +
1
0.5 10
3
10
3
(10 20)d
o
t

10 +10(10
2
40 )
1
t
10t
2
40t + 40 (V)
L&C Probs, 11/15/01 P:7-15 R. A. DeCarlo, P. M. Lin
The calculations for v
in
(t) for t = 0s to 3s are summarized in the following table:
Time Interval i
in
(t), mA v
in
(t), V
0s< t 1s 10t 10t
2
1s< t 3s 10t-20 10t
2
- 40t + 40
Below is the plot of v
in
(t) vs. time.
SOLUTION 7.15. Part 1 Using the capacitor v-i relationship,
i
in
(t) C
1
d
dt
(v
s
(t)) 20 6 1500 cos(1500t) 0.18cos(1500t) A
Then, we can find v
out
by applying (7.6),
v
out
(t) v
out
(0) +
1
0.5m
2i
in
( )d
0
t

10 +
2 0.18
0.5m1500
sin(1500t)
10 + 0.48sin(1500t) V
Part 2 The instantaneous power delivered by the independent source is given by
p(t) 2i
in
(t) v
out
(t) 0.36cos(1500t) [10+ 0.48sin(1500t)] W
Part 3 Applying (7.11), the energy stored in the capacitor over the interval [0,t] is given by
W
C1
[0, t]
1
2
C
1
v
s
2
(t) v
s
2
(0)
[ ]
360sin
2
(1500t) J
L&C Probs, 11/15/01 P:7-16 R. A. DeCarlo, P. M. Lin
SOLUTION 7.16. Part 1 From the solution of problem 7.15,
v
out
(t) v
out
(0) +
1
0.5m
2i
in
( )d
0
t

v
out
(0) +
1
0.5m
2 20
d
d
v
s
( ) ( )d
0
t

v
out
(0) + 0.08
d
d
v
s
( ) ( )d
0
t

v
out
(0) + 0.08v
s
( )
If we assume v
out
(0) = 0V, then
v
out
(t) 0.08 v
s
( )
The following is a plot of v
out
(t) vs time.
Part 2 The instantaneous power delivered by the dependent source is given by
p(t) v
out
(t) 2i
in
(t) ( v
out
(0) + 0.08 v
s
(t)) 2 20
d
dt
v
s
(t) ( )
40( v
out
(0) + 0.08 v
s
(t))
d
dt
v
s
(t) ( ) W
If we assume v
out
(0) = 0V, then
p(t) 3.2 v
s
(t)
d
dt
v
s
(t) ( ) W
The calculations for d/dt (v
S
(t)) and p(t) for t = 0s to 6ms are summarized in the following table:
L&C Probs, 11/15/01 P:7-17 R. A. DeCarlo, P. M. Lin
Time Interval d/dt (v
S
(t)), Vs
-1
p(t), W
0ms< t 1ms 2 10
3
12.8 t
1ms< t 3ms -2 10
3
12.8 (t-2)
3ms< t 4ms 2 10
3
12.8 (t-4)
4ms< t 6ms 0 0
The following is a plot of p(t) vs time.
Part 3 The energy stored in the 0.5-mF capacitor is given by
W
C
(t)
1
2
0.5m v
out
2
(t) 0.25( v
out
(0) + 0.08 v
s
(t))
2
mJ
If we assume v
out
(0)=0V, then
W
C
(t) 1.6(v
s
(t))
2
J
In the interval 0s < t 1ms, v
s
(t) 2 10
3
t(V). Thus, in this interval
W
C
(t) 6.4t
2
J
In the interval 1s < t 3ms, v
s
(t) 2 10
3
(2 t)(V). Hence, in this interval
W
C
(t) 6.4 (2 t)
2
J
In a similar way W
C
(t) can be computed for the interval 3ms < t 4ms.
The following is a plot of W
C
(t) vs time.
L&C Probs, 11/15/01 P:7-18 R. A. DeCarlo, P. M. Lin
SOLUTION 7.17. Part 1 From the solution of problem 7.16,
v
out
(t) v
out
(0) + 0.08 v
s
( )
If we assume v
out
(0)=0V, then
v
out
(t) 0.08 v
s
( )
The following is a plot of v
out
(t) vs time.
Part 2 The instantaneous power delivered by the dependent source is given by.
p(t) 40( v
out
(0) + 0.08 v
s
(t))
d
dt
v
s
(t) ( ) W
If we assume v
out
(0)=0V, then
p(t) 3.2 v
s
(t)
d
dt
v
s
(t) ( ) W
The calculations for d/dt (v
S
(t)) and p(t) for t = 0s to 6ms are summarized in the following table:
L&C Probs, 11/15/01 P:7-19 R. A. DeCarlo, P. M. Lin
Time Interval d/dt (v
S
(t)), Vs
-1
p(t), W
0ms< t 1ms 2 10
3
12.8 t
1ms< t 2ms 0 0
2ms< t 4ms -2 10
3
12.8 (t-3)
4ms< t 5ms 0 0
5ms< t 6ms 2 10
3
12.8 (t-6)
The following is a plot of p(t) vs time.
Part 3 The energy stored in the 0.5-mF capacitor is given by
W
C
(t) 0.25( v
out
(0) + 0.08 v
s
(t))
2
mJ
If we assume v
out
(0) = 0V, then
W
C
(t) 1.6(v
s
(t))
2
J
In the interval 0ms < t 1ms, v
s
(t) 2 10
3
t(V). In this interval
W
C
(t) 6.4t
2
J
In the interval 1ms < t 2ms, v
s
(t) 2(V). In this interval
W
C
(t) 6.4( J )
In a similar fashion W
C
(t) can be computed for the remaining intervals.
The following is a plot of W
C
(t) vs time.
L&C Probs, 11/15/01 P:7-20 R. A. DeCarlo, P. M. Lin
SOLUTION 7.18. Part 1 Applying the capacitor v-i relationship for the equivalent capacitor of the series
combination of 0.3 mF and 0.6 mF capacitors
v
in
(t) v
in
(0) +
1
0.3 10
3
0.6 10
3
i
s
( )d
0
t

(0) +
1
0.310
3
0.6 10
3
60 10
3
sin(100 )d
0
t


3 3cos(100t) V
Therefore,
i
out
(t) (0.2 10
3
+ 0.8 10
3
)
d
dt
(10v
in
(t)) 3sin(100t) V
Below is a sketch of i
out
(t) vs time.
Part 2 The instantaneous power delivered by the dependent source is given by

p(t) i
out
(t) 10v
in
(t) 90sin(100t) [1 cos(100t)] W
L&C Probs, 11/15/01 P:7-21 R. A. DeCarlo, P. M. Lin
Part 3 The instantaneous energy stored in the 0.8mF capacitor is given by
W
C
(t)
1
2
Cv
C
2
(t)
1
2
(0.8mF) 10v
in
(t) [ ]
2
0.361 cos(100t) [ ]
2
J
SOLUTION 7.19.
Part 1 Since Q = CV, the charge that resides on each plate of the capacitor = 10F 100V = 1mC
Part 2 Since V = Q/C, the required voltage = 1mC/5F = 200V
Part 3 Since V = Q/C, the required voltage = 50C/1F = 50V
Part 4 The energy required = 0.5 10F (100V)
2
= 0.05J
SOLUTION 7.20. When 0s t <2s, v
C
(t) = 25V. Conservation of charge requires that
q
1
(2

) + q
2
(2

) q
1
(2
+
) + q
2
(2
+
). Since q
2
(2

) = 0 C it follows that
25 V 150 mF = v
C
(2
+
)(150 mF + 100 mF)
Hence
v
C
(2
+
)
25V 150mF + 0V 100mF
150mF +100mF
15 V
Thus, for t > 2s, v
C
(t) = 15 V.
SOLUTION 7.21. For this solution consider the figure below in which C
1
and C
2
are labeled.
There are two cases to consider: (i) t < 2 ms and (ii) t > 2 ms.
Case 1. t < 2 ms. Here, since the current source is zero for t < 0, and C
1
is uncharged at t = 0,
v
C
(t)
1
C
1
i
s
( )d

v
C
(0) +
1
C
1
i
s
( )d
0
t

1
C
1
i
s
( )d
0
t

12
15 10
3
500
e
500
[ ]
0
t
1.6 1 e
500t
( )
V
L&C Probs, 11/15/01 P:7-22 R. A. DeCarlo, P. M. Lin
Note that v
C
(2

ms) 1.0114 V. Hence the energy stored over [0, 2 ms] is


W
C1
(0 < t < 2ms) 0.0192 1 e
500t
( )
2
J
and
W
C2
(0 < t < 2ms) 0
Case 2. 2 ms < t. At t = 2 ms the switch closes, forcing a discontinuity in the capacitor voltages. To
calculate the capacitor voltages at 2
+
ms, we use conservation of charge. Here, the relevant equation is:
q
C1
(2
+
ms) + q
C2
(2
+
ms) q
C1
(2

ms) + q
C2
(2

ms) + i
s
( )d
2

ms
2
+
ms

Note that since v


C2
(2

ms) 0, q
C2
(2

ms) C
2
v
C2
(2

ms) 0 and the integral of the bounded


continuous function i
s
(t) over an infinitesimal interval is zero, this equation reduces to
q
C1
(2
+
ms) + q
C2
(2
+
ms) q
C1
(2

ms)
or equivalently, since for t > 2 ms, v
C1
(t) v
C2
(t) v
C
(t) ,
C
1
v
C
(2
+
ms) + C
2
v
C
(2
+
ms) C
1
v
C
(2

ms)
Therefore
v
C
(2
+
ms)
C
1
C
1
+ C
2
v
C
(2

ms)
15
15 + 25
1.0114 0.37927 V
and it follows that
v
C
(t) v
C
(2
+
ms) +
1
C
eq
i
s
( ) d
2ms
t

0.379 +
1
C
1
+ C
2
i
s
( )d
2ms
t

0.379 +
12
40 10
3
500
e
500
[ ]
0.002
t
0.379 + 0.6 0.36788 e
500t
( )
V
Hence the energy stored in the two capacitors over the interval [2
+
ms, t] is
W
C
eq
(2
+
ms < t)
1
2
C
eq
v
C
2
(t)
1
2
C
eq
v
C
2
(2
+
ms) J
whereas the instantaneous stored energy, i.e., the energy stored over (-,t > 2 ms] is given by
W
C
eq
(t)
1
2
C
eq
v
C
2
(t)
What happens between 2
-
ms and 2
+
ms is beyond the scope of the material in this chapter. Please
refer to problem 51 in chapter 8 for an explanation.
L&C Probs, 11/15/01 P:7-23 R. A. DeCarlo, P. M. Lin
SOLUTION 7.22. Part 1 Applying the inductor v-i relationship,
v
L
(t) L
d
dt
i
in
(t)
2.5mH 200te
10t
+ 20e
10t
( )
0.05e
10t
(10t +1)
Applying the capacitor v-i relationship, we have:
v
C
(t) v
C
(0) +
1
C
i
in
( )d
0
t

20
1m
te
10
0
t

20 10
3
0.1 e
10
0.01e
10
[ ]
0
t
2 10
3
te
10t
0.1e
10t
+ 0.1
[ ]
V
and
v
in
(t) v
L
(t) + v
C
(t)
0.05e
10t
(10t +1)+ 2000 te
10t
0.1e
10t
+ 0.1
[ ]
2000.5 te
10t
199.95 e
10t
+ 200(V)
The sketches of v
L
(t), v
C
(t) and v
in
(t) are shown below.
L&C Probs, 11/15/01 P:7-24 R. A. DeCarlo, P. M. Lin
Part 2 The energy stored in the inductor is given by
W
L
(t)
1
2
L[i
in
(t)]
2
0.5t
2
e
20t
J
The sketch of W
L
(t) vs. time is shown below.
Part 3 The energy stored in the capacitor is given by
W
C
(t)
1
2
C v
C
(t) [ ]
2
2 te
10t
0.1e
10t
+ 0.1
[ ]
2
kJ
The sketch of W
C
(t) vs time is shown below.
L&C Probs, 11/15/01 P:7-25 R. A. DeCarlo, P. M. Lin
SOLUTION 7.23. Applying the inductor v-i relationship we have that
v
L
(t) 0.25
d[i
s
(t)]
dt
4cos(4t) (V)
Applying the capacitor v-i relationship it follows that
i
C
(t) 0.25
d[2v
2
(t)]
dt
0.25 2 4 4 [sin(4t)] 8sin(4 t) A
SOLUTION 7.24.
Part 1. By KCL i
s
(t) i
C1
(t) + i
C2
(t). Applying the v-i relationship for capacitor C
1
and C
2
we have:
i
S
(t) (10mF + 20mF)
d
dt
v
s
(t)
30 10
3
(12te
5t
30t
2
e
5t
)
0.36te
5t
0.9t
2
e
5t
A
Part 2. Applying the v-i relationship for the 20mH inductor, it follows that
v
out
(t) (20mH)
d
dt
[12i
s
(t)]
0.24 4.5t
2
e
5t
3.6te
5t
+ 0.36e
5t
( )
1.08t
2
e
5t
0.864te
5t
+ 0.0864e
5t
(V)
Part 3 The energy stored in the 20mH inductor for t > 0 is given by
W
L
(t)
1
2
(20mH)[12i
S
(t)]
2
1.44 0.36te
5t
0.9t
2
e
5t
( )
2
J
1.44 t
2
e
10t
(0.36 0.9t)
2
J
SOLUTION 7.25. We denote by v
C
(t) the voltage across the capacitors C
1
and C
2
and by i
L
(t) the current
through the inductors L
1
and L
2
. The equivalent capacitance of the parallel combination of C
1
and C
2
is
(C
1
+ C
2
) and thus:
i
S
(t) (C
1
+ C
2
)
dv
C
(t)
dt
Using the v-i relationship for the capacitor C
2
it follows that
i
C2
(t) C
2
dv
C
(t)
dt
By replacing
dv
C
(t)
dt
, it follows that
L&C Probs, 11/15/01 P:7-26 R. A. DeCarlo, P. M. Lin
i
C2
(t)
C
2
C
1
+ C
2
i
S
(t)
2
3
i
s
(t)
The equivalent inductance of the series combination of L
1
and L
2
is (L
1
+ L
2
) and thus:
9i
C2
(t) (L
1
+ L
2
)
di
L
(t)
dt
Using the v-I relationship for the inductor L
2
it follows that
v
out
(t) v
L2
(t) L
2
di
2
(t)
dt
By replacing
di
L
(t)
dt
, it follows that
v
out
(t)
L
2
L
1
+ L
2
9i
C2
(t)
2
3
9i
C2
(t) 6i
C2
(t) 4i
s
(t)
SOLUTION 7.26. Using the v-i relationship for the capacitor we can write:
v
C2
(t)
1
C
2
i
C
( )d

v
C1
(t)
1
C
1
i
C
( )d

where i
C
(t) is the current through the capacitors C
1
and C
2
. Since C
1
and C
2
are connected in series:
v
in
(t) v
C1
(t) + v
C2
(t)
1
C
1
+
1
C
2



_
,

i
C
( )d

It follows that i
C
( )d

v
in
(t)
C
1
C
2
C
1
+ C
2



_
,

. Hence
v
C2
(t)
1
C
2
i
C
( )d

1
C
2

C
1
C
2
C
1
+ C
2
v
in
(t)
C
1
C
1
+ C
2
v
in
(t)
Since L
1
and L
2
are combined in parallel it follows that
v
out
(t)
1
L
1
+
1
L
2



_
,

1
d(Av
C2
(t))
dt

L
1
L
2
L
1
+ L
2
A
dv
C2
(t)
dt

L
1
L
2
L
1
+ L
2
A
C
1
C
1
+ C
2
dv
in
(t)
dt
L&C Probs, 11/15/01 P:7-27 R. A. DeCarlo, P. M. Lin
where the last equality follows by replacing v
C2
(t) with
C
1
C
1
+ C
2
v
in
(t)
SOLUTION 7.27. Observe first that the 0.3H and 0.6H parallel inductances combine to make a 0.2H
inductance. Also the 0.4H and 1.2H parallel inductances combine to make a 0.3H inductance. Finally, the
0.2H and 0.3H inductances are combined in series and the equivalent inductance is 0.5H. Shortly, all the
above steps can be written as: L
eq
= (0.3H || 0.6H) + (0.4H || 1.2H) = 0.5H.
SOLUTION 7.28. Observe first that the 1mH and 5mH inductors combine to make a 6mH inductance.
This inductance combines in parallel with the 3mH inductance to make a 2mH inductance. The next step
is to combine in series the 2mH inductance with the 10mH inductance. The equivalent inductance is
12mH. This inductance is combined in parallel with the 36mH inductance and the result is 9mH. Finally,
the 9mH inductance is combined in series with the 4mH inductance and the result is 13mH. Shortly, all
the above steps cam be written as: L
eq
= [(5mH + 1mH) || 3mH + 10mH] || 36mH + 4mH = 13mH.
SOLUTION 7.29. Step 1. The parallel combination of the 0.6mH and 1.2mH inductors is equivalent to a
0.4mH inductor.
Step 2. The series combination of the 2.4mH and 0.4mH inductors is equivalent to a 2.8mH inductor.
Step 3. The parallel combination of the 2.8mH and 7mH inductors is equivalent to a 2mH inductor.
SOLUTION 7.30. The three inductors can be arranged in the seven fashions as shown below.
(a) (b) (c)
(d) (e)
(f) (g)
L
eq
a
= 1mH
L
eq
b
= 1mH + 1mH = 2mH
L
eq
c
= 1mH || 1mH = 0.5mH
L
eq
d
= 1mH + 1mH + 1mH = 3mH
L
eq
e
= 1mH + (1mH || 1mH) = 1.5mH
L
eq
f
= 1mH || (1mH + 1mH) = 0.667mH
L
eq
g
= 1mH || 1mH || 1mH) = 0.333mH
L&C Probs, 11/15/01 P:7-28 R. A. DeCarlo, P. M. Lin
SOLUTION 7.31. For Fig. P7.31a, L
eq
= 2 + 15 + 10 + 10 + 40 +30 + 20 + 8 = 135mH
The circuit in Fig. P7.31b is equivalent to the following circuit.
Therefore, L
eq
= 2 + 15 + 10 + 8 = 35mH
The circuit in Fig. P7.31c is equivalent to the following circuit.
Therefore, L
eq
= 2 + [(15+10) || (10+40) || (20+30)] + 8 = 22.5mH
SOLUTION 7.32. When the switch is open, the circuit in Fig. P7.32 can be rearranged as the following.
Therefore, L
eq
= 8L || 8L = 4L
When the switch is closed, the circuit in Fig. P7.32 can be rearranged as the following.
Therefore, L
eq
= 4L || 4L + 4L || 4L = 4L
L&C Probs, 11/15/01 P:7-29 R. A. DeCarlo, P. M. Lin
SOLUTION 7.33. L
eq1
=L
eq2
Without going into a detailed analysis, we present the following intuitive
argument. Note that the points a and b represent points on a balanced bridge circuit meaning that the
voltage between a and b would be zero. Therefore, no current will flow through the additional inductance
L. Therefore the presence of L does not affect the equivalent inductance value.
SOLUTION 7.34. L
eq1
>L
eq2
Without going into a detailed analysis, we present the following intuitive
argument. Note that the points a and b represent points on an unbalanced bridge circuit, meaning that the
voltage between a and b would not be zero. Also note that when two inductors are placed in parallel, the
equivalent inductance becomes smaller than either inductance. The addition of the inductor L in circuit 2
essentially creates an internal parallel inductance resulting in an L
eq2
lower than L
eq1
.
SOLUTION 7.35. First we add an i
in
label to the circuit as shown below.
From KVL and the derivative definition of the capacitor
v
in
(t) v
L1
(t) + v
L2
(t) L
1
di
in
(t)
dt
+ L
2
di
in
(t)
dt
L
1
+ L
2
( )
di
in
(t)
dt
Equivalently,
di
in
(t)
dt

1
L
1
+ L
2
( )
v
in
(t)
It follows that
v
L
k
(t) L
k
di
in
(t)
dt

L
k
L
1
+ L
2
v
in
(t)
which is the required voltage division formula.
SOLUTION 7.36. L
eq
= (11mH) || (19.25mH) + 3mH = 10mH. Applying the voltage division formula,
v
L1
(t) v
in
(t)
3mH
L
eq
60te
t
mV
and
v
L2
(t) v
in
(t) v
L1
(t) 140te
t
mV
L&C Probs, 11/15/01 P:7-30 R. A. DeCarlo, P. M. Lin
SOLUTION 7.37. First consider the circuit below which contains the additional label of v
in
(t).
From KCL and the integral definition of the inductor,
i
in
(t) i
L1
(t) + i
L2
(t)
1
L
1
v
in
( )d
0
t

+
1
L
2
v
in
( ) d
0
t

Equivalently
v
in
( )d
0
t

1
1
L
1
+
1
L
2





_
,



i
in
(t)
Therefore
i
L
k
(t)
1
L
k
v
in
( )d
0
t

1
L
k
1
L
1
+
1
L
2





_
,



i
in
(t)
L
1
L
2
L
k
L
1
+ L
2





_
,



i
in
(t) for k = 1,2
SOLUTION 7.38. In the circuit illustrated in Fig. 7.38,
L
eq
= (12mH + 27mH) || (130mH) = 30mH
Applying the current division formula,
i
L1
(t)
12 + 27
12 + 27 +130
i
in
(t) 0.231e
t
2
mA
i
L2
(t)
130
12 + 27 +130
i
in
(t) 0.769e
t
2
mA
Also,
v
in
(t) L
eq
d
dt
i
in
(t) 60te
t
2
V
The instantaneous energy stored in the 130-mH is given by
W
L1
(t)
1
2
L
1
i
L1
(t) [ ]
2
0.8e
2t
2
nJ
L&C Probs, 11/15/01 P:7-31 R. A. DeCarlo, P. M. Lin
SOLUTION 7.39. Using the inductor v-i relationship,
v
in
(t) L
eq
d
dt
i
in
(t) 30mH
d
dt
i
in
(t)
The calculations for v
in
(t) for t = -2s to 7s are summarized in the following table:
Time Interval d/dt (i
in
(t)), mAs
-1
v
in
(t), mV
-2s< t <=0s 100 3
0s< t <=2s -200 -6
2s< t <=3s 0 0
3s< t <=4s 200 6
4s< t <=6s 100 3
6s< t <=7s -200 -6
From the solutions to problem 7.38,
i
L1
(t) 0.231 i
in
(t)
i
L2
(t) 0.769 i
in
(t)
Below are the plots for v
in
(t), i
L1
(t) and i
L2
(t).
L&C Probs, 11/15/01 P:7-32 R. A. DeCarlo, P. M. Lin
SOLUTION 7.40. Part 1 Using the current division formula,
i
L2
(t)
18
18 + 6
i
in
(t) 90cos(300 t) mA
Part 2
L
eq
=1.5mH + 6mH || 18mH = 6mH
v
in
(t) L
eq
d
dt
i
in
(t) -0.679sin(300 t) V
Part 3 Instantaneous power delivered by the source is given by
p(t) v
in
(t) i
in
(t) [-0.679sin(300 t)][120cos(300 t)] mW
-40.7sin(600 t) mW
Below is a plot of p(t)
SOLUTION 7.41. Part 1 L
eq
= 0.3 || 0.9 + 0.4 || 0.4 = 0.425 H
For computing i
in
(t) we need to apply
i
in
(t) i
in
(t
0
) +
1
L
eq
v
in
( )d
t
0
t

to each interval [0, 1], [1, 2], , [n, n+1],


The initial condition for the interval [n, n+1] for n even is:
i
in
(n) i
in
(0) +
1
L
eq
v
in
( )d
0
n

We assume that i
in
(0) = 0 and it follows that i
in
(n) = 0 for n even, since
L&C Probs, 11/15/01 P:7-33 R. A. DeCarlo, P. M. Lin
v
in
( )d 0
0
n

for n even. The initial condition for the interval [n, n+1] for n odd is:
i
in
(t) i
in
(n 1) +
1
L
eq
v
in
( )d
n1
n

1
0.425
(16 )
n1
n
37.6 A
For the interval [n, n+1] with n even:
i
in
(t) i
in
(n) +
1
L
eq
v
in
( )d
n
t

0 +
1
0.425
16
n
t
37.6( t n) A
For the interval [n, n+1] with n odd:
i
in
(t) i
in
(n) +
1
L
eq
v
in
( )d
n
t

37.6 +
1
0.425
(16 )
n
t
37.6( n +1 t) A
For the interval [n, n+1] with n even:
i
in
(t) i
in
(n) +
1
L
eq
v
in
( )d
n
t

0 +
1
0.425
(16 )
n
t
37.6(t n) A
Hence,
i
in
(t)
37.6(t 2n), 2n < t 2n +1
37.6(2n + 2 t), 2n +1< t 2n + 2

'

where n is a non-negative integer. Below is a sketch for the input current vs. time.
Part 2 The total energy stored in the set of four inductors is given by
L&C Probs, 11/15/01 P:7-34 R. A. DeCarlo, P. M. Lin
W(t)
1
2
L
eq
i
in
(t) [ ]
2

300(t 2n)
2
J for 2n < t 2n +1
300(2n + 2 t)
2
J for 2n +1< t 2n + 2

'



where n is a non-negative integer.
Below is a sketch of W(t) vs. t.
Part 3 By the current division formula, the current through the 0.9H inductor is given by
i
L0.9H
(t)
0.3
0.3 + 0.9
i
in
(t)

9.4(t 2n) A for 2n < t 2n +1


9.4(2n + 2 t) A for 2n +1< t 2n + 2

'

where n is a non-negative integer.
Below is a sketch of the i
L0.9H
vs. time.
L&C Probs, 11/15/01 P:7-35 R. A. DeCarlo, P. M. Lin
SOLUTION 7.42.. a)
Step 1. The parallel combination of the 1F and 2F capacitances is equivalent to a 3F capacitance.
Step 2. The series capacitors 1.5F and 3F combine to make a 1F capacitance.
Step 3. Finally, the 2F capacitance is combined in parallel with the 1F capacitance that was the result of
Step 2 to make a 3F capacitance.
Shortly, the above steps can be written as:
C
eq
= [(1F + 2F) || 1.5F] + 2F] = 3F
b) Proceeding in a similar fashion as in part a:
C
eq
= 30mF + [9mF || (9mF + 18mF) || 5.4mF] = 33mF
SOLUTION 7.43. a)
Step 1. Combine the parallel capacitances of 1F and 2F to make a 3F capacitance.
Step 2. Combine the series capacitances of 1.5F and 3F to make 1F capacitance.
Step 3. Combine the parallel capacitances of 2.5F and 1F (the result of Step 2) to make a 3.5F
capacitance.
Shortly, the above steps can be written in a condensed form as:
C
eq
= 2.5F + [1.5F || (1F + 2F) = 3.5F
b) Proceeding in a similar fashion as in part a:
C
eq
= (1mF || 2 mF) + (1.2mF || 2mF || 1.5mF) + (4mF || 2.6667mF || 3.2mF || 1.6mF) = 2.1667mF
SOLUTION 7.44.
Part 1 C
eq
= ((( 30 || 60 + 40 ) || 30 + 40 ) || 30 + 40 ) || 30 + 40 = 60mF
Part 2 The value of v
in
(t) is given by
v
in
(t) v
in
(0) +
1
C
i
in
( )d
0
t

0 +
1
0.06
200sin(20 )d
0
t

166.7 166.7cos(20t) mV
SOLUTION 7.45. When the switch is open, the circuit in figure P7.45 can be represented by the
following.
L&C Probs, 11/15/01 P:7-36 R. A. DeCarlo, P. M. Lin
Therefore, C
eq
= 6.4C
When the switch is closed, the circuit in figure P7.45 can be represented by the following.
Therefore, C
eq
= ( 4 + 4 ) || ( 16+ 16 ) = 6.4C
SOLUTION 7.46. C
eq1
<C
eq2
Without going into a detailed analysis, we present the following intuitive
argument. Note that the points a and b represent points on an unbalanced bridge circuit, meaning that the
voltage between a and b would not be zero. Also note that when two capacitors are placed in parallel, the
equivalent capacitance becomes bigger than either capacitance. The addition of the capacitor C in circuit 2
essentially creates an internal parallel capacitance resulting in a C
eq2
higher than C
eq1
.
SOLUTION 7.47. The three capacitors can be arranged in the seven fashions as shown below.
For configuration (a), C
eq
= 1 || 1 || 1 = 0.3333F
For configuration (b), C
eq
= 1 || (1 + 1) = 0.6667F
For configuration (c), C
eq
= 1 || 1 + 1 = 1.5F
For configuration (d), C
eq
= 1 + 1 + 1 = 3F
For configuration (e), C
eq
= 0 + 1 + 1 = 2F
For configuration (f), C
eq
= 1 || 1 + 0 = 0.5F
For configuration (g), C
eq
= 1 + 0 + 0 = 1F
L&C Probs, 11/15/01 P:7-37 R. A. DeCarlo, P. M. Lin
SOLUTION 7.48. As Q = CV for any capacitor, C=Q/V. The equivalent capacitance of the network is
given by
C
eq

1C
2V
||
1C
3V
||
1C
5V
0.1 F
SOLUTION 7.49. First consider the circuit below which contains the additional label of i
in
(t).
From KVL and the integral definition of the capacitor,
v
in
(t) v
C1
(t) + v
C2
(t)
1
C
1
i
in
( )d +
0
t

1
C
2
i
in
( )d
0
t

Equivalently,
i
in
( )d
0
t

1
1
C
1
+
1
C
2
v
in
(t)
Therefore,
v
Ck
(t)
1
C
k
i
in
( )d
0
t

1
C
k
1
C
1
+
1
C
2
v
in
(t)
C
1
C
2
C
k
C
1
+ C
2
v
in
(t)
Hence,
v
C1
(t)
C
2
C
1
+ C
2
v
in
(t)
v
C2
(t)
C
1
C
1
+ C
2
v
in
(t)
which is the required voltage division formula.
SOLUTION 7.50. By the voltage division formula,
v
C3
(t)
6 ||6
6|| 6 + 9
v
in
(t) 21(1 e
20t
) V
SOLUTION 7.51. Part 1 By the voltage division formula,
L&C Probs, 11/15/01 P:7-38 R. A. DeCarlo, P. M. Lin
v
C1
(t)
0.05 + 0.15
0.05 + 0.15 + 0.1
v
in
(t) 6.667sin(120 t) V
v
C2
(t) v
in
(t) v
C1
(t) 3.333sin(120 t) V
Part 2 Let the 0.1F capacitor be C
1
and the 0.15F capacitor be C
2
. The energies stored in the 0.1F
and the 0.15F capacitors are given by
W
C1
[0, t]
1
2
C
1
v
C1
2
(t) v
C1
2
(0)
[ ]
2.222sin
2
(120 t) J
W
C2
[0,t]
1
2
C
2
v
C2
2
(t) v
C2
2
(0)
[ ]
0.8333sin
2
(120 t) J
SOLUTION 7.52. Part 1 As both the terminals of 0.08F are tied to the voltage source v
in
(t),
v
C1
(t) v
in
(t) 100e
2t
V
By the voltage division formula,
v
C2
(t)
0.06
0.03 + 0.06
v
in
(t) 66.67e
2t
V
Part 2 Let the 0.03-F capacitor be C
2
. The energy stored in the 0.03-F capacitor over the interval [0,t] is
given by
W
C2
[0,t]
1
2
C
2
v
C2
2
(t) v
C2
2
(0)
[ ]
66.67 e
4t
1
[ ]
J
SOLUTION 7.53. First consider the circuit below which contains the additional label of v
in
(t).
From KCL and the derivative definition of the capacitor
i
in
(t) i
C1
(t) + i
C2
(t) C
1
dv
in
(t)
dt
+ C
2
dv
in
(t)
dt
C
1
+ C
2
( )
dv
in
(t)
dt
Equivalently,
dv
in
(t)
dt

1
C
1
+ C
2
( )
i
in
(t)
It follows that
i
C
k
(t) C
k
dv
in
(t)
dt

C
k
C
1
+ C
2
i
in
(t)
which is the required current division formula.
L&C Probs, 11/15/01 P:7-39 R. A. DeCarlo, P. M. Lin
SOLUTION 7.54. Part 1 By the current division formula,
i
C1
(t)
0.08
0.08 + 0.03 0.06
i
in
(t) 80e
2t
A
i
C2
(t)
0.03 0.06
0.08 + 0.03 0.06
i
in
(t) 20e
2t
A
Part 2 Let the 0.03-F capacitor be C
2.
v
C2
(t) v
C2
(0) +
1
C
2
i
C2
( )d
0
t

333.3(1 e
2t
) V
The energy stored in the 0.03-F capacitor over the interval [0,t] is given by
W
C2
(0, t)
1
2
C
2
v
C2
2
(t) v
C2
2
(0)
[ ]
1666.5(1 e
2t
)
2
J
SOLUTION 7.55. Part 1 By the current division formula,
i
C2
(t)
2
1 + 2
i
s
(t) 20sin(250t) mA
Part 2 By the voltage division formula,
v
out
(t)
2
1+ 2
9i
c2
(t) 0.12sin(250t) V
Part 3 The current in the 2mH inductor is given by
i
L2
(t) i
L2
(0) +
1
L
2
v
out
( )d
0
t

0.24 cos(250t) 1 [ ] A
The energy stored in the 2-mH inductor is given by
W
L2
(t)
1
2
L
2
i
L2
2
(t) 57.6 cos(250t) 1 [ ]
2
J
SOLUTION 7.56. Part 1 The dual network N* is shown below.
Part 2 Since the equivalent conductance seen by I
s
* is 500S, the equivalent resistance seen by I
s
* is equal
to 0.002.
L&C Probs, 11/15/01 P:7-40 R. A. DeCarlo, P. M. Lin
SOLUTION 7.57. Part 1 The dual network N* is shown below.
Part 2 S*
eq
= 350S I*
out
= 8A.
SOLUTION 7.58. First we draw the graph, given below, of the circuit in figure P7.58. We construct the
graph associated with dual network graph for N* from the graph of N. The dual network graph is given
by the dashed lines.
This circuit and its dual have the branch characteristics given in the following table:
ORIGINAL NETWORK DUAL NETWORK
V
1
= 40 V I
1
*
= 40 A
V
2
= 500 I
2
I
2
*
= 500 V
2
*
I
3
= 2 I
2
V
3
*
= 2 V
2
*
V
4
= 400I
4
I
4
*
= 400V
4
*
V
5
= 200 I
5
I
5
*
= 200 V
5
*
Finally replace the branches by the elements given in the table above. This produces the dual network
below.
SOLUTION 7.59. The dual for the circuit in figure P7.59 is shown below.
L&C Probs, 11/15/01 P:7-41 R. A. DeCarlo, P. M. Lin
SOLUTION 7.60. First we draw the graph, given below, of the circuit in figure P7.60.
This circuit and its dual have the branch characteristics given in the following table:
ORIGINAL NETWORK DUAL NETWORK
V
1
= 5 I
1
I
1
*
5V
1
*
I
2
= 8 A V
2
*
8 V
I
3
= 6 A V
3
*
6 V
V
4
= 4 V I
4
*
4 A
V
5
= 2 I
5
I
5
*
2V
5
*
I
6
= 7 A V
6
*
7 V
Now we construct the graph associated with dual network graph for N* from the graph of N. The dual
network graph is given by the dashed lines.
L&C Probs, 11/15/01 P:7-42 R. A. DeCarlo, P. M. Lin
The graph of the dual network then is pulled out and flipped vertically to produce the graph topology of the
dual network, N*.
Finally replace the branches by the elements given in the table above. This produces the dual network
below.
SOLUTION 7.61. Part 1 We first redraw the circuit to eliminate branch crossing. The resultant
schematic is shown below.
L&C Probs, 11/15/01 P:7-43 R. A. DeCarlo, P. M. Lin
Then we draw the graph, given below, of the above circuit. We construct the graph associated with dual
network graph for N* from the graph of N. The dual network graph is given by the dashed lines.
The resultant dual network given by the above graph is shown below.
Part 2 The equivalent resistance R
eq
* seen by the current source in the dual circuit is equal to 1/R
eq
= 1/
.
SOLUTION 7.62. Part 1 We first redraw the circuit to eliminate branch crossing. The resultant
schematic is shown below.
L&C Probs, 11/15/01 P:7-44 R. A. DeCarlo, P. M. Lin
Then we draw the graph, given below, of the above circuit. We construct the graph associated with dual
network graph for N* from the graph of N. The dual network graph is given by the dashed lines.
The resultant dual network given by the above graph is shown below.
Part 2 If we label the conductances in the dual network by their corresponding resistance values, we have
the circuit below.
L&C Probs, 11/15/01 P:7-45 R. A. DeCarlo, P. M. Lin
By comparing to the original network, we conclude that R
eq
* = R
eq
.
Part 3 Since the general relationship between R
eq
and R
eq
* is given by R
eq
= 1/R
eq
*. We can write
R
eq
= 1/R
eq
* =1/R
eq
Solving the above equation, we know that R
eq
= 1 .
Part 4 To retain the special properties of parts (b) and (c), we require one resistance value must be a
reciprocal of the other one.
SOLUTION 7.63. The dual network is shown below.
SOLUTION 7.64. The dual network is shown below.
From the answer of problem 7.55,
i
C2
(t) = 20 sin (250t) mA, and v
out
(t) = 0.12 sin (250t) V
Therefore, v
C2
*(t) = 20 sin (250t) mV and i
out
*
(t) = 0.12 sin (250t) A
The energy stored in the 2mF capacitor is given by
W
C2
*
(t)
1
2
C
2
*
v
out
*
(t)
[ ]
2

1
2
L
2
i
L2
(t) [ ]
2
57.6[cos(250t) 1]
2
J
SOLUTION 7.65. The dual network is shown below.
L&C Probs, 11/15/01 P:7-46 R. A. DeCarlo, P. M. Lin
The equivalent inductance of the dual circuit (L
eq
*) = (1+2) || 1.5 + 2.5 = 3.5H.
SOLUTION 7.66. By voltage division
v
1
(t)
1
0.4
1
0.4
+
1
0.2
v
s
(t)
0.2
0.2 + 0.4
v
s
(t)
1
3
v
s
(t)
Note that the 0.4 F resistor in parallel with the dependent voltage source has no effect on i
1
(t) and hence is
redundant. By the definition of a capacitor,
i
1
(t) 0.2
dv
1
(t)
dt

1
15

dv
s
(t)
dt
Finally, since the last capacitor is initially uncharged,
v
out
(t)
1
0.5
i
1
( )d
0
t

2
15
dv
s
( )
d
d
0
t

2
15
v
s
(t)
A sketch is given below.
SOLUTION 7.67. Following the method outlined in page 269 of the text, we require
v
C
(t
0
)
1
C
i
0
( )d 14
0
1/200

With i
0
() = 2A, v
C
(0) = 20 V, it follows that
20 14
1
C
2A
1
200
s
C 1.667mF
Therefore, the standard capacitor value of 1.8mF should be chosen for this application.
L&C Probs, 11/15/01 P:7-47 R. A. DeCarlo, P. M. Lin
SOLUTION 7.68. From the relationships given in Problem 7.67, we have
v
L
(t)
d
dt
L(t)i
L
(t) [ ] I
0
d
dt
L(t) [ ] I
0
L
1
4
1
T
1

t
T
1
2



_
,





1
]
1
1
Below is a plot of v
L
(t) vs t.
From the plot, we notice that when t=T
1
, the value of v
L
(t) changes from positive to negative. This means
that v
L
(t) will change sign when a car is inside the loop. Therefore, we can make a circuit to monitor the
voltage v
L
(t) and whenever a negative voltage is detected, the left turn signal should be initiated during the
next traffic light change.
1st Order Circuit Probs 11/26/01 P8-1 R. A. DeCarlo, P. M. Lin
SOLUTIONS PROBLEMS CHAPTER 8
SOLUTION 8.1. (a) By KCL, C
dv
C
(t)
dt

v
C
(t)
R
or
dv
C
(t)
dt
+
v
C
(t)
RC
0 . Using 8.12
v
C
(t) v
C
(0)e
t /
10e
t
V where RC 1s. Plotting this from 0 to 5 sec
t = 0:.05:5;
vc = 10*exp(-t);
plot(t,vc)
grid
xlabel('time in s')
ylabel('vc(t) in V')
0 1 2 3 4 5
0
2
4
6
8
10
time in s
v
c
(
t
)

i
n


V
TextEnd
(b) The solution has the same general form as for (a),i
C
(t) i
C
(0+)e
t /

10
5 10
3
e
t
2e
t
mA =
v
C
(t)
R
.
1st Order Circuit Probs 11/26/01 P8-2 R. A. DeCarlo, P. M. Lin
0 1 2 3 4 5
-2
-1.5
-1
-0.5
0
time in s
i
c
(
t
)

i
n


m
A
TextEnd
(c) By linearity, if v
C
(0) is cut in half, all resulting responses are cut in half. If v
C
(0) is doubled, then all
resulting responses are doubled. Alternately, one view this as a simple change of the initial condition
with the same conclusion reached from linearity.
SOLUTION 8.2. (a) From inspection of the general form, 8.12, 0.1/ 1 0.1/ RC C = 0.1/R = 5
F.
(b) Since = RC = 0.1, v
C
(t) 10e
10t
V.
SOLUTION 8.3. (a) The general solution form is v
C
(t) v
C
(0)e
t /
v
C
(0)e
t /R
eq
C
. Using the
given data, take the following ratio,
v
C
(0.001)
v
C
(0.002)

18.394
6.7668
2.7183
v
C
(0)e
(0.001)/
v
C
(0)e
(0.002)/
e
0.001/
. Hence,
K = 18.394/6.7668
K =2.7183
tau = 1e-3/log(K)
tau =0.0010
C = 5e-6;
Req = tau/C
Req =200.0008
% Req = R*4e3/(R+4e3)
1st Order Circuit Probs 11/26/01 P8-3 R. A. DeCarlo, P. M. Lin
R = Req*4e3/(4e3-Req)
R =210.5272
vC0 = 6.7668/exp(-0.002/tau)
vC0= 49.9999 V
(b)
% v
C
(t) 50e
1000t
V
t = 0:tau/100:5*tau;
vc = vC0*exp(-t/tau);
plot(t,vc)
grid
xlabel('Time in ms')
ylabel('vC(t) in V')
SOLUTION 8.4. After one time constant the stored voltage, 8 V, decays to 8/e = 2.943 V. From the
graph, the time at which the output voltage is 2.94 V is approximately 0.19 s. Thus 0.19 s, and R =
/C = 190 .
SOLUTION 8.5. (a) The general form of the inductor current is i
L
(t) i
L
(0)e
t /
0.15e
t /
A where
L / R 2 10
3
s. Plotting i
L
(t) 0.15e
500t
A from 0 to 10 msec yields:
t = 0:.01e-3:10e-3;
iL = 0.15*exp(-t/2e-3);
plot(t,iL)
grid
1st Order Circuit Probs 11/26/01 P8-4 R. A. DeCarlo, P. M. Lin
xlabel('Time in s')
ylabel('Inductor Current')
0 0.001 0.002 0.003 0.004 0.005 0.006 0.007 0.008 0.009 0.01
0
0.02
0.04
0.06
0.08
0.1
0.12
0.14
0.16
Time in s
I
n
d
u
c
t
o
r

C
u
r
r
e
n
t
TextEnd
(b) Here v
L
(t) Ri
L
(t) 22.5e
500t
V.
0 0.001 0.002 0.003 0.004 0.005 0.006 0.007 0.008 0.009 0.01
-25
-20
-15
-10
-5
0
Time in s
I
n
d
u
c
t
o
r

V
o
l
t
a
g
e

i
n

V
TextEnd
(c) Using linearity, for i
L
(0) = 50 mA, then v
L
(t)
22.5
3
e
500t
7.5e
500t
V and for i
L
(0) = 250
mA, then v
L
(t) 22.5
250
150
e
500t
37.5e
500t
V.
SOLUTION 8.6. Since L / R, we can solve for L = 5 mH. Then solving i
L
(t) i
L
(0)e
t /
for i
L
(0)
at t = 4 sec yields i
L
(0) =15 mA.
1st Order Circuit Probs 11/26/01 P8-5 R. A. DeCarlo, P. M. Lin
SOLUTION 8.7. (a) We desire to solve i
L
(t) i
L
(0)e
t /
for i
L
(0) and R in 0.08 /(R +10
3
).
Using the following ratio,
i
L
(0.05 ms)
i
L
(0.15 ms)

9.197
1.2447

i
L
(0)e
(0.05m)/
i
L
(0)e
(0.15m)/
e
0.110
3
/
7.3889. Hence
K = 9.197/1.2447
K =7.3889
tau = 0.1e-3/log(K)
tau =5.0000e-05
L = 0.08;
Req = L/tau
Req =1.6000e+03
R = Req - 1000
R = 599.9862
iL0 = 9.197e-3/exp(-0.05e-3/tau)
iL0 = 0.0250 A
(b) 80m /(R +1000) 50 sec , i
L
(t) 0.025e
t /5010
6
A.
SOLUTION 8.8. By Ohms law, v
R
(0+) (4k ||16k)i
L
(0+) 32 V. The time constant
L /(4k ||6k) 25 sec , i.e.,
Req = (4e3*16e3/(4e3+16e3))
Req =3200
L = 0.08;
1st Order Circuit Probs 11/26/01 P8-6 R. A. DeCarlo, P. M. Lin
tau = L/Req
tau =2.5000e-05
Using the general equation, v
R
(t) v
R
(0+)e
t /
32e
t /25
V. Equivalently,
v
R
(t) R
eq
i
L
(t) R
eq
0.01e
410
4
t
V.
SOLUTION 8.9. (a) Note that the Thevenin resistance seen by the capacitor is Rth:
R1 = 360+60*120/(60+120)
R1 =400
Rth = 400*1200/1600
Rth =300
Hence, v
C
(t) v
C
(0)e
t /
80e
t /0.15
V where R
TH
C 300 0.5 10
3
0.15 s.
0 0.2 0.4 0.6 0.8 1
0
10
20
30
40
50
60
70
80
C
a
p
a
c
i
t
o
r

V
o
l
t
a
g
e

(
V
)
TextEnd
Time in s
(b) Here i
C
(0+) v
C
(0
+
) / R
TH
0.2667 . Therefore i
C
(t) i
C
(0+)e
t /
0.2667e
t /0.15
A.
Equivalently, i
C
(t)
v
C
(t)
R
th
0.2667e
t /0.15
A.
(c) By voltage division, v
R
(0+) v
C
(0)
60||120
(60||120) +160 + 200



_
,
8 V; for t > 0 v
R
(t) 8e
t /0.15
V.
1st Order Circuit Probs 11/26/01 P8-7 R. A. DeCarlo, P. M. Lin
SOLUTION 8.10. First, find the Thevenin equivalent seen at the left of the inductor. Introducing a test
source in place of the inductor we obtain the following KCL equation at that node.
i
test
v
test
/1k + v
test
200
v
test
1k


_
,



1
]
1
200 . Let vtest = 1 V. Then
itest = 1e-3 + (1 - 200*1e-3)/200
itest =0.0050
Rth = 1/itest
Rth =200
Thus R
th
200 , L / R
th
0.25 ms, and i
L
(t) 0.025e
4000t
A. Next from 8.13b find
v
L
(t) (R
TH
i
L
(0))e
4000t
5e
4000t
V, and from Ohms law i
x
(t) 5e
4000t
mA.
SOLUTION 8.11. For all parts it is necessary to find the Thevenin equivalent resistance seen by the
capacitor. To this end we apply an external test current to the remainder of the circuit to obtain:
v
test
R
1
i
test
+ R
2
(i
test
+ i
test
)
Thus
R
th
v
test
/ i
test
R
1
+ R
2
(1+ ) 120 + 70(1+ )
(a) With = 4, R
th
= 470 , R
th
C 0.1175 s, and v
C
(t) v
C
(0)e
t /
50e
8.51t
V.
0 0.1 0.2 0.3 0.4 0.5 0.6 0.7
0
10
20
30
40
50
Time in s
C
a
p
a
c
i
t
o
r

V
o
l
t
a
g
e

(
V
)
TextEnd
(b) With = -4, R
th
90 , R
th
C 0.0225 s, and v
C
(t) 50e
44.44t
V.
1st Order Circuit Probs 11/26/01 P8-8 R. A. DeCarlo, P. M. Lin
0 0.02 0.04 0.06 0.08 0.1 0.12
0
1000
2000
3000
4000
5000
6000
7000
8000
Time in s
C
a
p
a
c
i
t
o
r

V
o
l
t
a
g
e

(
V
)
TextEnd
Note how this is not a stable design as V increases exponentially without bound.
(c) From the general equation developed at the beginning, R
th
120 + 70(1+ ) > 0 requires that
> 2.7143.
SOLUTION 8.12. Find the Thevenin resistance left of the inductor. Forcing a test current source into
the output node,
v
test
R
1
i
test
+ R
2
i
test
R
1
i
test
( ) 100 + 50(1 100) ( )i
test
150 5000 ( )i
test
and
R
th
v
test
/ i
test
150 5000
(a) Using the above equation, R
th
350 and L / R
th
7.1429 10
-5
s. Hence,
i
L
(t) 0.1e
14000t
A, an unbounded response due to the presence of the negative equivalent resistance.
(b) R
th
100 , L / R
th
2.5 10
4
s, v
R
(0+) R
2
(1 R
1
)i
L
(0) 0 , but more importantly
v
R2
(t) R
2
(1 R
1
)i
L
(t) 0 i
L
(t) 0 .
(c) R
1
+ R
2
1 R
1
( ) 150 5000 ( ) > 0 implies < 0.03.
SOLUTION 8.13. Over a long period of time the inductor L, is seen as a short circuit. Thus at time 0
-
,
the current through the inductor is, by current division, I
s
/2. As such, i
L
(t) 0.5I
s
e
Rt /2L
A. A sketch
will reveal an exponentially decreasing current from an initial 0.5I
s
A.
1st Order Circuit Probs 11/26/01 P8-9 R. A. DeCarlo, P. M. Lin
SOLUTION 8.14. This is similar to problem 8.13. Here the current turns off at time zero instead of a
switch opening. By current division i
L
(0
+
) I
s
/2 . The difference between this problem and problem
8.13 is that the Thevenin resistance seen by the inductor is different. Here, R
TH
2R ||0.5R 0.4R. So
for t > 0, i
L
(I
s
/2)e
R
th
t /L
0.5I
s
e
Rt /2.5L
A. A sketch of this function plotted with respect to this
new time constant will be identical to the one in problem 8.13.
Define the time constant of problem 8.13 as
old
. The slower decay in the plot below represents
the fall of the inductor current for problem 8.14 relative to that of problem 8.13 which is the faster
decaying curve in the plot below.
1st Order Circuit Probs 11/26/01 P8-10 R. A. DeCarlo, P. M. Lin
SOLUTION 8.15. Over a long period of constant applied voltage, a capacitor looks like an open circuit.
By voltage division, v
C
(0
+
)
3
4
V
s
and = 3RC. Hence
v
C
(t) v
C
(0+)e
t /
0.75V
s
e
t /(3RC)
V
SOLUTION 8.16. Same problem as 8.15 for t < 0. For t > 0 the effective resistance changes.
R
th
3R / /R 0.75R and = 0.75RC. Thus, v
C
(t) 0.75V
s
e
t /(0.75RC)
V. Same behavior as in the
previous problem, except for a faster decay than in problem 8.15 due to a smaller effective resistance.
Note how the decreased resistance affects the RC and RL circuit differently.
SOLUTION 8.17. For t < 0 the inductor looks like a short circuit. Let R
1
1333/ /800 500 . The
current supplied by the source is I
s

12
100 + 500
0.02 A. By current division,
i
L
(0
+
) 0.02
1333
800 +1333
0.0125 A
For t > 0, the switch is opened and the inductor sees only the 800 resistor. Hence, L / R 25 sec
and
1st Order Circuit Probs 11/26/01 P8-11 R. A. DeCarlo, P. M. Lin
i
L
(t) 12.5e
40000t
mA
0 20 40 60 80 100
0
2
4
6
8
10
12
14
Time in micro seconds
I
n
d
u
c
t
o
r

c
u
r
r
e
n
t

i
n

m
A
TextEnd
SOLUTION 8.18. (a) For t < 0 the applied voltage is constant and at t = 0, the capacitor is like an open
circuit. By voltage division, v
C
(0
+
) 30
25k ||6.25k
(25k ||6.25k) +1k
25 V. For 0 < t < 1 ms, the source is off
and the capacitor discharges through three resistors in parallel; thus
R
th
C 833.33 0.6 10
6
0.5 ms and v
C
(t) 25e
2000t
V.
(b) From continuity v
C
(0.001) 25e
2000(0.001)
3.383. For t > 1ms, the capacitor keeps on
discharging through only one resistance, the 25 k resistor; thus the new time constant is
new
= 15 ms,
and v
C
(t) 3.383e
(t 0.001)/0.015
V.
(c) Plotv
C
(t) 25e
2000t
u(t) u(t 0.001) [ ] + 3.383e
(t 0.001)/0.015
u(t 0.001) V
t = 0:.01:12;
vc = 25*exp(-2*t) .*(u(t)-u(t-1))+3.3834*exp(-(t-1)/15) .*u(t-1);
plot(t,vc)
grid
xlabel('Time in milli-secs')
ylabel('Capacitor Voltage in V')
1st Order Circuit Probs 11/26/01 P8-12 R. A. DeCarlo, P. M. Lin
0 2 4 6 8 10 12
0
5
10
15
20
25
Time in milli-secs
C
a
p
a
c
i
t
o
r

V
o
l
t
a
g
e

i
n

V
TextEnd
SOLUTION 8.19. (a) From Ohms law i
L
(0
+
)
54
60 + 30
0.6 A. For t > 0, the Thevenin resistance
seen by the inductor is R
th
(60 + 30) || 720 80 and = L/R
th
= 1/160 s. Thus
i
L
(t) i
L
(0
+
)e
t /
0.6e
160t
A. From Ohms law and current division
v(t) 60
720
90 + 720
i
L
(t) 32e
160t
V
(b) From continuity property, i
L
(0.01t) 0.6
160(0.01)
121.14 mA. For t > 10 ms, the Thevenin
resistance seen by the inductor is R
th
(690 + 30) || 720 360 and
new
= L/R
th
= 1/720 s. Hence,
i
L
(t) 121.14e
720(t 0.01)
mA for t > 10 ms. From Ohms law and current division
v(t) 690
720
720 + 720
i
L
(t) 41.793e
720(t 0.01)
u(t 0.01) V
Therefore
i
L
(t) 0.6e
160t
u(t) u(t 0.01) [ ] + 0.12114e
720(t 0.01)
u(t 0.01) A
1st Order Circuit Probs 11/26/01 P8-13 R. A. DeCarlo, P. M. Lin
0 5 10 15
0
0.1
0.2
0.3
0.4
0.5
0.6
0.7
Time in milli-seconds
I
n
d
u
c
t
o
r

C
u
r
r
e
n
t

i
n

A
TextEnd
SOLUTION 8.20. For both circuits we first compute the Thevenin resistance seen to the right of the
capacitor for 0 t 60 ms. If we excite the circuit to the right of the capacitor over this time interval,
then i
test

v
test
200
+
(1 0.25)v
test
100
0.0125v
test
. Let R
th1

1
0.0125
80 .
(a) For t < 0, the capacitor acts as an open circuit. Using voltage division, v
C
(0
+
)
80
80 +133.3
80 30
V. For 0 t 60 ms, the time constant is
1
R
th1
C 40 ms, and
v
C
(t) v
C
(0
+
)e
t
30e
t (R
th1
C)
30e
25t
V
From continuity, v
C
(60
+
ms) 6.694 V. The new Thevenin resistance is R
th2
200 . Thus for t >
60 ms, the time constant is
2
200C 100 ms, and v
C
(t) 6.694e
10(t 0.06)
V. The resulting
capacitor voltage is plotted below.
1st Order Circuit Probs 11/26/01 P8-14 R. A. DeCarlo, P. M. Lin
0 50 100 150 200 250
0
5
10
15
20
25
30
time in ms
C
a
p
a
c
i
t
o
r

V
o
l
t
a
g
e

(
V
)
TextEnd
(b) It is the same circuit as above for t < 0; thus v
C
(0
+
) 30 V. However the Thevenin resistances seen
by the capacitor are different because there is no switch to disconnect the independent voltage source and
its series resistance. First for 0 t 60 ms, the Thevenin resistance to the right remains as
R
th1

1
0.0125
80 . However, for 0 t 60 ms, the Thevenin resistance seen by the capacitor
changes to R
th3
R
th1
//133.3 50 . Then new time constant is
3
R
th3
C 25 ms and for 0 t
60 ms
v
C
(t) 30e
40t
V
From continuity, v
C
(60
+
ms) 2.72 V. The new Thevenin resistance is R
th4
200/ /133.3 80 .
Thus for t > 60 ms, the time constant is
4
80C 40 ms, and v
C
(t) 2.72e
25(t 0.06)
V. The
resulting capacitor voltage is plotted below.
1st Order Circuit Probs 11/26/01 P8-15 R. A. DeCarlo, P. M. Lin
0 50 100 150 200 250
0
5
10
15
20
25
30
time in ms
C
a
p
a
c
i
t
o
r

V
o
l
t
a
g
e

(
V
)
TextEnd
(c) For t < 60 ms, the voltage decays faster in (b) due to the smaller time constant. Similarly, for t > 60
ms.
SOLUTION 8.21. Following, are the switching times with the time constants associated with them.
t 0 R
th
20 k 20 ms
t 5 ms R
th
4 k 4 ms
t 7.5 ms R
th
800 0.8 ms
It follows that with t in ms,
v
C
(t) 10e
0.050t
u(t) u(t 5) [ ] + 7.788e
0.250( t 5)
u(t 5) u(t 7.5) [ ] + 4.169e
1.25(t 7.5)
u(t 7.5) V
1st Order Circuit Probs 11/26/01 P8-16 R. A. DeCarlo, P. M. Lin
0 2 4 6 8 10 12
0
1
2
3
4
5
6
7
8
9
10
time in ms
C
a
p
a
c
i
t
o
r

V
o
l
t
a
g
e

(
V
)
TextEnd
*SOLUTION TO 8.22. This solution is done in MATLAB.
% Define switching times, inductance, and Thevenin equivalent resistances.
t1= 12e-6;
t2=18e-6;
t3=21e-6;
L= 0.1;
rth1= 800;
rth2= 8000;
rth3=1600;
rth4= 32000;
% Define time constants for each of the four time intervals.
tau1= L/rth1
tau2= L/rth2
tau3= L/rth3
tau4= L/rth4
tau1 = 1.2500e-04
tau2 = 1.2500e-05
tau3 = 6.2500e-05
tau4 = 3.1250e-06
% Compute initial inductor currents for each of the four time intervals.
il1= 100e-3;
il2=il1*exp(-t1/tau1)
1st Order Circuit Probs 11/26/01 P8-17 R. A. DeCarlo, P. M. Lin
il3=il2*exp(-(t2-t1)/tau2)
il4=il3*exp(-(t3-t2)/tau3)
il2 = 9.0846e-02
il3 = 5.6214e-02
il4 = 5.3580e-02
% Determine inductor currents for each of the four time intervals. Plot.
t = 0:0.5e-7:36e-6;
seg1= il1*exp(-t/tau1) .*(ustep(t)-ustep(t-t1));
seg2 =il2*exp(-(t-t1)/tau2) .*(ustep(t-t1)-ustep(t-t2));
seg3=il3*exp(-(t-t2)/tau3) .*(ustep(t-t2)-ustep(t-t3));
seg4= il4*exp(-(t-t3)/tau4) .* ustep(t-t3);
iL=seg1 + seg2 +seg3 + seg4;
plot(t,iL)
grid
SOLUTION 8.23.
For circuits with a forced voltage, equation 8.19c is used as a general solution,
v
C
(t) v
C
() + v
C
t
o
+
( )
v
C
( )
[ ]
e

t
R
TH
C



_
,

.
(a) At time zero the voltage is 0 V. As time approaches infinity, the capacitor looks like an open with
voltage 10 V. The Thevenin resistance is 10 k. Thus for t > 0.
1st Order Circuit Probs 11/26/01 P8-18 R. A. DeCarlo, P. M. Lin
v
C
(t) 10 + 10 [ ]e

t
2



_
,

10(1 e
0.5t
) V.
(b) With vin(t) = 0 and v
C
(0
+
) 5 V, v
C
(t) 5e

t
2



_
,

5e
0.5t
V.
Part (a) Part (b)
(c) From linearity, v
C
(t) 10(1 e
0.5t
) + 5e
0.5t
10 5e
0.5t
V. Using Ohms law,
i
C
(t)
v
in
(t)
10
4

v
C
(t)
10
4
. Thus i
C
(t) 1 1 0.5e
0.5t
( )
0.5e
0.5t
mA.
0 2 4 6 8 10
0
0.1
0.2
0.3
0.4
0.5
time in s
C
a
p
a
c
i
t
o
r

C
u
r
r
e
n
t

(
m
A
)
TextEnd
(d) This is the same as (a), under the condition that the input is 1.5 times larger. Hence by linearity,
v
C
(t) 1.5 10(1 e
0.5t
) 15(1 e
0.5t
) V.
(e) By linearity, ANSWER = 2x(ANSWER to (b)) + 3x(ANSWER to (a)):
1st Order Circuit Probs 11/26/01 P8-19 R. A. DeCarlo, P. M. Lin
v
C
(t) 2 5e
0.5t
+ 310(1 e
0.5t
) 15(1 e
0.5t
) 30 40e
0.5t
V
SOLUTION 8.24.
(a) At t = 0-, the capacitor looks like an open circuit; therefore, by voltage division and the continuity
property, v
C
(0

) v
C
(0
+
)
3R
4R
V
s1
0.75V
s1
. Similarly, at t = , v
C
()
3R
4R
V
s2
0.75V
s2
. The
circuit time constant is (3R / /R)C 0.75RC. Hence
v
C
(t) 0.75V
s2
+ 0.75 V
s1
V
s2
[ ]e

t
0.75RC



_
,

.
(b) A sketch will show an exponentially varying voltage from 0.75V
s1
converging to 0.75V
s2
with the
computed time constant.
(c) The response to the initial condition when the inputs are set to zero, zero-input response, is
v
C
(t) V
s1
e

t
0.75RC



_
,

. The zero order response, the response with 0V initial condition to a forced
voltage, is v
C
(t) V
s2
V
s2
e

t
0.75RC



_
,

.
SOLUTION 8.25.
For RL circuits with a forced current, equation 8.19b is used as a general solution:
i
L
(t) i
L
() + i
L
t
o
+
( )
i
L
( )
[ ]
e

R
th
L
(t t
o
)
.
Since R
th
= R = 100 and L = 0.4 H, we have 4 ms and
i
L
(t) i
L
() + i
L
t
o
+
( )
i
L
( )
[ ]
e
250(t t
o
)
(a) Here, i
L
(0) 0 and as time approach infinity, the inductor becomes a short and
i
L
() 10 100 0.1 A. Thus i
L
(t) 0.11 e
250t
( )
A.
(b) Here i
L
(0) 50 mA and because the input is zero, i
L
() 0 . Thus, i
L
(t) 0.05e
250t
A. Plots
for parts (a) and (b) appear below.
1st Order Circuit Probs 11/26/01 P8-20 R. A. DeCarlo, P. M. Lin
0 5 10 15 20
-0.06
-0.04
-0.02
0
0.02
0.04
0.06
0.08
0.1
Time in ms
I
n
d
u
c
t
o
r

C
u
r
r
e
n
t

(
A
)
TextEnd
(c) By linearity i
L
(t) 0.11 e
250t
( )
0.05e
250t
0.1 0.15e
250t
A. Further, by KVL and Ohm's
law, v
L
(t) v
in
(t) 100i
L
(t) implies v
L
(t) 10 10 +15e
250t
15e
250t
V.
0 5 10 15 20
0
5
10
15
Time in ms
I
n
d
u
c
t
o
r

V
o
l
t
a
g
e

(
V
)
TextEnd
(d) Observe that the new initial condition is 0.5 times the old one and that the new input voltage is 1.5
times the old one. Therefore, by linearity,
i
L
(t) 1.5 0.11 e
250t
( )
+ (0.5) 0.05e
250t
( )
0.15 0.125e
250t
and thus
v
L
(t) 15 15 +12.5e
250t
12.5e
250t
V
The plot is similar to part (c) with initial point 12.5 instead of 15.
SOLUTION 8.26. For this problem R
th
2R / /0.5R 0.4R in which case L R
th
L 0.4R.
1st Order Circuit Probs 11/26/01 P8-21 R. A. DeCarlo, P. M. Lin
(a) At t = 0, the inductor looks like a short circuit. Hence by current division, i
L
(0

) i
L
(0
+
) 0.5I
s1
.
A similar argument yields i
L
() 0.5I
s2
. Using the general form of the solution,
i
L
(t) 0.5I
s2
+ 0.5 I
s1
I
s2
[ ]e

0.4Rt
L
.
(b) A sketch will show an exponentially varying current from 0.5I
s1
A converging to 0.5I
s2
.
(c) The response to the initial condition when the inputs are set to zero, zero-input response, is
i
L
(t) 0.5I
s1
e

0.4Rt
L
. The zero state response, the response with no initial condition, to the input
I
s2
u(t), is i
L
(t) I
s2
1 e

0.4Rt
L




_
,


.
SOLUTION 8.27. For this problem, v
C
(0) 20 10e
0.4t
( )
t 0
10 V and
v
C
() 10I
s
20
t
lim
20 10e
0.4t
( )
. Hence I
s
2 A. Further,
i
C
(t) C
dv
C
(t)
dt
4Ce
0.4t
0.4e
0.4t
which implies that C = 0.1 F. Since
1/0.4 2.5 (10+ R)C 0.1(10+ R) , it follows that R = 15 .
SOLUTION 8.28.
(a) The Thevenin resistance for this configuration is R
th
1000/ /1000 500 and R
th
C 0.25 s.
Hence v
C
(t) v
C
(0
+
)e
t
15e
4t
V is the zero-input response.
(b) Using a source transformation and voltage division, v
C
() 3 V. Thus v
C
(t) 3 1 e
4t
( )
V.
(c) Here v
C
() 4 V, thus v
C
(t) 4 1 e
4t
( )
V.
(d) This is the superposition of parts (b) and (c), i.e., v
C
(t) 7 1 e
4t
( )
(e) The complete response is the superposition of parts (d) and (a), i.e., v
C
(t) 7 + 8e
4t
V.
(f) From linearity,
v
C
(t) 0.5 7 1 e
4t
( )
+ 2 15e
4t
3.5 + 26.5e
4t
SOLUTION 8.29. Solution done in MATLAB
%Problem 8.29
%RTH= (60||120)+120
%tau=L/RTH
RTH=1/(1/60+1/120)+120;
tau=0.2/RTH;
%Using superposition at t<0
il01=36/120/2;
il02=(72/180)*(60/180);
il0= il01+il02;
1st Order Circuit Probs 11/26/01 P8-22 R. A. DeCarlo, P. M. Lin
% At t>0 36volts is off thus
ilinf=il02;
t=0:5*tau/1000:5*tau;
ilt=ilinf+(il0-ilinf)*exp(-t/tau);
plot(t,1000*ilt);
xlabel('Time in seconds');
ylabel('Current in mA');
0 1 2 3 4 5 6 7
x 10
-3
120
140
160
180
200
220
240
260
280
300
Time in seconds
C
u
r
r
e
n
t

i
n

m
A
*SOLUTION 8.30.
% Rth = 120 + 120\\60 = 160 kohm
% tau = Rth*C
tau = 160e3*0.5e-6
tau =
0.0800
% Initial capacitor voltage is computed by
% voltage division and superposition
vc0 = 24+24
vc0 =
48
% At t = , capacitor looks like an open circuit. Hence
vcinf=24;
t = 0:1e-3:5*tau;
vct = vcinf+(vc0-vcinf)*exp(-t/tau);
plot(t*1000,vct)
grid
xlabel('Time in msec')
ylabel('Capacitor voltage (V)')
1st Order Circuit Probs 11/26/01 P8-23 R. A. DeCarlo, P. M. Lin
0 50 100 150 200 250 300 350 400
20
25
30
35
40
45
50
Time in msec
C
a
p
a
c
i
t
o
r

v
o
l
t
a
g
e

(
V
)
*SOLUTION 8.31A
vc0 =0;
% Consider 0 t 0.5
vcinf = 50;
% Rth = 600\\300 = 200 ohms
% tau1 = Rth*C
tau1 = 300e-6*200
vc0 = 0;
t = 0:5e-3:1;
vct = (vcinf+(vc0-vcinf)*exp(-t/tau1)) .* (ustep(t)-ustep(t-.5));

% Consider 0.5 t 1
tau2 = tau1
vc5 = (vcinf+(vc0-vcinf)*exp(-.5/tau1))
vcinf2 = 80;
vct2 = (vcinf2+(vc5-vcinf2)*exp(-(t-0.5)/tau1)) .* ustep(t-0.5);
vca = vct+vct2;
plot(t,vca)
grid
xlabel('Time in sec')
ylabel('Capacitor Voltage (V)')
1st Order Circuit Probs 11/26/01 P8-24 R. A. DeCarlo, P. M. Lin
0 0.2 0.4 0.6 0.8 1
0
10
20
30
40
50
60
70
80
Time in sec
C
a
p
a
c
i
t
o
r

V
o
l
t
a
g
e

(
V
)
*SOLUTION 8.31B
vc0 =0;
% for 0 t 0.5
vcinf = 75;
tau1 = 300e-6*300
vc0 = 0;
t = 0:5e-3:1;
vct = (vcinf+(vc0-vcinf)*exp(-t/tau1)) .* (ustep(t)-ustep(t-.5));
% for 0.5 t 1
tau2 = 300e-6*200
vc5 = (vcinf+(vc0-vcinf)*exp(-.5/tau1))
vcinf2 = 80;
vct2 = (vcinf2+(vc5-vcinf2)*exp(-(t-0.5)/tau1)) .* ustep(t-0.5);
vcb = vct+vct2;
plot(t,vcb)
grid
xlabel('Time in sec')
ylabel('Capacitor Voltage (V)')
pause
plot(t,vca,t,vcb,'b')
grid
1st Order Circuit Probs 11/26/01 P8-25 R. A. DeCarlo, P. M. Lin
0 0.2 0.4 0.6 0.8 1
0
10
20
30
40
50
60
70
80
Time in sec
C
a
p
a
c
i
t
o
r

V
o
l
t
a
g
e

(
V
)
Comparison of the two responses.
0 0.2 0.4 0.6 0.8 1
0
10
20
30
40
50
60
70
80
SOLUTION 8.32.
%Problem 8.32
%Consider t < 0
%vin=-20V, thus
vc0=(8/10)*(-20);
%For 0<t<20ms
RTH=1/(1/2e3+1/8e3);
tau1=5e-6*RTH;
1st Order Circuit Probs 11/26/01 P8-26 R. A. DeCarlo, P. M. Lin
vcinf=(8/10)*20;
t=20e-3;
vc20ms=vcinf+(vc0-vcinf)*exp(-t/tau1);
%For 20ms<t
RTH=1/(1/2e3+1/8e3+1/1.6e3);
tau2=5e-6*RTH;
%By superposition
vcinf2=20*0.5+20*0.4;
t=0:(40e-3)/1000:40e-3;
vct= (ustep(t)-ustep(t-20e-3)).*(vcinf+(vc0-vcinf).*exp(-t./tau1)) ...
+ ustep(t-20e-3).*(vcinf2+(vc20ms-vcinf2).*exp(-(t-0.02)./tau2));
plot(t,vct);
grid;
xlabel('time in seconds');
ylabel('Volts');
0 0.005 0.01 0.015 0.02 0.025 0.03 0.035 0.04
-20
-15
-10
-5
0
5
10
15
20
time in seconds
V
o
l
t
s
SOLUTION 8.33.
(a)
%Problem 8.33
%(a)
%at t<0 only one source is contributing thus,
il0=24/60*0.5;
%For t>0
RTH=60+1/(1/30+1/60);
tau=16e-3/RTH;
%As t goes to infinity, by superposition,
ilinf=24/(60)*0.5+24/80*1/3;
t=0:5*tau/1000:5*tau;
ilt=ilinf+(il0-ilinf).*exp(-t/tau);
figure(1);
plot(t,1000.*ilt);
grid;
xlabel('time in seconds');
ylabel('Current in mA');
1st Order Circuit Probs 11/26/01 P8-27 R. A. DeCarlo, P. M. Lin
0 0.1 0.2 0.3 0.4 0.5 0.6 0.7 0.8 0.9 1
x 10
-3
200
210
220
230
240
250
260
270
280
290
300
time in seconds
C
u
r
r
e
n
t

i
n

m
A
(b) Using 8.23, The time constant remains the same as in (a). In order to find the inductor voltage, we
must do so indirectly by solving for v
L
(t) V
A
60I
L
. At t=0
+
,
i
L
(0
+
) 200mA
V
A
24
60
90
+ 24
30
90
i
L
(0
+
)(60|| 30) 20V
v
L
(0
+
) x(t
0
+
) V
A
60i
L
(0
+
) 8V
For t > 0, X
e
v
L
() 0V , Thus v
L
(t) 8e
5000t
V .
(c) By linearity,
i
L
(t) 600 200e
5000t
mA
v
L
(t) 16e
5000t
V
V
A
SOLUTION 8.34.
(a) Since the voltage has been constant for a long time, the capacitor acts as an open circuit. Thus by
voltage division and continuity, v
C
(0

) v
C
(0
+
) 0.75V
o
.
(b) R
TH
6R ||18R || 3R 1.8R.
(c) For that period of time the switch is closed, v
C
(t) 0.9V
o
0.15V
o
e
t /1.8RC
.
(d) Using the previous equation, and by continuity, v
C
(T

) v
C
(T
+
) 0.9V
o
0.15V
o
e
T /1.8RC
.
(e) The time constant remains the same as the only difference is the source turning off.
(f) For t > T, v
C
(t) v
C
(T
+
)e
(t T )/1.8RC
.
(g)
1st Order Circuit Probs 11/26/01 P8-28 R. A. DeCarlo, P. M. Lin
0 2 4 6 8 10 12 14 16 18
0
0.1
0.2
0.3
0.4
0.5
0.6
0.7
0.8
0.9
Normalized time in terms of tau
N
o
r
m
a
l
i
z
e
d

V
o
l
t
s

i
n

t
e
r
m
s

o
f

V
o
SOLUTION 8.35.
(a) For t < 0, The switch is closed, the current source off, and the voltage source has been providing a
constant voltage for a prolonged period of time. Thus, v
C
(0

) 50uV .
(b) Since voltage in continuous across a capacitor, v
C
(0
+
) v
C
(0

) 50uV .
(c) The thevenin resistance seen by the capacitor is R
TH
200||200 100, thus R
TH
C 0.2s.
(d) As t goes to infinity the capacitor voltage goes to 16V, thus v
C
(t) 16 + (50u 16)e
5t
V .
(e) Again using the continuous property of a capacitor, v
C
(0.5
+
) v
C
(0.5

) 14.687V .
(f) For t > 0.5s, the switch is open, thus R
TH
200. R
TH
C 0.4s.
(g) As t goes to infinity, the capacitor voltage goes to 32V. v
C
(t) 32 + (14.687 32)e
2.5(t 0.5)
V .
(h)
0 0.5 1 1.5 2 2.5
-5
0
5
10
15
20
25
30
35
Time in seconds
V
o
l
t
s
SOLUTION 8.36.
(a) v
C
(0
+
) 5V
(b) Doing so in matlab.
%Problem 8.36b
%Initial condition
vc0=-5;
%From 0<t<80us
RTH=300e3;
tau=RTH*(1/3)*1e-9;
1st Order Circuit Probs 11/26/01 P8-29 R. A. DeCarlo, P. M. Lin
%So the initial condition on Vout
vout0=(10+5)/300e3*60e3-5; %-2V
voutinf1=10;
t=80e-6;
vout80us= 10+(vout0-10)*exp(-t/tau);
%For t > 80us
%tau stays the same
voutinf2=-5;
t=0:160e-6/1000:160e-6;
vout= (ustep(t)-ustep(t-80e-6)).*(10+(vout0-10).*exp(-t./tau)) ...
+ustep(t-80e-6).*(-5+(vout80us+5).*exp(-(t-80e-6)./tau));
plot(t,vout);
xlabel('time in seconds');
ylabel('Volts');
grid;
%Notice that Vout is vin-vct times a constant 60/300 plus vct
%Thus Vout=vin(60/300)+vct(1-60/300)
0 0.2 0.4 0.6 0.8 1 1.2 1.4 1.6
x 10
-4
-2
-1
0
1
2
3
4
5
time in seconds
V
o
l
t
s
(c)
1st Order Circuit Probs 11/26/01 P8-30 R. A. DeCarlo, P. M. Lin
(d)
SOLUTION 8.37.
1st Order Circuit Probs 11/26/01 P8-31 R. A. DeCarlo, P. M. Lin
If the source voltage has been 10 V for a long time then the switch is open and v
C
(0
+
) 10V . The
time constant with the switch open is 5us. At t > 0, the input voltage changes to 20V. It then follows
that v
C
() 20V , and
v
C
(t) 20 30e
t /5u
V .
Using the elapsed time formula, 8.24, we wish to find when the switch closes.
t
a
0 5us ln
10 20
0 20


_
,
2.03us. At that time the input voltage is still 20V, and the switch closes.
The time constant is now 98ns, and v
C
() 0.39V . Note that because the voltage converges to a
value greater than zero, this time interval will be from 2.03us to 5us when the input changes back to
10V, thus
v
C
(t) 0.39 0.39e
t /98n
V .
v
C
(5us) 0.39V . At t > 5us, the voltage changes to 10 V, so
v
C
(t) 0.2 + (0.39 + 0.2)e
t /98n
V .
Using the elapsed formula, we get t
b
0.1us for the voltage to go down to 0V again and cause the switch
to open again. At this point the time constant becomes the original value again and
v
C
(t) 10 +10e
t /5u
V
for t > (0.1+5) us.
SOLUTION 8.38.
(a) Introduce a test current source at the output and write KVL,
i
test
v
test
/200 + (v
test
6V) /200 + (6V v
test
) / 400. Solving for i
test
v
test
3
400


_
,
0.015 . This
implies the following,
R
TH
400 /3
i
SC
15mA
v
OC
R
TH
i
SC
2V
(b) Using the general form, v
C
(t) 2 8e
15000t
.
SOLUTION 8.39.
(a) Introduce a test voltage and solve for KVL,
v
test
5ki
test
+101(v
test
+1 40i
test
) 1 + 40i
test
v
test
10i
test
1
v
OC
1V
R
TH
10
(b) The complete response is v
C
(t) 1 + e
t
V. Note that the voltage goes to infinity as t goes to infinity
because of the negative time constant.
SOLUTION 8.40.
Compute the thevenin equivalent seen by the inductor at t > 0. Using KCL write,
i
test
100 i
test
+ (v
test
100i
test
+ 25) /50. Then one obtains the following,
1st Order Circuit Probs 11/26/01 P8-32 R. A. DeCarlo, P. M. Lin
i
test
v
test
/125 +1/ 5 A
R
TH
125
i
SC
1/ 5 A
v
OC
25V
A t < 0, the applied voltage has been 25V for a long time. Using the previously obtained thevenin
equivalent and linearity, i
L
(0

) 1/5 A. i
L
() i
SC
1/5 A, So i
L
(t) 0.2 + 0.4e
6250t
A.
SOLUTION 8.41.
(a) Introducing a test source and using KCL,
i
test

v
test
1.5v
s
40
+
v
test
100
0.2
v
s
40
+
1.5v
s
v
test
40
Solving for the test source current in terms of the test voltage, i
test

v
test
50
0.12 A. Thus the thevenin
equivalent is,
R
th
50
v
oc
i
sc
* R
th
6V
(b) v
C
(t) 6 6e
200t
V.
(c) From linearity all the currents increase by the same ratio, thus v
C
(t) 15 15e
200t
V.
(d)
(e)
1st Order Circuit Probs 11/26/01 P8-33 R. A. DeCarlo, P. M. Lin
SOLUTION 8.42.
%Problem 8.42
C= 1e-6;
vc0=0;
%For 0 < t < 5ms
Rth= 20e3;
tau1= Rth*C;
vcinf1= 50e-3*Rth;
vc5ms= vcinf1+(vc0-vcinf1)*exp(-5e-3/tau1);
%For 5ms < t < 7.5ms
Rth= 4e3;
tau2= Rth*C;
vcinf2= 50e-3*Rth;
vc75ms= vcinf2+(vc5ms-vcinf2)*exp(-(7.5e-3-5e-3)/tau2);
%For t > 7.5ms
Rth=800;
tau3= Rth*C;
vcinf3= 50e-3*Rth;
t=0:12e-3/1000:12e-3;
vct= (ustep(t)-ustep(t-5e-3)).*(vcinf1+(vc0-vcinf1).*exp(-t/tau1)) ...
+ (ustep(t-5e-3)-ustep(t-7.5e-3)).*(vcinf2+(vc5ms-vcinf2).*exp(-(t-5e-3)/tau2)) ...
+ (ustep(t-7.5e-3)).*(vcinf3+(vc75ms-vcinf3).*exp(-(t-7.5e-3)/tau3));
plot(t,vct);
grid;
1st Order Circuit Probs 11/26/01 P8-34 R. A. DeCarlo, P. M. Lin
xlabel('time in seconds');
ylabel('Volts');
0 0.002 0.004 0.006 0.008 0.01 0.012
0
50
100
150
200
250
time in seconds
V
o
l
t
s
SOLUTION 8.43.
(a) From the thevenin resistance R
TH
800,
1
125 s.
(b) From the thevenin resistance R
TH
8k,
2
12.5 s.
(c) From the thevenin resistance R
TH
1.6k,
3
62.5 s.
(d) From the thevenin resistance R
TH
32k,
4
3.125 s.
(e) 0 mA.
(f) In MATLAB:
%Problem 8.43f
tau1= 125e-6;
tau2= 12.5e-6;
tau3= 62.5e-6;
tau4= 3.125e-6;
vs= 100;
il0= 0;
ilinf1= vs/800;
ilinf2= vs/8e3;
ilinf3= vs/1.6e3;
ilinf4= vs/32e3;
il12us= ilinf1+(il0-ilinf1)*exp(-12e-6/tau1);
il18us= ilinf2+(il12us-ilinf2)*exp(-(18e-6-12e-6)/tau2);
il21us= ilinf3+(il18us-ilinf3)*exp(-(21e-6-18e-6)/tau3);
t= 0:36e-6/1000:36e-6;
ilt= (ustep(t)-ustep(t-12e-6)).*(ilinf1+(il0-ilinf1).*exp(-t/tau1))+ ...
(ustep(t-12e-6)-ustep(t-18e-6)).*(ilinf2+(il12us-ilinf2).*exp(-(t-12e-6)/tau2))+ ...
(ustep(t-18e-6)-ustep(t-21e-6)).*(ilinf3+(il18us-ilinf3).*exp(-(t-18e-6)/tau3))+ ...
(ustep(t-21e-6)).*(ilinf4+(il21us-ilinf4).*exp(-(t-21e-6)/tau4));
plot(t,1000*ilt);
grid;
xlabel('time in seconds');
ylabel('Current in mA');
1st Order Circuit Probs 11/26/01 P8-35 R. A. DeCarlo, P. M. Lin
0 0.5 1 1.5 2 2.5 3 3.5 4
x 10
-5
0
5
10
15
time in seconds
C
u
r
r
e
n
t

in

m
A
SOLUTION 8.44.
The first stage is a differentiator, and from 8.25, the output of the first op-amp is
RC
dv
in
(t)
dt
0.25RCe
0.25t
. The second stage is an integrator and using 8.26,
v
out
(t)
1
2RC
(0.25RCe
0.25
d )
0
t

0.5 e
0.25
[ ]
0
t
0.5e
0.25t
SOLUTION 8.45.
(a) First note the following relationships,
v
out
(t) v
C2
(t)
1
C
2
i
C2
( )d
0
t

v
in
(t) v
C1
(t)
i
C2
(t) v
in
(t) / R C
1
dv
C1
(t)
dt

v
in
(t)
R
C
1
dv
in
(t)
dt
Doing the appropriate substitution, and solving,
v
out
(t)
1
C
2
R
v
in
( )d
0
t

C
1
C
2
v
in
(t) +
C
1
C
2
v
in
(0).
(b)
4
C
2
R
e
0.25t
1
( )
+
C
1
C
2
1 e
0.25t
( )
V.
(c)
1
C
2
R
sin( t) +
C
1
C
2
1 cos( t) ( ) V.
SOLUTION 8.46.
These are two integrator in cascade. Using 8.26, the output of the first stage is

1
RC
v
in
( )d 2 cos(50 ) [ ]
0
t
mV 2cos(50t) 2
0
t

mV. Using the same equation again,


v
out
(t)
1
RC
(2cos(50 )d 10
2
50
sin(50 )



1
]
1
0
t
mV 2cos(50t)mV
0
t

1st Order Circuit Probs 11/26/01 P8-36 R. A. DeCarlo, P. M. Lin


*SOLUTION 8.47. The following solution is done in MATLAB
c= 1e-6;
rf= 10e6;
rs=1e6;
tau=c*rf;
vgain= -10/1;
vofinal= -3*vgain;
% Part (a)
voinit= 0;
t= 0: tau/100: tau;
vout= vofinal + (voinit -vofinal).*exp(-t/tau);
% Time at which output voltage reaches saturation is tsat
tsat= tau*log((0 - vofinal)/(15- vofinal))
plot (t,vout)
grid
xlabel('Time in secs')
ylabel('Output Voltage in volts')
tsat =
6.9315e+00
% Part (b)
voinit=-5;
vout= vofinal + (voinit -vofinal).*exp(-t/tau);
tsat= tau*log((-5 - vofinal)/(15- vofinal))
plot (t,vout)
grid
xlabel('Time in secs')
1st Order Circuit Probs 11/26/01 P8-37 R. A. DeCarlo, P. M. Lin
ylabel('Output Voltage in volts')
tsat =
8.4730e+00
(c) v
C
(0
-
) = 0. Observe saturation at about 6.4 seconds.
V(IVM)
Time (s) Leaky Integrator-Transient-7
(V)
+0.000e+000
+5.000
+10.000
+15.000
+0.000e+000 +2.000 +4.000 +6.000 +8.000 +10.000
v
C
(0
-
) = -5 V. Observe that the time of saturation is 8.198 seconds.
1st Order Circuit Probs 11/26/01 P8-38 R. A. DeCarlo, P. M. Lin
V(IVM)
Time (s) Prb Sol 8.47-Transient-16
(V)
-5.000
+0.000e+000
+5.000
+10.000
+15.000
+0.000e+000 +2.000 +4.000 +6.000 +8.000 +10.000
Note that in both cases the time of saturation is much lower than in the MATLAB computations which
assume an ideal op amp. In the Burr Brown model used by the SPICE simulation of this circuit, the
input resistance is 2 M which is comparable with the external input resistance. Hence the assumption
of an infinite input resistance is not valid for the SPICE simulation and causes the discrepancy in the
time of saturation. However, if the external input resistance is changed to 10 k and the feedback
resistance to 100 k with a corresponding change in the capacitor to 100 F, one obtains results
comparable to the MATLAB computations.
SOLUTION 8.48.
Since the op-amps do not load the first stage of the circuit, we can find its transfer function for the op-
amp stage as
v
out
(t)
v
+
(t)
(1+ K) .
(a) From the problem statement, we know that the overall function, is a scaled integrator. As the op-amp
stage only provides gain it is logical to assume that the R-C stage will perform the integration of the input
times some constant, G. With this in mind we have
v
out
(t) G(1+ K) 10sin( )d
10G(1+ K)
cos( t) 1 ( )
0
t

, where G must be negative.


(b) Using the same reasoning, v
out
(t) G(1+ K)
d(10sin( t))
dt
10G(1+ K) cos( t)where G is
positive.
For low frequency (a) yields a big output, while (b) a small one. For high frequency the reverse happens.
1st Order Circuit Probs 11/26/01 P8-39 R. A. DeCarlo, P. M. Lin
SOLUTION 8.49.
For (b) the integral i-v relationship is
v(t) v
Ceq
(0
+
) +
1
C
eq
i(
0
t

)d v
C1
(0
+
) + v
C2
(0
+
) +
C
1
+ C
2
C
1
C
2
i( )d
0
t

. Repeating the same for (a),


v
C1
(t) v
C1
(0
+
) +
1
C
1
i( )d
0
t

v
C2
(t) v
C2
(0
+
) +
1
C
2
i( )d
0
t

By KVL the two capacitor voltage can be added together, thus give the same relationship as for (b).
SOLUTION 8.50.
First calculate C
eq
C
1
|| C
2
0.2F . Then find the initial voltage
v
Ceq
(0
+
) v
C1
(0
+
) + v
C2
(0
+
) 30V . The final voltage will be 12V, and the time constant is
R
eq
C
eq
0.4s. Thus v
out
(t) 12 + (30 12)e
2.5t
V .
*SOLUTION 8.51. (a) After the switch closes, we have the circuit shown below.
From Chapter 7, Ceq = 0.5 F and v
C
(0
+
) v
C1
(0
+
) v
C2
(0
+
) 2 0 2 V. Hence,
i
R
(t) i
R
(0
+
)e
t /
i
R
(0
+
)e
t /RC
eq

v
C
(0
+
)
R
e
t /RC
eq
4e
4t
u(t) A
(b) For this part we apply the integral definition of the capacitor. Specifically,
1st Order Circuit Probs 11/26/01 P8-40 R. A. DeCarlo, P. M. Lin
v
C1
(t) v
C1
(0
+
) +
1
C
1
i
C1
( ) d
0
t

2 i
R
( ) d
0
t

2 4 e
4
d
0
t

1+ e
4t
V
and
v
C2
(t) v
C2
(0
+
) +
1
C
2
i
C2
( ) d
0
t

0 + i
R
( )d
0
t

4 e
4
d
0
t

1 e
4t
V
(c) The energy stored at t = 0
+
for each capacitor is:
W
C1
(0
+
) 0.5C
1
v
C1
2
(0
+
) 2 J
and
W
C2
(0
+
) 0.5C
2
v
C2
2
(0
+
) 0 J
Further at t = ,
W
C1
() 0.5C
1
v
C1
2
() 0.5 J
and
W
C2
() 0.5C
2
v
C2
2
() 0.5 J
Computing total instantaneous stored energies, we have
W
Ctot
(0
+
) 2 J and W
Ctot
() 1 J
Hence the decrease in stored energy from 0
+
to is 1 J.
(d) Computing the energy dissipated in the resistor over [0
+
, ) is
W
R
(0,) R i
R
2
( )d
0

R
v
C
2
(0
+
)
R
2



_
,

e
2t /RC
eq
d
0

v
C
2
(0
+
)
R

RC
eq
2
e
2t /RC
eq
]
0

C
eq
v
C
2
(0
+
)
2
1 J
(e) From the expressions developed in part (d), the dissipated energy is independent of the value of R.
R only affects the rate at which energy is dissipated. Clearly, the energy stored at 0 is 2 J while
the energy dissipated over [0,) is 1 J and the remaining energy at t = is 1 J. Hence
conservation of energy is verified.
1st Order Circuit Probs 11/26/01 P8-41 R. A. DeCarlo, P. M. Lin
SOLUTION 8.52. (a) Using the relations developed in P8.49,
C
eq
0.2F
v
Ceq
1 0.5 0.5V
i
R
(t) i
R
(0
+
)e
t /
i
R
(0
+
)e
t /RC
eq

v
Ceq
(0
+
)
R
e
t /RC
eq
e
10t
u(t)
(b) For this part we apply the integral definition of the capacitor. Specifically,
v
C1
(t) v
C1
(0
+
) +
1
C
1
i
C1
( ) d
0
t

1 i
R
( ) d
0
t

1 e
10
d
0
t

1
10
(9 + e
10t
) V
and
v
C2
(t) v
C2
(0
+
) +
1
C
2
i
C2
( ) d
0
t

0.5 + 4 i
R
( )d
0
t

0.5 + 4 e
10
d
0
t

1
10
(9 4e
10t
) V
(c) The energy stored at t = 0
+
for each capacitor is:
W
C1
(0
+
) 0.5C
1
v
C1
2
(0
+
) 0.5 J
and
W
C2
(0
+
) 0.5C
2
v
C2
2
(0
+
) 31.25mJ
Further at t = ,
W
C1
() 0.5C
1
v
C1
2
() 405mJ
and
W
C2
() 0.5C
2
v
C2
2
() 101.25mJ
Computing total instantaneous stored energies, we have
W
Ctot
(0
+
) 531.25mJ and W
Ctot
() 506.25mJ
Hence the decrease in stored energy from 0
+
to is 25 mJ.
(d) Computing the energy dissipated in the resistor over [0
+
, ) is
1st Order Circuit Probs 11/26/01 P8-42 R. A. DeCarlo, P. M. Lin
W
R
(0,) R i
R
2
( )d
0

R
v
Ceq
2
(0
+
)
R
2




_
,


e
2t /RC
eq
d
0

v
Ceq
2
(0
+
)
R

RC
eq
2
e
2t /RC
eq
]
0

C
eq
v
Ceq
2
(0
+
)
2
25mJ
(e) From the expressions developed in part (d), the dissipated energy is independent of the value of R.
R only affects the rate at which energy is dissipated. Clearly, the energy stored at 0 is 531.25 mJ
while the energy dissipated over [0,) is 506.25 mJ and the remaining energy at t = is 25 mJ.
Hence conservation of energy is verified.
SOLUTION 8.53. As all the switches are open initially, the initial current through the inductors is 0A.
For 0 < t < 50 ms, i
L
(t) 54.54 54.54e
20000t
mV. At t > 50 ms, the equivalent inductance is 10 mH,
the initial current through the 110 mH inductance is 54.54 mA, and through the 11 mH inductance 0 A.
So assuming the current splits equally between the two branches in steady state,
i
L1
27.27 + (54.54 27.27)e
220000t
i
L2
27.27 27.27e
220000t
SOLUTION 8.54. (a) Charges will distribute in order to achieve equal voltage by KVL. Since q=CV,
v
R
(0

) 0V , due to equal capacitance the charges will distribute half and half, v
R
(0
+
) 0.5V .
(b) The equivalent capacitance is 2 F, thus v
R
(t) 0.5e
0.5t
.
*SOLUTION 8.55. (a) Writing a node equation at v we have for all t,
4
dv
dt
+
v
4
+ 4
d
dt
v v
s
( ) +
v v
s
( )
2
0 (*)
Equivalently,
8
dv
dt

3v
4
+ 4
dv
s
dt
+
v
s
2
(**)
Grouping terms and dividing by 8 yields when t > 0,
dv
dt

3
32
v +
1
16
(***)
1st Order Circuit Probs 11/26/01 P8-43 R. A. DeCarlo, P. M. Lin
Notice that v
s
= 1 for t > 0 and for t > 0,
dv
s
(t)
dt

du(t)
dt
0.
(b) By inspection v(0
-
) = 0; both capacitors are uncharged at 0
-
. Recall from part (a) that KCL at the
node for v yields (*) which is equivalent to (**). Since conservation of charge follows by
integrating (*) or equivalently integrating (**) we have
8
dv
d
0

0
+

d
3
4
v
0

0
+

d + 4
dv
s
d
0

0
+

d + 0.5 v
s
0

0
+

d
Since the integral of a finite integrand over an infinitesimal interval is zero, we have equivalently,
8
dv
d
0

0
+

d 0 + 4
dv
s
d
0

0
+

d + 0
Evaluating these integrals we obtain
8 v(0
+
) v(0

)
( )
4 v
s
(0
+
) v
s
(0

)
( )
4u(0
+
) 4
(c) Since v(0
-
) = 0, v(0
+
) = 0.5 V. Since v satisfies (***), i.e.,
dv
dt

3
32
v +
1
16

1
v + F (***)
from equation 8.17,
v(t) F + v(0
+
) F
[ ]
e
t /

2
3

1
6
e
3t /32

_
,
u(t) V
Using v(0
-
) would have led to an incorrect answer.
SOLUTION 8.56. (a)
x(t) K
1
e
t /
+ K
2
dx(t)
dt

K
1
e
t /
doing the substitution,
K
1
e
t /

1
(K
1
e
t /
+ K
2
) + F . In order to satisfy the equality, K
2
F .
1st Order Circuit Probs 11/26/01 P8-44 R. A. DeCarlo, P. M. Lin
(b) x(t
0
+
) K
1
e
t
0
/
+ F , and K
1
x(t
0
+
) F
[ ]
e
t
0
/
.
(c) x(t) K
1
e
t /
+ K
2
x(t
0
+
) F
[ ]
e
(t t
0
)/
+ F .
SOLUTION 8.57. (a) From the graph, the initial and final values are 0 and 80 V respectively. That sets
the following constraint, 100
R
2
R
1
+ R
2
80V . From v
C
( ) 80 80e
1
50.57V . Looking at the Graph
5ms . Thus R
1
|| R
2
C 5ms . Solving, R
1
6250 and R
2
25k.
(b) Using the same equalities, R
1
0.25R
2
2k, and C
2k || 8K
3.125uF .
SOLUTION 8.58. (a) From the graph, the initial and final values are 0 and 100 mA respectively. Thus
R
1
200 /100m 2k. From i
L
( ) 100 100e
1
mA 63.21mA, the graph shows a 20ms . Thus
L /(R
1
|| R
2
) 20ms, and R
2
0.25m.
(b) R1 stays the same, L 20ms(R
1
|| R
2
) 20H.
SOLUTION 8.59. This question is done in matlab
%Problem 8.59
tau1= 20*1;
tau2= 1;
%For 0 < t < ta
vo= 0;
vinfa= 10;
%Using the elapsed time formula,
ta=tau1*log((0-10)/(9-10));
%For ta < t < tb
vinfb=0;
tb=tau2*log((9-0)/(1-0));
%For tb < t < tc
vinfc=vinfa;
tc=tau1*log((1-10)/(9-10));
1st Order Circuit Probs 11/26/01 P8-45 R. A. DeCarlo, P. M. Lin
%Next switching is just a repeat of ta < t < tb
t1=ta;
t2=ta+tb;
t3=t2+tc;
t4=t3+tb;
t5=t4+tc;
t=0:t5/1000:t5-1/1000;
vt= (ustep(t)-ustep(t-t1)).*(10-10.*exp(-t/tau1))+ ...
(ustep(t-t1)-ustep(t-t2)).*(9.*exp(-(t-t1)/tau2))+ ...
(ustep(t-t2)-ustep(t-t3)).*(10-9.*exp(-(t-t2)/tau1))+ ...
(ustep(t-t3)-ustep(t-t4)).*(9.*exp(-(t-t3)/tau2))+ ...
(ustep(t-t4)-ustep(t-t5)).*(10-9.*exp(-(t-t4)/tau1));
Frequency= 1/(tb+tc)
plot(t,vt);
grid;
xlabel('time in seconds');
ylabel('Volts');
(b) The frequency is 0.0217 Hz.
0 20 40 60 80 100 120 140
0
1
2
3
4
5
6
7
8
9
time in seconds
V
o
l
t
s
SOLUTION 8.60. When the switch is in position A, 18.18ms . In position B it is 99.5us. Using
the elapsed time formula, find ta, when the output voltage reaches 90 V.
1st Order Circuit Probs 11/26/01 P8-46 R. A. DeCarlo, P. M. Lin
t
a
18.18msln
60 136.36
90 136.36


_
,
9.07ms . At this point the switch goes to B, and the elapsed time until the
voltage reaches 60 V is t
b
99.5usln
90 59.45
60 59.45


_
,
0.4ms . Adding both time, F 105.6Hz .
SOLUTION 8.61. (a) The circuit can be rearranged in a series of one V
solar
V voltage source, one L
store
inductor, and one R
store
+R
solar
resistor.
(b) i
L
(t)
V
solar
R
solar
+ R
store
1 e
(Rstore+Rsolar )t /L
[ ]
A.
(c) In this time period the circuit reduces to an L
sotre
inductor in series with a R
store
+R
1
resistor.
(d)
i
L
(T
1

)
V
solar
R
solar
+ R
store
i
L
(t) i
T
1
e
(Rstore+R1)(t T1)/L
A
.
(e) The two elements in series are an L
store
inductor and a resistor R
eq
R
store
+ (R
1
|| R
2
).
(f)
i
L
(T
2

) i
T
1
e
(Rstore+R1)(T
2
T1)/ L
sotre
A
i
L
(t) i
T
2
e
(Re q)(t T 2)/ Lstore
A
(g)
P
Vsolar
V
solar
i
L
(t)
V
2
solar
R
solar
+ R
store
1 e
(Rstore+Rsolar)t /L
[ ]
W
P
Rsolar
R
solar
i
L
2
(t)W
P
Rstore
R
store
i
L
2
(t)W
P
Lstore
L
store
i
L
(t)
di
L
(t)
dt
W
(h) W
L
(0,t)
1
2
Li
L
2
(t)J
SOLUTION 8.62. The light turns off when the current through it goes down to 0.5 mA. This
corresponds to i
b
10uA, and a voltage across the capacitor of v
C
i
b
(R
1
+ 2k) + 0.5 0.7V . The time
constant of this circuit is (R
1
+ 2k) || 5k 1000uF 4s. The final voltage across the capacitor is by
voltage division, 0.1V. Thus using the elapsed time formula t
1
4ln
1.5 0.1
0.7 0.1


_
,
3.39s.
1st Order Circuit Probs 11/26/01 P8-47 R. A. DeCarlo, P. M. Lin
SOLUTION 8.63. (a) Since RC = 10
-3
s, from equation 8.25, v
a
(t) RC
dv
s
(t)
dt
cos(1000t) V.
Hence v
b
(t) RC
dv
a
(t)
dt
sin(1000t) V, and v
out
(t)
R
f
R
v
b
(t) sin(1000t) v
s
(t) V.
(b) With the switch moved to position B, there is no source in the circuit. But the output at the
switching instant is sin(1000t) V which coincides with vs(t). Hence, the input to the first amp remains
the same and the circuit continues to produce v
out
(t) sin(1000t) V, i.e., the circuit becomes an
oscillator.
*SOLUTION 8.64. Before attacking the problem proper, consider driving an ideal unity gain integrator
with the square wave of figure P8.64b. If we start the integration when the square wave goes positive,
then we have a triangular waveform as follows:
On the other hand, if we start the integration when the square wave goes negative, we get the following
waveform
One concludes that without some further physical assumptions, there is no unique solution to this
problem.
Physically speaking all capacitors have a leakage resistance. Hence, in modeling the capacitor we
put a very large resistance in parallel with an ideal C, producing a nearly ideal leaky integrator circuit.
The leaky integrator circuit has a first order response. Hence over time, when the circuit reaches steady
state, the dc level of the resulting output waveform will be proportional to the average level of
1st Order Circuit Probs 11/26/01 P8-48 R. A. DeCarlo, P. M. Lin
the square wave which is zero in this case.
Comments: Actually the proportionality constant in the above statement is the overall dc gain of the
integrator-inverter. See the formulas given in P22.16. Adding up the two formulas, we have
Output(t)
max
+ Output(t)
min
= K (V
max
+ V
min
)
and K = H(0), i.e., the dc gain of the first order low pass system. This leads to
Average of output = (dc gain) (average of input levels)
See the analysis in example 8.7 and later an exact analysis is given in problems 22.15 and 22.16. In
other words, one would expect that the output of our (leaky) integrator in steady state to be given by the
waveform below.
Now we can start to solve the problem. The first part is to design a (leaky) integrator circuit to
produce a triangular waveform of value 2 V peak-to-peak. For this we consider the following figure
which consists of the leaky integrator followed by an inverter.
1st Order Circuit Probs 11/26/01 P8-49 R. A. DeCarlo, P. M. Lin
To handle this analysis, recall that i q/t in which case q = Cv. Hence, to have a peak-to-peak
voltage at v
2
(t) and v
3
(t) of 2 V, we require that
v
i
in
t
C

9
R
1

50 10
6
C
2
Hence R
1
C = 2.2510-4. If we choose R
1
= 10 k, then C = 22.5 nF. At this point the waveform of
v
3
(t) is given below.
In order to complete the design, we must raise the portion of the curve with positive slope by 1 V and
lower the portion with negative slope by 1 V. This can be done by adding one-ninth of v
in
(t) to v
3
(t).
This can be done by using the following circuit. In this circuit, there is a voltage-divider at the non-
1st Order Circuit Probs 11/26/01 P8-50 R. A. DeCarlo, P. M. Lin
inverting terminal of the second op amp. Here V
+
equals one-eighteenth of v
in
(t). However the gain of
the non-inverting portion is 2; therefore one-ninth of the input is added to v
3
(t) as desired.
CHAPTER 9 PROBLEM SOLUTI ONS
SOLUTI ON TO PROBLEM 9.1. If we can compute expressions for K and q that are real,
then these quantities exist by construction. Consider that A, B, K and q must satisfy the
following relationship:
Kcos(t + ) Kcos() ( )cos(t) + Ksin() ( )sin(t) Acos(t) + Bsin(t)
Therefore Kcos() A and Ksin() B. Consequently,
Kcos() ( )
2
+ Ksin() ( )
2
K
2
A
2
+ B
2
in which case K A
2
+ B
2
. Further,
Ksin()
Kcos()
tan()
B
A
in which case
tan
1
B
A


_
,
with due regard to quadrant.
SOLUTI ON TO PROBLEM 9.2. For the inductor,
W
L
(t)
1
2
L i
L
2
(t)
1
2
L V
o
C
L
sin
1
LC
t



_
,




1
]
1
2

CV
0
2
2
sin
2
1
LC
t



_
,

and for the capacitor,
W
C
(t)
1
2
C v
C
2
(t)
1
2
C V
o
cos
1
LC
t



_
,




1
]
1
2

CV
0
2
2
cos
2
1
LC
t



_
,
.
Hence,
W
C
+ W
L

1
2
C v
C
2
(t) +
1
2
L i
L
2
(t)
CV
0
2
2
sin
2
1
LC
t



_
,
+ cos
2
1
LC
t



_
,




1
]
1

CV
0
2
2
SOLUTI ON TO PROBLEM 9.3. Since x(t) (K
1
+ K
2
t)e
t
,
x' (t) K
1
e
t
+ K
2
e
t
t K
2
e
t
and
x' ' ( t)
2
K
1
e
t
K
2
e
t
K
2
e
t
+
2
t K
2
e
t
Substituting into the differential equation, we have

2
K
1
e
t
2 K
2
e
t
+
2
t K
2
e
t
+ 2 K
1
e
t
+ K
2
e
t
t K
2
e
t
[ ]
+
2
K
1
e
t
+ K
2
te
t
[ ]
0
This means that the solution form satisfies the differential equation.
SOLUTI ON TO PROBLEM 9.4.
(a) Suppose x(T) 0 at some T. Then K
1
e
s
1
T
K
2
e
s
2
T
. Since e
s
i
T
> 0 whenever s
i
is real and T is finite, K
1
& K
2
must have opposite signs.
(b) For this we solve for T and show there can only be one solution. Since
K
1
e
s
1
T
K
2
e
s
2
T
and e
s
i
T
> 0,
K
1
K
2

e
s
2
T
e
s
1
T
implies

ln
K
1
K
2



_
,

ln
e
s
2
T
e
s
1
T



_
,

s
2
s
1
( )T
Hence the unique solution is given by

T ln
K
1
K
2



_
,

s
2
s
1
( )
provided s
2
s
1
which is the case for distinct roots.
SOLUTI ON TO PROBLEM 9.5. Suppose x(T) 0 at some T >0. This is true if and only if
K
1
e
s
1
T
K
2
Te
s
1
T
(*)
Since e
s
1
T
> 0 and T >0, (*) is true if and only if K
1
K
2
T which is true if and only if
K
1
& K
2
have opposite signs.
SOLUTI ON 9.6. (a) Denote one period of oscillation by T. Then by definition
9950T 2. Hence, T 0.63148 ms. The time constant of decay is 1 ms. Therefore,
NT N 0 . 6 3 1 4 8 =1 . Hence N 1.5836 cycles.
(b) Here observe that the time constant of decay is 1/ s. Hence NT N
2

d
=
1

. One
concludes that N =

d
2
.
SOLUTI ON 9.7. The differential equation for the capacitor voltage is
d
2
v
C
(t)
dt
2
+
1
LC
v
C
(t)
1
LC
V
s
For t >0, the characteristic equation is s
2
+
1
LC
0. Hence from table 9.2, the solution
for either the inductor current or capacitor voltage has the general form
v
C
(t) Acos(
d
t) + Bsin(
d
t) + X
F
where
d

1
LC
. From table 9.2, X
F

V
s
LC


_
,
1
LC


_
,
V
s
. Further,
v
C
(0
+
) A + X
F
A+ V
s
0
Hence A V
s
. Also,
v
C
'
(0
+
)
d
B
1
LC
B
i
L
(0
+
)
C
0
Hence B 0. Therefore
v
C
(t) V
s
cos(
d
t) +V
s
V
s
1 cos
t
LC



_
,




_
,

V
To obtain the expression for iL(t) (=iC(t)), we can either repeat the above derivation or
differentiate and multiply by C. We choose the latter. Therefore
i
L
(t)
CV
s
LC
sin
t
LC



_
,

V
s
L
C
sin
t
LC



_
,
A
SOLUTI ON TO PROBLEM 9.8. Essentially this is example 9.7, case 1, with literals and R =
. Clearly the circuit is a driven parallel LC circuit having characteristic equation
s
2
+
1
LC
s +
j
LC



_
,
s
j
LC



_
,
s + j
d
( ) s j
d
( ) 0
Thus we obtain
i
L
(t) Acos(
d
t) + Bsin(
d
t) + X
F
Acos
1
LC
t



_
,
+Bsin
1
LC
t



_
,
+X
F
Here X
F
I
S
is the value of the current when the inductor is shorted and the capacitor is
open. Applying the initial conditions,
i
L
(0+) A + I
s
0A I
s
Further
i
L
'
(0+)
v
L
(0+)
L

v
C
(0+)
L
0
1
LC
B
because capacitor voltage is continuous and because
i
L
'
(0+)
d
dt
Acos
1
LC
t



_
,
+ Bsin
1
LC
t



_
,
+ X
F



1
]
1
t 0

B
LC
Hence B 0. Therefore,
i
L
(t) I
s
1 cos(
1
LC
t)



1
]
1
Rather than repeat the above derivation,
v
C
(t) v
L
(t) L
di
L
(t)
dt
LI
s
d
dt
1 cos
1
LC
t



_
,




1
]
1
I
s
L
C
sin
1
LC
t



_
,

SOLUTI ON TO PROBLEM 9.9. Observe that the circuits of figures (a) and (b) are dual
circuits. Hence the numerical value of v
out
(t) and i
out
(t) are the same for the same
excitation. Since the circuit is linear, when the excitation is doubled, the response is
doubled (given zero initial conditions) by linearity. Therefore, i
out
(t) 2g(t) .
SOLUTI ON TO PROBLEM 9.10.
(a) The initial conditions are:
v
c
(0) 0 v
c
( 0+)
i
L
(0) 10 / 0.5 20 i
L
( 0+)
(b)
W
L
( 0)
1
2
L i
L
2
( 0) 0.5 J
W
C
(0) 0
(c) Maximum value of v
C
when all energy is in capacitor must satisfy
1
2
C v
C,max
2
0.5
or equivalently v
C,max
1000 V.
(d) From the text development, the parallel LC circuit has the differential equation,
d
2
v
C
dt
2
+
1
LC
v
C
0
The solution form is:
v
C
(t) A cos
1
LC
t



_
,
+B sin
1
LC
t



_
,

The initial conditions are:
v
C
( 0) A 0
and
v
C
' ( 0+)
1
LC
B
i
C
(0+)
C

i
L
( 0+)
C
Hence
B 1000
Thus
v
C
(t) 1000 sin
1
LC
t



_
,
V
SOLUTI ON TO PROBLEM 9.11.
The switch has been closed for a long time which means that the inductor acts like a short
and the capacitor like an open. Hence, at t =0
-
, v
L
=0 and i
C
=0. Hence I
s
divides
equally between the resistors, i.e.,
i
L
(0) i
L
( 0+) I
s
/ 2 and v
C
(0) v
C
( 0+) 0
For t 0, the differential equation is
d
2
v
C
dt
2
+
1
LC
v
C
0
with corresponding response
v
C
(t) A cos
1
LC
t



_
,
+B sin
1
LC
t



_
,

Evaluating at the initial conditions,
v
C
( 0) A 0
and
v
C
' ( 0 ) B
1
LC

i
C
(0+)
C

i
L
(0+)
C
in which case
B
I
S
2
L
C
Therefore,
v
C
(t)
I
s
2
L
C
sin
1
LC
t



_
,

SOLUTI ON TO PROBLEM 9.12.
Natural frequency is
1
LC
5000 2 . Hence,
C =100e-9;
L = 1/((10e3 *pi)^2 *C)
L =
1.0132e-02
By voltage divider,
v
C
(0) v
C
( 0+) 20 mV
Current through L,
i
L
(0) i
L
( 0+) 0
Voltage across capacitor satisfies,
v
C
(t) Acos
1
LC
t



_
,
+ Bsin
1
LC
t



_
,

Using the ICs,
v
C
(0+) A 20 mV
and
v
C
' ( 0 ) B
1
LC

i
C
(0+)
C

i
L
( 0+)
C
0
Hence
v
C
(t) 20 cos 10, 000t ( ) mV
SOLUTI ON TO PROBLEM 9.13. By definition

1
LC
2 40
in which case
C =0.1e-3;
w =2*pi*40;
L =1/(w^2 *C)
L =
1.5831e-01 (rad/s).
Observe that
i
L
(0) i
L
( 0+) 1A
From the given circuit, the capacitor is never connected to a source. Therefore,
v
C
(0) v
C
( 0+) 0. Also, since
d
2
i
L
dt
2
+
1
LC
i
L
0
it follows that
i
L
(t) Acos(t) + Bsin(t)
From the initial conditions
i
L
( 0) A 1
and
i
L
' (0
+
) B
v
C
(0
+
)
L
0
i
L
(t) cos(80t) A
SOLUTI ON TO PROBLEM 9.14.
(a) From the continuity property, the capacitor voltage and inductor current remain the
same,
v
C
(0) v
C
( 0+) 5 V
i
L
(0) i
L
( 0+) 1
However, i
C
(0) 0 but i
C
(0+) i
L
(0+) 1 A and v
L
(0) 0 but v
L
(0+) 5 V.
These values change to maintain satisfaction of KVL and KCL.
(b)
v
C
(t) Acos
1
LC
t



_
,
+ Bsin
1
LC
t



_
,

where
v
C
(0+) 5 A
v
C
' ( 0+) B
1
LC

i
C
(0+)
C

1
C
B

2
v
C
(t) 5cos(2t)

2
sin(2t) V
Alternately, from equation 9.4,
v
C
(t) Kcos(2t + )
and from equation 9.17b K A
2
+ B
2
and tan
1
B
A


_
,
in which case
A =5; B =-pi/2;
K =sqrt(A^2 +B^2)
K =
5.2409e+00
theta =atan2(-B,A)
theta =
3.0440e-01
thetadeg =theta*180/pi
thetadeg =
1.7441e+01
Hence, K =5.24 and =17
o
.
(c)
0 0.2 0.4 0.6 0.8 1 1.2 1.4 1.6 1.8 2
-6
-4
-2
0
2
4
6
C
a
p
a
c
i
t
o
r

V
o
l
t
a
g
e

(
V
)
TextEnd
Time in s
SOLUTI ON TO PROBLEM 9.15. As before,
1
LC
2 40 in which case L =0.158
H.
Now,
v
C
(0) v
c
(0+)
100
125
25 20 mV
and
i
L
(0) i
L
( 0+) 10 mA
The solution form is:
i
L
(t) Acos(80t) + Bsin(80t)
where
i
L
(0+) A 10 mA
and
i
L
' (0+)
v
L
( 0+)
L

v
C
(0+)
L
80B B =0.50265 mA
SOLUTI ON TO PROBLEM 9.16.
(a) Note: v
C
(0) v
C
( 0+) 1 V and i
L
( 0) 5 A. The characteristic equation is:
s
2
+
1
RC
s +
1
LC
s
2
+ 5s + 4 (s +1)(s + 4 ) 0
As stated, the circuit is overdamped. Hence,
v
C
(t) K
1
e
s
1
t
+ K
2
e
s
2
t
K
1
e
t
+ K
2
e
4t
v
C
(0+) K
1
+ K
2
1
v
C
' ( 0+) s
1
K
1
+ s
2
K
2
K
1
4K
2

i
C
(0+)
C
0
K
1

s
2
s
2
s
1
1.3333 and K
2

s
1
s
2
s
1
0.3333.
Finally,
v
C
(t) 1.3333e
t
+ 0.3333e
4t
V
Using MATLAB to plot:
t =0:.02:4.5;
vc =-1.33333*exp(-t) + 0.33333*exp(-4*t);
plot(t,vc)
grid
ylabel('Capacitor Voltage in V')
xlabel('Time in s')
0 0.5 1 1.5 2 2.5 3 3.5 4 4.5
-1
-0.9
-0.8
-0.7
-0.6
-0.5
-0.4
-0.3
-0.2
-0.1
0
C
a
p
a
c
i
t
o
r


V
o
l
t
a
g
e

i
n

V
TextEnd
Time in s
(b)
K
1
+ K
2
1
v
C
' ( 0+) s
1
K
1
+ s
2
K
2
K
1
4K
2

i
C
(0+)
C

10
C
10
Hence,
A =[1 1;-1 -4]
A =
1 1
-1 -4
b =[1;-10]
b =
1
-10
K =A\b
K =
-2
3
yielding
v
C
(t) 2e
t
+ 3e
4t
V
0 0.5 1 1.5 2 2.5 3 3.5 4 4.5
-1
-0.8
-0.6
-0.4
-0.2
0
0.2
0.4
0.6
0.8
1
C
a
p
a
c
i
t
o
r


V
o
l
t
a
g
e

i
n

V
TextEnd
Time in s
Obviously, there is only one zero crossing.
SOLUTI ON TO PROBLEM 9.17.
(a) First,
v
C
(0) v
C
( 0+)
100
300
30 10 V
i
L
(0) i
L
( 0+) 0.1 A
Clearly, i
C
(0) 0 and v
L
(0) 0. However,
i
L
(0+) + i
C
(0+) +
v
C
(0+)
66.667
0.1 + i
C
(0+) + 0.15 0
Hence, i
C
(0+) 0.25 A. Further,v
L
(0+) 10 V.
(b) The characteristic equation is:
s
2
+
1
RC
s +
1
LC
0
From MATLAB,
R =66.667;C =25e-6; L =0.5;
b =1/(R*C)
b = 6.0000e+02
c =1/(L*C)
c = 80000
si =roots([1 b c])
si = -3.9999e+02
-2.0000e+02
We take the roots to be: s
1
400 and s
2
200.
(c) Overdamped response implies,
v
C
(t) K
1
e
s
1
t
+ K
2
e
s
2
t
K
1
e
400t
+ K
2
e
200t
(d)
v
C
(0+) K
1
+ K
2
10
v
C
' ( 0+) s
1
K
1
+ s
2
K
2
400K
1
200K
2

i
C
(0+)
C
10
4
A =[1 1;-400 -200];
b =[10;-1e4];
K =A\b
K =
40
-30
Finally,
v
C
(t) 40e
400t
30e
200t
V
(e)
t=0:0.01e-3:25e-3;
vc =40*exp(-400*t) - 30*exp(-200*t);
plot(t,vc)
grid
ylabel('Capacitor Voltage in V')
xlabel('Time in s')
0 0.005 0.01 0.015 0.02 0.025
-6
-4
-2
0
2
4
6
8
10
C
a
p
a
c
i
t
o
r

V
o
l
t
a
g
e

i
n

V
TextEnd
Time in s
SOLUTI ON TO PROBLEM 9.18.
(a)
i
L
(0) i
L
( 0+) 0
v
C
(0) v
C
(0+) 5 V
At t =0+, the circuit is a series RLC with R =12.5 , L = 2.5 H, and C=0.1 F. The
resulting characteristic polynomial is:
s
2
+
R
L
s +
1
LC
s
2
+ 5s + 4 (s + 4)(s +1) 0
Hence, s
1
, s
2
4, 1 and the form of the response is:
v
C
(t) K
1
e
4t
+ K
2
e
t
At t =0+,
v
C
(0+) K
1
+ K
2
5
and
v
C
'(0) 4K
1
K
2

i
C
(0+)
C

i
L
(0+)
C
0
Solve for K
1
and K
2
we obtain:
A =[1 1; -4 -1];
b =[5;0];
K =A\b
K =
-1.6667e+00
6.6667e+00
Hence,
v
C
(t) 1.66667e
4t
+ 6.66667e
t
V
(b)
i
L
(0+) i
L
(0) 0
Since, the stable (passive) circuit contains no source for t >0, all initial energy is
absorbed by the resistor. Hence lim
t
i
L
(t) 0, i.e., i
L
() 0.
i
L
(t) i
C
(t) C
dv
C
dt

2
3
e
4t

2
3
e
t
A
SOLUTI ON TO PROBLEM 9.19. This circuit is a series RLC in which case the
characteristic equation is always: s
2
+
R
L
s +
1
LC
0
(a) For this time period, R =2 k in which case the characteristic equation is found and
solved in MATLAB as follows:
R =2e3; L =0.1; C =0.1e-6;
b =R/L
b =
20000
c =1/(L*C)
c =
100000000
s12 =roots([1 b c])
s12 =
-10000
-10000
The roots are repeated and: s
1
, s
2
=10,000. The form of the response is:
i
L
(t) (K
1
+ K
2
t)e
10000t
To find K
1
and K
2
:
i
L
(0+) K
1
2.5
and
i
L
'(0+) 25000 + K
2

v
L
(0+)
L

v
C
(0+) Ri
L
(0+)
L

6 2000 2.5
0.1
49940
K
2
24940
Thus
i
L
(t) (2. 5 24940t)e
10000t
0 t 0.1msec
(b) After the switch closes, R =1 k,
i
L
(0.110
3
) (2.5 24940 0.110
3
)e
1
0.0022073
and
i
L
'(10
3
) 25000e
1
24940e
1
+ 24940e
1
9197
The new characteristic equation is computed as follows:
R =1e3; L =0.1; C =0.1e-6;b =R/L
b = 10000
c =1/(L*C)
c = 100000000
s12new =roots([1 b c])
s12new =
-5.0000e+03 +8.6603e+03i
-5.0000e+03 - 8.6603e+03i
Hence s
1
, s
2
5000 t j8660.25. The form of the new solution is:
i
L
(t') e
5000t'
Acos( t') + Bsin( t') [ ]
A
where t' t 0.110
3
and 8660.25 rad/sec. Observe that
i
L
(t' 0) 0.0022073 A
and
i
L
'( t' 0) 5000A + 8660.25B 9197
From MATLAB,
w =imag(s12new(1));
sig =real(s12new(1));
B =(-9197 -sig*2.2073e-3)/w
B =
-1.0607e+00
Hence for t >0.1 ms,
i
L
(t) e
5000(t 0.1ms)
0.0022073cos (t 0.1ms) ( ) 1.0607sin (t 0.1ms) ( ) [ ]
A
(c)
0 0.5 1 1.5 2 2.5 3 3.5 4
x 10
-3
-1
-0.5
0
0.5
1
1.5
2
2.5
i
L
(
t
)
,

i
n

A
TextEnd
time in s
K1 =2.5;
K2 =-24940;
t =0:0.01e-3:4e-3;
A =0.0022073;
B =(-9197 -sig*2.2073e-3)/w
B = -1.0607e+00
iL =(K1 +K2*t) .* exp(-10000*t) .* (u(t)-u(t-1e-4)) ...
+exp(-5000*(t - 1e-4)) .* (A*cos(w*(t - 1e-4))+B*sin(w*(t - 1e-4))) ...
.* u(t -1e-4);
plot(t,iL)
grid
iL =(K1 +K2*t) .* exp(-10000*t) .* (u(t)-u(t-1e-4))
(d)
K1 =2.5;
K2 =-24940;
t =0.1e-3;
iL =(K1 +K2*t) .* exp(-10000*t)
iL = 2.2073e-03
% The energy stored in the inductor over [0,0.1ms] is in J :
WL =0.5*0.1*(iL^2 - 2.5^2)
WL =
-3.1250e-01
% The energy stored in the capacitor over [0,0.1ms] first
% requires computation of vL and then vC.
vL =0.1*(K2*exp(-10000*t) - 10000*(K1 +K2*t) .* exp(-10000*t))
vL =
-9.1970e+02
vC =vL +2000*iL
vC =
-9.1528e+02
% The energy stored in the capacitor over [0,0.1ms] is in J :
WC =0.5*0.1e-6*(vC^2 - 6^2)
WC =
4.1885e-02
% To compute energy dissipated in resistor, we make
% use of conservation of energy: WR +WC +WL =0
WR =-WL - WC
WR =
2.7061e-01
SOLUTI ON TO PROBLEM 9.20. For this problem, i
L
(0) 8 A, v
C
(0) 20 V. For the
parallel RLC, the characteristic equation s
2
+
1
RC
s +
1
LC
0 is solved as follows:
R =20; C =0.01e-3; L =25e-3;
si =roots([1 1/(R*C) 1/(L*C)])
si =
-4.0000e+03
-1.0000e+03
s1 =si(1); s2 =si(2);
Hence
v
C
(t) K
1
e
4000t
+ K
2
e
1000t
To compute the constants,
v
C
(0) K
1
+ K
2
20
and
v
C
'(0) 4000K
1
1000K
2

i
C
(0+)
C

i
L
(0+) +
v
C
(0+)
R
C
iL0 =8; vC0 =20;
vCprime =-(iL0 +vC0/R)/C
vCprime =
-9.0000e+05
A =[1 1;-4000 -1000];
b =[20;vCprime];
K =A\b
K =
2.9333e+02
-2.7333e+02
Hence,
v
C
(t) 293.33e
4000t
273.33e
1000t
V
Also,
i
L
(t)
v
C
(t)
R
Cv
C
'
(t) 2.933e
4000t
+10.933e
1000t
A
where
K1 =K(1); K2 =K(2);
KR1 =K1/R; KR2 =K2/R;
KCp1 =-4000*K1*C; KCp2 =-1000*K2*C;
KL1 =-KR1-KCp1
KL1 =
-2.9333e+00
KL2 =-KR2 - KCp2
KL2 =
1.0933e+01
t =0:0.01e-3:4e-3;
iL =KL1*exp(-4000*t) +KL2*exp(-1000*t);
plot(t,iL)
grid
xlabel('Time in s')
ylabel('Inductor Current in A')

0 0.5 1 1.5 2 2.5 3 3.5 4


x 10
-3
0
1
2
3
4
5
6
7
8
9
Time in s
I
n
d
u
c
t
o
r

C
u
r
r
e
n
t

i
n

A
TextEnd
SOLUTI ON TO PROBLEM 9.21.
(a) The circuit is critically damped and the characteristic polynomial is:
s
2
+
1
RC
s +
1
LC
(s + 20)
2
s
2
+ 40s + 400 0
R =2; C =1/80;
L =1/(400*C)
L =
2.0000e-01
(b) v
C
(0) 10 V and v
C
'(0
+
) 800
i
C
(0
+
)
C

i
L
(0
+
) v
C
(0
+
) / 2
C
.
vC0 =10; vCp0 =-20*10 -600
vCp0 =
-800
iL0 =C*800-vC0/2
iL0 =
5
Hence: i
L
(0-) =i
L
(0+) =5 A.
(c)
By simple KCL,
i
L
(t)
1
80
dv
C
dt

v
C
2
5e
20t
+150 t e
20t
A
SOLUTI ON TO PROBLEM 9.22.
(a) The characteristic equation for the series RLC
s
2
+
R
eq
L
s +
1
LC
0
For critically damped response, want (R
eq
/ L)
2
4 / (LC) 0. Solving yields R
eq
4
. Hence,
R
eq

5R
5 + R
implies that R =20 .
(b) Solving for the resulting roots implies that
si =roots([1 Req/L 1/(L*C)])
si =
-50
-50
s1 =50;
Hence
v
C
(t) (K
1
+ K
2
t)e
50t
From the initial conditions, v
C
(0) K
1
5 and
v
C
'(0) 250 + K
2

i
C
(0+)
C

i
L
(0+)
C

5
0.01
500
Hence, K
2
=250 and
v
C
(t) (5 250t)e
50t
V
Set to zero and solve for t =0.02. See MATLAB plot.
0 0.005 0.01 0.015 0.02 0.025 0.03 0.035 0.04 0.045 0.05
-1
0
1
2
3
4
5
SOLUTI ON TO PROBLEM 9.23.
(a) The circuit is a series RLC. Hence
R =40; C =0.25e-3; L =0.1;
si =roots([1 R/L 1/(L*C)])
si =
-200
-200
s1 =si(1);
Since the circuit is critically damped,
i
L
(t) (K
1
+ K
2
t)e
200t
A
Using initial conditions,
i
L
(0) K
1
1
i
L
'(0) s
1
K
1
+ K
2
200K
1
+ K
2

v
L
(0+)
L

v
C
(0+) 40i
L
(0+)
L
350
Hence
K
2
150
Thus
i
L
(t) (1150t)e
200t
A
Now using MATLAB,
R/L
ans = 400
1/(L*C)
ans = 40000
y =dsolve('D2y +400*Dy +40000*y =0, y(0) =1,Dy(0) =-350')
y =
exp(-200*t)-150*exp(-200*t)*t
This answer coincides with the analytical solution.
(b) As above, the form of the solution is
v
C
(t) (K
1
+ K
2
t)e
50t
Applying initial conditions,
v
C
(0) K
1
5
v
C
'(0) s
1
K
1
+ K
2
200K
1
+ K
2

i
C
(0+)
C

i
L
(0+)
C
4000
Hence, K
2
5000 and
v
C
(t) (5 + 5000t)e
200t
V
In MATLAB,
y =dsolve('D2y +400*Dy +40000*y =0, y(0) =5,Dy(0) =4000')
y =
5*exp(-200*t)+5000*exp(-200*t)*t
which verifies the analytical solution. Plotting we obtain,
0 0.005 0.01 0.015 0.02 0.025 0.03 0.035 0.04 0.045 0.05
0
2
4
6
8
10
12
Observe, there is no zero crossing as per problem 9.5.
SOLUTI ON TO PROBLEM 9.24.
Source has been on for a long time and turns off at t =0. Hence
i
L
(0) i
L
(0+) 0
v
C
(0) v
C
(0+) 0.1 40 4 V
At t 0, we have a series RLC circuit:
R =40; C =4e-3; L =2;
si =roots([1 R/L 1/(L*C)])
si =
-1.0000e+01 +5.0000e+00i
-1.0000e+01 - 5.0000e+00i
wd =imag(si(1))
wd =
5
sig =-real(si(1))
sig =
10
Hence
v
C
(t) e
10t
Acos(5t) + Bsin(5t) ( )
Applying initial conditions,
v
C
(0) A 4
v
C
'(0) 10A + 5B
i
C
(0+)
C

i
L
(0+)
C
0
Hence, B =8. The final form is:
v
C
(t) e
10t
4cos( 5t) + 8sin(5 t) ( ) V
Similarly,
i
L
(t) e
10t
Acos(5t) + Bsin(5t) ( ) A
Applying initial conditions,
i
L
(0) A 0
i
L
'(0) 10A + 5B
v
L
(0+)
L

v
C
(0+) 40i
L
(0+)
L
2
Thus
i
L
(t) 0.4e
10t
sin(5t) A
SOLUTI ON TO PROBLEM 9.25.
At t =0
-
, the capacitor is an open circuit and the inductor is a short circuit. The
resulting circuit is a simple resistive network. The first step in the solution is to solve this
network for the initial conditions on the capacitor and inductor. Specifically, solve for
the capacitor voltage (i.e. the voltage across the series connection of the 6 resistor and
the independent voltage source) and the inductor current (i.e. the current flowing through
the 4 resistor).
Verify that i
L
(0) i
L
(0+) 1 A and v
C
(0) v
C
(0+) 12 V.
When the switch opens, the branch containing the independent voltage source is
eliminated. So, we end up with a series RLC circuit. The equivalent resistance is
R
eq
4 + 24 / / 4 8 20 .
R =20; C =0.01; L =2;si =roots([1 R/L 1/(L*C)])
si =
-5.0000e+00 +5.0000e+00i
-5.0000e+00 - 5.0000e+00i
sig =-real(si(1))
sig =
5
wd =imag(si(1))
wd =
5
i
L
(t) e
5t
Acos(5t) + Bsin(5t) [ ] A
Applying ICs,
i
L
(0) A 1
i
L
'(0) 5A + 5B
v
L
(0+)
L

8
2
4
Hence B =0.2. It follows that
i
L
(t) e
5t
cos(5t) +
1
5
sin(5t)



1
]
1
A
SOLUTI ON TO PROBLEM 9.26. The series RLC circuit has characteristic equation
s
2
+
R
L
s +
1
LC
s
2
+
10
L
s +
1
LC
s
2
+ 2 s +
2
+
d
2
0
From the given response, 10
10
2L
which implies that L =0.5 H. Further,
1
LC

2
C

2
+
d
2
10
2
+ 10 3 ( )
2
400
Hence, C =5 mF.
Now, from given response,i
L
(0) 0 and i
L
'(0) 500 3
v
L
(0)
L
2v
L
(0) .
Hence
v
L
(0) 250 3 V. In addition, v
C
(0) 10i
L
(0) v
L
(0) 250 3 V and
v
C
'(0) i
L
(0) / C 0
To find the capacitor voltage we have
v
C
(t) e
10t
Acos(10 3t) + Bsin(10 3t) ( )
It follows that v
C
(0) 250 3 A and
v
C
'(0) 0 2500 3 +10 3 B
B =250. Therefore
v
C
(t) e
10t
250 3 cos(10 3t) + 250sin(10 3t) ( )
SOLUTI ON TO PROBLEM 9.27. (a) From the given information,
v
C
(t) Ae
t
cos( t + ) V
where 2 f 2 / T
2
0.5 10
3
4000 rad/s. f =2 kHz.
The time constant of the exponential term is 1/ which is 2 ms from the figure.
Hence 500. The characteristic equation of the parallel RLC is:
s
2
+
1
RC
s +
1
LC
s
2
+
1
10
4
C
s +
1
LC
s
2
+ 2 s +
2
+
d
2
0
Since
1
10
4
C
2 1000, C 0.1 F. Further,
1
LC

10
7
L

2
+
d
2
10
6
+16
2
10
6
implies that
L =1e7/(1e6 +16*pi^2 *1e6) or L =63 mH.
(b) From the figure and the above calculations, v
C
(0) =10 V and
v
C
(t) 10e
500t
cos(4000t) V,
Hence
v
C
'(t) 5000e
500t
cos(4000 t) 40000e
500t
sin(4000t)
implying that v
C
'
(0) 5000. Thus
i
L
(0) v
C
(0) /10
4
Cv
C
'
(0) 0.001 + 5000 0.110
6
0.5 10
3
A
SOLUTI ON TO PROBLEM 9.28. As given v
C
(t) Ae
3t
cos(4t + ) and R =10 . The
characteristic polynomial of the parallel RLC circuit is:
s
2
+
1
RC
s +
1
LC
s
2
+
1
10C
s +
1
LC
s
2
+ 2 s +
2
+
d
2
0
Hence 1/(RC) =6 implies C =1/60 F. Further
d

4 60
L
36
2
4. Hence L =2.4
H.
To change R to obtain a cricially damped circuit,
60
R


_
,
2

4
LC
100
Hence R
2
=36 or R =6 . It follows that 2 =10 or =5. The form of the response is:
v
C
(t) (K
1
+ K
2
t)e
5t
SOLUTI ON TO PROBLEM 9.29. For all cases, v
C
(0-)=v
C
(0+) =0 and i
L
(0-)=i
L
(0+) =
20/20 =1 A. Further for all cases the circuit is a parallel RLC with characteristic
equation:
s
2
+
1
RC
s +
1
LC
s
2
+ 2 s +
2
+
d
2
0
(a)
L =2e-3; C =5e-6;
c =1/(L*C);
R =10;
b =1/(R*C);
si =roots([1 b c])
si =
-10000
-10000
% Solution is cricitally damped.
Thus
v
C
(t) (K
1
+ K
2
t)e
s
1
t
(K
1
+ K
2
t)e
10
4
t
V
From ICs,
v
C
(0) K
1
0
v
C
'(0) s
1
K
1
+ K
2
K
2

i
C
(0+)
C

i
L
(0+)
C
2 10
5
Hence
v
C
(t) 2 10
5
te
10
4
t
V
(b)
R =100;
si =roots([1 1/(R*C) 1/(L*C)])
si =
-1.0000e+03 +9.9499e+03i
-1.0000e+03 - 9.9499e+03i
v
C
(t) e
1000t
Acos(9950t) + Bsin(9950t) ( ) V
From ICs.
v
C
(0+) A 0
v
C
'(0) 1000A + 9950B
i
C
(0+)
C

i
L
(0+)
C
2 10
5
in which case B =20.1. Thus
v
C
(t) 20.1e
1000t
sin(9950t) V
(c)
R =87/17;si =roots([1 1/(R*C) 1/(L*C)])
si =
-3.6328e+04
-2.7527e+03
We define s
1
,s
2
=2753, 36327. Thus
v
C
(t) K
1
e
s
1
t
+ K
2
e
s
2
t
K
1
e
2753t
+ K
2
e
36327t
V
From the IC's
v
C
(0) K
1
+ K
2
0
v
C
'(0) s
1
K
1
+ s
2
K
2
2 10
5
A =[1 1;si(2) si(1)];
b =[0; -2e5];
K =A\b
K =
-5.9568e+00
5.9568e+00
Therefore
v
C
(t) 5.9568 e
2753t
e
36327t
( )
V
SOLUTI ON TO PROBLEM 9.30. For all cases, v
C
(0-) =v
C
(0+) =0 and i
L
(0-) =i
L
(0+) =
10/100 =0.1 A. Further for all cases the circuit is a parallel RLC with characteristic
equation:
s
2
+
1
RC
s +
1
LC
s
2
+ 2 s +
2
+
d
2
0
(a)
R =50; C =0.04e-3; L =0.625;
si =roots([1 1/(R*C) 1/(L*C)])
si =
-400
-100
Define the two roots as:
s
1
=100
s
2
=400
Hence,
v
C
(t) K
1
e
100t
+ K
2
e
400t
V
From, the initial conditions
v
C
(0+) =0 =K
1
+K
2
and
v
C
'(0) s
1
K
1
+ s
2
K
2
100K
1
400K
2

i
L
(0+)
C
2500
A =[1 1;si(2) si(1)];
b =[0;-2500];
K =A\b
K =
-8.3333e+00
8.3333e+00
Therefore
v
C
(t) 8.3333 e
100t
e
400t
( )
V
(b)
L =0.4;
si =roots([1 1/(R*C) 1/(L*C)])
si =
-2.5000e+02
-2.5000e+02
Thus
v
C
(t) (K
1
+ K
2
t)e
250t
V
From IC's,
v
C
(0) K
1
0
v
C
'(0) s
1
K
1
+ K
2
K
2

i
C
(0+)
C

i
L
(0+)
C
2500
Therefore
v
C
(t) 2500te
250t
V
(c)
L =0.2;
si =roots([1 1/(R*C) 1/(L*C)])
si =
-2.5000e+02 +2.5000e+02i
-2.5000e+02 - 2.5000e+02i
v
C
(t) e
250t
Acos(250t) + Bsin(250t) [ ] V
From ICs.
v
C
(0+) A 0
v
C
'(0) 250A + 250B 250B
i
C
(0+)
C

i
L
(0+)
C
2500
in which case B =10. Thus
v
C
(t) 10e
250t
sin(250t) V
SOLUTI ON TO PROBLEM 9.31.
(a) The indicated behavior occurs when the resistance causes the circuit to be critically
damped, i.e.,
1
RC


_
,
2

4
LC

4
R


_
,
2
16 0
Thus R =1 and
R 1 R
0
+ e
t 5
0.8 + e
t 5
So
t =5+log(1-0.8)
t =
3.3906e+00 (years)
(b) Here, we have a series case: the indicated behavior occurs when the resistance
causes the circuit to be critically damped, i.e.,
R
L


_
,
2

4
LC
R
2
144 0
Thus R =12 and
R 12
R
0
1 + e
t 5

15
1 + e
t 5
t =5 + log((15-12)/12)
t =
3.6137e+00 (years)
SOLUTI ON TO PROBLEM 9.32. Step 1: Since the step functions are 0 from t = up to t =
0-,
v
C
(0-) =v
C
(0+) =0, i
L
(0-) =i
L
(0+) =0
Step 2: At t =0+, we have
i
C
(0+) 2
v
C
(0+)
20
i
L
(0+) 2 A
and
v
L
(0+) v
C
(0+) 5i
L
(0+) 50 50 V
SOLUTI ON TO PROBLEM 9.33. Since the step function is 0 from t = up to t =0-,
v
C
(0-) =v
C
(0+) =0, i
L
(0-) =i
L
(0+) =0
Since the circuit is a parallel RLC
s
2
+
1
RC
s +
1
LC
s
2
+100s +1600 0
R =4; L =0.25; C =2.5e-3;
si =roots([1 1/(R*C) 1/(L*C)])
si =
-80
-20
Hence s
1
,s
2
=-20, -80. The general form is:
v
C
(t) K
1
e
80t
+ K
2
e
20t
+ X
f
V
When the capacitor is open and the inductor is a short, X
f
=0. Thus,
v
C
(t) K
1
e
80t
+ K
2
e
20t
V
From the ICs
v
C
(0+) =0 =K
1
+K
2
and
v
C
'(0) 80K
1
20K
2

i
C
(0+)
C

i
L
(0+) v
C
(0+) 8 + (20 v
C
(0+)) 8
C

2.5
2.5 10
3
1000
A =[1 1;-80 -20];
b = [0; 1000];
K =A\b
K =
-1.6667e+01
1.6667e+01
Hence,
v
C
(t) 16.667 e
80t
e
20t
[ ]
V
SOLUTI ON TO PROBLEM 9.34. From the continuity property and the fact that at t =0-,
the capacitor looks like an open and the inductor looks like a short at t =0-,
i
L
(0-) =i
L
(0+) =1A
v
C
(0-) =v
C
(0+) =0
Since the circuit is a parallel RLC
s
2
+
1
RC
s +
1
LC
s
2
+ 500s + 40000 0
R =40; C =0.05e-3;L =0.5;
si =roots([1 1/(R*C) 1/(L*C)])
si =
-400
-100
Define the two roots as:
s
1
=100
s
2
=400
Hence, the general form is:
i
L
(t) K
1
e
100t
+ K
2
e
400t
+ X
f
When the capacitor is open and the inductor is a short, X
f
=1 A. Thus,
i
L
(t) K
1
e
100t
+ K
2
e
400t
1
From, the initial conditions
i
L
(0+) K
1
+ K
2
1 1
and
i
L
'(0) 100K
1
400K
2

v
L
(0+)
L

v
C
(0+)
L
0
A =[1 1;-100 -400];
b =[2;0];
K =A\b
K =
2.6667e+00
-6.6667e-01
Hence,
i
L
(t)
8
3
e
100t

2
3
e
400t
1 A
SOLUTI ON TO PROBLEM 9.35.
(a) From the problem specs and the continuity property,
i
L
(0-) =i
L
(0+) =0.008 A
v
C
(0-) =v
C
(0+) =2
At t =, the inductor looks like a short and the capacitor looks like an open; hence i
L
()
=0 and v
C
() =4000.006 =2.4 V. The circuit is a series RLC with characteristic
polynomial
s
2
+
R
L
s +
1
LC
0
R =400; C =6.6667e-6; L =0.2;
si =roots([1 R/L 1/(L*C)])
s1 =si(1); s2 =si(2);
si =
-1.5000e+03
-5.0000e+02
Hence
v
C
(t) K
1
e
1500t
+ K
2
e
500t
+ 2.4 V
Using initial conditions
v
C
(0) =K
1
+K
2
+2.4 =2
v
C
'(0) 1500K
1
500K
2

i
C
(0+)
C

i
L
(0+)
C

8 10
3
6.6667 10
6
1200
A =[1 1;-1500 -500];
b =[2-2.4;1200]
b =
-4.0000e-01
1.2000e+03
K =A\b
K =
-1.0000e+00
6.0000e-01
Hence
v
C
(t) e
1500t
+ 0.6e
500t
+ 2.4 V
(b) v
L
(t) is going to have the same form as v
C
(t) above except that v
L
() =0 since the
inductor is a short at t =. Alternately however, we have
v
L
(t) L
di
L
(t)
dt
L
di
C
(t)
dt
LC
d
2
v
C
(t)
dt
2
3e
1500t
+ 0.2e
500t
V
SOLUTI ON TO PROBLEM 9.36.
At t =0, inductor is a short circuit and the capacitor is an open circuit. Since the current
source is 0 at t =0-, and the continuity property,
i
L
(0-) =i
L
(0+) =1 A
v
C
(0-) =v
C
(0+) =65 V
For positive time, we have a series RLC circuit with characteristic equation
s
2
+
R
L
s +
1
LC
0
R =65; C =0.1e-3; L =0.1;
si =roots([1 R/L 1/(L*C)])
s1 =si(1); s2 =si(2);
si =
-4.0000e+02
-2.5000e+02
At t =, v
C
() =0.6*65 =39 V. Hence
v
C
(t) K
1
e
400t
+ K
2
e
250t
+ 39 V
Using initial conditions,
v
C
(0) =K
1
+K
2
+39 =65
v
C
'(0) 400K
1
250K
2

i
C
(0+)
C

i
L
(0+)
C

1
10
4
10
4
A =[1 1;s1 s2];
b =[65-39; -1e4];K=A\b
K =
2.3333e+01
2.6667e+00
Hence,
v
C
(t) 23.333e
400t
+ 2.6667e
250t
+ 39 V
SOLUTI ON 9.37.
(a) R
th
=200//50 +R =(40 +R). The characteristic equation is:
s
2
+
1
R
th
C
s +
1
LC



_
,

0
Critically damped means that both roots are the same, so the discriminant is zero, i.e.
1
R
th
C



_
,

2

4
LC
0
Equivalently,
R
th
40 + R 0.5
L
C
50
Thus R =10 .
(b) Short the inductor and open the capacitor. Because the capacitor is in parallel
with the shorted inductor at t =0-, v
C
(0
+
) =v
C
(0
-
) =0. The Thevenin equivalent
resistance seen by the LC-parallel combination is R
th
=50 from part (a). A simple
calculation indicates that V
oc
=0.8*50 =40 V. Therefore, i
L
(0
+
) =i
L
(0
-
) =40/50 =0.8
A. To find v
R
(0
+
) we use the following equivalent circuit:
Hence,
v
R
(0
+
) 40
10
10 + 40
8 V.
To compute the derivative of v
R
at 0
+
, consider that
d
dt
v
R
(t) ( )
d
dt
10 i
R
(t) ( ) 10
d
dt
40 v
C
(t)
50


_
,
0.2
d
dt
v
C
(t) ( ) 0.2
i
C
(t)
C
Hence
dv
R
(t)
dt
1
]
1
t 0
+
0.2
i
C
(0
+
)
C
4000i
C
(0
+
)
But
i
C
(0
+
)
40
50
i
L
( 0
+
) 0.8 0.8 1.6 A
Therefore,
dv
R
(t)
dt
1
]
1
t 0
+
4000i
C
(0
+
) 6400 V/s
(c) Since the circuit is critically damped the roots of the characteristic equation are
s
1,2

1
2R
th
C
200
According to table 9.2 for t 0,
v
R
(t) K
1
+ K
2
t ( )e
200t
+ X
F
It follows from the circuit and this equation that
X
F
v
R
() 8 V
K
1
v
R
(0
+
) X
F
0
dv
R
(t)
dt
1
]
1
t 0
+
200K
1
+ K
2
K
2
6400
Therefore
v
R
(t) 6400te
200t
8
( )
u(t) V
A plot of the waveform is given below
SOLUTI ON TO PROBLEM 9.38. For this problem we first compute the Thevenin
equivalent of the circuit to the left of the capacitor for t >0. Consider
Now observe
I
s
I
1
+ I
2

V
s
12
500
+
V
s
kv
1
500

V
s
12
500
+
V
s
k 12 V
s
( )
500
It follows that
V
s

500
2 + k


_
,
I
s
+
1 + k
2 + k


_
,
12 R
th
I
s
+V
oc
The parallel LC is now driven by this Thevenin equivalent.
L =0.01; C =1e-6;
% Critical damping means (1/(Rth*C))2 - 1/(L*C) =0
x =sqrt(4/(L*C));
Rthcrit =1/(C*x)
Rthcrit =
50
kcrit =(500 - 2*50)/50
kcrit =
8
% For parallel circuit, larger Rth means less damping
% Hence, smaller Rth means overdamped. Smaller Rth
% means larger k. Therefore k >8 is the ranger for
% overdamped response.
For the critically damped response we have R =R
th
=50 ; hence
R =50; C =1e-6; L =0.01;
si =roots([1 1/(R*C) 1/(L*C)])
si =
-10000
-10000
in which case
v
C
(t) (K
1
+ K
2
t)e
50t
+ X
f
At t =, v
C
() v
L
() 0 in which case X
f
0. From the initial conditions,
v
C
(0) K
1
0
v
C
'(0) s
1
K
1
+ K
2
K
2

i
C
(0+)
C

V
oc
R
thcrit
i
L
(0+)
C
Hence
K
2

10.8 50
10
6
2.16 10
5
Therefore
v
C
(t) 2.16 10
5
e
50t
V
SOLUTI ON TO PROBLEM 9.39. (a) The series RLC leads to a characteristic equation of
the form
s
2
+
R
L
s +
1
LC
s
2
+
24
0.2
s +
1
0.2C
s
2
+120s +
5
C
0
For a critically damped response, 120
2

20
C
. Hence, C =1.3889 mF.
(b)
C =20/120^2
C =
1.3889e-03
L =0.2; R =24;
si =roots([1 R/L 1/(L*C)])
si =
-60
-60
Hence
i
L
(t) (K
1
+ K
2
t)e
60t
+ 0.4 A
where i
L
() =0.4 because at t =, the capacitor looks like an open and the inductor like
a short. Hence all current from the source flows through the inductor.
Using the initial conditions,
i
L
(0) =K
1
+0.4 =0 K
1
= 0.4
i
L
'(0) s
1
K
1
+ K
2
60 0.4 + K
2

v
L
(0+)
L

v
C
(0+) 24 i
L
(0+) 0.4 ( )
L
48
Hence, K
2
=24 and
i
L
(t) (0.4 + 24t)e
60t
+ 0.4 A
SOLUTI ON TO PROBLEM 9.40. This problem differs from 39 in the initial condition
calculation. Specifically,
i
L
(0-) =i
L
(0+) =0.4
v
C
(0-) =v
C
(0+) =0
Again
i
L
(t) (K
1
+ K
2
t)e
60t
+ 0.4 A
and
i
L
(0+) =K
1
+0.4 = 0.4 K
1
= 0.8
i
L
'(0) 60 0.8 + K
2

v
L
(0+)
L

v
C
(0+) 24 i
L
(0+) 0.4 ( )
L
96
Hence,
K
2
=48
and
i
L
(t) (0.8 + 48t)e
60t
+ 0.4 A
SOLUTI ON TO PROBLEM 9.41.
(a) At 0-, the capacitor is an open circuit and inductor is a short circuit. So,
v
C
(0-) =v
C
(0+) =5 V
i
L
(0-) =i
L
(0+) =510
-3
410
-3
=1 mA
Now, at 0+, replace the capacitor by a 5 V voltage source and the inductor by a 1 mA
current source. Also, the original independent current source is turned off. Solve the
resulting circuit to obtain.
i
C
+510
-3
110
-3
=0
i
C
(0+) =4 mA
v
L
(0+) =0
(b) Since the circuit is a parallel RLC, the characteristic polynomial is
s
2
+
1
RC
s +
1
LC
s
2
+
1
10
4
C
s +
1
LC
s
2
+ 2 s +
2
+
d
2
0
R =1e3; C =0.5e-6; L =0.184;
si =roots([1 1/(R*C) 1/(L*C)])
si =
-1.0000e+03 +3.1416e+03i
-1.0000e+03 - 3.1416e+03i
wd =imag(si(1))
wd =
3.1416e+03
sig =-real(si(1))
sig =
1000
Also, at t =, i
L
() =510
-3
A. Therefore
i
L
(t) e
1000t
Acos( 10
3
t) + Bsin( 10
3
t)
[ ]
+ 5 10
3
A
Using initial conditions
i
L
(0) A + 5 10
3
10
3
A 4 10
3
and
i
L
'(0) 1000A + 10
3
B 4 + 10
3
B
v
L
(0+)
L
0
Hence,
B =-4e-3/pi
B =
-1.2732e-03
Finally,
i
L
(t) e
1000t
4c os ( 10
3
t) +1.2732sin( 10
3
t)
[ ]
+ 5 mA
SOLUTI ON TO PROBLEM 9.42. The response here coincides with that of problem 41 up to
time t =2s. At this point we need the new initial conditions on the circuit at t =2+.
However, at t =2, e
1000t
0 for all practical purposes. Hence, i
L
(2) 5 10
3
A.
Differentiating the expression for i
L
(t) and evaluating at t =2 yields zero by inspection.
This follows because Ke
2000
0 for K in the range of 1 to 10
4
. This can also be seen
from the circuit because at t =2 s, the capacitor has charged to 5 V, making
i
L
'
(2+)
v
L
(2+)
L
0.
To find steady state current, solve the circuit with the new current source value and with
the capacitor and inductor as open and short circuits, respectively:
i
L
() 5 10
3
+ 4 10
3
9 10
3

i
L
(t) e
1000(t 2)
Acos 10
3
(t 2)
( )
+ Bsin 10
3
(t 2)
( ) [ ]
+ 9 10
3
A
Using the new initial conditions
i
L
(2+) A + 9 10
3
5 10
3
A 4 10
3
and
i
L
'(2+) 1000A + 10
3
B 4 + 10
3
B
v
L
(0+)
L
0
Hence B =1.273210
-3
and for t 2s,
i
L
(t) e
1000(t 2)
4cos 10
3
(t 2)
( )
+1.2732sin 10
3
(t 2)
( ) [ ]
+ 9 mA
SOLUTI ON TO PROBLEM 9.43. At t =0-, i
L
(0-) =10/20 =0.5 A and v
C
(0-) =5 V by the
usual considerations. At t =0+, we have a parallel RLC circuit. Hence
s
2
+
1
RC
s +
1
LC
s
2
+
1
10
4
C
s +
1
LC
s
2
+ 2 s +
2
+
d
2
0
R =10; C =0.05e-3; L =0.01;
si =roots([1 1/(R*C) 1/(L*C)])
si =
-1.0000e+03 +1.0000e+03i
-1.0000e+03 - 1.0000e+03i
wd =imag(si(1))
wd =
1.0000e+03
sig =-real(si(1))
sig =
1000
Also, at t =, v
C
() =5 V. Therefore
v
C
(t) e
1000t
Acos(10
3
t) + Bsin(10
3
t)
[ ]
5 V
Using initial conditions
v
C
(0) A 5 5 A 0
and
v
C
'(0) 1000A +1000B 10
3
B
i
C
(0+)
C

0.5
0.05 10
3
10
4
Hence, B 10 and
v
C
(t) 10e
1000t
sin(10
3
t) 5 V
Further
v
L
(t) 5 v
C
(t) 10e
1000t
sin(10
3
t) V
SOLUTI ON TO PROBLEM 9.44. (a) The circuit is a driven series RLC. Hence
s
2
+
R
L
s +
1
LC
s
2
+ 2 s +
2
+
d
2
0
R =400; C =0.5e-6; L =0.2;
si =roots([1 R/L 1/(L*C)])
si =
-1.0000e+03 +3.0000e+03i
-1.0000e+03 - 3.0000e+03i
wd =imag(si(1))
sig =-real(si(1))
wd =
3000
sig =
1000
Also, at t =, v
C
() =4000.006 =2.4 V. Therefore
v
C
(t) e
1000t
Acos(3000t) + Bsin(3000t) [ ] + 2.4 V
Using initial conditions
v
C
(0) A + 2.4 2 A 0.4
and
v
C
'(0) 1000A + 3000B 400 + 3000B
i
C
(0+)
C

i
L
(0+)
C
1.6 10
4
Hence, B 5.2 and
v
C
(t) e
1000t
0.4cos(3000t) + 5.2sin(3000t) [ ] + 2.4 V
(b) Consistent with underdamped circuit behavior and because the inductor behaves as a
short and the capacitor as an open at t = (i
L
() =0),
i
L
(t) e
1000t
Acos(3000t) + Bsin(3000t) [ ] V
Using the initial conditions,
i
L
(0+) A 0.008 A
Further,
i
L
'(0) 1000A + 3000B 8 + 3000B
v
L
(0+)
L

400(0.006 0.008) 2
0.2
14
Hence, B 0.002 and
i
L
(t) e
1000t
0.008cos(3000t) 0.002sin(3000 t) [ ] V
Finally
v
L
(t) v
R
(t) v
C
(t) 400 0.006 i
L
(t) [ ] v
C
(t) 2.4 400i
L
(t) v
C
(t)
which implies that
v
L
(t) e
1000t
3.6cos(3000t) 13.2sin(3000t) [ ] V
SOLUTI ON TO PROBLEM 9.45. This circuit is the same series RLC as problem 44. Note
that at t = , v
C
() =2.4 V. Hence
v
C
(t) e
1000t
Acos(3000t) + Bsin(3000t) [ ] 2.4 V
Now, the initial conditions are:
v
C
(0-) =2.4 =v
C
(0+) V, i
L
(0-) =0 = i
L
(0+)
Thus
v
C
(0) A 2.4 2 . 4 A 4.8
Further,
v
C
'(0) 1000A + 3000B 4800 + 3000B
i
C
(0+)
C
0
Hence, B 1.6 and
v
C
(t) e
1000t
4.8cos(3000t) +1.6sin(3000 t) [ ] 2.4 V
SOLUTI ON TO PROBLEM 9.46. For all three cases, assuming i
L
is pointing downward,
v
C
(0) v
C
(0+) 0 and i
L
(0) i
L
(0+) 0.1 A
At t =,
v
C
() 0 and i
L
() 0.2 A
Further
i
C
(0+) i
L
(0+) +
10 v
C
(0+)
50
0.1 0.2 0.3 A
Lastly, all three cases are for a parallel RLC whose characteristic equation is:
s
2
+
1
RC
s +
1
LC
0
(a)
R =50; C =0.04e-3; L =0.625;
si =roots([1 1/(R*C) 1/(L*C)])
s1 =si(1); s2 =si(2);
si =
-400
-100
Hence,
v
C
(t) K
1
e
100t
+ K
2
e
400t
V
From, the initial conditions
v
C
(0+) =0 =K
1
+K
2
and
v
C
'(0) s
1
K
1
+ s
2
K
2
100K
1
400K
2

i
C
(0+)
C

0.3
0.04 10
3
7500
A =[1 1;si(2) si(1)];
b =[0;-7500];
K =A\b
K =
-25
25
Therefore
v
C
(t) 25 e
100t
e
400t
( )
V
(b)
R =50; C =0.04e-3; L =0.4;
si =roots([1 1/(R*C) 1/(L*C)])
s1 =si(1); s2 =si(2);
si =
-2.5000e+02
-2.5000e+02
Thus
v
C
(t) (K
1
+ K
2
t)e
250t
V
From IC's,
v
C
(0) K
1
0
v
C
'(0) s
1
K
1
+ K
2
K
2

i
C
(0+)
C
7500
Therefore
v
C
(t) 7500te
250t
V
(c)
L =0.2;
si =roots([1 1/(R*C) 1/(L*C)])
si =
-2.5000e+02 +2.5000e+02i
-2.5000e+02 - 2.5000e+02i
v
C
(t) e
250t
Acos(250t) + Bsin(250t) [ ] V
From ICs.
v
C
(0+) A 0
v
C
'(0) 250A + 250B 250B
i
C
(0+)
C
7500
in which case B =30. Thus
v
C
(t) 30e
250t
sin(250t) V
SOLUTI ON TO PROBLEM 9.47.
At t =0-, he capacitor is open and the inductor is a short. This together with the
continuity property implies v
C
(0-) =v
C
(0+) =10 V and i
L
(0-) =i
L
(0+) =1 A by
inspection.
Now, for t =0+, v
in
=0, replace capacitor and inductor with a voltage source and a
current source, respectively (values are those of the initial conditions). Solve for initial
capacitor current and initial inductor voltage to obtain:
v
L
(0+) =-10 V
i
C
(0+) =i
L
(0+) i
R1
i
R2
= 1 A
Notice that the resulting circuit is an undriven parallel RLC circuit with R
eq
=10//10 =5
.
R =5; C =0.01; L =4/3;
si =roots([1 1/(R*C) 1/(L*C)])
s1 =si(1); s2 =si(2);
si =
-15
-5
Hence,
v
C
(t) K
1
e
15t
+ K
2
e
5t
V
From, the initial conditions
v
C
(0+) =10 =K
1
+K
2
and
v
C
'(0) s
1
K
1
+ s
2
K
2
15K
1
5K
2

i
C
(0+)
C

1
0.01
100
A =[1 1;-15 -5];
b=[10; -100];
K =A\b
K =
5.0000e+00
5.0000e+00
v
C
(t) 5e
15t
+ 5e
5t
V
Verify with dsolve function in matlab: dsolve('D2y+20*Dy+75*y=0,y(0)=10,Dy(0)=-
100')
dsolve('D2y+20*Dy+75*y=0,y(0)=10,Dy(0)=-100')
ans =
5*exp(-5*t)+5*exp(-15*t)
0 0.1 0.2 0.3 0.4 0.5 0.6 0.7 0.8 0.9 1
0
1
2
3
4
5
6
7
8
9
10
SOLUTI ON TO PROBLEM 9.48.
For t =0-, there is no source present nor has there been a non-zero excitation. Hence,
v
C
(0-) =v
C
(0+) =0 and i
L
(0-) =i
L
(0+) =0
At t =0+, replace capacitor and inductor by 0-valued voltage and current sources to
obtain:
v
L
(0+) =10 V, i
C
(0+) =1 A
For t >0, we have a driven parallel RLC circuit with v
C
() =10 V. Thus
R =5; C =0.01; L =4/3;
si =roots([1 1/(R*C) 1/(L*C)])
s1 =si(1); s2 =si(2);
si =
-15
-5
and
v
C
(t) K
1
e
15t
+ K
2
e
5t
+10 V
From, the initial conditions
v
C
(0+) =0 =K
1
+K
2
+10
and
v
C
'(0) s
1
K
1
+ s
2
K
2
15K
1
5K
2

i
C
(0+)
C

1
0.01
100
A =[1 1;-15 -5];
b=[-10; 100];
K =A\b
K =
-5.0000e+00
-5.0000e+00
in which case
v
C
(t) 5e
15t
5e
5t
+10 V
To compute i
L
, note that i
L
() =1 A,
i
L
(t) K
1
e
15t
+ K
2
e
5t
+1 A
From, the initial conditions
i
L
(0+) =0 =K
1
+K
2
+1
and
i
L
'(0) s
1
K
1
+ s
2
K
2
15K
1
5K
2

v
L
(0+)
L

10
4 / 3
7.5
A =[1 1;-15 -5];
b=[-1; 7.5];
K =A\b
K =
-2.5000e-01
-7.5000e-01
i
L
(t) 0.25e
15t
0.75e
5t
+1 A
SOLUTI ON TO PROBLEM 9.49.
Input to this circuit is a superposition of the inputs in problems 9.47 and 9.48. So, the
output of the circuit here is a superposition of the output of the circuit in problems 9.47
and 9.48:
v
C
(t) 10e
15t
+10e
5t
10 V
For t >0, by linearity this is the difference of the zero-input circuit response (i.e., due to
the IC's as per problem 47) and the zero-state (zero ICs) as per problem 48.
SOLUTI ON TO PROBLEM 9.50.
(a)
v
C
(0-) =60 V, i
L
(0-) =0.1 A
(b) By continuity property,
v
C
(0+) =60, i
L
(0+) =0.1
(c)
Replace capacitor by voltage source of value 60 V and inductor by current source of
value 0.1 A.
v
L
(0+) +v
C
(0+) =60
v
L
(0+) =120
and
i
C
(0+) +i
R1
(0+) 1 i
L
(0+) =0
i
C
(0+) =1
(d) Req =120//600 =100 .
R =100; C =1e-3; L =2;
si =roots([1 1/(R*C) 1/(L*C)])
si =
-5.0000e+00 +2.1794e+01i
-5.0000e+00 - 2.1794e+01i
sig =-real(si(1)); wd =imag(si(1));
(e) Note that if the excitation of 60 V had remained forever, then i
L
() would be 0.1 A.
Therefore for the interval 0 <t <1,
i
L
(t) e
5t
Acos(21.794t) + Bsin(21.794t) [ ] + 0.1 A
(f)
i
L
(0+) =A +0.1 =0.1
A =0.2
i
L
'
(0+) 5A + 21.794B 1 + 21.794B
v
L
(0+)
L
60
Hence, B =2.707 and
i
L
(t) e
5t
0.2cos(21.794t) + 2.707sin(21.794t) [ ] + 0.1 A
(g) For t >1, the forcing function is zero iL() =0. Thus,
i
L
(t) e
5(t 1)
Acos 21.794(t 1) ( ) + Bsin 21.794(t 1) ( ) [ ]
A
From part (f) of the exp(-5t) term, we can guess that i
L
(1) approximates 0.1 and also that
v
C
(1) approximates its steady state value of 60 V. These may be off by percent or two,
but are good enough for our engineering calculations. It follows that v
L
(1+)
approximates 60 V.
i
L
(1+) A 0.1
and
i
L
'
(1+) 5A + 21.794B 0.5 + 21.794B
v
L
(1+)
L
30
Here B =1.35. Hence
i
L
(t) e
5(t 1)
0.1cos 21.794(t 1) ( ) 1.35sin 21.794(t 1) ( ) [ ]
A
*SOLUTI ON 9.51. To find the initial conditions, use the following equivalent circuit at t =
0

.
By inspection i
L
(0
+
) i
L
( 0

) 1 A and v
C
(0
+
) v
C
( 0

) 5 V.
To find the characteristic roots, set independent source to zero which means open
circuit the independent current source in figure P9.51. This leaves a series RLC with R
th
=10 . Hence
s
2
+
R
th
L
s +
1
LC
s
2
+100s + 2.5 10
4
0
Using MATLAB, we find
Rth =10;C =0.4e-3; L =0.1;
s12=roots([1 Rth/L 1/(L*C)])
s12 =
-50
-50
Since for t >0, the source is off, we use table 9.1, case 3 to obtain
i
L
(t) K
1
+ K
2
t ( )e
50t
A
It follows that 1 i
L
(0
+
) K
1
and
di
L
dt
(0
+
) 50K
1
+ K
2
50 + K
2

1
L
v
L
(0
+
) 10v
L
(0
+
)
To find v
L
(0
+
) we consider the equivalent circuit valid at 0
+
:
It follows that
v
L
(0
+
) 5 10 1 5 V
Hence
50 + K
2
50
or K
2
=0. Finally
i
L
(t) e
50t
u(t) A
SOLUTI ON TO PROBLEM 9.52. To find the initial conditions, use the following equivalent
circuit at t =0

.
By inspection i
L
(0
+
) i
L
( 0

) 1 A and v
C
(0
+
) v
C
( 0

) 5 V.
To find the characteristic roots, set independent source to zero which means open
circuit the independent current source in figure P9.51. This leaves a series RLC with R
th
=10 . Hence
s
2
+
R
th
L
s +
1
LC
s
2
+ 20s + 5 10
3
0
Using MATLAB, we find
Rth =10;C =0.4e-3; L =0.5;
s12=roots([1 Rth/L 1/(L*C)])
s12 =
-1.0000e+01 +7.0000e+01i
-1.0000e+01 - 7.0000e+01i
Since for t >0, the source is off, we use table 9.1, case 2 to obtain
i
L
(t) e
10t
Acos(70t) + Bsin(70t) [ ] A
It follows that 1 i
L
(0
+
) A and
di
L
dt
(0
+
) 10A + 70B 10 + 70B
1
L
v
L
(0
+
) 2v
L
(0
+
)
To find v
L
(0
+
) we consider the equivalent circuit valid at 0
+
:
It follows that
v
L
(0
+
) 5 10 1 5 V
Hence 10 + 70B 10 or B =0. Finally
i
L
(t) e
10t
cos(70t)u(t) A
0 0.05 0.1 0.15 0.2 0.25 0.3 0.35 0.4 0.45 0.5
-0.8
-0.6
-0.4
-0.2
0
0.2
0.4
0.6
0.8
1
SOLUTI ON TO PROBLEM 9.53. (a) v
C
(0) v
C
(0+) 150 10
3
80 12 V.
(b) i
L
(0-) =i
L
(0+) =150 mA
(c) For these values at 0+, the independent current source becomes an open circuit.
Replace the inductor and capacitor with current and voltage sources to represent the
initial conditions. Solve the resulting simple circuit to obtain:
v
L
(0+) =6 =12 120*0.15 V, i
C
(0+) =i
L
(0+) =150 mA
(d)
Rth =120;C =2/9 * 1e-3; L =0.6;
s12=roots([1 Rth/L 1/(L*C)])
s12 =
-1.5000e+02
-5.0000e+01
(e)
v
C
(t) K
1
e
150t
+ K
2
e
50t
V
From, the initial conditions
v
C
(0+) =12 =K
1
+K
2
and
v
C
'(0) s
1
K
1
+ s
2
K
2
150K
1
50K
2

i
C
(0+)
C
675
A =[1 1;-150 -50];
b=[12; -675];
K =A\b
K =
7.5000e-01
1.1250e+01
in which case
v
C
(t) 0.75e
150t
+11.25e
50t
V
(f)
i
L
(t) i
C
(t) C
dv
C
(t)
dt

2
9
10
3
150 0.75 ( )e
150t
+ 50 11.25 ( )e
50t
[ ]
0.025e
150t
+ 0.125e
50t
A
SOLUTI ON TO PROBLEM 9.54. (a) At t =0-, the independent current source is off, the
inductor is a short circuit, and the capacitor is an open circuit. By voltage division,
v
C
(0-) =v
C
(0+) =(80/100)50 =40 V
(b)
i
L
(0-) =i
L
(0+) =50/100 =0.5 A
(c) At t =0+, we have an independent current source. Also, we replace the inductor with
a current source and the capacitor with a voltage source to represent the initial conditions.
1.5 i
L
(0+) =i
C
(0+)
i
C
(0+) =1 A
Further,
v
C
(0+) +40 i
C
(0+) =80 i
L
(0+) +v
L
(0+)
v
L
(0+) =40 V
(d)
Rth =120;C =2/9 * 1e-3; L =0.6;
s12=roots([1 Rth/L 1/(L*C)])
s12 =
-1.5000e+02
-5.0000e+01
(e) In steady state, the capacitor is open and the inductor is a short in which case, X
f
=80
* 1.5 =120 V.
v
C
(t) K
1
e
150t
+ K
2
e
50t
+120 V
From, the initial conditions
v
C
(0+) =40 =K
1
+K
2
+120
and
v
C
'(0+) 150K
1
50K
2

i
C
(0+)
C
4.5 10
3
A =[1 1;-150 -50];
b=[-80; 4.5e3];
K =A\b
K =
-5
-75
in which case
v
C
(t) 5e
150t
75e
50t
+120 V
(f)
i
L
(t) 1.5 i
C
(t) 1.5 C
dv
C
(t)
dt
0.16667e
150t
0.83333e
50t
+1.5 A
SOLUTI ON TO PROBLEM 9.55.
(a) At t =0-, we replace the inductor by a short circuit and the capacitor by an open
circuit; hence
i
L
(0-) =i
L
(0+) =1 A
and
v
C
(0-) =v
C
(0+) =(40)(1) 20 =20 V
(b) At t =0+, we replace the capacitor by a voltage source of value 20 V and the
inductor by a current source of value 1 A. Since the inductor current is 1 A and the
independent current source outputs 1 A, no current flows through the branch containing
the capacitor. Therefore,
i
C
(0+) =0
Also, because of the zero current in the branch containing the capacitor, no voltage drop
occurs across the resistance in series with the capacitor. Therefore, the voltage across the
independent current source is v
C
(0+). Therefore,
v
L
(0+) =v
C
(0+) 40*i
L
(0+) = 20 V
(c) At steady state (large t), the capacitor becomes an open circuit and the inductor
becomes a short circuit. By inspection,
v
C
() =40 V
(d)
Rth =80;C =1/15 * 1e-3; L =0.1;
s12=roots([1 Rth/L 1/(L*C)])
s12 =
-5.0000e+02
-3.0000e+02
v
C
(t) K
1
e
500t
+ K
2
e
300t
+ 40 V
(e) From, the initial conditions
v
C
(0+) =20 =K
1
+K
2
+40
and
v
C
'(0+) 500K
1
300K
2

i
C
(0+)
C
0
A =[1 1;-500 -300];
b=[-20; 0];
K =A\b
K =
3.0000e+01
-5.0000e+01
in which case
v
C
(t) 30e
500t
50e
300t
+ 40 V
SOLUTI ON TO PROBLEM 9.56.
(a) At t =0-, we replace the inductor by a short circuit and the capacitor by an open
circuit; hence
i
L
(0-) =i
L
(0+) =1 A
and
v
C
(0-) =v
C
(0+) =(40)(1) 20 =20 V
(b) At t =0+, we replace the capacitor by a voltage source of value 20 V and the
inductor by a current source of value 1 A. Since the inductor current is 1 A and the
independent current source outputs 1 A, no current flows through the branch containing
the capacitor. Therefore,
i
C
(0+) =0
Also, because of the zero current in the branch containing the capacitor, no voltage drop
occurs across the resistance in series with the capacitor. Therefore, the voltage across the
independent current source is v
C
(0+). Therefore,
v
L
(0+) =v
C
(0+) 40*i
L
(0+) = 20 V
(c) At steady state (large t), the capacitor becomes an open circuit and the inductor
becomes a short circuit. By inspection,
v
C
() =40 V
(d)
Rth =80;C =62.5e-6; L =0.1;
s12=roots([1 Rth/L 1/(L*C)])
s12 =
-400
-400
v
C
(t) K
1
+ K
2
t ( )e
400t
+ 40 V
(e) From, the initial conditions
v
C
(0+) =20 =K
1
+40 K
1
= 20
and
v
C
'(0+) 400K
1
+ K
2
8000 + K
2

i
C
(0+)
C
0
in which case
v
C
(t) 20 + 8000t ( )e
400t
+ 40 V
SOLUTI ON TO PROBLEM 9.57.
Step 1:
dv
C
/dt =i
C
/C =2i
C
di
L
/dt =v
L
/L =0.5v
L
Step 2: i
R
=v
C
/2. From KCL, i
L
i
R
i
c
=0. Therefore, i
c
=i
L
v
c
/2.
Step 3: Similarly, v
in
v
L
v
c
=0, which implies v
L
=v
in
v
C
. Hence,
dv
C
/dt =2 i
L
v
C
di
L
/dt =0.5 v
in
0.5 v
C
Step 4. Eliminate terms in v
C
(see equation 9.47 in text) to obtain:
d
2
i
L
/dt
2
+di
L
/dt +i
L
=0.5 dv
in
/dt +0.5 v
in
SOLUTI ON TO PROBLEM 9.58.
(a) At t =0-,
v
C
(0-) =v
C
(0+) =0
i
L
(0-) =i
L
(0+) =0
For t between 0 and 1, we have a parallel RLC circuit, with R
eq
being the parallel
combination of the two 21.1333 resistors.
R =21.1333/2; C =15.7729e-3; L =0.1;
si =roots([1 1/(R*C) 1/(L*C)])
si =
-3.0000e+00 +2.5000e+01i
-3.0000e+00 - 2.5000e+01i
If there were no further switchings, then X
f
= 1 A. Hence,
i
L
(t) e
3t
Acos(25t) + Bsin(25t) [ ] +1 A
Applying the initial conditions,
i
L
(0+) =A +1 =0 A =1
and
i
L
'
(0+) 3A + 25B 3 + 25B
v
L
(0+)
L
0
Hence, for 0 t <1,
i
L
(t) e
3t
cos(25t) + 0.12sin(25 t) [ ] +1 A
Now, for the next interval, we need initial conditions. These are obtained from the above
equation for i
L
(t) at t =1.
i
L
(1) =e
-3
[-cos(25) 3/25 sin(25)] +1 =0.9514
and
i
L
'
(1) 3e
3
cos(25) + 0.12sin(25) [ ] e
3
25sin(25) + 0.12 25cos(25) [ ] 0.1671
The circuit is still a parallel RLC circuit, but now there is no source and R =1.268:
R =1.268; C =15.7729e-3; L =0.1;
si =roots([1 1/(R*C) 1/(L*C)])
si =
-2.5000e+01 +3.0002e+00i
-2.5000e+01 - 3.0002e+00i
Hence,
i
L
(t) e
25(t 1)
Acos(3(t 1)) + Bsin(3(t 1)) [ ]
Using initial conditions
A =0.9514
0.1671 =25A+3B B =7.8726
Thus, for 1 t <2
i
L
(t) e
25(t 1)
0.9514cos(3(t 1)) + 7.8726sin(3( t 1)) [ ] A
(b)
In period between 1 and 2 seconds, the response has a time constant of 1/25. So, when t =
2, 25 time constants would have passed from the time the switch is turned (t =1). This
means that the L and C currents and voltages would have settled almost identically to
their values at 0. A similar argument can be made for the other cycle. Thus the overall
response effectively becomes a periodic response equal to the response over 0 t <2
that reflects the periodicity of the switching.
(c)
0 1 2 3 4 5 6
0
0.2
0.4
0.6
0.8
1
1.2
1.4
1.6
1.8
Time in s
I
n
d
u
c
t
o
r

c
u
r
r
e
n
t

i
n

A
TextEnd
SOLUTI ON TO PROBLEM 9.59.
(a)
dv
c
/dt =i
c
/C =0.707i
c
di
L
/dt =v
L
/L =0.707v
L
To find expressions for iC and vL we use the following figure.
From this resistive circuit, i
C
= v
C
/1 +i
L
and v
L
=v
in
1i
L
v
C
v
C
0.707v
C
+ 0.707i
L

i
L
0.707v
C
0.707i
L
+ 0.707v
in
Using equation 9.47,
d
2
v
C
dt
2
+1.414
dv
C
dt
+ v
C
0.5v
in
si =roots([1 sqrt(2) 1])
si =
-7.0711e-01 +7.0711e-01i
-7.0711e-01 - 7.0711e-01i
(b) At steady state (large t), v
C
=0.5
v
C
(t) =e
-0.707t
[A cos(0.707t) +B sin(0.707t)] +0.5 V
At t =0,
v
C
(0-) =v
C
(0+) =A +0.5 =0 A =0.5
v
C
'
(0)
i
C
(0)
C

i
L
(0) v
C
(0)
C
0 0.707A + 0.707B
B =0.5
Thus
v
c
(t) =e
-0.707t
[0.5 cos(0.707t) 0.5 sin(0.707t)] +0.5 V
SOLUTI ON TO PROBLEM 9.60.
(a)
dv
C
dt
3i
C
di
L
dt

v
L
3
To eliminate i
C
and v
L
, consider
Hence
i
C
=i
in
v
C
/2 i
L
v
L
=v
C
12i
L
v
C
1.5v
C
3i
L
+ 3i
in

i
L

1
3
v
C
4i
L
Using equation 9.47,
d
2
v
C
dt
2
+ 5.5
dv
C
dt
+ 7v
C
3i
in
'
+12i
in
si =roots([1 5.5 7])
si =
-3.5000e+00
-2.0000e+00
Note: i
in
(t) =0 for t>0.
At t =0-, current source is off, inductor is a short circuit, and capacitor is an open circuit.
v
c
(0-) =v
c
(0+) =0
i
L
(0-) =i
L
(0+) =0
At t =0+, the current source is on. Replace the inductor and capacitor by current and
voltage sources to represent the initial conditions. Hence, i
C
(0+) =1 A. At t =,
v
C
() 1 12 / / 2 ( ) 1.7143. Thus,
v
C
(t) K
1
e
3.5t
+ K
2
e
2t
+1.7143 V
To find the constants,
v
C
(0) K
1
+ K
2
+1.7143 0
and
v
C
'
(0)
i
C
(0)
C
3 3.5K
1
2K
2
A =[1 1;-3.5 -2];
b =[-1.7143;3];
K =A\b
K =
2.8573e-01
-2.0000e+00
Therefore
v
C
(t) 0.28573e
3.5t
2e
2t
+1.7143
(b) From equation 9.47b,
d
2
i
L
dt
2
+ 5.5
di
L
dt
+ 7v
C
i
in
Since i
L
() 0.14286 A,
i
L
(t) K
1
e
3.5t
+ K
2
e
2t
+ 0.14286 A
Using the initial conditions,
i
L
(0) K
1
+ K
2
+ 0.14286 0
and
i
L
'
(0)
v
L
(0)
L

v
C
(0) 12i
L
(0)
L
0 3.5K
1
2K
2
A =[1 1;-3.5 -2];
b =[-1/7; 0];
K =A\b
K =
1.9048e-01
-3.3333e-01
Therefore,
i
L
(t) 0.19048e
3.5t
0.3333e
2t
+ 0.14286 A
SOLUTI ON TO PROBLEM 9.61.
(a)
dv
C1
/dt =0.5i
C1
dv
C2
/dt =0.5i
C2
Writing a node equation,
i
C1
+v
C1
/0.5 +(v
C1
v
C2
)/0.5 =0
i
C1
=4v
C1
+2v
C2
By symmetry,
i
C2
=4v
C2
+2v
C1
Hence
v
C1
2v
C1
+ v
C2
v
C2
v
C1
2v
C2
From equation 9.47a,
d
2
v
C
dt
2
+ 4
dv
C
dt
+ 3v
C
0
(b)
si =roots([1 4 3])
si =
-3
-1
Hence
v
C
(t) K
1
e
3t
+ K
2
e
t
(c)
v
C1
(0) =2 K
1
+K
2
=2
and
v
C1
'
(0)
i
C1
(0)
C

2(v
C2
(0) v
C1
(0)) 2v
C1
(0)
C

8 8
C
0 3K
1
K
2
Hence
v
C
(t) e
3t
+ 3e
t
V
*SOLUTI ON P9.62. (a) For this problem we need to define a voltage at the output of the
first op amp as shown below.
Also, let us relate the input and output voltages for an arbitrary leaky integrator as shown
below.
We now write a node equation at the inverting terminal of the op amp. Here
v
a
R
a
+
v
b
R
b
+ C
dv
b
dt
0
Equivalently
dv
b
dt

v
b
R
b
C

v
a
R
a
C
(1)
Now we apply the formula of (*) to the second stage of our given op amp circuit to
obtain:
dv
out
dt

v
out
RC
2

v
1
RC
2
(2)
where R =1 M and C
2
is to be determined.
Applying the formula of (*) to the first stage we obtain:
dv
1
dt

v
1
RC
1

v
s
R
1
C
1
(3)
The equations (2) and (3) form a coupled set of state equations which we can write as
v
out
v
1



1
]
1

1
RC
2

1
RC
2
0
1
RC
1






1
]
1
1
1
1
v
out
v
1



1
]
1
+
0
1
R
1
C
1




1
]
1
1
v
s
Using equation 9.47a of text we can write down the characteristic equation as
s
2
+
1
RC
2
+
1
RC
1



_
,

s +
1
RC
1
RC
2
s +
1
RC
2



_
,

s +
1
RC
1



_
,

0
We require that the natural frequencies be 4 and 12 in which case
1
RC
2
4,
1
RC
1
12
From MATLAB
R =1e6;
C2 =1/(R*4)
C2 = 2.5000e-07
C1 =1/(R*12)
C1 = 8.3333e-08
(b) For this part, the overall dc gain must be 10. The dc gain of the second stage is 1.
The dc gain of the first stage must be 10
10
6
R
1
; hence R
1
100 k.
(c) Since the roots are distinct and real, for t >0
v
out
(t) K
1
e
4t
+ K
2
e
12t
+ X
f
K
1
e
4t
+ K
2
e
12t
+ 10 V
where X
f
10V by part (b). The problem states that the capacitor voltages are initially
zero. Hence
0 v
C2
( 0) v
out
(0) K
1
+ K
2
+10
Equivalently
K
1
+ K
2
10 (1)
Also,
dv
out
dt
(0)
1
C
2
i
C2
( 0) 4K
1
12K
2
0 (2)
because i
C2
( 0) 0. This is so because v
C1
( 0) 0 v
1
( 0) means no current flows
through the 1 M input resistor to stage 2. This fact and the fact that v
C2
(0) 0, means
that no current flows through C
2
.
From equation 2, K
1
=3 K
2
. Substituting into equation (1) yields K
2
=5 and
hence K
1
=15. Finally,
v
out
(t) 15e
4t
+ 5e
12t
+10 V
*SOLUTI ON 9.63. This problem requires the characteristic equation in terms of A. For
this we may set v
in
=0 and the circuit becomes the one given below. Note the new label
v
1
.
The first step is to write a node equation at v
1
:
v
1
100
+10
4
d
dt
v
1
v
out
( ) +
1
100
v
1

v
out
A


_
,
0
Equivalently
dv
1
dt

dv
out
dt

100
A
v
out
200v
1
(*)
Now we write a node equation at v
g
=v
out
/A. Here,
10
4
A
dv
out
dt
+
1
100
v
out
A
v
1


_
,
0
Equivalently,
dv
out
dt
100v
out
+100 Av
1
(**)
Let us put (*) and (**) in matrix form to obtain:
1 1
0 1



1
]
1
v
1
v
out



1
]
1

200 100 / A
100A 100



1
]
1
v
1
v
out



1
]
1
We can now solve this to obtain the state equations
v
1
v
out



1
]
1

1 1
0 1



1
]
1
200 100 / A
100A 100



1
]
1
v
1
v
out



1
]
1

200 +100A 100 +100 / A


100 A 100



1
]
1
v
1
v
out



1
]
1
Compare these equations with equation 9.37 and use the formula of 9.47b to obtain the
following second order differential equation in v
out
:
d
2
v
out
dt
2
+ 300 100A ( )
dv
out
dt
+10
4
v
out
0
The discriminant of this characteristic equation is plotted below for 0 <A <3. For values
of A >3, the circuit is unstable. A negative value of the discriminant indicates
underdamped (1 <A <3) and a positive value overdamped (0 <A <1). For A =1, we
have critical damping.
Solution 9.64 We can write two state equations as follows:
(i) From the definition of a capacitor,
dv
C1
dt
10
7
i
C1
dv
C2
dt
10
9
i
C2
(ii) From KVL and Ohm's law
dv
C1
dt
10
6
v
i
v
C1
v
C2
( ) 10
6
v
C1
10
6
v
C2
+10
6
v
i
dv
C2
dt
10
8
v
i
v
C1
v
C2
0.01v
C2
( ) 10
8
v
C1
1.0110
8
v
C2
+10
8
v
i
Casting these two equations into a second order differential equation, as described in the
text:
d
2
v
C2
dt
2
+1.02 10
8
dv
C2
dt
+10
12
v
C2
10
8
dv
i
dt
The characteristic equation for this differential equation has real roots:
si =roots([1 1.02e8 1e12])
si =
-1.0199e+08
-9.8049e+03
Since the capacitors become open circuits, v
C2
() =0 and v
C1
() =v
i
.
v
C2
t ( ) K
1
e
1.0210
8
t
+ K
2
e
9.810
3
t
V
Applying IC's:
v
C2
0 ( ) K
1
+ K
2
0
Also,
v
C2
'
(0)
i
C2
(0+)
C
2

0.1
10
9
1.02 10
8
K
1
9.8 10
3
K
2
Thus
v
C2
'
(0) 1 1.02K
1
9.8 10
5
K
2
b =[0; 1];
A =[1 1; -1.02 -9.8e-5];
K =A\b
K =
-9.8049e-01
9.8049e-01
v
C2
t ( ) 0.9805e
1.0210
8
t
+ 0.9805e
9.80510
3
t
V
t =0:1/(abs(s1)*100):100/abs(s1);vc =-0.9805*exp(s1*t) +0.9805*exp(s2*t);
plot(t,vc)
grid
xlabel('Time in s')
ylabel('vout in V')
0 0.2 0.4 0.6 0.8 1
x 10
-6
0
0.2
0.4
0.6
0.8
1
Time in s
v
o
u
t

i
n

V
TextEnd
Solution 9.65. First, derive the differential equation by writing state equations:
dv
C
dt
3i
C
di
L
dt

v
L
3
Now, assume that the capacitor is a voltage source and the inductor is a current source,
and write by KCL
i
C

v
C
2
+
v
C
R
N
i
L
And by KVL:
v
L
v
C
12i
L
Substitute into the differential equations:
dv
C
dt

3
2
+
3
R
N



_
,

v
C
3i
L
di
L
dt

1
3
v
C
4i
L
Using equation 9.47 we obtain
d
2
v
C
dt
2

3
2
+
3
R
N



_
,

4




1
]
1
1
dv
C
dt
+
3
2
+
3
R
N



_
,

4 ( ) +1




1
]
1
1
v
C
0
or equivalently
d
2
v
C
dt
2
+ 5.5
3
R
N



_
,

dv
C
dt
+ 7
3
R
N



_
,

v
C
0
For constant amplitude oscillations, the middle term should be zero, which means that R
N
=3/5.5 =0.54545 . Thus the negative resistance is R
N
=0.54545 .
SOLUTI ON 9.66. The problem data is
i
1
(t) =I
m
sin(t +) A
R
1
=500 +100(I
m
0.01), R
2
=500
Suppose it starts out with exponentially growing amplitude. R
1
will increase with
increasing amplitude. This changes the location of the roots until equilibrium is reached
where the roots and the amplitude are stable. This is achieved when the roots of the
characteristic equation describing the output voltage are purely imaginary, i.e.,
(R
1
R
2
)/(R
1
R
2
C) =0
R
1
=R
2
=500 =500 +100(I
m
0.01)
I
m
=0.01
(a)

0
=1/[(500)(1)] =2 k-rad/s
(b) Amplitude of i
1
is 0.01. Thus i
1
=0.01 sin(
0
t) A. Let v
1
=V
m
cos(
0
t +) V.
Then
dv
1
/dt =
0
V
m
sin(
0
t +) =(0.01/C) sin(
0
t)
V
m
=0.01/(
0
C) =5 V and = rad
Now, v
2
+v
1
+i
1
R
1
3v
2
=0 v
2
=v
1
/2 +i
1
R
1
/2. Finally,
v
out
3v
2
1.5v
1
+1.5R
1
i
1
7.5cos(
0
t + ) + 7.5sin(
0
t) V
Hence the amplitude of v
out
(t) is: 7.5 2 V.
SOLUTI ON 9.67. (a)
n

1
RC
10
4
rad/s
(b) From equation 9.47a,
d
2
v
1
dt
2
+
1
R
2
C
2
v
1
0
Hence,
v
1
t ( ) Acos
1
RC
t


_
,
+ Bsin
1
RC
t


_
,
V
Using the value of v
1
(0) we have,
v
1
0 ( ) A 5
To compute the second initial condition,
v
1
'
0 + ( )
i
C1
(0+)
C

i
R1
(0+)
0.110
6
10
4
B
But,
i
R1
(0+)
3v
2
(0+) v
2
(0+) v
1
(0+)
R
1

2v
2
(0+) v
1
(0+)
10
3
0.005 Amps
Hence
v
1
'
0 + ( )
5 10
3
0.110
6
10
4
B
Hence B =5.
v
1
t ( ) 5cos 10,000t ( ) 5sin 10,000 t ( ) V
0 0.2 0.4 0.6 0.8 1
x 10
-3
-8
-6
-4
-2
0
2
4
6
8
Time in s
V
o
l
t
a
g
e

v
1

i
n

V
TextEnd
*SOLUTI ON 9.68. For this problem, R
2
=10 k should be R
f
=10 k and R
1
=1 k
should be R
2
=1 k.
(a) For sustained sinusoidal oscillation, R1 =R2 =1 k. From equation 9.59,

1
R
1
C
10
4
rad/s or 1.5915 kHz
(b) From figure P9.68, to obtain an R1 =1 k, I
R1,peak
0.2mA. Therefore,
i
R1
(t) 0.2sin(
0
t + ) mA for appropriate . Since C
dv
1
dt
i
C1
i
R1
0.2sin(
0
t + )
mA, we know that v1 has the following form:
v
1
(t) V
1m
cos(
0
t + )
In which case
C
dv
1
dt
C
d
dt
V
1m
cos(
0
t + ) ( ) CV
1m

0
sin(
0
t + + ) 0.2sin(
0
t + ) mA
Therefore CV
1m

0
0.2 10
3
. It follows that V
1m
0.2 volts. Here V
1m
is the peak
value of the sinusoid. However, the op amp peak output voltage with respect to ground,
as shown in problem 66, is 1.5 2V
1m
. Also, for such a small amplitude, we expect the
output waveform to be quite close to sinusoidal. By choosing a different lamp (R
1
) with
a different characteristic, we can obtain larger peak output voltages.
SOLUTI ON 9.69. (a) Note that the capacitor is like an open circuit and the inductor is
like a short circuit at t=0-. Thus, we can obtain the capacitor voltage by voltage division:
v
C
0 ( ) 10
4
5
8 v
C
0 + ( )
Similarly, the inductor current is obtained by applying Ohms Law:
i
L
0 ( )
10
5
2 i
L
0 + ( )
(b) Here, we note that the new initial conditions are just 2.5 times the values that we just
obtained in part (a). This can be achieved by simply changing the input voltage source,
from 10 to 25 V.
1
PROBLEM SOLUTIONS CHAPTER 10
SOLUTION 10.1 Using KCL, we can write
C
dv
C
dt
+
v
C
R
= i
in
t ( )
Dividing by C:
dv
C
dt
+
1
RC
v
C
=
i
in
t ( )
C
We know that i
in
t ( ) = 20sin 400t ( ) mA, which can be represented by a complex exponential,
i
in
t ( ) = Re 20e
j 400t
e
j/2
[ ]
mA. For convenience we will simply let i
in
t ( ) = 20e
j 400t
e
j/2
mA, knowing
that we must take the real part to complete our solution. The output voltage will also be reparesented as a
complex exponential:
v
C
t ( ) = V
m
e
j 400t + ( )
= V
m
e
j 400t
e
j
Substituting this expression into the differential equation and canceling e
j400t
:
j400V
m
e
j
+
V
m
RC
e
j
=
20 10
3
e
j/2
C
Thus
V
m
e
j
1
RC
+ j 400




=
20 10
3
C
V
m
e
j
=
j4000
1000 + j400
= 3.714 111.8
o
where the values for R = 100 and C = 5 mF were substituted in. Thus,
V
m
= 3.714
= 0 tan
1
400
1000




= 111.8
Taking into account a 90
o
phase shift we obtain
v
C
t ( ) = 3.714cos 400t 111.8 ( ) = 3.714sin 400t 21.8 ( ) V
and
i
out
t ( ) = 18.57sin 400t 21.8 ( ) mA
SOLUTION 10.2 From KCL and component definitions:
2
i
in
t ( )
v
L
25
i
L
= 0
0.1
25
di
L
dt
+ i
L
= i
in
t ( )
di
L
dt
+ 250i
L
= 250i
in
t ( )
We represent the input signal by the complex exponential: i
in
t ( ) = 0.2e
j 250t
A and the unknown current
can be represented as i
L
(t) = I
L
e
j 250t + ( )
.
Substituting this into the differential equation and canceling e
j250t
:
j250I
L
e
j
+ 250I
L
e
j
= 50
Thus
I
L
e
j
j250 + 250 ( ) = 50 I
L
e
j
=
50
250 + j250
= 0.14142 45
o
and
I
L
= 0.141, = 45 i
L
t ( ) = 0.141cos 250t 45 ( ) A
SOLUTION 10.3. Construct differential equation by KVL and device definitions:
v
in
t ( ) 0.5
di
L
dt
200i
L
= 0
di
L
dt
+ 400i
L
= 2v
in
t ( )
We represent v
in
t ( ) as the complex exponential function, v
in
t ( ) = 20e
j 400t
V. The current in the inductor
has the form: i
L
= I
L
e
j 400t + ( )
. Substituting into the differential equation and canceling e
j400t
:
j400I
L
e
j
+ 400I
L
e
j
= 40
Thus
I
L
e
j
j + 400 ( ) = 40 I
L
e
j
=
40
400 + j 400
= 0.070711 45
o
and
I
L
= 0.0707, = 45, i
L
t ( ) = 70.7cos 400t 45 ( ) mA
Hence,
v
out
t ( ) = 14.14cos 400t 45 ( ) V
SOLUTION 10.4. Construct differential equation using KVL and device definitions:
3
v
in
t ( ) v
C
C
dv
C
dt
R = 0 RC
dv
C
dt
+ v
C
= v
in
t ( )
The output voltage is defined as:
v
out
t ( ) = v
in
t ( ) v
C
t ( )
This means that finding v
C
is enough to be able to obtain the output voltage. The input voltage is
represented by the complex exponential:
v
in
t ( ) = 20e
j 250t
e
j/2
V
and v
C
t ( ) = V
m
e
j 250t + ( )
. Substituting into the differential equation, dividing by e
j250t
, and rearranging:
j250RCV
C
e
j
+ V
C
e
j
= j20 V
C
e
j
j250RC +1 ( ) = j20
V
C
e
j
=
j20
1 + j
= 14.142 135
o
Now
V
out
e
j
= j20 V
C
e
j
V
out
e
j
=10 j10 =14.142 45
o
Thus, in the time-domain,
v
out
t ( ) = 14.142cos 250t 45 ( ) V
SOLUTION 10.5. The circuit is identical to that of problem 10.1. Thus,
RC
dv
s
dt
+ v
s
= Ri
s
t ( ) Ri
s
t ( ) RC
dv
s
dt
= v
s
Moreover, the complex exponential solution is given by
v
s
(t) = V
m
e
j
= 223.6e
j 63.43
o
V
Hence
R jRC223.6e
j 63.43
o
= 223.6e
j 63.43
o
= 100 j200
i.e.,
R jRC(100 j200) = R 200RC j100RC =100 j200
4
Thus RC = 2 s, R =100 + 200RC = 500 , and C = 4 mF.
SOLUTION 10.6. First use KVL
v
in
v
L
v
c
v
out
= 0 v
in
L
di
L
dt

1
C
i
L
dt

Ri
L
= 0
Differentiating this equation, rearranging, and dividing by L,
d
2
i
L
dt
2
+
R
L
di
L
dt
+
1
LC
i
L
=
1
L
dv
in
dt
We represent the input signal with = 10
4
as v
in
t ( ) =100e
j t
e
j/2
V and i
L
(t) = I
L
e
j t
e
j
A.
Substituting these two expressions into the differential equation and dividing out e
j t
:

2
I
L
e
j
+ j
R
L
I
L
e
j
+
1
LC
I
L
e
j
=
j100 j
L
I
L
e
j
1
LC

2
+ j
R
L




=10
8
I
L
e
j
=
10
8
75 10
6
+ j10
8
= 0.48 j0.64 = 0.8 126.87
o
Solving for the magnitude and angle (by hand or using MATLAB):
i
L
t ( ) = 0.8cos 10,000t 126.87 ( ) = 0.8sin 10,000t 36.87 ( ) A
and
v
out
t ( ) = 80sin 10,000t 36.87 ( ) V
SOLUTION 10.7. Using standard reference directions, from KCL and component definitions,
i
in
= i
R
+ i
L
+ i
C
=
v
out
R
+ i
L
(0) +
1
L
v
out
( ) d
0
t

+ C
dv
out
dt
Taking a second derivative and dividing by C yields
d
2
v
out
dt
2
+
1
RC
dv
out
dt
+
1
LC
v
out
=
1
C
di
in
dt
5
We now let = 2500 rad/s and represent i
in
(t) by the real part of the complex exponential 0.02e
j t
.
Further we represent v
out
(t) as the real part of the complex exponential V
m
e
j( t + )
= V
m
e
j t
e
j
.
Substituting these expressions into the differential equation and taking the indicated derivatives yields
V
m
( j )
2
e
j t
e
j
+
V
m
RC
j e
j t
e
j
+
1
LC
V
m
e
j t
e
j
=
0.02
C
j e
j t
Observe that e
j t
cancels out on both sides of this equation producing
V
m
e
j
1
LC

2
+
j
RC




=
j0.02
C
Hence
V
m
e
j
=
j0.02
C
1
LC

2
+
j
RC




=1.28 + j0.96
This was obtained using MATLAB as follows:
w = 2500;
L = 40e-3;
C = 1e-6;
R = 100;
Vout = j*0.02*w/(1/L - C*w^2 +j*w/R)
Vout = 1.2800e+00 + 9.6000e-01i
magVout = abs(Vout)
magVout = 1.6000e+00
angVout = angle(Vout)*180/pi
angVout = 3.6870e+01
Therefore
v
out
(t) = 1.6cos(2500t + 36.87
o
) V
SOLUTION 10.8. (a) From KCL: 315 545 I = 0 I = 315 545. In MATLAB,
Ibar = 3*exp(j*15*pi/180) - 5*exp(j*45*pi/180)
6
Ibar =-6.3776e-01 - 2.7591e+00i
abs(Ibar)
ans =2.8318e+00
angle(Ibar)*180/pi
ans =-1.0302e+02
Therefore,
i t ( ) = Re Ibar { } = 2.83cos t 103 ( ) A
(b) From KCL,
Ibar = (1 + 2*j) - (-2 + j*6)
Ibar =
3.0000e+00 - 4.0000e+00i
abs(Ibar)
ans = 5
angle(Ibar)*180/pi
ans =-5.3130e+01
Therefore i t ( ) = 5cos 50t 53 ( ) A.
SOLUTION 10.9. We define a Gaussian surface encompassing the three bottom nodes. Thus, KCL
dictates that the sum of 4 currents be zero:
Ibar = -(-2-j*8) + (3 +j*12) + 10
Ibar =
1.5000e+01 + 2.0000e+01i
abs(Ibar)
ans =
25
angle(Ibar)*180/pi
ans =
5.3130e+01
Therefore i t ( ) = 25cos 1000t + 53.13 ( ) A.
SOLUTION 10.10. First represent the time-domain functions as phasors:
7
V
1
= 20, V
2
= 2 2 45
Then, by KVL
V1 = 2; V2 = 2*sqrt(2)*exp(-j*pi/4);
VL = V1 - V2
VL =
0 + 2.0000e+00i
Therefore, V
L
= 290 V and v
L
t ( ) = 2cos t + 90 ( ) = 2sin( t) V
SOLUTION 10.11. Apply KVL by simply following the loop defined by the independent voltage sources:
V
x
= 4 j 2 j 11+ (1 j) (1+ j) = 2
SOLUTION 10.12. First note that I
out
=
V
R
R
. V
R
= j10 + (5 j5) 10 V
R
= 5 j15 V.
Thus, I
out
=
5 j15
5
= 1 j 3 = 3.16 108.4

i
out
(t) = 3.16cos 500 t 108.4 ( ) A.
SOLUTION 10.13. Using a Gaussian surface,
I
y
= (2 + j3) (1+ j2) + (1 j5) = 2 j10 = 10.2 101.3
o
A.
Therefore, i t ( ) =10.2cos 2000 t 101.3 ( ) A. Now applying KVL,
V
x
= (2 + j2) + (2 + j 3) (1 4 j) = 3 + 9 j
Vx=2+2*j+2+j*3-1+4*j
Vx =
3.0000e+00 + 9.0000e+00i
abs(Vx)
ans =
9.4868e+00
angle(Vx)*180/pi
8
ans =
7.1565e+01
Thus, v
x
t ( ) = 9.487cos 2000 t + 71.6 ( ) V.
SOLUTION 10.14. (a) At = 1000 rad/s,
Y
C
( j ) = j C = j4.7 10
3
C =1.496 F
So, at = 50 , Y
C
( j ) = j C = j2.3499 10
4
S Z
C
(j ) = j4255 .
(b) At = 1000 rad/s, Z
L
= j L = j18.85 L = 6 mH. Since impedance is proportional to
frequency, multiplying the frequency by 20 means the impedance is multiplied by 20. Thus, at
= 20 (10
3
) rad/s,
Z
L
= j18.85 20 = j 377 Y
L
= j2.652 mS
SOLUTION 10.15. The input voltagephasor as 20. By inspection:
I
1
= j C V
C
= j10 0.1 2 = j2
From the definition of a dependent V-source and an inductor:
I
2
=
5I
1
j L
=
j10
j2
= 5
Finally,
V
out
= 5 2 I
1
+ I
2
( ) = 10 5 + j2 ( ) = 50 + j20 = 53.8521.8
v
out
t ( ) = 53.85cos 10t + 21.5 ( ) V
SOLUTION 10.16. Using KCL and the definition of a resistor:
V
10
= 10 60 390 ( ) = 10(6 3j) = 60 j 30 V
Thus,
Z j
0
( ) =
60 j 30
j3
= 10 j20
9
And, the combination of this Z( j
0
) with the 10 resistance, at this frequency, is 10 j5.
Z=-10-j*20; R = 10;
ZZ = R*Z/(R+Z)
ZZ =1.0000e+01 - 5.0000e+00i
C = 1/(-5*j*j*2000*pi)
C =3.1831e-05
This is equivalent to a 10 resistor in series with a 31.83 F capacitor.
SOLUTION 10.17.
w = 2*pi*60;
VL = 3 +12*exp(-j*30*pi/180) + 6 -12*exp(j*30*pi/180)
VL =9.0000e+00 - 1.2000e+01i
ZL = j*w/60
ZL =0 + 6.2832e+00i
IL = VL/ZL
IL =-1.9099e+00 - 1.4324e+00i
abs(IL)
ans =2.3873e+00
angle(IL)*180/pi
ans =-1.4313e+02
Therefore, i
L
(t) = 2.387cos(120t 143.1
o
) A.
Solution 10.18 (a)
First represent the inputs with their phasors: I
s1
=1030 = 8.66 + j5 and V
s2
=1000.
The admittance of the RC combination is: Y
1
=
1
R
+ j C = 0.1+ j0.1. Using superposition and noting that
a 0-volt V-source is a short circuit and a 0-amp current source is an open circuit,
I
x
= Y
1
V
s2
I
s1
=1.3397 + j5 = 5.176475
o
A
Therefore,
i
x
t ( ) = 5.176cos 100t + 75 ( )
(b) Let i
x1
be the contribution to i
x
generated by the current source and i
x2
the contribution generated by
the voltage source. Then i
x1
(t) = 10cos(50t + 30
o
) A, and i
x2
(t) = 14.142cos(100t + 45
o
) A since
10
Vs2 = 100;
w = 100;
R = 10; C = 1e-3;
Y1 = 1/R + j*w*C
Y1 =1.0000e-01 + 1.0000e-01i
Ix2 = Y1*Vs2
Ix2 =1.0000e+01 + 1.0000e+01i
abs(Ix2)
ans =1.4142e+01
angle(Ix2)*180/pi
ans =45
Therefore i
x
(t) = 10cos(50t + 30
o
) 14.142cos(100t + 45
o
) A.
Solution 10.19 First of all, write out the given values: = 200 rad/s, I
1
= 0.590 = 0.5 j A, and
V
s2
= 40. From KVL V
s1
= 3I
1
+ j LI
1
+
I
1
j C
+ V
s2
which leads to:
w = 200; I1 = 0.5j; Vs2 = 4;
Vs1 = 3*I1 + j*w*0.04*I1 + I1/(j*w*1e-3) +Vs2
Vs1 =2.5000e+00 + 1.5000e+00i
abs(Vs1)
ans =
2.9155e+00
angle(Vs1)*180/pi
ans =
3.0964e+01
Therefore v
s1
(t) = 2.9155cos(200t + 30.964
o
) V.
Solution 10.20 The phasor for the input can be written as = 1000 rad/s and V
in
= 1060 = 5 + j8.66
V. The currents can be obtained easily by applying Ohms law for phasors:
11
I
R
=
1060
500
= 0.0260 A, I
L
=
1060
j1000 0.25
= 0.04 30 A, and
I
C
= 1060 j1000 2 10
6
( )
= 0.02150 A. Thus
I
in
= 0.0260 + 0.04 30 + 0.02150 = 0.028315 A
and
i
in
t ( ) = 0.0283cos 1000t +15 ( ) A
using the following MATLAB code:
Vin = 10*exp(j*60*pi/180)
Vin =5.0000e+00 + 8.6603e+00i
R = 500; L = 0.25; C = 2e-6;
w = 1e3;
IR = Vin/500
IR =1.0000e-02 + 1.7321e-02i
IL = Vin/(j*w*L)
IL =3.4641e-02 - 2.0000e-02i
IC = j*w*C*Vin
IC =-1.7321e-02 + 1.0000e-02i
Iin = IR + IL + IC
Iin =2.7321e-02 + 7.3205e-03i
abs(Iin)
ans =2.8284e-02
angle(Iin)*180/pi
ans =1.5000e+01
Solution 10.21 In MATLAB
Iin = -100*j*1e-3; R = 100;
L = 0.04; C = 2e-6; w = 2500;
VR = R*Iin
VR =0 - 1.0000e+01i
VL = j*w*L*Iin
VL =10
VC = Iin/(j*w*C)
VC =-20
Vin = VR + VL + VC
12
Vin =-1.0000e+01 - 1.0000e+01i
abs(Vin)
ans =1.4142e+01
angle(Vin)*180/pi
ans =-135
Therefore, v
in
t ( ) =14.14cos 2500t 135 ( ) V.
Solution 10.22 (a) Here, i
1
t ( ) = 0.6cos 200t ( ) A and v
out
t ( ) = 20sin 200t ( ) = 20cos 200t 90 ( ) V.
For = 200 rad/s, the phasors are by inspection: I
1
= 0.60 A, V
out
= 20 90 V .
(b) Write down the resistor, inductor, and capacitor current phasors, given V
out
:
I
R
=
20 90
1/0.03
= 0.6 90 = j0.6
I
L
=
20 90
j200 0.1
=1 180 = 1
I
C
= 20 90 j200 0.4 10
3
( )
= 1.60
Now, by KCL
I
2
= I
R
+ I
L
+ I
C
I
1
= j0.6
where we have substituted the above values of branch currents. The time-domain function is:
i
2
t ( ) = 0.6cos 200t 90 ( ) = 0.6sin 200t ( ) A
Solution 10.23 (a) = 400 rad/s and V
in
= 200 V. We can easily use the voltage divider formula for
phasors and substitute values to obtain:
V
out
=
200
200 + j L
V
in
=
20
1 + j
= 14.14 45
in which casev
out
t ( ) = 14.14cos 400t 45 ( ) V
(b) = 250 rad/s and V
in
= 20 90 = j20 V. Again, we can easily use voltage division:
V
out
=
400
400 +
1
j C
V
in
=
20 90
1 j
=14.14 45 V
Thus, in the time-domain,
v
out
t ( ) = 14.142cos 250t 45 ( ) V
13
Solution 10.24 (a) = 10,000 rad/s and V
in
= 100 90 = j100 V. Apply the voltage divider
formula:
V
out
=
100
100 + j L +
1
j C
V
in
=
100
100 + j100 j0.25
V
in
= 80 126.87
The steady-state response is thus,
v
out
t ( ) = 80sin 10,000t 36.87 ( ) V
The phasor method provides for a much easier way of obtaining the steady-state response.
(b) Here, = 2500 rad/s and I
in
= 0.020 A. Now, apply current division:
I
R
=
0.01
0.01 + j C +
1
j L
I
in
=
0.01
0.01 + j0.0025 j0.01
I
in
= 0.01636.86
By Ohms law:
V
out
= 100I
R
=1.636.87 V
Therefore
v
out
(t) = 1.6cos(2500t + 36.87
o
) V
Solution 10.25 (a) It is easier to find the admittance first:
Y
in
( j100) =
1
j L
+ j C = j0.2 + j12.5 = j12.3
Z
in
( j100) = j0.0813
(b) Y
in
( j100) =
1
25 j
=
1
j100 0.05
+ j100C. Hence, in MATLAB,
C = (1/(25*j) - 1/(j*w*L) )/(j*100)
C = 1.6000e-03
Solution 10.26 (a)
Z
in
( j ) =
1
j C
+
R j L
R + j L
=
j
C
+
j RL
R + j L
= 9.975 0.0072
R = 10; L = 0.1; C = 1e-3; w = 2e3;
Zin = -j/(w*C) + j*w*R*L/(R + j*w*L)
Zin = 9.9751e+00 - 1.2469e-03i
14
abs(Zin)
ans = 9.9751e+00
angle(Zin)*180/pi
ans = -7.1620e-03
(b) As the frequency increases, the capacitor becomes a short circuit and the inductor becomes an open
circuit. Thus, the impedance approaches R. Analytically,
lim

Z
in
( j ) = lim

1
j C
+ lim

R
R
j L
+1
= R
(c) Z
in
( j ) =
j
C
+
j RL(R j L)
R
2
+
2
L
2
= j
LR
2
R
2
+
2
L
2

1
C






+
R
2
L
2
R
2
+
2
L
2
. It follows that w must satisfy
2
LR
2
C = R
2
+
2
L
2
or equivalently
2
=
R
2
LR
2
C L
2
=
1
LC 1
L
CR
2






. For the given component
values 1
L
CR
2





= 0; hence there is no finite value of frequency for which the impedance is real.
SOLUTION 10.27. We note that the input admittance is given by:
Y
in
( j ) =
1
100 + j 0.1
+ j 10
6
=
100 j 0.1
10
4
+ 0.01
2
+ j 10
6
=
100
10
4
+ 0.01
2
+ j 10
6
j
0.1
10
4
+ 0.01
2
Thus, must satisfy 10
5
=10
4
+ 0.01
2
or equivalently, = 10
7
10
6
= 3000 rad/s.
SOLUTION 10.28. (a) As usual we will use MATLAB.
R1 = 20; R2 = 10;
L = 0.04; C = 0.6e-3;
w = 250;
15
Yin = 1/R1 + 1/(R2 + j*w*L) + j*w*C
Yin = 1.0000e-01 + 1.0000e-01i
Hence
Y
in
( j250) = 0.1+ j0.1 S
(b) For this part we observe that
Y
in
( j250) =
1
R
1
+
1
R
2
+ jL
+ jC = 0.1 0.05 j + j250C
For this to be real, the imaginary part must be zero, i.e.,
C =
0.05
250
=
1
5000
= 0.2 mF
Solution 10.29 (a) We can derive an expression for the input impedance by noting that it is the series
combination of the resistance and the inductor/capacitor pair connected in parallel. Thus,
Z
in
( j ) = R +
1
j C
j L
1
j C
+ j L
= R j
L
C
L
1
C
Equating the real and imaginary parts of the given impedance, R = 4 and
L
LC1/
=
L
2L 0.25
= 2 L = 0.1 H
(b) At zero inductance, the above reactance is zero. Also, at L = 0.125, the denominator of the above
reactance is zero, which means that the reactance is infinite.
Solution 10.30 (a) First derive an expression for the input impedance as a function of frequency:
Z
in
j ( ) = 5 + j L +
1
j C
= 5 + j L
1
C




We want the imaginary part to be equal to zero. Thus,
L
1
C
= 0
2
=
1
LC
= 2500 rad/s
16
where we have substituted the values of L and C.
The magnitude of the impedance is minimum when the imaginary part is zero, and Z
in
(j2500) = 5.
(b) Derive an expression for the admittance:
Y
in
( ) =
1
5
+ j C +
1
j L
=
1
5
+ j C
1
L




Again, the imaginary part is equal to zero when =
1
LC
or = 2500, at which point the admittance is
0.2.
Solution 10.31 The input admittance is Y
in
( ) =
1
R
+ j C = 0.008+0.004. Equating this to the given
admittance at = 500yields R = 125 , C = 0.004 or C = 8 F.
Solution 10.32 Derive an expression for the admittance at = 1000:
Y
in
( j ) =
1
j L
+
0.25 j C
0.25 + j C
= j0.5 +
j0.125
0.25 + j0.5
= 0.2 j0.4 S
Note that this is equivalent to a 0.2 S conductance (i.e. 5 resistance) in parallel with a 2.5 mH inductance
(at the given frequency!). Now, the impedance is:
Z
in
=
1
0.2 j0.4
= 1+ j2
This is equivalent to a resistance of 1 in series with a 2 mH inductance (at the given frequency).
Solution 10.33 The current is zero when the input impedance of the parallel combination of inductor and
capacitor is infinite. The latter is given by:
Z
LC
( j ) =
1
j C
j L
1
j C
+ j L
= j
L
C
L
1
C
The magnitude of this is infinite when
r
L
1
C
= 0
r
2
=
1
LC

r
=15,811 rad/s. Observe that
I
s
=
V
s
R + Z
LC
( j
r
)
= 0
17
Hence i
s
(t) = 0 at
r
= 15,811 rad/s. At this frequency, the voltage across the LC tank is equal to the input
voltage (since there is no drop across the resistor).
Solution 10.34 (a) By inspection:
Z
in
=
2 j 0.4
2 + j 0.4
=
j 0.8
2 + j 0.4
Y
in
=
1
R
+
1
j L
= 0.5 j
2.5
(b) Use the current divider formula, and substitute the given frequency, to obtain:
I
L
=
R
R + j L
2 =
2 2
2 + j2
=1 45
and i
L
(t) = cos(5t 45
o
) A.
Solution 10.35 (a) Using voltage division, V
out
=
R
R
j
C
V
in
=
j RC
1+ j RC
V
in
which
V
out
= 90
o

in
tan
1
RC ( ) and V
out
=
RCV
in
1+
2
R
2
C
2
(b) For this part, we need to make sure that tan
1
RC ( ) = 45 RC = 1 =1/ RC .
(c) At this frequency,
V
out
V
in
=
1
RC
RC
1+
1
R
2
C
2
R
2
C
2
=
1
2
Solution 10.36 Here, = 1/ RC and
V
C
V
in
=
1
j C
1
j C
+ R
=
1
1 + j RC
=
1
1 + j
. Therefore
V
C
=
1
1 + j
V
in
= 0.707V
m
45
The time-domain function is
v
C
t ( ) = 0.707V
m
cos
1
RC
t 45




V
18
Solution 10.37 (a) The magnitude of the capacitor voltage is 10/14.14 = 0.707 times the magnitude of the
input signal. We just showed in the above problem that
V
C
V
in
=
1
1+ j10RC

V
C
V
in
=
1
1 +100R
2
C
2
And we also showed that the ratio is 0.707 when the frequency is 1/RC. So, C = 1/(10R) = 0.01 F.
(b) Again, from the results of the previous problem, the angle is 45 degrees.
Solution 10.38 = 1000 rad/s and I
in
= 245 A. The equivalent admittance is
Y
eq
j ( ) =
1
R
+ j C
1
L




= 0.25 + j 0.25 0.25 ( ) = 0.25 S
In MATLAB
Yeq = 0.25;
Iin = 2*exp(j*pi/4)
Iin =
1.4142e+00 + 1.4142e+00i
Vout = Iin/Yeq
Vout =
5.6569e+00 + 5.6569e+00i
abs(Vout)
ans = 8
angle(Vout)*180/pi
ans = 4.5000e+01
% Using Current Division
IL = (1/(j*1000*4e-3)/Yeq)*Iin
IL =
1.4142e+00 - 1.4142e+00i
abs(IL)
ans = 2
angle(IL)*180/pi
ans = -45
19
Therefore, v
out
(t) = 8cos(1000t + 45
o
) V and i
L
(t) = 2cos(1000t 45
o
) A.
(b) If = 618 rad/s, then
Y
eq
( ) =
1
4
j0.405 + j0.1545 = 0.25 j0.25 = 0.3536 45
and
Vout = Iin/Yeq
Vout =
-3.4779e-04 + 5.6565e+00i
abs(Vout)
ans =
5.6565e+00
angle(Vout)*180/pi
ans =
9.0004e+01
IL = Iin*(1/(j*618*4e-3))/Yeq
IL =
2.2882e+00 + 1.4069e-04i
abs(IL)
ans =
2.2882e+00
angle(IL)*180/pi
ans =
3.5228e-03
Therefore, v
out
(t) = 5.657cos(1000t + 90
o
) = 5.657sin(1000t) V and i
L
(t) = 2.288cos(1000t) A.
Solution 10.39 Write the input phasor: = 1000 rad/s and I
in
= 0.01 260A.
w =1000;
Iin = 0.01*sqrt(2)*exp(j*60*pi/180)
Iin =
7.0711e-03 + 1.2247e-02i
Yeq = 1/500 +1/(j*w*0.25) +j*w*2e-6
Yeq =
2.0000e-03 - 2.0000e-03i
Vin = Iin/Yeq
20
Vin =
-1.2941e+00 + 4.8296e+00i
abs(Vin)
ans =
5
angle(Vin)*180/pi
ans =
1.0500e+02
IR = Iin*(1/500)/Yeq
IR =
-2.5882e-03 + 9.6593e-03i
abs(IR)
ans =
1.0000e-02
angle(IR)*180/pi
ans =
1.0500e+02
IL = Iin*(1/(j*w*0.25))/Yeq
IL =
1.9319e-02 + 5.1764e-03i
abs(IL)
ans =
2.0000e-02
angle(IL)*180/pi
ans =
1.5000e+01
IC = Iin*j*w*2e-6/Yeq
IC =
-9.6593e-03 - 2.5882e-03i
abs(IC)
ans =
1.0000e-02
angle(IC)*180/pi
ans =
-1.6500e+02
21
Therefore, v
in
t ( ) = 5cos 1000t 105 ( ) V, i
C
t ( ) = 0.01cos 1000t 165 ( ) A, i
L
t ( ) = 0.02cos 1000t +15 ( )
A, and i
R
t ( ) = 0.01cos 1000t +105 ( ) A.
SOLUTION 10.40. = 4 . Using voltage division
V
1
=
R
R +
1
j C
+ j L
V
in
=
2
2 4 j + 2 j
8 j ( ) = 5.657 45
o
Converting back to time:
v
out
t ( ) = 5.657cos 4t 45 ( ) V
SOLUTION 10.41. = 25 rad/s, V
s
= 100 V. By voltage division,
V
C
=
j
0.02 25
10 + j0.08 25
j
0.02 25
100 = 2 j = 2 90
Thus, v
C
t ( ) = 2cos 25t 90 ( ) V.
SOLUTION 10.42.
V
out
V
in
=
1
3 + j8 +
1
j 4C
= 0.2 25 = 3 + j 8
1
4C




2
.
Thus 8
1
4C




2
= 25 9 C =
1
16
= 0.0625 F.
SOLUTION 10.43. Here, = 3.33 10
3
rad/s and V
in
= 500 V. Using phasors,
V
R
=
400
400
j
3.3310
3
10
6
50 = 4036.897 V
and
V
out
=
100V
R
j C
100 +
1
j C
=
100V
R
j100 C +1
= 398532.1 V
22
Hence, v
out
(t) = 3985cos(3.3310
3
t + 32.1) V.
SOLUTION 10.44. Here, = 10
4
rad/s, I
in
= 0.010 A.
Z
1
=
100 j 0.1
100 + j 0.1
= 1+ j1000 V
L
= Z
L
I
in
= 0.01 Z
L
= 0.01 + j10
Now, in MATLAB
w=1e4; R = 100;L = 0.1; C = 0.1e-6;
Z1 = R*j*w*L/(R+j*w*L)
Z1 = 9.9010e+01 + 9.9010e+00i
Iin = 0.01;
VL = Z1*Iin
VL =9.9010e-01 + 9.9010e-02i
Z2 = 1/(1/R + j*w*C)
Z2 = 9.9010e+01 - 9.9010e+00i
VC = Z2*VL
VC = 9.9010e+01 + 1.7764e-15i
abs(VC)
ans = 9.9010e+01
angle(VC)*180/pi
ans = 1.0280e-15
Thus, v
C
t ( ) = 99cos 10000t ( ) V.
SOLUTION 10.45. Here = 40 rad/s and V
in
= 1200. This problem is best done in MATLAB using
parallel impedance computation, voltage division, and Ohm's law for phasors:
R1 = 500; R2 = 80;
C = 0.1e-3; L = 2;
w = 40; Vin = 120;
Z1 = R1/(j*w*C)/(R1 + 1/(j*w*C))
Z1 = 1.0000e+02 - 2.0000e+02i
Z2 = R2*j*w*L/(R2 + j*w*L)
Z2 = 4.0000e+01 + 4.0000e+01i
Use voltage division
VC = Z1*Vin/(Z1+Z2)
VC = 1.2212e+02 - 3.1858e+01i
abs(VC)
ans = 1.2621e+02
angle(VC)*180/pi
ans = -1.4621e+01
23
Use voltage division and Ohm's law for inductors
VL = Z2*Vin/(Z1+Z2)
VL =-2.1239e+00 + 3.1858e+01i
IL = VL/(j*w*L)
IL =3.9823e-01 + 2.6549e-02i
abs(IL)
ans = 3.9911e-01
angle(IL)*180/pi
ans = 3.8141e+00
Therefore, v
C
(t) = 126.21cos(40t 14.621
o
) V and i
L
(t) = 0.399cos(40t + 3.814) A.
SOLUTION 10.46. Here = 40 rad/s and I
in
= 0.1200 A. This problem is best solved using
MATLAB.
w = 40; Iin = 0.12;R = 5;
C = 0.004; L = 0.1;
Y1 = 1/R + j*w*C
Y1 = 2.0000e-01 + 1.6000e-01i
Y2 = 1/R + 1/(j*w*L)
Y2 = 2.0000e-01 - 2.5000e-01i
% USING CURRENT DIVISION
IL = Iin*Y2/(Y1 + Y2)
IL = 7.3171e-02 - 5.8537e-02i
abs(IL)
ans = 9.3704e-02
angle(IL)*180/pi
ans = -3.8660e+01
% AGAIN USING CURRENT DIVISION
IC = Iin*Y1/(Y1 + Y2)
IC = 4.6829e-02 + 5.8537e-02i
% USING OHM'S LAW FOR CAPACITORS
VC = IC/(j*w*C)
VC = 3.6585e-01 - 2.9268e-01i
abs(VC)
ans = 4.6852e-01
angle(VC)*180/pi
ans = -3.8660e+01
SOLUTION 10.47 Here, = 25 rad/s and I
S
= 20 A. Now perform a source transformation. The
combination of current source in parallel with resistor is changed into a voltage source in series with the
same resistor. The voltage source value is: V
S
= 1 I
S
= 20 V. Apply Ohms law to obtain:
I
1
=
20
2 +
1
j25 0.02
= 0.5 + j0.5 = 0.70745
24
Hence, i
1
(t) = 0.707cos(25t + 45
o
) A.
SOLUTION 10.48 Apply a source transformation to obtain:
The impedance of the inductor branch is Z
L
( j 4) = 2 + j4 0.5 = 2 + j2 and
Y
L
( j 4) =
1
Z
L
( j4)
= 0.25 j0.25 S. Now, the total admittance seen by the source:
Y
eq
( j 4) =
1
2
+ j0.25 + Y
L
( j4) = 0.75 S
Thus, by inspection, V
C
=
60
0.75
= 80 V and I
C
= 0.2590 80 = 290 A. Therefore
i
C
(t) = 2cos(4t + 90
o
) = 2sin(4 t) A.
SOLUTION 10.49. Apply a source transformation to obtain:
Then, by inspection, Y
eq
( j500) =
1
2
+
1
1
+
1
j2

1
j2
=1.5 S. Thus,
25
V
C
=
2.50
1.5
= 1.6670 V = V
L
I
L
= 1.667/ j2 = 0.833 90 A
In the time-domain:
v
C
t ( ) = 1.667cos 500t ( ) V and i
L
t ( ) = 0.833cos 500t 90 ( ) = 0.833sin 500t ( ) A.
1
PROBLEM SOLUTIONS CHAPTER 10
SOLUTION 10.50. The input voltage phasor is 2000 rad/s and V
S
200 V. Now, do a source
transformation on the phasor circuit:
where
I
S

V
S
Z
L

200
j2000 10 10
3
1 90 j A
Now,
Y
eq

1
20
+
1
j20
+ j2000 50 10
6
0.070745 S
and
V
C

I
S
Y
eq

1 90
0.070745
14.14 135 V
SOLUTION 10.51. Use superposition. First, find response to current source using circuit below:
V
x_1
I
s1
Z
RC
I
s1
3 ( j 3)
3 j3
20 2.121 45 4.242 45
Now, find the response due to the voltage source using the following circuit:
2
The voltage across the inductor is the same as the input source, and this voltage divides between the series
combination of capacitor and resistor:
V
x_2

j3
3 j3
V
s2

j 3
3 j3
390
o
2.121 135
o
V
Combining the two contributions implies that:
V
x
4.242 45
o
+ 2.121 135
o
4.74 71.6
o
V
SOLUTION 10.52. (a) As stated, V
L
= aV
s1
+ bI
s2
. To find a, set I
s2
= 0 and use voltage division:
V
L

j30
j30 + j30
V
s1
0.5V
s1
aV
s1
To find b, set V
s1
= 0 and use parallel impedance and Ohm's law:
V
L
j30/ / j30 ( )I
s2
j15I
s2
bI
s2
Hence
V
L
= 0.5V
s1
+j15I
s2
(b) For this part, V
s1
10 and I
s2
0.5 90
o
. Hence from the formula,
V
L
0.5V
s1
+ j15I
s2
0.5 10 + j15 0.5( j) 5 + 7.5 2.5 V
3
Therefore v
L
(t) = 2.5 cos(100t) V.
SOLUTION 10.53. In this problem, we can make use of the linearity property for phasors. Specifically,
from the given information, we can write
V
1
I
in

2045
100
245 a and
V
1
V
in

590
1045
0.545 b
Hence,
V
1
= aI
in
+ bV
in
Substituting the new values of input current and voltage, we obtain:
V
1
100 +1045 18.4822.5 V
SOLUTION 10.54. (a) For V
out
to be zero, we want
R
C
R
C

j
C

j L
R
L
+ j L
R
C
R
L

L
C
(b) Substituting R
C
C 2 s and R
L
3 into the above expression gives: L R
C
CR
L
6 H.
(c) The bridge circuit can be represented by an impedance Z
bridge
(j). The voltage that appears across the
bridge, say V
bridge
, is obtained by voltage division. Hence, by the voltage substitution theorem, the problem
may be solved as in part (a) with this new source voltage V
bridge
appearing across Z
bridge
(j).
SOLUTION 10.55 The input phasor is: 1000 rad/s and I
in
245 A, assuming peak value. First
compute
Y
th
( j10
3
) 0.25
j
1000 4 10
3
+ j1000 0.25 10
3
0.25S Z
th
( j10
3
) 4
Then,
V
oc
I
in
4 845 V
4
in which case v
oc
(t) 8cos(10
3
t + 45
o
) V. The final equivalent is a voltage source (having value V
oc
) in
series with a resistance of 4 .
SOLUTION 10.56. (a) First note that the frequency is given in Hz, so, 1281.77rad/s and I
in
100
A. Then, in MATLAB,
R = 0.25; L = 1.17e-3; C = 520e-6;
w = 2*pi*204;
Yin = j*w*C +1/(R + j*w*L)
Yin =
1.0815e-01 + 1.7737e-02i
Zin = 1/Yin
Zin =
9.0039e+00 - 1.4766e+00i
abs(Zin)
ans =
9.1242e+00
angle(Zin)*180/pi
ans =
-9.3134e+00
Therefore Z
th
9.1242 9.313 . Finally, V
oc
Z
th
I
in
91.2 9.313 V.
(b) Now, the circuit looks like the following:
Simple voltage division can yield:
V
L

Z
L
Z
L
+ Z
TH
V
oc
46.2190
o
v
L
t ( ) 46.2cos 1281.77t ( ) V
5
SOLUTION 10.57. For this problem we short the V-source and compute Z
th
and use voltage division to
find V
oc
. Specifically
Z
th

1
jC +
1
0.1+ jL
10
3
j10
and
V
oc

1
jC
1
jC
+ 0.1 + jL
2 200 j V
where
w=1000;
L = 0.01;
C = 0.1e-3;
Zth = 1/(j*w*C + 1/(0.1 + j*w*L))
Zth = 1.0000e+03 - 1.0000e+01i
Voc = 2*(1/(j*w*C))/(0.1 + j*w*L + 1/(j*w*C))
Voc = 0 - 2.0000e+02i
To compute the load voltage, define
Z
load
10
3
+ j10
2
10
3
+ j10
Again using voltage division and MATLAB we have
Zload = 1e3 + j*10;
Vload = Voc*Zload/(Zload + Zth)
Vload =
1.0000e+00 - 1.0000e+02i
magVload = abs(Vload)
magVload =
1.0000e+02 (volts)
6
SOLUTION 10.58. Here, 3.33 10
3
rad/s and V
in
500 V. Here we note that we already found
V
oc
in Problem 10.43. Thus, V
oc
398532.1 V. In order to find Z
th
, we introduce a fictitious 1 A
current source at the A and B terminals:
Now, we note that the V
R
voltage phasor is zero. Thus, the dependent source has zero volts across it. This
way, the temporary current source sees the parallel combination of a resistor and a capacitor:
V
AB

100
j
3.3310
3
0.2510
6
100 j1200
99.65 4.77 V
Thus, Z
th
99.65 4.77 . The Thevenin equivalent consists of V
oc
in series with
Z
th
99.65 4.77 99.3 j8.2865 . Thus the Norton equivalent is the parallel combination of Z
th
and the current source with value
I
sc

V
oc
Z
th

398532.1
99.65 4.77
39.9936.91
o
A.
SOLUTION 10.59. 10,000 rad/s and V
in
100 V. Again, we have already found V
oc
in Problem
10.44: V
oc
990 V. Now, to compute the impedance, we introduce the temporary current source of 1
A:
7
Again, the inductor has no voltage across it. So, the dependent source generates no current. Hence, the
independent source sees the parallel combination of a resistor and a capacitor:
V
AB

100 j1000 ( )
100 j1000
99.5 5.71 V
Thus Z
th
( j10
4
) 99.5 5.71 . So, the Thevenin equivalent is the series combination of the V
oc
source and the above Z
th
( j10
4
) 99.5 5.71 .
SOLUTION 10.60. Inject a current source at terminals A and B. Then, write a KCL equation at node A:
I
R
I
S
I
1
V
R
I
S
I
1
V
C
V
AB
V
R
+ I
1
I
S
I
1
+ I
1
I
S
Since the voltage across the current source is equal to its current, the equivalent impedance across this
current source is 1 .
SOLUTION 10.61. Inject a current source as usual. Then, write Ohms law for phasors for the equivalent
series RLC circuit. Note that the controlling current for the dependent source is the input current:
V
AB
j0.01 200 I
S

j
200 0.005
I
S
2I
S
2 + j ( )I
S
Therefore, Z
th

V
AB
I
s
2 + j .
SOLUTION 10.62. Inject a current source I
s
:
8
Apply KCL:
j2I 6I
6
I
s
I I
6
j2
I
s
. Now, V
AB
is the voltage across the inductor:
V
AB

6 j2
j2
I
s
6I
s
Z
th
6 .
Solution 10.63. 2000 rad/s and V
S
250 V. Now, inject a current source, and express V
AB
as a
function of this current source and V
oc
:
Now, write two nodal equations at A and the top of the dependent current source:
V
1
V
S
10
3
+ g
m
V
AB
+ j C V
1
V
AB
( ) 0
j C V
AB
V
1
( ) +
V
AB
j L
I
S
Substituting values, these can be cast into the following matrix equation:
9
10
3
1 + j 3 j
j j



1
]
1
V
1
V
AB



1
]
1

10
-3
V
S
I
S




1
]
1
1
This can be solved in MATLAB to obtain:
V
1
V
AB



1
]
1
100
2.5 j2.5 5 + j10
2.5 + j2.5 5



1
]
1
10
-3
V
S
I
S




1
]
1
1
Thus
V
AB
(0.25 + j0.25)V
s
+ 500I
s
500I
s
+ 6.25 + j6.25
Therefore Z
th
500 and V
oc
6.25 + j6.25 8.83945
o
V. For the Norton equivalent we need
I
sc

V
oc
Z
th

8.839
500
45
o
0.0176845
o
A.
SOLUTION 10.64. Inject an upward current source, I
S2
, at V
AB
. Then, write the following two nodal
equations at V
x
and V
A
: let R
1
= R
2
= R, then
V
x
R
+ j C
1
V
x
+ j C
2
V
x
V
AB
( ) I
S
j C
2
V
AB
V
x
( ) + g
m
V
x
+
V
AB
R
I
S2
which after grouping terms becomes
1
R
+ j C
1
+ j C
2


_
,
V
x
j C
2
V
AB
I
S
j C
2
+ g
m
( )V
x
+ j C
2
+
1
R


_
,
V
AB
I
S2
In MATLAB
R = 100e3; C1 = 1e-9; C2 = 1e-10;
gm = 0.1e-3; w = 1e3;
Nodal = [1/R+j*w*C1+j*w*C2 -j*w*C2;-j*w*C2+gm j*w*C2+1/R]
Nodal =
1.0000e-05 + 1.1000e-06i 0 - 1.0000e-07i
10
1.0000e-04 - 1.0000e-07i 1.0000e-05 + 1.0000e-07i
Nodalinv = inv(Nodal)
Nodalinv =
9.5680e+04 - 2.0070e+04i 2.1024e+02 + 9.5470e+02i
-9.5449e+05 + 2.1120e+05i 9.7783e+04 - 1.0523e+04i
Thus
V
x
V
AB



1
]
1
10
3

95.68 j20 0.21+ j0.955


954.5 + j211.2 97.78 j10.52



1
]
1
40 10
-6
I
S2




1
]
1
1
Therefore
V
AB
38.18 + j8.448 + 97.8 j10.5 ( ) 10
3
I
S2
from which we identify
V
oc
38.18 + j8.448 39.1167.5
o
V
and Z
th
97.8 j10.5 ( ) 10
3
98.35 10
3
6.14
o
.
SOLUTION 10.65. We solve this problem by the method illustrated in example 6.3 where a fictitious
source is applied and the repsonse is calculated. One can either apply a voltage source or a current source
(see figure 6.10 a and b). Generally speaking, neither choice can be claimed as superior to the other. But
for a specific circuit, one choice can lead to a much simpler solution than the other. To illustrate this point,
we solve the problem with both choices below. Note that although the first method is much simpler than
the second, it lacks the generality. If one more resistor were inserted into the circuit, the simplicity of
solution of solution may disappear totally, whereas the second method will proceed with very few changes.
Method 1. For this solution we apply an arbitrary voltage source, labeled in phasor form as V
s
as
indicated in the circuit below.
For this circuit we will compute an equation of the form ofequation 6.11:
11
I
A

1
Z
th
V
s
I
sc
(1)
By inspection of the circuit,
I
A
I
C
+ I
L
1 jCV
s
+
V
L
jL
1 j2V
s
jV
L
1 (2)
But
V
L
0.25I
C
+ V
s
0.25 jCV
s
+ V
s
0.5 j +1 ( )V
s
(3)
Substituting (3) into (2) produces
I
A
0.5 + j ( )V
s
1 (4)
By comparing (4) with (1), we obtain the answers for the Norton equivlent circuit: I
s c
= 1 A and Z
th
=1/(0.5 + j) =
0.4 j0.8 .
Method 2: For this method we apply an arbitrary current source, labeled in phasor form as I
s
as indicated
in the circuit below.
Notice that we have added a current label I
x
as we plan to use modified nodal analysis method to obtain the
desired answer. For this we will compute an equation of the form
V
AB
Z
th
I
s
+ V
oc
Because of the addition of I
x
, we can write the modified nodal equations more or less by inspection:
1/ jL 0 1
0 jC 1
1 (1+ j0.25C) 0





1
]
1
1
1
V
L
V
AB
I
x





1
]
1
1
1

j 0 1
0 2 j 1
1 1 0.5 j 0





1
]
1
1
1
V
L
V
AB
I
x





1
]
1
1
1

1
I
s
0





1
]
1
1
1
where from MATLAB,
12
w=2000;
L = 0.5e-3;
C = 1e-3;
Y11 = 1/(j*w*L)
Y11 = 0 - 1.0000e+00i
Y22 = j*w*C
Y22 = 0 + 2.0000e+00i
Y32 = -(1+j*0.25*w*C)
Y32 = -1.0000e+00 - 5.0000e-01i
Solving the equations in MATLAB produces,
A = [-j 0 1;0 2*j -1; 1 -1-0.5*j 0];
Ainv = inv(A)
Ainv =
8.0000e-01 - 6.0000e-01i 8.0000e-01 - 6.0000e-01i 1.6000e+00 + 8.0000e-01i
4.0000e-01 - 8.0000e-01i 4.0000e-01 - 8.0000e-01i 8.0000e-01 + 4.0000e-01i
1.6000e+00 + 8.0000e-01i 6.0000e-01 + 8.0000e-01i -8.0000e-01 + 1.6000e+00i
Multiplying the second row of Ainv times the right-most vector of our equations produces
V
AB
(0.4 j 0.8)I
s
+ (0.4 0.8j )
This implies that Z
th
= 0.4 j0.8 and V
oc
= 0.4j0.8 V. For the Norton equivalent we need
I
sc

V
oc
Z
th
1 A
SOLUTION 10.66. (a) 1000 rad/s and V
S
2045 V. By voltage division:
V
out

1
1+ j
2045 14.140 V I
L
14.140 A
(b) At dc, V
out
= V
in
. We want the frequency at which V
out
= 0.1V
in
. Thus, we want:
13
V
out
V
in

1
1+ j 0.001
0.1
1
1+
2
10
6

1
100
9950 rad/s
SOLUTION 10.67. Z 25 j20 . By KCL:
V
Z
140
j15
+
V
Z
25 j20
+
V
Z
+ 890
j20
0
This equation simply needs to be manipulated in order to obtain:
V
Z
41.35 73.46
SOLUTION 10.68. In this problem, we note that the impedance, jX, is in series with the parallel RLC
circuit to the right. Thus, all we need to do is to find an expression for the equivalent impedance of the
parallel RLC circuit:
Y
RLC

1
30

j
20
+ j0.025 Z
RLC
19.2 + j14.4
Now, the total impedance seen by the source is 19.2 + j14.4 +jX. Therefore, for this to be real, the
unknown reactance has to be 14.4 . Also, the input current now is I = 96/19.2 = 5 A. Hence
i(t) 5cos(10t) A.
SOLUTION 10.69. First, compute the current through the series RC section: I
C

1
1 j
A.
Now, by KCL, we can write
0 = I
C
I
x
+
1 2I
x
j

1
1 j
I
x
+
1 2I
x
j
I
x
0.3 + j0.1 0.31618.44
o
A
SOLUTION 10.70. 20 rad/s and Y
in
0.05 + j0.0866 S.
(a) Since Y
in

1
R

j
L
+ j C, equating the real parts of the above two expressions implies that R =
1/0.05 = 20 .
(b) Similarly, equating the imaginary parts and substituting, we obtain:
14
j C
1
L


_
,
j0.0866 L 3.73 H
(c)
V
C

I
in
Y
in

2030
0.05 + j0.0866
200 30 V v
C
t ( ) 200cos 20t 30
o
( )
V
(d)
I
L

20030
j20 3.73
2.68 120A i
L
t ( ) 2.68cos 20t 120
o
( )
A
(e) V
oc
is just the voltage V
C
, which was obtained in (c), and Z
th

1
Y
in
10 60 .
(f) Z
th
= 5 j8.66 . This is equivalent to a series combination of a 5 resistance and a 5.77 mF
capacitance at the given frequency = 20 rad/s.
SOLUTION 10.71. (a) V
s
2 + j0V, 1000 rad/s. By KCL
V
A
j1.33
+
V
A
2 + j2
+
V
A
V
s
4
0 V
A
0.5 j0.5 0.707 45 V
The time-domain expression is:
v
C
t ( ) v
A
t ( ) 0.707cos 1000t 45
o
( )
V
(b) I
L

0.707 45
2 + j2
0.25 90 A implies that i
L
t ( ) 0.25cos 1000t 90
o
( )
0.25sin(1000t) A.
(c) We already determined V
oc
in part (a). Now, turn off the source to compute the equivalent impedance:
Y
th
0.75 j +
1
2 + j2
+ 0.25 0.5 + j0.5 Z
th
1 j . This is the series connection of a 1 resistor
with a capacitor of value C
1
10
3
F. This completes the definition of the Thevenin equivalent.
SOLUTION 10.72. In MATLAB,
R = 5; L = 1e-3; C = 20e-6;Vs1 = 5; Is2 = 0.5*j;
G = 1/R; w = 10e3;
YL = 1/(j*w*L)
YL =
0 - 1.0000e-01i
15
YC = j*w*C
YC =
0 + 2.0000e-01i
Hence
G(V
C
V
s1
) + Y
C
V
C
+ Y
L
(V
C
V
s1
2V
R
) I
s2
Substituting for V
R
,
G(V
C
V
s1
) + Y
C
V
C
+ Y
L
(V
C
V
s1
2(V
s1
V
C
)) I
s2
Therefore
G+ Y
C
+ 3Y
L
( )V
C
G+ 3Y
L
( )V
s1
+ I
s2
Again, using MATLAB,
a = G + YC + 3*YL
a =
2.0000e-01 - 1.0000e-01i
b = (G+3*YL)*Vs1 + Is2
b =
1.0000e+00 - 1.0000e+00i
VC = b/a
VC =
6.0000e+00 - 2.0000e+00i
abs(VC)
ans =
6.3246e+00
angle(VC)*180/pi
ans =
-1.8435e+01
Hence, v
C
(t) 6.325cos(10
4
t 18.44
o
) V.
SOLUTION 10.73. Denote by v
C1
, the node voltage of the 2.5 mF capacitor . Note that at = 800 rad/s,
V
s
= 200
o
V. From this we write a set of nodal equations by inspection after observing the following
from MATLAB:
16
w = 800;
C1 = 2.5e-3;
L = 1.25e-3;
Y1 = 0.5+j*w*C1-j/(L*w)
Y1 = 5.0000e-01 + 1.0000e+00i
Yoff=j/(L*w)
Yoff = 0 + 1.0000e+00i
Y2 = 0.25+j*w*3.75e-3 - j*w*1.25e-3
Y2 = 2.5000e-01 + 2.0000e+00i
This information leads to the following matrix nodal equation:
0.5 + j j
j 0.25 + j2



1
]
1
V
C1
V
out



1
]
1

0.5V
s
0.25V
s



1
]
1
To solve these equations we again use MATLAB:
A = [Y1, Yoff;Yoff, Y2]
A =
5.0000e-01 + 1.0000e+00i 0 + 1.0000e+00i
0 + 1.0000e+00i 2.5000e-01 + 2.0000e+00i
b = [0.5*20;0.25*20]
b =
10
5
Vnodes = inv(A)*b
Vnodes =
7.1141e+00 - 6.9799e+00i
-3.6242e+00 + 5.3691e-01i
magVnodes = abs(Vnodes)
magVnodes =
9.9664e+00
17
3.6637e+00
angVnodes = angle(Vnodes)*180/pi
angVnodes =
-4.4454e+01
1.7157e+02
Therefore
v
out
(t) 3.664cos(800t +171.57
o
) V
SOLUTION 10.74. For this problem we use loop analysis with loops indicated in the figure below.
Since there are no controlled sources, we can write down the loop equations by inspection:
10 + 9 j 6 + 5j
6 + 5 j 12 + 9 j



1
]
1
I
A
I
C



1
]
1

120 120 120


o
120120
o
120 120
o




1
]
1
1

180 +103.92
j207.85



1
]
1
The solution of this equation is done in MATLAB as follows:
b1=120-120*exp(-j*2*pi/3)
b1 = 1.8000e+02 + 1.0392e+02i
b2=120*exp(j*2*pi/3)-120*exp(-j*2*pi/3)
b2 = 0 + 2.0785e+02i
18
A = [10+j*9, 6+j*5;6+j*5,12+j*9]
A =
1.0000e+01 + 9.0000e+00i 6.0000e+00 + 5.0000e+00i
6.0000e+00 + 5.0000e+00i 1.2000e+01 + 9.0000e+00i
I=inv(A)*[b1;b2]
I =
1.4472e+01 - 1.3469e+01i
4.2926e-01 + 1.7703e+01i
% Please note that using the commands I=inv(A)*[b1,b2]'
% will lead to the wrong answer because a conjugate is
% inserted along with the transpose.
magI = abs(I)
magI =
1.9770e+01
1.7708e+01
angleI = angle(I)*180/pi
angleI =
-4.2944e+01
8.8611e+01
IB = -I(1)-I(2)
IB = 1.8288e+00 + 1.7234e+01i
magIB = abs(IB)
magIB = 1.7331e+01
angleIB = angle(IB)*180/pi
angleIB = 8.3943e+01
Changing the sign on each source amounts to multiplying its value by "1". This means that all
magnitudes remain the same, but there is a 180
o
phase shift for each current, i.e., add 180
o
to each current
angle.
19
SOLUTION P10.75. For this problem we have both a transient component to the response and a steady
state component. The steady state component is computed in the usual way because the circuit is stable,
i.e., the time constant is positive. Once the steady state part is computed, we use initial conditions to obtain
the coefficient B in the response.
Part 1: Compute steady state response. For this we use MATLAB,
R = 0.5; L = 0.866;
Vs = 10;
w = 1;
Zin = R + j*w*L
Zin = 5.0000e-01 + 8.6600e-01i
IL = Vs/Zin
IL = 5.0002e+00 - 8.6604e+00i
magIL = abs(IL)
magIL = 1.0000e+01
angIL = angle(IL)*180/pi
angIL = -5.9999e+01
Hence
i
L
(t) 10cos(t 60
o
) + Be
0.577t
A
Part 2: From the initial conditions we have
i
L
(0) 1 10cos(t 60
o
) + Be
0.577t
[ ]
t 0
10cos(60
o
) + B 5 + B
Hence B = 4. It follows that
i
L
(t) 10cos(t 60
o
) 4e
0.577t
A
SOLUTION 10.76. KCL dictates that:
I
L
+ I
R
+ I
C
= I
S
I
L
= I
S
I
R
I
C
We can perform this sum graphically as follows:
20
j2
-I
C
-I
R
I
L
=-j
SOLUTION 10.77. (a) Note that V
S
= V
R
+ V
L
, but that V
L
leads V
R
by 90 degrees. Similarly, I
S
= I
R
+
I
C
, but I
C
leads V
S
by 90 degrees. Also note that the inductor current is also I
R
, and the capacitor voltage is
V
S
.
V
R
I
R
V
L
V
S
I
C
I
S
(b) Using graph paper to construct the phasor diagram to scale, we find the difference between the phase
angles of I
s
and V
s
is zero.
Solution 10.78. First note that V
C
, the capacitor voltage, will lag I
S
by 90 degrees. Now, V
C
plus the
unknown element voltage should result in a vector that runs diagonally between the two vectors. From the
following illustration, it follows that the unknown voltage should have the same phase as the input current:
V
C
I
S
V
u
V
S
V
C
21
This means that the unknown element is a resistor. The 45o phase difference implies that V
C
V
u
or
I
s
C
I
s
R. Therefore R
1
C

1
10
3
10
6
1000 .
SOLUTION 10.79. The student can construct the phasor diagram using graph paper. The diagram is going
to look like that in the problem statement, except that the proper lengths and angles will be used.
SOLUTION 10.80. As the frequency approaches infinity, the capacitor shorts and the inductor opens. So,
the output voltage is zero. As the frequency approaches zero, the capacitor opens, but the inductor shorts,
so the output is also zero. A plot of the complete response is shown below. (Note that the magnitude
response at 10rad/s is infinite):
L = 0.04; C = 0.25;
w = 0: 30/300:30;
% Vout = Zin * Iin
Zin = j*w*L ./(j*w*L*j.*w*C + 1);
plot(w, abs(Zin))
grid
ylabel('Magnitude Zin')
xlabel('Frequency in rad/s')
plot(w,angle(Zin)*180/pi)
grid
xlabel('Phase in degrees')
ylabel('Phase in degrees')
xlabel('Frequency in rad/s')
22
0 5 10 15 20 25 30
0
5
10
15
20
25
M
a
g
n
i
t
u
d
e

Z
i
n
TextEnd
Frequency in rad/s
0 5 10 15 20 25 30
-100
-80
-60
-40
-20
0
20
40
60
80
100
Frequency in rad/s
P
h
a
s
e

i
n

d
e
g
r
e
e
s
TextEnd
SOLUTION 10.81. At infinite frequency, the resistor current is zero (because the inductor opens). So, the
output voltage is zero. At DC, the inductor is short, and the output voltage is equal to the input voltage. The
plot of the frequency response is shown below (a logarithmic x-axis is used):
SOLUTION 10.82. The circuit inside the black box is
23
At DC, the capacitor is an open circuit. Thus, the voltage across the resistor is 1mAR. But we know that
this voltage is 1 from the graph. This means that R = 1 k. Now, in general for the above diagram:
V
I

R
1+ j RC
The magnitude of this function is R/sqrt(2) when 1/ RC. Substituting the frequency from the graph
(1000 rad/s), we get C = 1 F.
Solution 10.83. The admittance of the parallel RLC circuit is:
Y
in

1
R
+
1
j L
+ j C
Z
in

1
Y
in

V
s
I
s
The function we want to find the frequency response for is nothing but the input admittance of the circuit.
Using MATLAB, the following plot can be obtained:
R = 100; L = 0.1; C = 1e-3;
w = 0:0.5:300;
w = 0.01:0.5:300;
Yin = 1/R + 1. ./(j*w*L) + j*w*C;
Zin = 1 ./Yin;
plot(w,abs(Zin))
grid
xlabel('Frequency in rad/s')
ylabel('Magnitude Zin')
24
0 50 100 150 200 250 300
0
10
20
30
40
50
60
70
80
90
100
Frequency in rad/s
M
a
g
n
i
t
u
d
e


Z
i
n
TextEnd
plot(w,angle(Zin)*180/pi)
grid
xlabel('Frequency in rad/s')
ylabel('Phase Zin in degrees')
0 50 100 150 200 250 300
-100
-80
-60
-40
-20
0
20
40
60
80
100
Frequency in rad/s
P
h
a
s
e

Z
i
n

i
n

d
e
g
r
e
e
s
TextEnd
25
SOLUTION 10.84. The circuit inside the box is a series RLC circuit. It cannot be a parallel RLC, because
as per problem 83, the admittance of a parallel RLC does not approach zero as w approaches infinity. Thus,
I
V
Y
in

1
R +
1
j C
+ j L
The resonance frequency is 50 rad/s and is determined by 1/sqrt(LC). Given L = 0.4 H, C = 1 mF.
To obtain R, we make use of the fact that, from the given graph at = 57 rad/s, the current
magnitude is approximately 0.2 times the peak magnitude. Therefore
1
R
2
+ 57L
1
57C


_
,
2

0.2
R
2
+ 50L
1
50C


_
,
2

0.2
R
Hence
R
2
+ 57 0.4
1
57 10
3



_
,

2

R
2
0.04
25R
2
R
2
+ 57x0.4 -
1
57x0.001
2

R
2
0.04
= 25R
2
From which R
5.2561
24
1.0728 .
SOLUTION 10.85. Create three mesh currents in the three planar loops. All currents are clockwise: I
1
in
the voltage source loop, I
2
in the top bridge loop, and I
3
in the bottom one. The three mesh equations are:
V I
1
R
1
R
2
I
1
I
2
( ) R
3
I
1
I
3
( ) 0
R
2
I
2
I
1
( ) +
1
j C
1
I
2
+ R
meter
I
2
I
3
( ) 0
R
3
I
3
I
1
( ) + R
meter
I
3
I
2
( ) +
1
j C
2
I
3
0
The plots of the magnitude and phase of V
B
V
C
R
meter
I
3
I
2
( ) are shown in the text.
26
SOLUTION 10.86. We can see all ranges by plotting on a logarithmic scale:
Note that the output will decay when we start to reach the bandwidth of the op-amp. In other words, the
inverting amplifier says that the output is 1 times the input (provided the op amp works properly). Once
the op amps gain starts dropping, the output voltage also decays with it.
SOLUTION 10.87. Correction: Change the 0.01 F capacitor to 1 F. (a) For this part consider the
diagram below,
From the problem statement, = 320 rad/s, and I
in
= 0.010
o
A. Observe that the 50 k resistor input
to the inverting op amp terminal is in parallel with the 100 resistor because of the virtual ground at the op
amp terminals. However, for all practical purposes, this has no effect on the 100 resistor, hence from
Ohm's law
27
V
L

100 j 0.1
100 + j0.1
I
in
50.265 + j50 ( ) 0.01 0.50265 + j0.5
From the inverting op amp configuration,

V =-10V
L
5.0265 j5
From voltage division,
V
C

10
3
10
3
+ j10
9

V 0.49735 j0.5 ( ) 5.0265 j5 ( ) 5179.7
o
V
Therefore
v
C
(t) 5cos(320t +179.7
o
)V
Parts (b) and (c). For the SPICE simulation we have the following circuit in B
2
-SPICE:
which leads to the response below
28
MAG(V(IVM))
Frequency (Hz) Prb10-87-Small Signal AC-2
(V)
+0.000e+000
+100.000m
+200.000m
+300.000m
+400.000m
+500.000m
+1.000 +10.000 +100.000 +1.000k +10.000k
The magnitude at 160 Hz is 0.499 for a 1 mA current input. Thus a 10 mA input current should lead to
4.99 V by linearity which approximates the 5 V computed analytically in part (a). Hence with a 15 V
saturation limit, the input magnitude may increase by a factor of 3 to 30 mA.
SOLUTION 10.88. (a) 400 rad/s and V
in
10
3
90 V. By the virtual short property:
I
in

V
in
1/ j C
j CV
in
All this current flows through the 1 M resistor: V
in
j C10
6
V
in
0.4 180. Thus,
v
out
(t) = 0.4cos(400t) V
(b) 200 rad/s, V
in
10
3
0 V, and V
out
10
3
90 V. Again,
V
out
j10
3
j C 4 10
5
V
in
j80 10
3
C C 12.5 nF
29
SOLUTION 10.89. (a) 800rad/s and V
in
1 90V I
in

V
in
10
6
. No current flows into Op-
Amp terminals:
V
out
I
in
1
j C

j ( j)
800
1.25 10
3
V
Thus, v
out
(t) = 1.25cos(800t) mV.
(b) Again, I
in

V
in
200 10
3
A, and V
out
I
in
1
j C
10
j ( j)
C2 10
5
C 2.5 nF .
Solution 10.90 (a) 2 700rad /s and V
in
10 V I
in

1
150 10
3
A. Further,
V
out

150 10
3 j
C
150 10
3

j
C
I
in

1
150 10
3

150 10
3 j
C
150 10
3

j
C
0.001590
o
V
Thus, v
out
(t) = 1.5cos(2700t + 90
o
) mV.
(b)
(c) The output lags the DC response by 45 degrees (note that at DC, the amplifier is inverting, or has a
phase of 180 degrees). Now, the frequency response is really determined by the RC circuit in the
feedback path of the op-amp. The first resistance at the input simply converts the input into a current that
30
drives this RC circuit. It can be shown that a 45 degree phase shift occurs in an RC circuit when the
frequency is 1/R
f
C (directly from the results of an analysis on an RC circuit). So,
R
f
C = 1/2000 C =0.016 nF (We know R
f
=10 M.)
Also, at this frequency, the response is 0.707DC response. The DC response is 10
7
/R. So, the DC gain
is 14.14. Thus, R = 7.07 k.
Solution 10.91 (a) Use the virtual short property: I
in

V
in
10
3
+
1
j C
. All of current flows through the
feedback path: V
out
10
4
I
in

10
4
V
in
10
3

j
C

V
out
V
in
7.07 135
o
.
(b)
(c) The spice result looks the same at low frequencies. However, at high frequencies, the response falls
back to zero as the op-amp non-ideal frequency response starts to affect the behavior of the circuit.
SOLUTION 10.92. (a) The negative terminal of the op-amp is at V
s
. This implies I
R

V
S
R
. By KVL,
31
V
S
+ Z
RC
I
R
V
o

V
o
V
S
1 +
1
R
j 3R
C
3R
j
C
1 +
3
j3 RC +1
The MATLAB plot for the given values is:
(c) The spice result looks pretty much the same, especially since the cut-off frequency of this circuit is
much lower than the frequency at which the op-amp ceases to operate as an ideal op-amp.
SOLUTION 10.93. To compute the gain as a function of we observe that by the properties of an ideal op
amp,
Gain
V
out
V
in

2 10
5
+ j0.5 10
6

10
4
+ j0.1 10
6

20 + j0.5
100 + j 0.1
In MATLAB
G1 = 1/50e3;
G2 = 1/10e3;
C1 = 0.5e-6;
C2 = 0.1e-6;
w = logspace(-1,5,1500);
Y1 = G1 +j*w*C1;
Y2 = G2 + j*w*C2;
H = Y1 ./ Y2;
semilogx(w,abs(H))
grid
32
SOLUTION 10.94. (a) The equations are:
V
1
V
in
1000
+
V
1
V
2
1000
+
V
1
V
2
1/ jwC
f
0
V
2
V
1
1000
+
V
2
1/ j C
2
0
V
2
V
out
Substituting values and solving for V
1
and V
2
,
V
1
0.8535 82
o
V
in
V
2
0.723 114.7
o
V
in
The second one is the relation that we are looking for.
(b) At 100 Hz,
V
1
1.00 3.68
o
V
in
V
2
1.00 7.3
o
V
in
At 3000 Hz,
V
1
0.15 102
o
V
in
V
2
0.0728 164
o
V
in
33
(c) The response is that of a low-pass filter, as predicted from the results of part (b) above.
SOLUTION 10.95. (a) The two nodal equations:
V
x
V
in
200
+ j C
1
V
x
+ j C
2
V
x
V
out
( ) 0
j C
2
0 V
x
( )
V
out
28 10
3
0
In MATLAB,
w = 2*pi*1.34e3; C = 0.05e-6; R1 = 200; R2 = 28e3;
A = [1/R1+j*w*C+j*w*C -j*w*C;-j*w*C -1/R2]
A =
5.0000e-03 + 8.4195e-04i 0 - 4.2097e-04i
0 - 4.2097e-04i -3.5714e-05
b = [1/R1; 0];
V = A\b
V =
2.6664e-01 - 5.9266e+00i
-6.9859e+01 - 3.1429e+00i
Vout = V(2)
Vout =
-6.9859e+01 - 3.1429e+00i
abs(Vout)
ans =
6.9929e+01
angle(Vout)*180/pi
ans =
-1.7742e+02
Hence,
V
out
V
in
69.93 177.4
o
.
When the capacitors are shorts, the output is shorted to the virtual ground input, at 0 V. Similarly, when
they are opens, the virtual ground makes sure that v
out
is zero, since there is no drop across the feedback
resistor.
34
(b) The band-pass response can be computed using any SPICE program.
SOLUTION 10.96. First, note the input-output relationship:
V
out
V
in

Z
f
R
1
where Z
f
is the impedance of the parallel RLC circuit. We have studied this circuit extensively earlier, and
we have shown that at resonance, the impedance of this circuit is going to be real and equal to the value
of resistance, in this case R
2
. Thats exactly the requirement of this problem, since we want
V
out
V
in

Z
f
R
1
to simply be equal to R
2
/R
1
. It remains to note that this resonance occurs at a frequency = 1/sqrt(LC).
SOLUTION 10.97. (a) First, analyze the feedback amplifier circuit. The output of this op-amp circuit is:
V
op2

1/ j C
2 10
6
V
out
Also, by voltage division, the voltage at the resistive voltage divider (+ terminal of first op-amp):
V
RR

2 10
3
2 10
5

1
j C
2
2 10
6
V
out

0.01
j C
2
2 10
6
V
out
Now, the first op-amp circuit is an inverting amplifier, but its + terminal is at VRR now. Thus,
V
out
10 V
in
V
RR
( ) + V
RR
Substituting the above V
RR
means that:
V
out

10
1+
1.1
j
V
in
Since the input voltage has unity magnitude and zero phase, the above expression gives the required
magnitude and phase of the output voltage.
(b)
35
0 0.2 0.4 0.6 0.8 1 1.2 1.4 1.6 1.8 2
0
1
2
3
4
5
6
7
8
9
10
Frequency Hz
M
a
g
n
i
t
u
d
e
TextEnd
(c) As can be seen, the response to zero frequency (i.e. DC) is zero. Also, the circuit goes back very quickly
(less than 2 Hz) to provide the required operation, which is to achieve a gain of 10.
Complex Power Prbs 2/5/00 P11-1 @ DeCarlo & P. M. Lin
CHAPTER 11. PROBLEM SOLUTIONS
SOLUTION 11.1. Using equation 11.3, P
av

1
2
(e
t
1)
2
Rdt
e
2t
2
+ t 2e
t




1
]
1
1
0
1
0.758
0
1

W.
SOLUTION 11.2.
(a) From 11.6, P
av

V
m
2
2R
50 mW for a sinusoidal input.
(b) From 11.3,
P
av

R
2
(10cos(10t))
2
dt
0
/20

+ (10cos(10t))
2
dt
/20
3 /20

+ (10cos(10t))
2
dt
3 /20
2 /10





1
]
1
1

I
m
2
R
2
50 mW
just as the previous case since the square of the absolute cos(10t) is the same as the square of cos(10t).
(c)
P
av

10R
2
0.01cos
2
(10t)
[ ]
2
dt
0
2 /10

10
3
2
R 0.5cos(20t) + 0.125cos(40t) + 0.375 [ ]dt
0
2 /10

1
2
0.5
20
sin(20t) +
0.125
40
sin(40t) + 0.375t



1
]
1
0
2 /10
37.5 mW
(d)
t=0:1/1000:1;
R=1e3;
pta= (0.01*cos(10.*t)).^2.*R;
ptb= (0.01*abs(cos(10.*t))).^2.*R;
ptc= (0.01*cos(10.*t)).^4.*R;
subplot(3,1,1);
plot(t,pta);
grid
ylabel('W');
subplot(3,1,2);
plot(t,ptb);
grid
ylabel('W');
subplot(3,1,3);
plot(t,ptc);
grid
ylabel('W');
xlabel('time in seconds');
Complex Power Prbs 2/5/00 P11-2 @ DeCarlo & P. M. Lin
0 0.2 0.4 0.6 0.8 1
0
0.05
0.1
W TextEnd
0 0.2 0.4 0.6 0.8 1
0
0.05
0.1
W TextEnd
0 0.2 0.4 0.6 0.8 1
0
0.5
1
x 10
-5
W TextEnd
time in seconds
SOLUTION 11.3. (a) For figure a, the period is 2, and P
av

1
2
v
2
(t)
R 0
2

dt
1
2R
400 +100 ( ) 25 W.
In figure b, the period is 1, and P
av

20t ( )
2
R 0
1

dt
400
R
t
3
3




1
]
1
1
0
1
13.3 W.
(b)
Complex Power Prbs 2/5/00 P11-3 @ DeCarlo & P. M. Lin
0 0.5 1 1.5 2 2.5 3 3.5 4
10
15
20
25
30
35
40
Time in s
I
n
s
t
a
n
t
a
n
e
o
u
s

P
o
w
e
r

i
n

W
TextEnd
0 0.5 1 1.5 2 2.5 3 3.5 4
0
5
10
15
20
25
30
35
40
Time in s
I
n
s
t
a
n
t
a
n
e
o
u
s

P
o
w
e
r

i
n

W
TextEnd
SOLUTION 11.4. (a) For (a), looking the definition for the effective voltage, one sees graphically that
the integral over one period, 2, of the squared waveform, is 500. Dividing by the period, and taking the
square root, V
eff
15.81 V. For fig. b, V
eff
20t ( )
2
dt
0
1

400
1
3
11.55 V
(b) P
av
I
eff
2
R
V
eff
10



_
,

2
8 20 W.
(c) P
av

V
eff
10



_
,

2
8 10.67 W
Complex Power Prbs 2/5/00 P11-4 @ DeCarlo & P. M. Lin
SOLUTION 11.5. (a) This can be done graphically quite easily. The period of fig a, is 9s. The total
area of one period of the squared waveform is 75. This yields I
eff

75
9
2.89 A. In fig. b, the area
over one period is 25 which yields I
eff

25
3
2.89 A.
(b) Using current division, P
av
I
eff
60
90


_
,
2
30 111.36 W.
(c) The same result as (b) is obtained since the effective current is the same.
SOLUTION 11.6. (a) I
eff

1
2
e
t
1
( )
2
0
1

dt 0.615 A.
(b) P
av
I
eff
2
R 0.758 W.
SOLUTION 11.7. (a)
V
eff
2

20
2
10 + 2cos(20t) ( )
2
0
2 /20

dt
20
2
100t + 2t +
2
40
sin(40t) + 2sin(20t)



1
]
1
0
2 /20
102.01
Hence V
eff
10.1 V.
(b)
V
eff
2

1
10cos(2t) + 5cos(4t) ( )
2
0

dt
1
62.5t [ ]
0
62.568
Hence V
eff
7.91 V.
(c)
Without going into detailed calculation, note the following fact about v
3
2
(t) . Only the product terms that
have the same frequency will produce a non-zero result when integrated. Thus the integral reduces
to the following:
V
eff
2

1
100cos
2
(2t) + 25cos
2
(4t) + 25cos
2
(4t /4) + 50cos(4t)cos(4t /4)
( )
0

dt
Hence
V
eff
50 +12.5 +12.5 + 25cos( / 4) [ ] 9.627 V
SOLUTION 11.8. The voltage is V 500 V and the impedance Z
eq
100 100 j 141.42 45
o
. Thus I
V
Z
eq
353.645
o
mA. Hence, P
av
R I
eff
2
100
0.3536
2



_
,

2
6.2516 W.
Complex Power Prbs 2/5/00 P11-5 @ DeCarlo & P. M. Lin
SOLUTION 11.9. (a) The equivalent load seen by the source, Z
eq

1
j C +1/ R
2 36.87
o
. Thus
V
L
IZ
eq
10 36.87
o
V and v
L
(t) 10cos(30t 36.87
o
) V.
(b) P
av

V
m
I
m
2
cos(
v

i
)
10 5
2
cos(36.87
o
) 20 W, and from 11.4
p
L
(t) 25cos(36.87
o
) + 25cos(60t 36.87
o
) W
SOLUTION 11.10. (a) First find the impedance Z
eq
3 + j4 553.13
o
, then
I
s

V
s
Z
eq
10 143.13
o
A. The magnitude is 10 A rms or 14.14 A peak-value.
(b) P
av
10 50cos(53.13
o
) 300 W.
(c) 3I
s
2
300 W
SOLUTION 11.11. (a) By KVL, V
s
10I
L
+ j100I
L
+ 9 10I
L
and
I
L

V
s
100 + j100

1000
141.4245
o

1
2
45
o
A.
(b) P
avVs
V
s
I
L
cos(45
o
) 50 W and P
av9V1
9 10I
L
I
L
cos(180
o
) 45 W.
SOLUTION 11.12. (a) By KCL the current through the resistor is 5I. So by KVL,
V
s
j1000I j500I + 200 5I and hence
I
V
s
1000 + j500
107.33 26.57
o
A
The power absorbed by the resistor is, P
av
200 5I 5I 57.6 W.
(b) P
avVs
V
s
I cos(26.57
o
) 11.52 W and P
av4I
4I 200 5I cos(0) 46.08 W.
SOLUTION 11.13. First find the input impedance, Z
eq
2 j8 + 6 + j2 8 j6 . Then calculate the
current I
L

100
8 j6
8 + j6 A. The complex power is then computed:
ZL = 6 + j*2;
IL = 8 + j*6;
VL = ZL*IL
VL =
3.6000e+01 + 5.2000e+01i
SL = VL * conj(IL)
SL =
6.0000e+02 + 2.0000e+02i
Complex Power Prbs 2/5/00 P11-6 @ DeCarlo & P. M. Lin
abs(SL)
ans =
6.3246e+02
Thus the apparent power is 790 VA, the average power 600 W, the reactive power 200 var, and the
apparent power is 632.46 VA.
SOLUTION 11.14. (a) Using MATLAB
Vseff = 100*exp(j*pi/6)
Vseff = 8.6603e+01 + 5.0000e+01i
ZL = 350 +j*1*300;
Zin = 50 + ZL
Zin = 4.0000e+02 + 3.0000e+02i
ILeff = Vseff/Zin
ILeff = 1.9856e-01 - 2.3923e-02i
ILpk = sqrt(2)*abs(ILeff)
ILpk = 2.8284e-01
ILang = angle(ILeff)*180/pi
ILang = -6.8699e+00
SL = ZL*ILeff*conj(ILeff)
SL = 1.4000e+01 + 1.2000e+01i
Therefore, i
L
(t) 2 0.2 ( )cos(300t 6.87
o
) 0.2828cos(300t 6.87
o
) A. S
L
14 + j12 VA, and the
average power is 14 W.
SOLUTION 11.15. First use voltage division and observe that V
L

1
2
V
s
6060 V. Now the
complex power is S
L
V
L
V
L
*
(6 + j8)
*
36053.13 216 + j288 VA. Thus the average power is 216
W.
*SOLUTION 11.16. (a) To find V
2
we write a node equation. First we note that Y
i
= 1/Z
i
is the
corresponding admittance. Hence
Y
1
V
2
V
a
( ) +Y
2
V
2
+ Y
3
V
2
V
b
( ) 0
Hence
V
2

Y
1
V
a
+ Y
3
V
b
Y
1
+Y
2
+Y
3
100 + j50 111.826.57
o
V
obtained using MATLAB as follows
Z1 = 0.1+j*0.1; Z2 = 0.4+j*2.2;
Z3 = 0.2 + j*0.2; Va = 104 + j*50; Vb = 106 + j*48;
Y1 = 1/Z1
Complex Power Prbs 2/5/00 P11-7 @ DeCarlo & P. M. Lin
Y1 = 5.0000e+00 - 5.0000e+00i
Y2 = 1/Z2
Y2 = 8.0000e-02 - 4.4000e-01i
Y3 = 1/Z3
Y3 = 2.5000e+00 - 2.5000e+00i
V2 = (Y1*Va + Y3*Vb)/(Y1 + Y2 + Y3)
V2 = 1.0000e+02 + 5.0000e+01i
magV2 = abs(V2)
magV2 = 1.1180e+02
angleV2 = angle(V2)*180/pi
angleV2 = 2.6565e+01
(b) Again working strictly in MATLAB we have the following complex powers of the loads and the two
sources:
Sz1 = (V2 - Va)*conj((V2-Va)*Y1)
Sz1 =
8.0000e+01 + 8.0000e+01i
Sz2 = V2*conj(V2*Y2)
Sz2 =
1.0000e+03 + 5.5000e+03i
Sz3 = (V2 - Vb)*conj((V2-Vb)*Y3)
Sz3 =
1.0000e+02 + 1.0000e+02i
Sva = Va*conj((Va - V2)*Y1)
Sva =
1.0800e+03 + 3.0800e+03i
Svb = Vb*conj((Vb - V2)*Y3)
Svb =
1.0000e+02 + 2.6000e+03i
(c) To verify conservation of power observe that:
TotSrsPwr = Sva + Svb
TotSrsPwr =
1.1800e+03 + 5.6800e+03i
TotLdPwer = Sz1 + Sz2 + Sz3
TotLdPwer =
1.1800e+03 + 5.6800e+03i
which provides the desired verification.
SOLUTION 11.17. Use MATLAB and refer to the following figure:
Complex Power Prbs 2/5/00 P11-8 @ DeCarlo & P. M. Lin
Z1
Z2 Z3
Z4
%(a)
%Bundle the impedances as per the following figure and
%obtain the following.
Z1=2+2*j;
Y1=1/Z1;
Y2=2+0.5*j;
Z2=1/Y2;
ZL3=4*j;
YL3=1/ZL3;
Y3=4+0.25*j;
Z3=1/Y3;
Z4=4+4*j;
Y4=1/Z4;
V1=10+2*j;
V2=12+2*j;
%Write out KCL for node 1 and 2
%(Va-V1)*Y1=V1*Y2+(V1-V2)*YL3
%(Vb-V2)*Y4=V2*Y3+(V2-V1)*YL3
Va= (V1*Y2+(V1-V2)*YL3)/Y1+V1
Vb= (V2*Y3+(V2-V1)*YL3)/Y4+V2
Va = 29.0000 +59.0000i
Vb = 1.6000e+02+ 2.3400e+02i
(b)
Sr3=( (Va-V1)*Y1*2)*conj((Va-V1)*Y1)
Sl1= ((Va-V1)*Y1*(2*j))*conj((Va-V1)*Y1)
Sc1= V1*conj(V1*0.5*j)
Sr1= V1*conj(V1/0.5)
Sl3= (V1-V2)*conj((V1-V2)*YL3)
Sc2= V2*conj(V2*0.25*j)
Sr2= V2*conj(V2/0.25)
Sl2= (V2-Vb)*Y4*4*j*conj((V2-Vb)*Y4)
Sr4= (V2-Vb)*Y4*4*conj((V2-Vb)*Y4)
SVA = Va * conj((Va-V1)*Y1)
SVB = Vb * conj((Vb-V2)*Y4)
Sr3 = 9.0250e+02
Sl1 = 0 + 9.0250e+02i
Sc1 = 0 - 5.2000e+01i
Complex Power Prbs 2/5/00 P11-9 @ DeCarlo & P. M. Lin
Sr1 = 208
Sl3 = 0 + 1.0000e+00i
Sc2 = 0 - 3.7000e+01i
Sr2 = 592
Sl2 = 0 + 9.4660e+03i
Sr4 = 9466
SVA = 1.1115e+03 + 8.4550e+02i
SVB = 1.0057e+04 + 9.4350e+03i
(c) Take the real part of each of the complex power found in (b). The only components with non-zero
average power will be the resistors which have 208 W, 592 W, 902.5 W, and 9466 W average power
respectively.
%(d)
Total_passive=Sr1+Sr2+Sr3+Sr4+Sl1+Sl2+Sl3+Sc1+Sc2
Total_active= Va*conj((Va-V1)*Y1)+Vb*conj((Vb-V2)*Y4)
Total_passive =
1.1168e+04+ 1.0280e+04i
Total_active = 1.1168e+04+ 1.0280e+04i
which verifies the conservation of power.
SOLUTION 11.18. (a) From conservation of energy, the complex power is the sum of the complex
power absorbed by every circuit elements. Thus S
s
1240 + j145 VA, and the apparent power is 1248.4
VA. The average power is 1240 W.
(b) From S
s
V
s
I
s
*
, I
s

S
s
V
s
5.428 A.
SOLUTION 11.19. (a) The complex power delivered by the source is the sum of the complex power
consumed by the circuit elements. Thus S
s
44 + j28 kVA.
(b) I
s

S
s
V
s
22.675 A
(c) The total power delivered to the three groups of impedance following V
1
is S
1
41.5 + j22. From
the current obtained in (b) V
1

S
1
+ S
2
+ S
4
I
s
2071.5 V.
(d) From V
1
, and the total power delivered to Z
4
and Z
2
, I
2

S
4
+ S
2
V
1
12.385 A. Finally
V
2

S
2
I
2
1805.5 V.
Complex Power Prbs 2/5/00 P11-10 @ DeCarlo & P. M. Lin
SOLUTION 11.20. (a) I
s

44 + j28
2.3


_
,
*
= 22.675 (32.47 0) 22.675 32.47 A.
(b) V
1

S
1
+ S
2
+ S
4
I
s
*




1
]
1
1
2071.5 4.54 V
(c) Similarly as before I
2

S
1
+ S
2
V
1



1
]
1
*
12.385 37.61 A, and V
2
Z
2
I
2
1805.5 11.05 V.
SOLUTION 11.21. From equation 11.30, we find Q P
1
pf
2
1 455.61 var; thus
S 1000 + j455.61 109924.5 VA.
SOLUTION 11.22. (a) S
1
1000 W, and S
2
800 + j600 VA. Thus the total power delivered by the
source is S
tot
1800 + j600 1897.3718.43 VA, and I
s

S
tot
*
V
s
*
15.81 18.43 15 j5 A rms.
(b) V
1

S
1
I
s
*
63.25 18.43 V.
(c) V
2

S
2
I
s
*
63.2518.43 V.
SOLUTION 11.23. Using 11.25, find S
L

P
ave
pf

3000
0.75
4000 VA, and the load current
I
L

S
L
V
L

4000
120
33.33 A. The power absorbed by the transmission line is then from
P
line
RI
L
2
0.2 33.33 ( )
2
222.2 W.
SOLUTION 11.24. The capacitor must absorb a reactive power of Q
new
Q
old
17.9 kvar. Thus
jQ
C
j17.9 j CV
s
2
, and C
Q
C
V
s
2
0.897 mF.
SOLUTION 11.25. From equation 11.30, Q
new
86.6
1
(0.94)
2
1 31.43 VA. Thus the reactive
power absorbed by the capacitor is 18.57 var. Hence C
Q
C
V
s
2
3.714 F.
Complex Power Prbs 2/5/00 P11-11 @ DeCarlo & P. M. Lin
SOLUTION 11.26. Device 1 has a complex power of S
1
= P
1
+ jQ
1
= 360 + j 0 VA. Recall equation
11.29, pf
P
ave
S
, and equation 11.30 Q
new
P
ave
1
(pf )
2
1
where with a lagging pf, Q > 0, and with a leading pf for Q < 0. Using equation 11.30, we have for
device 2:
Q
2
1440
1
(0.8)
2
1 1080 var
S
2
= 1440 + j 1080 VA.
S
1,2
S
1
+ S
2
1800 + j1080 VA
As an aside we compute the magnitude of the current without the capacitor attached.
I
s

S
1,2
120
17.493 A
The capacitor is used to achieve a lower source current with the same average power. The first step is to
find the desired QC. Here
S
tot
S
1
+ S
2
+ S
C
1800 + j1080 + jQ
C
Hence
S
tot
120

1800 + j1080 + jQ
C
120
15
In MATLAB we have:
QC = sqrt((15*120)^2 - 1800^2) - 1080
QC = -1080
From the formula on page 451 of the text,
Q
C
CV
source
2
Hence C = 0.2 mF. Finally
pf
P
ave
S
1
+ S
2
+ S
C

1800
1800
1
SOLUTION 11.27. (a) From equation 11.30, Q
old
7
1
(0.65)
2
1 8.1839 kvar. Therefore, the
power absorbed without the capacitor bank is S
old
7 + j8.1839 kVA. When the bank is added we
want Q
new
7
1
(0.8)
2
1 5.25 kvar, and henceS
new
7 + j5.25 kVA. Thus the reactance that must
be absorbed by the bank is 2.934 kvar, and C
eq

Q
C
V
s
2
0.13511 mF.
(b) As was just determined 5.25 kVA.
Complex Power Prbs 2/5/00 P11-12 @ DeCarlo & P. M. Lin
(c) S
old

7
0.65
10.77 kVA. S
new

7
0.8
8.75 kVA. The kVA saving is: 10.77 8.75 = 2.02
kVA.
Savings = 20*2.02*12
Savings =
4.8480e+02
i.e., $484.80.
SOLUTION 11.28. (a) The apparent power is simply 94kW/0.78=120.51 kVA.
(b) S
m
120.5138.74
o
94 + j 75.41 kVA.
(c) 75.41 kvar.
(d) I
eff

S
m
*
V
eff
*
523.96 38.74 A.
(e) By KVL, V
s
R
line
I
eff
+ V
eff
516.08 j229.52 V.
(f) S
s
V
s
I
eff
*
295.9414.76 kVA.
(g) The efficiency is
120.51cos(38.74)
295.94cos(14.76)
100 32.96%. Note that the line resistance of 0.7 is much
to large for practical usage. This value is chosen for pedagogical reasons.
(h) With a power factor of 0.94, S
m
new
94 + j 34.12 kVA. The average power of the motor must be kept
the same. The reactance that must be provided by the capacitor is, Q
new
Q
old
Q
C
41.29 kvar. The
proper capacitor current will be I
C

j42.29k
V
eff
*
, and Z
C

1
j C

V
eff
I
C

V
eff
2
j 42.29 10
3
. Solving for
C
42.29 10
3
V
eff
2
2.12 mF.
(i) I
eff
new

S
m
new
V
eff



_
,

*
434.79 19.95 A
(j) By KVL, V
s
R
line
I
eff
new
+ V
eff
516.1 j103.84 526.43 11.38
o
V.
(k) S
s
new
V
s
new
I
eff
new
( )
*
228.898.57 226.33 + j 34.12 kVA, and the efficiency is 41.5%.
SOLUTION 11.29. (a) The Thevenin equivalent seen at the output is, Z
th
5 j /(0.1 ). For
maximum power transfer, Z
L
Z
th
*
5 + j . Note that V
OC
50 V rms, and that I
eff

V
OC
Z
th
+ Z
L
5 A
rms. Thus S
L
Z
L
I
eff
I
eff
*
125 + j25 VA, and the average power is 125 W.
SOLUTION 11.30. First find the following Thevenin equivalent,
Z
th
2 + j 4 , V
oc

10 2
3
V.
By the maximum transfer property, R
L
2, and C 1/(4 ) 0.25 mF.
Complex Power Prbs 2/5/00 P11-13 @ DeCarlo & P. M. Lin
SOLUTION 11.31. (a) The thevenin equivalent left of the load is by KCL,
I
test
+ I
s
V
R
( j0.001+ 0.001) and V
test
3V
R
+ j500I
test
V. Substituting for V
R
,
V
test
I
test
1500 j1000 [ ] + I
s
1500 j1500 [ ] V, and Z
th
1500 j1000 with
V
oc
I
s
2121.3 45 V. The value of the load for maximum power transfer is then,
Z
L
1500 + j1000 .
(b) The complex power absorbed by the load is, S
L
Z
L
V
oc
Z
th
+ Z
L
2
750 + j500 VA, and the average
power 750 W.
SOLUTION 11.32. Consider
V1 V2
(a) Using nodal analysis get the following equations:
V
1
V
2
4(V
1
/8)
I
s
+ I
test
V
1
(1/8 + j0.25) + V
2
(1/16 + j0.5)
V
test
V
2
+ 8I
test
Using MATLAB we get the following expression,
V
test
(8.2847- j0.9110)I
test
+ (0.2847- j0.9110)I
s
From this expression V
oc
28.47 j91.1 V, and Z
th
8.2847- j0.9110 . The phasor equivalent of
this circuit is a V
oc
source in series with a 8.28 resistor and a 7.32 mF capacitor.
(b) Z
L
8.2847+ j0.9110, which is a 8.28 Ohm resistor in series with a 6.07mH inductor. Same
configuration as before.
(c) S
L
Z
L
V
oc
Z
th
+ Z
L
2
275.2 + j30.26 VA and the average power is 275.2 W.
SOLUTION 11.33. (a) The Norton equivalent may be found by inspection as Z
th
10 + j20 and
I
sc
10 A. Thus for maximum power transfer, Z
L
20 j20 . This is a 10 resistor and a 0.005 F
capacitor in series. The maximum power is S
L
Z
L
I
sc
Z
th
Z
th
+ Z
L
2
1250 j2500 VA. The maximum
average power is 1250 W.
(b) If R is set to 20 , the closest that can be achieved to maximum power transfer is Z
L
20 j20 , or
C equal to 0.005 F. With Z
L
as above, by current division
Complex Power Prbs 2/5/00 P11-14 @ DeCarlo & P. M. Lin
I
load
10
10 + j20
(10 + j20) + (20 j20)
3.333 j6.6667 7.4563.434
o
A
The maximum average power then 20 I
load
2
1111 W.
(c) Using 11.38, R
L
31.62 . P
av
R
L
I
sc
Z
th
Z
th
+ Z
L
2
600.63 W.
SOLUTION 11.34. (a) From Thevenin Z
th
19.2 j14.4 , and from 11.38, set R
L
24 .
V
oc

V
s
( j 40)
30 j40
80 36.87. The maximum power is P
av

V
oc
Z
th
+ R
L
2
R
L
74.07 W.
(b) The voltage is V V
oc
R
L
Z
th
+ R
L
, from this relationship, one sees that as the load resistance increase
to infinity the output voltage goes to V
oc
, which is the maximum output voltage.
SOLUTION 11.35. Correction: the inductor symbol in the load should be a resistor. Since the source
resistance is variable, example 6.21 serves as a reference suggesting that R = 0 is the answer. To see this
consider that
P 2I
load
2

2 100
(R + 2)
2
+ L
1
C


_
,
2

2 100
(R + 2)
2
+ (2 2)
2
Hence, decreasing R produces increasing power and the maximum power is transferred when R = 0 with
P
max
50 W assuming that the source voltage is given in rms V.
SOLUTION 11.36. As per problem 35,
P 10I
load
2

10 50 2
( )
2
(R +10)
2
+ L
1
C


_
,
2

12.5 10
3
(R +10)
2
+ L
1
C


_
,
2
Here, again R = 0 with C chosen to eliminate the reactive term maximizes power transfer. Hence
C
1
2
L
0.01 F
with P
max
125 W.
SOLUTION 11.37. (a) By the maximum power transfer theorem, P
1
is maximized when Z
L
is chosen
as the conjugate of Z
source
, i.e.
Z
L
10 + j1000
(b) To find the appropriate values of L and C observe that
Complex Power Prbs 2/5/00 P11-15 @ DeCarlo & P. M. Lin
Z
L
( j ) j L +
1
10
4
+ j C
j10
7
L +
10
4
j C
10
8
+10
14
C
2

10
4
10
8
+10
14
C
2
+ j 10
7
L
C
10
8
+10
14
C
2



_
,
10 + j1000
Equating real parts leads to:
%10^-4 = 10^-7 + 10^15*C^2

C = sqrt(1e-4 - 1e-7)/sqrt(1e15)
C = 3.1607e-10
Thus C = 0.31607 nF.
Equating imaginary parts using the above value of C leads to:
w = 1e7;
L = (1e3 + w*C/(1e-8 + 1e14*C^2))/1e7
L = 1.3161e-04
Thus L = 0.1316 mH.
(c) In part (b), L and C are chosen to maximize P
1
, the power delivered to Z
L
. Since L and C consume
no average power, this maximum power is transferred to the 10 k fixed resistor with the
computed values of L and C. Thus Z
L
is the same as in part (b) or Z
L
10 + j1000 .
Since we know Z
L
,
P
max

(0.1)
2
4 10
0.25 mW
This is the average power consumed by the 10 k resistor. Therefore
V
2
2
0.25 10
3
10
4
2.5
It follows that V
2
1.5811 V. Power to the 10 k fixed resistor is
P
10k

V
2
2
10
4
Thus if P
10k
is maximized, then V
2
is maximized.
SOLUTION 11.38. (a) From equation 11.4, if we substitute
2
T
, then the resulting instantaneous
power will be p(t)
V
m
I
m
2
cos(
v

i
) +
V
m
I
m
2
cos(
4
T
t +
v
+
i
) W where it is clearly seen that the
fundamental period will now be halved. Note that by the same argument the fundamental frequency of
the instantaneous power is double that of the voltage and current.
Complex Power Prbs 2/5/00 P11-16 @ DeCarlo & P. M. Lin
(b) As a sinusoid, the fundamental period is 2/10, any integer multiple of this period will also be
periodic.
(c) This is the same as (b) with an offset of 1 V added.
SOLUTION 11.39. First, F
eff

1
T
f
2
(t)dt
t
0
t
o
+T

. Without going into detailed calculation, f


2
(t) will
give a summation of two types of products, a product of each element with themselves, and products of
each element with the other element. In the later case, we know that two cosines multiplied with one
another and integrated over one period will yield zero if their angular frequency are different. As for the
former case the integral will yields the result we are looking for. For example look at the first two terms,
F
eff

1
T
f
2
(t)dt
t
0
t
o
+T

1
T
F
o
2
+ 2F
1
2
cos
2
(
1
t +
1
) + ...
( )
dt
t
0
t
0
+T

1
T
F
o
2
+ F
1
2
+ F
1
2
cos(2
1
t + 2
1
) + ...
( )
dt
t
0
t
0
+T

1
T
F
o
2
t + F
1
2
t + F
1
2
sin(2
1
t + 2
1
) + ...
[ ]
0
T
F
o
2
+ F
1
2
...
SOLUTION 11.40. (a) We are given that v
C
(t) V
m
sin( t) V. Hence
i
C
(t) C
dv
C
dt
CV
m
cos( t) A
It follows that
p(t) v
C
(t)i
C
(t) CV
m
2
sin( t)cos( t) 0.5 CV
m
2
sin(2 t) Watts
Clearly, p(t) has a peak value of 0.5 CV
m
2
and the integral of the sign over one period is zero implying
that the average value of p(t) is zero.
(b) Here
W
C
(t) 0.5Cv
C
2
(t) 0.5CV
m
2
sin
2
( t) 0.25CV
m
2
1 cos(2 t) ( ) J
Here the peak value occurs when cos(2t) = 1 in which case the peak value is 0.5CV
m
2
. Further, the
average value of cos(2t) over one period, T = /, is zero whereas the average of a constant over the
same period is simply the constant. Hence, W
C,ave
0.25CV
m
2
J.
(c) From example 11.6,
Q
C
I
C,eff
V
C,eff
CV
C,eff
2
0.5 CV
m
2
2 0.25CV
m
2
( )
2 W
C,ave
.
Therefore, W
C,ave

Q
C
2
.
Complex Power Prbs 2/5/00 P11-17 @ DeCarlo & P. M. Lin
SOLUTION 11.41. (a) We are given that i
L
(t) I
m
sin( t) A. Hence
v
L
(t) L
di
L
dt
LI
m
cos( t) V
It follows that
p(t) v
L
(t)i
L
(t) LI
m
2
sin( t)cos( t) 0.5 LI
m
2
sin(2 t) watts
Clearly, p(t) has a peak value of 0.5 LI
m
2
and the integral of the sign over one period is zero implying
that the average value of p(t) is zero.
(b) Here
W
L
(t) 0.5Li
L
2
(t) 0.5LI
m
2
sin
2
( t) 0.25LI
m
2
1 cos(2 t) ( ) J
Here the peak value occurs when cos(2t) = 1 in which case the peak value is 0.5LI
m
2
. Further, the
average value of cos(2t) over one period, T = /, is zero whereas the average of a constant over the
same period is simply the constant. Hence, W
L,ave
0.25LI
m
2
J.
(c) As an extension to example 11.5,
Q
L
I
L,eff
V
L,eff
LI
L,eff
2
0.5 LI
m
2
2 0.25CI
m
2
( )
2 W
L,ave
.
Therefore, W
L,ave

Q
L
2
.
SOLUTION 11.42. (a) The complex power absorbed by the load is, S
L
VI
*
ZII
*
Z I
2
. Now note
that the average power is the real part of the complex power. Also note that a complex number
multiplied by its complex conjugate will yield a real value. Therefore the real part of ZI
2
is just the
real part of Z, R, multiplied by I
2
, P
av
RI
2
. With the same reasoning, the reactance is the
imaginary part of Z, X, multiplied by I
2
, Q XI
2
.
(b) The complex power absorbed by the load is S
L
VI
*
V(YV)
*
Y V
2
. The same reasoning as in
(a) holds thus the real part of the admittance times V
2
, yields P
av
GV
2
. Using the imaginary
part, Q BV
2
.
SOLUTION 11.43. (a) The equivalent resistance seen by the source is R
eq
= 6 j9 . So the current
delivered by the source is:
I
s

V
s
6 j9

j110
6 j9
10.17 33.69 A
or i
L
(t) 10.17 2 cos(120 t 33.69
o
) A. Similarly
V
C
j15 I
s

15 110
6 j9
152.54 123.69 V
Complex Power Prbs 2/5/00 P11-18 @ DeCarlo & P. M. Lin
or v
C
(t) 152.54 2 cos(120 t 123.69
o
) V. Further, L
6
15.9 mH and C
1
15
176.8 F.
The instantaneous energy stored in the inductor is
W
L
(t) 0.5Li
L
2
(t) 1.646cos
2
(120 t 33.69
o
) J
and the instantaneous energy stored in the capacitor is
W
C
(t) 0.5Cv
C
2
(t) 4.115cos
2
(120 t 123.69
o
) J
The source voltage is zero when t = 0. Therefore
W
L
(0) 1.646cos
2
(33.69
o
) 1.1395 J
and
W
C
(0) 4.115cos
2
(123.69
o
) 1.266 J
(b) and (c) Observe that i
L
(t) and v
C
(t) are 90
o
out of phase. When one is zero, the other has a peak
value. Therefore whenW
C
0 implies v
C
(t
0
) = 0 for appropriate t
0
; hence W
L
(t
0
) 1.646 J.
Similarly, when W
L
= 0, say at t
0
, then W
C
(t
0
) 4.115 J.
SOLUTION 11.44. In order to solve this problem, we want to express the power in terms or R's and L's
in both circuits. First, looking at the circuit with just the coil and the 110 V source: I V / Z
coil
,
Z
coil
R + j L, and
P
coil
I
2
R
110
2
R
2
+
2
L
2
R 300 watts (*)
Next, looking at the circuit when a resistance is added in series with the coil, I V /(8 + Z
coil
),
Z
coil
R + j L,
P
coil
I
2
R
220
2
(8 + R)
2
+
2
L
2
R 300 watts (**)
To find R, solve equation (*) for R
2
+
2
L
2
and substitute into equation (**) to obtain
R = 300*64/(220^2 - 300*16 -110^2)
R = 6.0952e-01
Substituting R into equation (*) yields L 13.05 mH.
SOLUTION 11.45. The average power consumed by the 2.7 resistor is 250 watts. This allows us to
compute the magnitude of | I
coil
|. We know that | V
coil
| is 150 Vrms. Thus we can compute
the magnitude of the coil impedance and hence L as follows:
magIcoil = sqrt(250/2.7)
magIcoil = 9.6225e+00
magZcoil = 150/magIcoil
magZcoil = 1.5588e+01
Complex Power Prbs 2/5/00 P11-19 @ DeCarlo & P. M. Lin
% magZcoil^2 = 2.7^2 + (w*L)^2

w = 2*pi*60;
L = sqrt( magZcoil^2 - 2.7^2)/w
L =
4.0725e-02
% Magnitude of impedance seen by 220 V source is:

magZin = 220/magIcoil
magZin = 2.2863e+01
% magZin^2 = (R + 2.7)^2 + (w*L)^2

Realpart = sqrt(magZin^2 - (w*L)^2)


Realpart = 1.6941e+01
R = Realpart - 2.7
R = 1.4241e+01
9/26/01 P12-1 @ DeCarlo & P. M. Lin
CHAPTER 12. PROBLEM SOLUTIONS
SOLUTION P12.1. By conservation of energy, the instantaneous power consumed by each load when
summed together is equivalent to the total power consumed by the three phase load. Thus writing out
p
tot
(t) p
AB
(t) + p
BC
(t) + p
CA
(t) v
AB
(t)i
AB
(t) + v
BC
(t)i
BC
(t) + v
CA
(t)i
CA
(t)
V
L
2 cos( t +
v
)
V
L
2
Z
cos( t +
i
) +V
L
2 cos( t +
v
120
o
)
V
L
2
Z
cos( t +
i
120
o
)
+ V
L
2 cos( t +
v
+120
o
)
V
L
2
Z
cos( t +
i
+120
o
)

V
L
2
Z
cos(
v

i
) + cos(2 t +
v
+
i
) ( ) +
V
L
2
Z
cos(
v

i
) + cos(2 t +
v
+
i
+120
o
)
( )
+
V
L
2
Z
cos(
v

i
) + cos(2 t +
v
+
i
120
o
)
( )

3V
L
2
Z
cos(
v

i
)
3V
L
2
Z
pf
SOLUTION P12.2. To justify the point of this problem we equate the following two equations:
(i) For the 3 phase system:
P
loss
'
3 I
L
2
R
'
3
3
9

P
L
2
V
L
2
R
'

P
L
2
V
L
2
R
'
(ii) For the single phase system:
P
loss
2 I
L
2
R
( )

P
L
2
V
L
2
2R
It follows that R' = 2R. Since both systems have the same distance of transmission and the resistance of
a wire is inversely proportional to the cross sectional area, the condition R' = 2R implies that the cross
section A' of each wire in the three-phase system need only be half of the area A of the wire in the single
phase system. But there are two wires in the single phase system and three wires in the three-phase
system. Therefore the ratio of the materials used is:
material in 3 phase system
material in 1 phase system

3A
'
2A

3
2

1
2
75%
SOLUTION P12.3. For row 1 of table 12.1, the impedance in (a) seen between each pair of terminal is
Z
ik
Z

|| 2Z

2
Z

3
. In (b) the impedance seen between any two terminals is
Z
jk

3
+
Z

3
2
Z

3
.
9/26/01 P12-2 @ DeCarlo & P. M. Lin
In row 2, the impedance between any two terminals is Z
jk
Z
Y
+ Z
Y
2Z
Y
for (c), and
Z
ik
3Z
Y
|| 6Z
Y
2Z
Y
for (d).
SOLUTION P12.4. Consider the -Y relationship of the figures below (row 3 of table 12.1):
Let us consider the terminal pair (1,2):
(i) For the Y-connected case, Z
th
2
Z

3
and
V
oc

V
p
3
30
o
( )

V
p
3
150
o
( )

V
p
3
1 30
o
1 150
o
( )
V
p

(ii) For the D-connected case,


Z
th
Z

/ / 2Z

( )
2(Z

)
2
3Z

2
3
Z

Now note that there is no load connected to the -configuration. Applying KVL to the indicated loop
implies that:
0 3Z

I
loop
+V
p
+V
p
120
o
+ V
p
+120
o
3Z

I
loop
Hence I
loop
0 . Finally, V
oc
V
12
Z

I
loop
+V
p
V
p
. Therefore, looking into terminals 1-2,
both the -configuration and the Y-configuration have the same Thevenin equivalent. For terminal pairs
(1-3) and (2-3), the proof is virtually the same. Hence this establishes the equivalence in row 3 of table
12.1.
To establish the equivalence in row 4 of table 12.1, we do all the same computations with the
slightly different labeling to obtain the same result, i.e., the circuits are equivalent.
SOLUTION 12.5. For (a) first note the following relationship V
N

V
1
Z
1
+
V
2
Z
2



_
,

Z
1
|| Z
2
|| Z
3
( ) , which is
obtained by KCL at the center node, N, with node 3 as the reference node.. Write out KCL at terminal 1,
I
1

V
1
Z
1

V
1
Z
1
Z
1
+
V
2
Z
2
Z
1



_
,

Z
1
|| Z
2
|| Z
3
( ) V
1
Z
3
+ Z
2
Z
2
Z
3
+ Z
1
Z
3
+ Z
1
Z
2



_
,

V
2
Z
3
Z
1
Z
2
+ Z
2
Z
3
+ Z
3
Z
1




_
,


. Do the
9/26/01 P12-3 @ DeCarlo & P. M. Lin
same for terminal 1 in (b), I
1
V
1
1
Z
31
+
1
Z
12



_
,

V
2
Z
12
. Now substitute the formulas in the problem
statement into the later equation, I
1
V
1
Z
2
+ Z
3
Z
1
Z
2
+ Z
2
Z
3
+ Z
3
Z
1




_
,


V
2
Z
3
Z
1
Z
2
+ Z
2
Z
3
+ Z
3
Z
1




_
,


, which is
the same as the equation for (a).
Using the same method, the same result is obtained for node 2. So the fact that the substitution
of the equivalence in (b) yields the same equation as in (a) proves that the equivalences are accurate.
SOLUTION 12.6. By Ohms law I
A

V
p
Z

2000
10 + j5
17.89 26.57 A. By the same relationship
I
B
17.89 146.57 A, and I
C
17.8993.43 A. The neutral line current is the sum of the other
three and is zero. The power of each phase is the same, as they have the same load, and current
magnitude. Using the current at terminal A, the total power is, P 3RI
A
2
9602 W.
SOLUTION 12.7. For this balanced Y-Y connection,we follow the method of example 12.6. Using
MATLAB:
Z = 20 + j*10'
Z = 2.0000e+01 + 1.0000e+01i
VAN = 200;
IA = VAN/Z
IA = 8.0000e+00 - 4.0000e+00i
IAmag = abs(IA)
IAmag = 8.9443e+00
IAangle = angle(IA)*180/pi
IAangle = -2.6565e+01
% Use phase inference to obtain:
IBmag = IAmag
IBmag = 8.9443e+00
IBangle = IAangle - 120
IBangle = -1.4657e+02
ICmag = IAmag
ICmag = 8.9443e+00
ICangle = IAangle + 120
ICangle = 9.3435e+01
Ptotave = 3*real(Z)*IAmag^2
Ptotave = 4.8000e+03
9/26/01 P12-4 @ DeCarlo & P. M. Lin
SOLUTION 12.8. Using the same approach as in the example 12.6 we compute
V
AN
V
sA

Z
Z + Z
g
120
4 + j3
4 + j3 + 0.1 + j0.2
116.94 V
Thus the drop is:
120 116.94
120
100 2.55 % .
SOLUTION 12.9. A 5.5 % drop corresponds to V
AN
113.4 V. This means
Z
Z + Z
1
+ Z
g

V
AN
120

0.945 120
120
0.945
From this equation,
Z + Z
1
+ Z
g

Z
0.945
5.291.
So 4.05 + 0.1 ( )
2
+ 3.15 + 0.2 ( )
2
27.99 . Solving the quadratic equation results in 0.7893.
SOLUTION 12.10. Solving the single phase equivalent circuit for phase A,
I
A

V
p
V
p
120
10 + j5
58.87 56.57
Using phase inference,
I
B
I
A
120 58.87 176.57
I
C
I
A
120 58.8764.43
Alternately, using much more work,
I
B

V
p
120 V
p
10 + j5
58.87 176.57
I
C

V
p
120 V
p
120
10 + j5
58.8764.43
The total power, using the previously derived relationship, is
P 3V
L
I
L
pf 3 380 58.87 pf 34.655 kW
where
pf cos tan
1
5
10



1
]
1



_
,
0.8944
9/26/01 P12-5 @ DeCarlo & P. M. Lin
SOLUTION 12.11. (a) By KVL and Ohm's law,
I
AB

V
p
30 + j15
6.57 26.57
By phase inference,
I
BC
I
AB
120 6.57 146.57
I
CA
I
AB
120 6.5793.43
Alternately,
I
BC

V
p
120
30 + j15
6.57 146.57
I
CA

V
p
120
30 + j15
6.5793.43
To compute the line currents we have in Amps:
I
A
I
AB
I
CA
11.36 56.57
I
B
I
BC
I
AB
11.36 176.57
I
C
I
CA
I
BC
11.3663.43
(b) The average power is P 3 30 I
AB
2
3.88 kW, and the total reactive power is
Q 3 15 I
AB
2
1.94 kvar.
SOLUTION 12.12. From the efficiency we know that
25 746
P
deliverd
0.85 , so the total power being
delivered to the load is 21941 W. Using 12.4, I
L

21941
3 V
L
pf
63.31 A.
SOLUTION 12.13. (a) Using the power efficiency relationship, P
tot

300 746
0.935
239.36 kW.
(b) Using 12.4, I
L

239.36 10
3
3 V
L
pf
68.28 A.
SOLUTION 12.14. The magnitude of the power delivered to each delta connected load is 79.787 kW.
We can now perform the analysis on a single phase. Using equation 11.30, we obtain
9/26/01 P12-6 @ DeCarlo & P. M. Lin
Q
old
79787
1
0.88


_
,
2
1 43064 vars
and
Q
new
79787
1
0.95


_
,
2
1 26225 vars
Hence
S
old
79787 + j 43064
S
new
79787 + j26225
So the reactive power to be supplied by the capacitor is Q
C
= 16839 = 43064 26225. The
capacitance
C
Q
C
V
l
2
8.44 uF.
SOLUTION 12.15. Replacing the delta configuration of the source by its Y-equivalent, the following
voltages (in V) are obtained,
V
AN

V
p
3
30, V
BN

V
p
3
150, V
CN

V
p
3
90
From the single phase equivalent,
I
A

V
AN
Z
19.63 56.57
From phase inference
I
B
I
A
120 19.63 176.57
I
C
I
A
120 19.6363.43
The power factor of the load is
pf cos tan
1
5
10



1
]
1



_
,
0.8944
Hence, the total power in the balanced system is
P 3V
L
I
L
pf 3 380 19.63 0.8944 11.556 kW.
SOLUTION 12.16. First replace the delta connected load by it Y-equivalent. Then analyzing a single
phase and using phase inference, we obtain in amps,
9/26/01 P12-7 @ DeCarlo & P. M. Lin
I
A

V
p
(Z / 3)
34.08 26.56
I
B
I
A
120 34.08 146.56
I
C
I
A
120 34.0893.43
The power factor of the load is
pf cos tan
1
5
10



1
]
1



_
,
0.8944
The total power is P 3V
L
I
L
pf 3 127 34.08 pf 11.61 kW. The voltage across each load is
3 V
p
V
L
219.97 V.
SOLUTION 12.17. (a) Referring to figure 12.2, the following relationships may be pointed out:
V
AB
V
AN
+ V
NB
V
AN
V
BN
V
BC
V
BN
+ V
NC
V
BN
V
CN
V
CA
V
CN
+ V
NA
V
CN
V
AN
(b) Perform a delta to Y-transformation and use Ohms law and phase inference to obtain in amps:
I
A

220 30
3 Z
11.36 56.57
I
B
11.36 176.57
I
C
11.3663.43
(c) First,
pf cos tan
1
5
10



1
]
1



_
,
0.8944
The average power, P
av
3V
L
I
L
pf 3 220 I
A
0.8944 3.87 kW.
The reactive power may be found as follows, Q 3 5I
L
2
1.936 kvar.
SOLUTION 12.18. Part (a) Apply Y transformation to the source.
Apply the transformation of row 3, Table 12.1 to the source. The result is the circuit shown in figure
12.19 of example 12.8. Therefore we may use many of the calculated values in example 12.9. In
particular, the line currents are
I
A
= 19.49/-68.9
o
A, I
B
= 19.49/-188.9
o
A,

and I
C
= 19.49/51.09
o
A
Part (b). Compute line-to-neutral voltage. From figure 12.21 and by Ohm's law
9/26/01 P12-8 @ DeCarlo & P. M. Lin
V
AN
= I
A
Z
Y
= 19.49/-68.9
o
536.87
o
=

97.4532.03
o
V
By inference on phase angles
V
BN
=

97.45152.03
o
V and V
CN
=

97.4587.97
o
V
Part (c). Compute line voltages V
AB
, V
BC
, and V
CA
. From the voltage phasor diagram of figure 12.2.
V
AB
3 V
AN
e
j 30
o
168.8e
j 2.03
o
V
By inference on phase angles,
V
BC
V
AB
e
j120
o
168.8e
j122.03
o
V
V
CA
V
AB
e
j120
o
168.8e
j117.97
o
V
Part (d). Compute the total power. From equation 12.4
P
total
3 V
L
I
L
pf 3 168.8 19.49 0.8 4558 W
SOLUTION 12.19. Perform a delta to Y transformation on the source. This yields
V
1N

V
p
3
30
o

380
3
30
o
V. Perform a delta to Y-transformation on the load. This yields a
new impedance Z
new

Z
3
4 + j 3 . Now analyzing a single phase and finding the others by
inference yields, in amps,
I
A

V
1N
Z
1
+ Z
new
42.18 67.97
I
B
42.18 187.97
I
C
42.1852.03
The average power delivered to the load is P
av
3 I
A
2
4 21.35 kW.
SOLUTION 12.20. Performing a delta-to-Y-transformation on the source and the load, the new source
magnitudes are multiplied by 1 3 , and the loads are divided by 3. Observe that Z
g
0.15 + j0.45 ,
Z
1
0.1 + j0.2 , and Z 12 + j9 . Using Ohms law on phase one, and then inference on the
phase angles for the other currents, yields in amps,
I
A

V
p
30
3
Z
g
3
+ Z
1
+
Z
3



_
,

41.14 68.91
I
B
41.14 188.91
I
C
41.1451.09
9/26/01 P12-9 @ DeCarlo & P. M. Lin
The total power may be calculated as follows, P
av
3
12
3


_
,
I
L
2
12 (41.14)
2
20.310 kW.
SOLUTION 12.21. Perform delta-to-Y-transformation on the delta load. Because of the property of the
neutral to be ground in balance circuit, the two loads in the same phase combine in parallel. Thus the new
impedance seen in one branch is
Z
Ynew
Z
Y
||
Z

(5 + j5) (5 j 3)
(5 + j5) + (5 j3)
4.044 + j0.1923
Now looking at a single phase and inferring for the others,
I
A

V
p
30
3 Z
Ynew

230 30
3 4.044 + j0.1923 ( )
32.84 32.73 A
I
B
32.84 152.73 A
I
C
32.8487.27 A
The complex power is S 3 Z
Ynew
I
L
2
13.07 + j0.615 kVA.
SOLUTION 12.22. For this problem we do not need to use any 3-phase knowledge if we are clever.
We will provide a clever solution here. V
DN
= V
DB
V
NB
. Using B as the reference we write a node
equation at N to obtain:
0
V
NB
Z
1
+
V
NA
Z
1
+
V
NC
Z
1

V
NB
Z
1
+
V
NB
V
AB
Z
1
+
V
NB
V
CB
Z
1
Therefore
3V
NB
V
AB
+ V
CB
115 115 120
o
172.5 + j99.59
From voltage division
V
DB

Z
3
Z
3
+ Z
2
V
AB

2 + j4
2 + j4 + 4 j2
115 57.5 + j57.5 V
Now using MATLAB we have
115-115*exp(-j*2*pi/3)
ans = 1.7250e+02 + 9.9593e+01i
VNB =ans/3
VNB = 5.7500e+01 + 3.3198e+01i
VDB = 115*(2+j*4)/(6+j*2)
VDB = 5.7500e+01 + 5.7500e+01i
VDN = VDB - VNB
VDN = 7.1054e-15 + 2.4302e+01i
magVDN = abs(VDN)
magVDN = 2.4302e+01
angVDN = angle(VDN)*180/pi
angVDN = 9.0000e+01
9/26/01 P12-10 @ DeCarlo & P. M. Lin
SOLUTION P12.23. First we write the following two loop equations,
V
sA
V
sA
120 I
A
Z
g
+ Z
1
+ Z
A ( )
+ I
C
+ I
A
( ) Z + Z
1
+ Z
g ( )
V
sA
120 V
sA
120 I
A
+ 2I
C
( ) Z
g
+ Z
1
+ Z
( )
and solve in MATLAB:
(a)
%Problem 12.23a
Zg=0.05+0.15i;
Z=4+3i;
Z1=0.1+0.2i;
ZA=5+2i;
VA= 120;
VB= 120*exp(-120*i*pi/180);
VC= 120*exp(120*i*pi/180);
%Write mesh equations in matrix form and solve:
% i.e. [IA;IC] = A - 1*C
A=[ 2*(Z1+Zg)+ZA+Z Z+Z1+Zg; Zg+Z1+Z 2*(Zg+Z1+Z)];
Ainv= A^(-1);
C=[VA-VB;VC-VB];
B=Ainv*C;
IA= B(1);
IC= B(2);
IB= -IA-IC;
IA
abs(IA)
180*angle(IA)/pi
IB
abs(IB)
180*angle(IB)/pi
IC
abs(IC)
180*angle(IC)/pi
%And By Ohm's law,
VAN= IA*ZA
abs(VAN)
180*angle(VAN)/pi
VBN= IB*Z
abs(VBN)
180*angle(VBN)/pi
VCN= IC*Z
abs(VCN)
180*angle(VCN)/pi
IA = 21.7629.12 A
IB = 23.88155.60 A
IC = 20.64 82.39 A
9/26/01 P12-11 @ DeCarlo & P. M. Lin
VAN = 117.20-7.32 V
VBN = 119.4118.73 V
VCN = 103.2119.26 V
(b)
% From phasor diagram
VAB = VAN - VBN
abs(VAB)
180*angle(VAB)/pi
VBC = VBN - VCN
abs(VBC)
180*angle(VBC)/pi
VCA = VCN-VAN
abs(VCA)
180*angle(VCA)/pi
VAB = 195.5 27.34
VBC = 194.9 92.04
VCA = 196.99 147.8
(c) Calculating the average power delivered to each load using the general formula
P
tot
R I
2

5 21.76 ( )
2
+ 4 23.88 ( )
2
+ 4 20.64 ( )
2
6.353 kW
SOLUTION P12.24. Doing this problem in MATLAB,
%Problem 12.24
ZDg=0.15+0.45i;
ZD=12+9i;
Z1=0.1+0.2i;
ZDA=15+6i;
%Apply the delta to Y transformation to the load as follows,
ZA=(ZD*ZDA)/(2*ZD+ZDA);
ZB=(ZDA*ZD)/(2*ZD+ZDA);
ZC=(ZD*ZD)/(2*ZD+ZDA);
%Apply delta to Y trans. to the source,
V1= 180/sqrt(3)*exp(-30*i*pi/180);
V2= 180/sqrt(3)*exp(-150*i*pi/180);
V3= 180/sqrt(3)*exp(90*i*pi/180);
Zg= ZDg/3;
%Write nodal equations in matrix form and solve
%i.e. [IA;IC]=A-1*C
A=[ 2*(Z1+Zg)+ZA+ZB ZB+Z1+Zg; Zg+Z1+ZB 2*(Zg+Z1)+ZC+ZB];
Ainv= A^(-1);
C=[V1-V2;V3-V2];
B=Ainv*C;
IA= B(1);
IC= B(2);
IB= -IA-IC;
IA
abs(IA)
9/26/01 P12-12 @ DeCarlo & P. M. Lin
180*angle(IA)/pi
IB
abs(IB)
180*angle(IB)/pi
IC
abs(IC)
180*angle(IC)/pi
IA = 17.4 63.1 A
IB = 20.1 178.9 A
IC = 19.5 51 A
(b)
% By Ohm's law,
VAN= IA*ZA;
VBN= IB*ZB;
VCN= IC*ZC;
%From phasor diagram
VAB=VAN - VBN
abs(VAB)
180*angle(VAB)/pi
VBC = VBN-VCN
abs(VBC)
180*angle(VBC)/pi
VCA =VCN - VAN
abs(VCA)
180*angle(VCA)/pi
VAB = 170.40 2.5 V
VBC = 168.58 122.4 V
VCA = 169.75 118.1 V
(c)
% by Ohm's law
IAB=VAB/ZDA
abs(IAB)
180*angle(IAB)/pi
IBC=VBC/ZD
abs(IBC)
180*angle(IBC)/pi
ICA=VCA/ZD
abs(ICA)
180*angle(ICA)/pi
IAB = 10.55 24.3
o
A
IBC = 11.24 159.3 A
ICA = 11.32 81.2 A
(d)
P
tot
R I
2

15 10.55 ( )
2
+12 11.24 ( )
2
+12 11.32 ( )
2
4.721 kW
9/26/01 P12-13 @ DeCarlo & P. M. Lin
SOLUTION P12.25. The average power that the load draws is P 440 40 pf 8.8 kW. From this we
obtain the old and new complex powers:
S
old
8.8 + j15.242 kVA and S
new
8.8 + j15.242 + jQ
C
kVA
Here the reactance supplied by the capacitor is Q
C
CV
l
2
7299 var. So the new power factor is
pf cos tan
1
Q
new
8800


_
,



_
,
0.742 . From S V
l
I
l
11.855 kVA, we solve for
I
l
I
A
I
B
26.94 A.
SOLUTION P12.26. Using Ohms law and the power factor of the load,
I
C
440 C90 16.59 90 A, I
A
4060 A, and I
B
I
C
I
A
26.94 137.9 A.
SOLUTION P12.27. By simply applying Ohms law to each load and then KCL at node N, we get the
following:
I
A

V
A
50
4.4 A, I
B

V
B
50
4.4 120 A, I
C

V
C
250
0.88120 A, and
I
N
I
A
+ I
B
+ I
C
3.52 60 A.
Finally,
P
A
50 4.4 ( )
2
968 W, P
B
50 4.4 ( )
2
968 W, P
C
250 0.88 ( )
2
193.6 W.
SOLUTION P12.28. Using MATLAB,
VA = 220
VB = 220*exp(-120*pi*i/180);
VC = 220*exp(120*pi*i/180);
%Use KVL to solve for the different currents
IA = (VA - VB)/50 + (VA - VC)/50
abs(IA)
180*angle(IA)/pi
IB = (VB - VC)/100 + (VB - VA)/50
abs(IB)
180*angle(IB)/pi
IC = (VC - VB)/100 + (VC - VA)/50
abs(IC)
180*angle(IC)/pi
IA = 13.2 A
IB = 10.1 130.9 A
9/26/01 P12-14 @ DeCarlo & P. M. Lin
IC = 10.1 130.9 A
The total power delivered to each resistor may be calculated using the following relationship P
V
l
2
R
,
where V
l
2
3V
p
2
using Y to Delta source transformation. Thus P
AB
2904 W, P
BC
1452 W, and
P
CA
2904 W.
SOLUTION P12.29. Consider the circuit of figure 12.29 with two additional node labeling:
Choose N as the reference node and apply KCL to node M. This yields the following node equation:
0.02 V
M
220 ( ) + 0.02 V
M
220e
j120
o


_
,
+ 0.004 V
M
220e
j120
o


_
,
0
Equivalently
2.2V
M
88 j152.42 or V
M
40 j69.282 V
The remaining calculations proceed in MATLAB as follows:
% Compute node voltage VM
X = +220 +220*exp(-j*2*pi/3) + 0.2*220*exp(j*2*pi/3)
X = 8.8000e+01 - 1.5242e+02i
VM = X/2.2
VM = 4.0000e+01 - 6.9282e+01i
% Compute line currents
IA = 0.02*(220 - VM)
IA = 3.6000e+00 + 1.3856e+00i
magIA = abs(IA)
magIA = 3.8575e+00
angIA = angle(IA)*180/pi
angIA = 2.1052e+01
IB = 0.02*(220*exp(-j*2*pi/3) - VM)
IB = -3.0000e+00 - 2.4249e+00i
magIB = abs(IB)
magIB = 3.8575e+00
angIB = angle(IB)*180/pi
angIB = -1.4105e+02
IC = 0.004*(220*exp(j*2*pi/3) - VM)
IC = -6.0000e-01 + 1.0392e+00i
magIC = abs(IC)
9/26/01 P12-15 @ DeCarlo & P. M. Lin
magIC = 1.2000e+00
angIC = angle(IC)*180/pi
angIC = 1.2000e+02
% Compute total power
Ptot = 50*magIA^2 + 50*magIB^2 + 250*magIC^2
Ptot = 1.8480e+03
SOLUTION P12.30. (a) The equivalent resistance of a 40 ft size #14 wire is
R
eq1
40
2.575
1000
0.103. The voltage at the light and appliance is then
V
app
115 2 R
eq1
10 112.94 V, which is equivalent to a 1.8% drop, thus the wire gauze is
appropriate.
(b) Repeating the previous calculations, R
eq2
50
1.619
1000
0.08095 .
V
app
115 2 R
eq2
14 112.7334 V, which corresponds to 1.97 %. So the wire gauze is appropriate.
(c) Under normal operating condition the current in the live wire should equal the current in the neutral
wire; thus no current should be in the ground wire.
(d) Note the following relationship, V
AB
V
AN
+ V
NB
V
AN
V
BN
. This means that
V
AN
115 V and V
BN
115180 V.
SOLUTION P12.31. Assuming a very large resistance for the person who touches the prong. For prong
A, the voltage is approximately V 115 V. For prong N, the voltage is approximately V 0. For the
ground prong G, no current flows through the wire, so the voltage should be ground or zero.
SOLUTION P12.32. For an approximate analysis, we use the circuit models below given the following
assumptions:
(i) The resistance from body to ground is very large (possibly due to rubber shoes);
(ii) The resistances of all connecting wires are negligible.
(iii) The motor winding is represented by R
1
in series with R
2
. Here we can further estimate
that. R
1
+ R
2
= 115/3 = 38.3 , and R
1
= R
2
= 19.15 .
(iv) The trigger switch of the drill has been depressed.
(a) The point P is connected to the metal case as shown below. It is obvious that V
case
= V
p
= 0.
9/26/01 P12-16 @ DeCarlo & P. M. Lin
(b) The point M is connected to the metal case as shown below. The current through the hot wire is
115/19.15 = 6 A. If the fuse capacity is smaller than 6 A, then it will blow and V
case
= V
M
= 0. On the
other hand , if the fuse capacity is greater then 6 A, then R
2
is nearly shorted and V
case
= V
M
= 0.
(c) The point Q is connected to the metal case as shown below.
9/26/01 P12-17 @ DeCarlo & P. M. Lin
Both R
1
and R
2
are nearly shorted. The line current is so large that it will blow the fuse. Hence V
case
= V
Q
= 0.
CONCLUSION: In all three cases, the person touching the metal case of the defective appliance will
experience zero or a very low voltage. The circuit is safe.
SOLUTION P12.33. For an approximate analysis, we use the circuit models below given the following
assumptions:
(i) The resistance from body to ground is very large (possibly due to rubber shoes);
(ii) The resistances of all connecting wires are negligible.
(iii) The motor winding is represented by R
1
in series with R
2
. Here we can further estimate
that. R
1
+ R
2
= 115/3 = 38.3 , and R
1
= R
2
= 19.15 .
(iv) The trigger switch of the drill has been depressed.
(a) The point P is connected to the metal case as shown below. It is obvious V
case
= V
p
= 0.
(b) The point M is connected to the metal case as shown below. Simple voltage divider action leads to
9/26/01 P12-18 @ DeCarlo & P. M. Lin
V
case
= V
M
= 0.5115 = 57.5 V
(c) The point Q is connected to the metal case as shown below.
It is obvious that V
case
= V
Q
= 115 V.
CONCLUSION: In all three cases, the fuse will not blow. For cases 2 and 3, the voltage appearing on
the metal case may cause serious injury to the person.
Prbs Ch 13 March 18, 2002 P13-1 R. A. DeCarlo, P. M. Lin
1
PROBLEM SOLUTIONS CHAPTER 13
SOLUTION 13. 1. Given
i (t) +16 i (t) + 4Bi(t) v (t) + 8v(t)
(a) with v(t) v (t) 0 and i
1
(0) + i
2
(0) 0
(i) the characteristic equation is
s
2
+16s + 48 0
(ii) the characteristic equation has factors
(s + 4) (s + 12) = 0
and hence
s
1
, s
2
4,12
(iii) Equivalent circuit at t = 0
(iv) Here by KCL
i
1
(0) i
2
(0) 6A
At point (1)
i
1
(0) + i
2
(0) i
2
(0)
6A + i
2
(0) 6A
i
2
(0) 0
and
v
L1
v
R2
(0) 2i
2
(0) 0V
Then by KVL at t = 0
v
L1
(0) v
6
(0) + v
L2
(0) 0 v
L2
36V
L
2
di
2
dt
36V
i (0) i
2
(0) 36A / s
(v) from part ii
i(t) Ae
4t
+ Be
12t
and then
i (t) 4 Ae
4t
12Be
12t
Then using the initial conditions
A + B 6
4A 12B 36
Prbs Ch 13 March 18, 2002 P13-2 R. A. DeCarlo, P. M. Lin
2
solving yields B = 1.5 and A = 4.5. Then
i(t) 4.5e
4t
+1.5e
12t
A
(b) If v(t) 12V and i(0) i (0) 0 , then v (t) 0 and
i (t) +16 i (t) + 48i(t) 8v(t)
with
i(t) Ae
4t
+ Be
12t
i (t) 4Ae
45
12Be
12t
i (t) 16Ae
4t
+144Be
12t
Then at t = when v() = 12V
i() C, i () 0 and i () 0
Thus
48C 96 C 2
Then
i(t) Ae
4t
+ Be
12t
+ 2
and
i(0) 0 A + B + 2
i (0) 0 4 A12B
Multiply the first of these by 4 yields
4 A + 4B 8
4A 12B 0
Solving yields B = 1 and A = -3. Thus for t > 0
i(t) 3e
4t
+ e
12t
+ 2 A
(c) If v(t) is as in fig. 13.1(b) then
v(t)
2t 0 < t 2
0 t 2

'

The value of C in part (b) will change and at t = 2s, a new set of initial conditions will be required
(obtainable from the solution at t = 2s) and these would be used in the decay portion described by
i (t) +16 i (t) + 48i(t) 0 t 2s
Prbs Ch 13 March 18, 2002 P13-3 R. A. DeCarlo, P. M. Lin
3
SOLUTION 13. 2.
(a) Use the figure with the currents i
1
through i
5
designated in the circuit below.
Then work from v
0
to v
in
using repeated applications of KVL, KCL and the elemental equations:
i
1
2v
o
i
2
2 v
o
i
3
i
1
+ i
2
2 v
o
+ v
o
( )
with
v
2
v
o
v
1
0.5i
3
+ v
2
0.5 2 v
o
+ v
o
( ) + v
o
[ ] v
o
+ 2v
o
then
i
4
2 v
1
2
d
dt
v
o
1
+ 2v
o ( )
2 v
o
+ 2 v
o
( )
i
5
i
3
+ i
4
2 v
o
+ v
o
( ) + 2 v
o
+ 2 v
o
( ) 2 v
o
+ 6 v
o
+ 2v
o
Finally
v
in
0.5i
5
+ v
1
0.5 2 v
o
+ 6 v
o
+ 2v
o
( ) + v
o
+ 2v
o
v
o
+ 4 v
o
+ 3v
o
Hence
v
out
(t) + 4 v
out
(t) + 3v
out
(t) v
in
(t)
(b) Note from part (a) that
v
out
(t) v
2
(t)
and
v
1
(t) v
out
(t) + v
out
(t)
Hence
v
out
(o) v
2
(o) 1V
v
out
(o) v
1
(o) v
out
(o) 7 1 6V
(c) The characteristic equation
s
2
+ 4s + 3 0
has factors
s +1 ( ) s + 3 ( ) 0
and roots
s
1
,v
2
1,3
Prbs Ch 13 March 18, 2002 P13-4 R. A. DeCarlo, P. M. Lin
4
Thus, because of the input, v
in
(t) 6V
v
out
(t) Ae
t
+ Be
3t
+ C
v
out
(t) Ae
t
3Be
3t
v
out
(t) Ae
t
+ 9Be
3t
and at t =
v
out
() + 4v
out
() + 3v
out
() 6V
0 + 0 + 3C = 6V
C = 2V
and
v
out
(t) Ae
t
+ Be
3t
+ 2V
SOLUTION 13. 3. (a) From the given differential equation, the characteristic equation is
s
3
+14s
2
+ 52s + 24 (s + 6)(s
2
+ 8s + 4) 0
Therefore the roots a 6, b 4 + 2 3 0.5359,c 4 2 3 7.4641.
(b) (i) v(0) = v
C3
(0) = 6 V, as given.
(ii) To compute v' (0) we write a nodal equation at node 3. In particular
(v v
C1
) + (v v
C2
) + 0.5v' +v 0 which implies that
v'(t) 2v
C1
(t) + 2v
C2
(t) 6v(t) (*)
Hence v'(0) 2v
C1
(0) + 2v
C2
(0) 6v(0) 24 +18 36 6 V/s.
(iii) To compute v' ' (0) we first differentiate equation (*). This yields
v''(t) 2v'
C1
(t) + 2v'
C2
(t) 6 2v
C1
(t) + 2v
C2
(t) 6v(t) ( ) (**)
To express v'
C1
(t) and v'
C2
(t) in terms of the node voltages we write node equations at nodes 1and
2 respectively. At node 1
(v
C1
(t) v
C2
(t)) + (v
C1
(t) v(t)) + 0.5v
C1
'
(t) i(t)
and at node 2
(v
C2
(t) v
C1
(t)) + (v
C2
(t) v(t)) + 0.5v
C2
'
(t) 0
Hence
v'
C1
(t) 4v
C1
(t) + 2v
C2
(t)) + 2v(t) + 2i(t)
Prbs Ch 13 March 18, 2002 P13-5 R. A. DeCarlo, P. M. Lin
5
and
v
C2
'
(t) 2v
C1
(t)) 4v
C2
(t) + 2v(t)
Substituting these two equations into (**) yields the desired result when t is set to 0. However, this
quantity has no direct physical meaning.
v''(t) 2 4v
C1
(t) + 2v
C2
(t)) + 2v(t) + 2i(t) ( ) + 2 2v
C1
(t)) 4v
C2
(t) + 2v(t) ( )
6 2v
C1
(t) + 2v
C2
(t) 6v(t) ( )
Hence, v' ' (0) = 2(48 + 18 + 12 + 2i(0)) + 2(24 36 + 12) 6(24 + 18 36) = 72 + 4i(0) V/s
2
.
Finally, using the characteristic roots found in part (a) and assuming a constant input, the form
of the solution is
v(t) Ae
6t
+ Be
0.5359t
+ Ce
7.4641t
+ D
Following the methods of example 13.2,
A = 0, B = (7.3301 1.0774i(0)), C = (1.3301 + 0.0774i(0)), and D = i(0).
(c) (i) Not proportional to a single voltage but it is proportional to i
C3
(0).
(ii) Much more complex.
(iii) No.
(iv) No. This is why we use the Laplace transform approach.
SOLUTION 13. 4.
(a) f (q + T
0
)]
qt
f (t + T
0
)
(b) e
5q
cos 0.5q + 0.25 ( )
]
q2t
e
10t
cos t + 0.25 ( )
SOLUTION 13. 5.
(a) Let T = 1.
Prbs Ch 13 March 18, 2002 P13-6 R. A. DeCarlo, P. M. Lin
6
-1 -0.5 0 0.5 1 1.5 2 2.5 3
0
0.5
1
1.5
2
(b) Again let T = 1.
-1 -0.5 0 0.5 1 1.5 2 2.5 3
-2
-1.5
-1
-0.5
0
(c)
-1 -0.5 0 0.5 1 1.5 2 2.5 3
0
0.05
0.1
0.15
0.2
0.25
(d)
Prbs Ch 13 March 18, 2002 P13-7 R. A. DeCarlo, P. M. Lin
7
-1 -0.5 0 0.5 1 1.5 2 2.5 3
0
0.2
0.4
0.6
0.8
1
(e)
-1 -0.5 0 0.5 1 1.5 2 2.5 3
0
1
2
3
4
5
6
7
(f) i (t i)
i 0
r

(g)
-1 -0.5 0 0.5 1 1.5 2 2.5 3
0
1
2
3
4
5
(h) Pulses of height 1 and width T.
Prbs Ch 13 March 18, 2002 P13-8 R. A. DeCarlo, P. M. Lin
8
SOLUTION 13. 6.
(a) L[ f
1
(t)] F
1
(s) f
1
(t)e
st
dt
0

e
st
dt
T
1
T
2

e
st
s
1
]
1
1
T
1
T
2

1
s
e
sT
1
e
sT
2
( )
(b) f
2
(t) f
1
(t) . Hence, the answer is the same as in (a).
(c) L[ f
3
(t)] F
3
(s) f
3
(t)e
st
dt
0

2 (t)cos(4t 0.25)e
st
dt
0

2
2
2
2
(d) F
4
(s) f
4
(t)e
st
dt
0

2 (t T)cos(4t 0.25)e
st
dt
0

2cos(4T 0.25)e
sT
(e) F
5
(s) f
5
(t)e
st
dt
0

(t) (t T) [ ]e
st
dt
0

1 e
sT
SOLUTION 13. 7.
(a) F(s) f (t)e
st
dt
0

5e
4t
e
st
dt
0

5
e
(s+4)t
s + 4
1
]
1
1
0

5
s + 4
0 0.2 0.4 0.6 0.8 1 1.2 1.4 1.6 1.8 2
0
0.5
1
1.5
2
2.5
3
3.5
4
4.5
5
t
f
(
t
)
TextEnd
Prbs Ch 13 March 18, 2002 P13-9 R. A. DeCarlo, P. M. Lin
9
(b) F(s) f (t)e
st
dt
0

5e
4t
e
st
dt
1

5
e
(s+4) t
s + 4
1
]
1
1
1

5e
(s+4)
s + 4
e
s
5e
4
s + 4
0 0.2 0.4 0.6 0.8 1 1.2 1.4 1.6 1.8 2
0
0.01
0.02
0.03
0.04
0.05
0.06
0.07
0.08
0.09
0.1
t
f
(
t
)
TextEnd
(c) F(s) f (t)e
st
dt
0

5e
4(t 1)
e
st
dt
1

5e
4
e
(s+4)t
s + 4
1
]
1
1
1

e
s
5
s + 4
0 0.2 0.4 0.6 0.8 1 1.2 1.4 1.6 1.8 2
0
0.5
1
1.5
2
2.5
3
3.5
4
4.5
5
t
f
(
t
)
TextEnd
(d) F(s) f (t)e
st
dt
0

5e
4( t 1)
(t)e
st
dt
0

5e
4
(e) F(s) f (t)e
st
dt
0

5e
4( t 1)
(t 1)e
st
dt
0

5e
s
Prbs Ch 13 March 18, 2002 P13-10 R. A. DeCarlo, P. M. Lin
10
(f)
F(s) f (t)e
st
dt
0

2 u(t)u(1 t) [ ]e
0.25t
e
st
dt
0

2 e
0.25t
e
st
dt
0

2
s + 0.25
1 e
(s+0.25)
( )
2 e
0.25t
e
st
dt
0

2 e
0.25t
e
st
dt
1

2
s + 0.25
1 e
s
( )
0 0.2 0.4 0.6 0.8 1 1.2 1.4 1.6 1.8 2
0
0.2
0.4
0.6
0.8
1
1.2
1.4
1.6
1.8
2
t
f
(
t
)
TextEnd
F(s) f (t)e
st
dt
0

SOLUTION 13. 8.
(a) F(s) f (t)e
st
dt
0

Ae
t
e
st
dt
0

Ae
(s )t
dt
0

A
s
(b) F(s) f (t)e
st
dt
0

Ae
(t 1)
u(t 1)e
st
dt
0

Ae

e
(s )t
dt
1

e
s
A
s
(c) F(s) f (t)e
st
dt
0

Ae
(t 1)
(t)e
st
dt
0

Ae

(d) F(s) f (t)e


st
dt
0

Ae
t
e
st
dt
0

A
e
(s )t
(s )
1
]
1
1
0

A
s
1 e
(s )
( )
Prbs Ch 13 March 18, 2002 P13-11 R. A. DeCarlo, P. M. Lin
11
SOLUTION 13. 9. Consider the following in which q = at and t = q/a.
L[ f (at)] f (at)e
st
dt
0

1
a
f (q)e
(s/a)q
dq
0

1
a
F
s
a


_
,
SOLUTION 13. 10.
(a)
e
j t
cos t + j sin t
e
j t
cos t j sin t
Add these equations and divide by 2 to obtain
cos( t)
e
j t
+ e
j t
2
Similarly, subtract the equations and divide by 2j to obtain
sin( t)
e
j t
e
j t
2 j
Note that
L e
j t
[ ]

1
s j
, L e
j t
[ ]

1
s + j
(b)
L cos t [ ] L
e
j t
+ e
j t
2




1
]
1
1

0.5
s j
+
0.5
s + j

s
s
2
+
2
and
L sin t [ ]
0.5 j
s j
+
0.5 j
s + j

s
2
+
2
SOLUTION 13. 11. From the time frequency scaling property,
Prbs Ch 13 March 18, 2002 P13-12 R. A. DeCarlo, P. M. Lin
12

L sin( t) u(t) [ ]
1 1
s

_
,
2
+1

s
2
+
2
Using the time differentiation property,
L[ cos( t)u(t)] L
d
dt
sin( t)u(t) ( )



1
]
1
sL[sin( t) u(t)] sin(0)
s
s
2
+
2
Finally using the multiplication by t property
L[ t cos( t)u(t)]
d
ds
s
s
2
+
2



_
,

s
2
+
2

2 s
2
s
2
+
2
( )
2
Hence
L[sin( t)u(t) t cos( t)u(t)]
2
s
2
+
2

2 s
s
2
+
2
( )
2

2 s
2
+
2
( )
2 s
2
s
2
+
2
( )
2

2
3
s
2
+
2
( )
2
SOLUTION 13. 12. (a)
g
1
(t) At sin( t), f (t) sin(t)u(t), F(s)
1
s
2
+1
By the frequency scaling property
L f ( t) [ ]
1 1
s

_
,
2
+1







1
]
1
1
1
1
1

1
2
s
2
+
2




1
]
1
1

s
2
+
2
By the multiplication by t property
L t sin( t)u(t) [ ]
d
ds
s
2
+
2



1
]
1

2 s
s
2
+
2
( )
2





1
]
1
1
1

2 s
s
2
+
2
( )
2
By the linearity property
L At sin( t) [ ] A
2 s
s
2
+
2
( )
2
Prbs Ch 13 March 18, 2002 P13-13 R. A. DeCarlo, P. M. Lin
13
(b)
g
2
(t) Ae
at
sin( t)u(t), f (t) sin(t)u(t), F(s)
1
s
2
+1
By the frequency scaling property,
L sin( t) [ ]
s
2
+
2
By the damping property
L e
at
sin( t)
[ ]

s a ( )
2
+
2
and by the linearity property
L g
2
(t) [ ] A
s a ( )
2
+
2
(c) Before beginning, note that
sin t + ( ) cos( )sin( t) + sin( )cos( t)
Here
g
3
(t) Ae
at
sin t + ( )u(t) cos( )Ae
at
sin( t)u(t) + sin( )Ae
at
cos( t)u(t)
f (t) sin(t)u(t), F(s)
1
s
2
+1
From the linearity property and part (b),
G
3
(s) Acos( )
s a ( )
2
+
2
+ L{sin( )Ae
at
cos( t)u(t)}
By the differentiation in the time domain property cos(t)
d
dt
sin(t) which implies
L cos(t) [ ] s L sin(t) [ ] { } f (0)
s
s
2
+1
0
s
s
2
+1
By the frequency scaling property
L cos t [ ]
1
s
s
( )
2
+1





1
]
1
1
1

1
2
s
2
s
2
+
2




1
]
1
1

s
s
2
+
2
Then by the linearity property
Prbs Ch 13 March 18, 2002 P13-14 R. A. DeCarlo, P. M. Lin
14
L Asin( )cos( t) [ ] Asin( )
s
s
2
+
2
Then by the frequency-shift property
L Asin( )e
at
cos( t)
[ ]
Asin( )
s a
(s a)
2
+
2
G
3
(s) L Ae
at
sin t + ( )
[ ]
Acos( )
s a ( )
2
+
2
+ Asin( )
s a
(s a)
2
+
2

A
s a ( )
2
+
2
cos( ) + sin( )(s a) ( )
SOLUTION 13. 13. (a) We are given that
f t ( ) sin t ( ) F(s)
1
s
2
+1
And must find the transform of
g
1
(t) At cos( )tu(t)
By the time differentation property
cos t ( )
d
dt
sin t ( ) L cos t ( ) [ ]
s
s
2
+1
sin 0

( )

s
s
2
+1
By the frequency scaling property with
L cos t ( ) [ ]
1
F
s

_
,

1
s
s
( )
2
+1





1
]
1
1
1

s
2
2
s
2
+
2
( )

s
s
2
+
2
Using the multiplication-by-t property
L t cos t ( ) [ ]
d
ds
s
s
2
+
2



1
]
1

s
2
+
2
s(2s)
s
2
+
2
( )
2

s
2

2
s
2
+
2
( )
2
Finally, by the linearity property
L At cos t ( )u(t) [ ]
A
2
s
2
( )
s
2
+
2
( )
2
Prbs Ch 13 March 18, 2002 P13-15 R. A. DeCarlo, P. M. Lin
15
(b) Let g
2
(t) Ae
at
cos t ( )u(t) . Recall from part (a) that
L Acos t ( ) [ ] A
s
s
2
+
2
By the Frequency-shift property
L Ae
at
cos( t)
[ ]
A
s a
s a ( )
2
+
2
(c) From Trig identities,
cos t + ( ) cos( t)cos( ) sin( t)sin( )
Recall from part (b) that
L Ae
at
cos( t)
[ ]
A
s a
s a ( )
2
+
2
Hence
L Ae
at
cos( )cos( t)
[ ]
Acos( )
s a
s a ( )
2
+
2
It follows that
L Ae
at
cos t + ( )
[ ]
Acos( )
s a
s a ( )
2
+
2
Asin( )L e
at
sin t ( )
[ ]
Acos( )
s a
s a ( )
2
+
2
Asin( )
(s a)
2
+
2
A
cos( )(s a) sin( )
s a ( )
2
+
2




1
]
1
1
SOLUTION 13. 14. (a) We are given that
L cos t ( ) [ ]
s
s
2
+1
which implies by the frequency scaling property that
L cos t ( ) [ ]
s
s
2
+
2
Using the multiplication-by-t property
L t cos t ( ) [ ]
d
ds
s
s
2
+
2



1
]
1

s
2
+
2
2s
2
s
2
+
2
( )
2

s
2

2
s
2
+
2
( )
2
Prbs Ch 13 March 18, 2002 P13-16 R. A. DeCarlo, P. M. Lin
16
(b) If g(t) te
at
cos( t), then using part (a) and frequency shift property,
L te
at
cos( t)
[ ]

s a ( )
2

2
s a ( )
2
+
2
[ ]
2
SOLUTION 13. 15. (a) With
sinh(at)
e
at
e
at
2
L sinh at ( ) [ ]
1
2
1
s a

1
s + a



1
]
1

1
2
s + a s a ( )
s
2
a
2



1
]
1

1
2
2a
s
2
a
2



1
]
1

a
s
2
a
2
(b) With
cosh(at)
e
at
+ e
at
2
L cosh(at) [ ]
1
2
1
s + a
+
1
s a



1
]
1

1
2
s a + s + a
s
2
a
2



1
]
1

1
2
2s
s
2
a
2



1
]
1

s
s
2
a
2
SOLUTION 13. 16. (a) From Problem 15
L sinh at ( ) [ ]
a
s
2
a
2
So that by the multiplication by t property
L t sinh at ( ) [ ]
d
ds
a
s
2
a
2



1
]
1

a(2s)
s
2
a
2
( )
2

2as
s
2
a
2
( )
2
(b) From Problem 15
L cosh(at) [ ]
s
s
2
a
2
So that by the multiplication-by-t property
Prbs Ch 13 March 18, 2002 P13-17 R. A. DeCarlo, P. M. Lin
17
L t cosh(at) [ ]
d
ds
s
s
2
a
2



1
]
1

s
2
a
2
( )
s(2s)
s
2
a
2
( )
2

s
2
+ a
2
+ 2s
2
s
2
a
2
( )
2

s
2
+ a
2
s
2
a
2
( )
2
SOLUTION 13. 17. Here
F(s)
s + 2
s +1
(a) Since g
1
(t) 5 f (t 2), use the time shift and linearity properties to obtain
L g
1
(t) [ ] 5e
2s
s + 2
s +1



1
]
1
(b) Since g
2
(t) 5e
2t
f (t), use the frequency shift and linearity properties,
L g
2
(t) [ ] 5
s + 2 ( ) + 2
s + 2 ( ) +1



1
]
1
5
s + 4
s + 3



1
]
1
(c) From part (a),
L 5 f (t 2) [ ] 5e
2s
s + 2
s +1



1
]
1
Therefore, since g
3
(t) 5e
2t
f (t 2) , by the frequency shift property
L g
3
(t) [ ] G
1
(s + 2) 5e
2(s+2)
(s + 2) + 2
(s + 2) +1



1
]
1
5e
2(s+2)
s + 4
s + 3



1
]
1
(d) Since g
4
(t) 5tf (t) , use the multiplication-by-t and linearity principle to obtain
L g
4
(t) [ ]
d
ds
5 s + 2 ( )
s +1



1
]
1

5 (s +1) (s + 2) [ ]
(s +1)
2

5
(s +1)
2
SOLUTION 13. 18. In all parts
L f (t)u(t) [ ] F(s)
s
s
2
+ 4
Prbs Ch 13 March 18, 2002 P13-18 R. A. DeCarlo, P. M. Lin
18
(a) L Af (t T)u(t T) [ ] A
e
Ts
s
s
2
+ 4
(b) L Atf (t)u(t) [ ]
d
ds
Af (s) A
d
ds
s
s
2
+ 4



1
]
1
A
s
2
+ 4
( )
s(2s)
s
2
+ 4
( )
2





1
]
1
1
1
A
s
2
4
( )
s
2
+ 4
( )
2
(c) Note that the answer is simply a time shift of the function given in (b).
L A(t T) f (t T)u(t T) [ ] Ae
sT
s
2
4
( )
s
2
+ 4
( )
2
(d) This function is that of part (a) multiplied by t. Hence, by the multiplication by t property,
L Atf (t T)u(t T) [ ] A
d
ds
e
Ts
s
s
2
+ 4




1
]
1
1
A
e
Ts
Tse
Ts
( )
(s
2
+ 4) 2s se
Ts
s
2
+ 4
( )
2





1
]
1
1
1
Ae
Ts
2s
2
1 Ts ( )(s
2
+ 4)
s
2
+ 4
( )
2





1
]
1
1
1
Ae
Ts
Ts
3
+ s
2
+ 4Ts 4
s
2
+ 4
( )
2





1
]
1
1
1
SOLUTION 13. 19. In all parts
F(s) L f (t)u(t) [ ]
s
2
+
2
(a) By the time shift property, L Af (t T)u(t T) [ ] A
e
Ts
s
2
+
2
( )
Using the multiplication-by-t property,
L Atf (t)u(t) [ ] A
d
ds
F(s) [ ] A
d
ds
s
2
+
2



1
]
1
A
(2s)
s
2
+
2
( )
2
A
2 s
s
2
+
2
( )
2
Prbs Ch 13 March 18, 2002 P13-19 R. A. DeCarlo, P. M. Lin
19
The answer here is an application of the time shift property to the answer of part (b).
L A(t T) f (t T)u(t T) [ ] Ae
sT
2 s
s
2
+
2
( )
2
(d) The answer here uses the multiplication-by-t property applied to the answer of part (a).
L Atf (t T)u(t T) [ ] A
d
ds
e
Ts
s
2
+
2
( )





1
]
1
1
1
A
Te
Ts
(s
2
+
2
) + 2se
Ts
s
2
+
2
( )
2





1
]
1
1
1
A e
Ts
Ts
2
+ 2s + T
2
s
2
+
2
( )
2





1
]
1
1
1
Solution 13. 20. (a)
f (2t) (2t) + (2t 1)
For the first term on the right, the peak occurs at t = 0 and
(2t)dt
0

0
+

0.5 ( )d 0.5
0

0
+

under the transformation = 2t. For the second term, the function peaks at t = 0.5 and
(2t 1)dt
0.5

0.5
+

0.5 ( )d 0.5
0

0
+

under the transformation = 2t 1. Hence


f (2t) (2t) + (2t 1) 0.5 (t) + (t 0.5) [ ]
Therefore, a = 0.5 = b.
Prbs Ch 13 March 18, 2002 P13-20 R. A. DeCarlo, P. M. Lin
20
(b) (i) Here F(s) L f (t) [ ] 1 + e
s
. By the time scaling property
L f (2t) [ ]
1
2
F(s /2) 0.5 1+ e
0.5s
( )
(b)-(ii) For this part,
L f (2t) [ ] 0.5L (t) + (t 0.5) [ ] 0.5 1+ e
0.5s
( )
SOLUTION 13. 21. (a)
L v(t) [ ] 2L g"(t) [ ] L g'(t) [ ] 2s
2
F(s) 2sg(0

) 2g'(0

) sF(s) + g(0

)
2s
2
s
( )
F(s) 2sg(0

) 2g'(0

) + g(0

)
(2s1)(s +1)
s
2
(b)
L v(t) [ ] 2L f "(t) [ ] L f '(t) [ ] 2s
2
F(s) sF(s) 2sf (0

) 2 f '(0

) + f (0

)
2s
2
s
( )
F(s) 2s 2 +1
2s1 ( )(s +1)
s
2
2s 2 +1
(c)
L v(t) [ ] L g'( t) [ ] L g(q)dq





1
]
1
1
sG(s) g(0

)
G(s)
s

1
s
g(q)dq

s
1
s


_
,
G(s)
s
1
s


_
,
s +1
s
3
s +1 ( )
1
s
2

1
s
4



1
]
1
(d)
L v(t) [ ] L f '( t) [ ] L f (q)dq





1
]
1
1
sF(s) f (0

)
F(s)
s

1
s
f (q)dq

s +1 ( )
1
s
2

1
s
4



1
]
1
1
1
s
assuming > 0. The expression is ill-defined if 0.
Prbs Ch 13 March 18, 2002 P13-21 R. A. DeCarlo, P. M. Lin
21
SOLUTION 13. 22.
(a) (i) If

F(s) L f (t)u(t) [ ] ln
s
2
+ 4
s
2



_
,

ln s
2
+ 4
( )
ln s
2
( )
Then by the multiplication-by-t property

L tf (t)u(t) [ ] +
d
ds
ln s
2
+ 4
( )
ln s
2
( ) [ ]

2s
s
2
+ 4

2s
s
2

2s
s
2
+ 4

2
s

2s
2
2s
2
8
s s
2
+ 4
( )

8
s s
2
+ 4
( )
(ii) Using the solution to (a)-(i), by the frequency shift property
L te
2t
f (t)u(t)
[ ]

8
s + 2 ( ) s + 2 ( )
2
+ 4
( )

8
s + 2 ( ) s
2
+ 4s + 8
( )

8
s
3
+ 6s +16s +16
(b) If
G(s)
8
s s
2
+ 4
( )
a partial fraction expansion may be employed
G(s)
8
s s
2
+ 4
( )

K
1
s
+
As + B
s
2
+ 4

2
s
+
2
s
2
+ 4
Hence,
g(t) 2cos(2t) 2 [ ]u(t)
and
f (t)
g(t)
t

2
t

2
t
cos(2t)
SOLUTION 13. 23: Part (a)-(i): From table 13.2, the multiplication by t property implies that
Prbs Ch 13 March 18, 2002 P13-22 R. A. DeCarlo, P. M. Lin
22

L[tf (t)u(t)]
d
ds
F(s)
d
ds
ln
s + a
s a



1
]
1

d
ds
ln s + a [ ] ln s a [ ] ( )
1
s + a

1
s a

2a
s
2
a
2
Part (a)-(ii): Let us make use of the answer to part (a)-1. Let G(s) L[tf (t)u(t)]
2a
s
2
a
2
.
Then by the frequency shift property in table 13.2,
L[te
at
f (t)u(t)] L[e
at
tf (t)u(t) ( )] G(s + a)
2a
(s + a)
2
a
2

2a
s(s + 2a)
Part (b): g(t) L
1
[G(s)] L
1
2a
s
2
a
2



1
]
1
L
1
1
s + a



1
]
1
L
1
1
s a



1
]
1
e
at
e
at
( )
u(t)
More specifically,
g(t) e
at
e
at
( )
u(t) 2
e
at
e
at
2



_
,

u(t) 2sinh(at) u(t)
Hence f (t)
g(t)
t

2sinh(at)
t
.
SOLUTION 13. 24.
(a)-(i) If

F(s) L f (t)u(t) [ ] ln
s + a
s + b


_
,
ln(s + a) ln(s + b)
Then by the multiplication-by-t property

L tf (t)u(t) [ ] +
d
ds
ln s + a ( ) ln s + b ( ) [ ]
1
s + a

1
s + b

s + b (s + a)
(s + a)(s + b)

b a
(s + a)(s + b)
(ii) By the frequency-shift property
L te
at
f (t)u(t)
[ ]

b a
(s + 2a)(s + a + b)

b a
s
2
+ s(3a + b) + 2a(a + b)
(b) If
Prbs Ch 13 March 18, 2002 P13-23 R. A. DeCarlo, P. M. Lin
23
G(s) L tf (t)u(t) [ ]
b a
(s + a)(s + b)
a partial fraction expansion may be employed
G(s)
b a
(s + a)(s + b)

K
1
s + a
+
K
2
s + b

1
s + a
+
1
s + b
Hence, g(t) e
at
e
bt
( )
u(t)
and
f (t)
g(t)
t

e
bt
t

e
at
t
SOLUTION 13. 25. The relationship is f (t)
d
dt
g(t) or equivalently, g(t) f (q)dq

.
Now we have that f (t) 6 (t) 12 (t 2) + 6 (t 4) . Therefore,
F(s) 6 12e
2s
+ 6e
4s
From the time integration property,
G(s)
F(s)
s

6
s

12e
2s
s
+
6e
4s
s
SOLUTION 13. 26. For 0 t < T
1
, we see that g(t) f (q)dq

. Thus one presupposes here that


the relationship is f (t)
d
dt
g(t) or equivalently, g(t) f (q)dq

. As such E = A B and D = A
B + C.
Further, f (t) A (t) B (t T
1
) + C (t T
2
) which implies that
F(s) A Be
T
1
s
+ Ce
T
2
s
Thus
Prbs Ch 13 March 18, 2002 P13-24 R. A. DeCarlo, P. M. Lin
24
G(s)
F(s)
s

A
s

Be
T
1
s
s
+
Ce
T
2
s
s
SOLUTION 13. 27.
(a) f (t) u(t) + u(t 1) f (s)
1
s
+
e
s
s

1
s
1 e
s
( )
(b)
f (t) u(t) + u(t 1) u(t 3) f (s)
1
s
+
e
s
s

e
3s
s

1
s
1+ e
s
e
3s
( )
(c)
f (t) u(t) + u(t 1) 2u(t 3) F(s)
1
s
+
e
s
s

2e
3s
s

1
s
1 + e
s
2e
3s
( )
(d)
f (t) 2u(t) u(t 2) u(t 3) F(s)
2
s

e
2s
s

e
3s
s

1
s
2 e
2s
e
3s
( )
SOLUTION 13. 28.
(a) f (t) 2r(t) 2r(t 1) F(s)
2
s
2

2e
s
s
2

2
s
2
1 e
s
( )
(b) f (t) 2r(t) 2r(t 1)+ r(t 2) F(t)
2
s
2

2e
s
s
2
+
e
2s
s
2

1
s
2
2 2e
s
+ e
2s
( )
(c) f (t) 2r(t) 2r(t 1) 2r(t 2) + 2r(t 3). It follows that
F(s)
2
s
2

2e
s
s
2

2e
2s
s
2
+
2e
3s
s
2

2
s
2
1 e
s
e
2s
+ e
3s
( )
Prbs Ch 13 March 18, 2002 P13-25 R. A. DeCarlo, P. M. Lin
25
SOLUTION 13. 29. (a) Here f (t)
3
2
r(t) 3r(t 2) +
3
2
r(t 4) . Thus,
F(s)
3
2s
2

3e
2s
s
2
+
3e
4s
2s
2

3
2s
2
1 2e
2s
+ e
4s
( )
(b) Here f (t)
V
o
T
r(t)
2V
o
T
r(t T) +
V
o
T
r(t 2T). Thus
F(s)
V
o
T
1
s
2

2e
T
s
2
+
e
2T
s
2



_
,

(c) Here f (t) 2r(t 1) 4r(t 2) + 4r(t 4) 2r(t 5)
F(s)
1
s
2
2e
s
4e
2s
+ 4e
4s
2e
5s
( )
SOLUTION 13. 30.
(a) Here f (t) 2r(t) 2r(t 1) 2u(t 4) implies
F(s)
2
s
2

2e
s
s
2

2e
4s
s

2
s
2
1 e
s
se
4s
( )
(b) Here f (t) 2u(t) r(t 2) + r(t 4) implies
F(s)
2
s

e
2s
s
2
+
e
4s
s
2

2
s
2
s e
2s
e
4s
( )
SOLUTION 13. 31. (a) Here f (t) 2u(t) r(t) + 2r(t 2) 2r(t 4) 2u(t 4)
Thus F(s)
2
s

1
s
2
+
2e
2s
s
2

2e
4s
s
2

2e
4s
s
.
(b) f (t) u(t) + r(t) r(t 2) u(t 2). Hence F(s)
1
s
+
1
s
2

e
2s
s
2

e
2s
s
.
Prbs Ch 13 March 18, 2002 P13-26 R. A. DeCarlo, P. M. Lin
26
(c) f (t) 2r(t) 2r(t 1) 2u(t 1) + 2u(t 2) 2r(t 2) + 2r(t 3).
Hence F(s)
2
s
2

2e
s
s
2

2e
s
s
+
2e
2s
s

2e
2s
s
2
+
2e
3s
s
2
.
2/23/02 page P14.1 R. A. DeCarlo, P. M. Lin
PROBLEM SOLUTIONS CHAPTER 14
SOLUTION 14. 1.
(a)
Z(s)
R(Ls +
1
Cs
)
R + Ls +
1
Cs

Cs(RLCs
2
+ R)
Cs(RCs + LCs
2
+1)

RLC(s
2
+1 LC)
LC(s
2
+
R
L
s +
1
LC
)

R(s
2
+1 LC)
s
2
+
R
L
s +
1
LC
(b)
Z(s) R +
(Ls)(
1
Cs
)
Ls +
1
Cs
R +
LCs
C(LCs
2
+1)
R +
Ls
LCs
2
+1

RLCs
2
+ Ls + R
LCs
2
+1

RLC(s
2
+
1
RC
s +
1
LC
)
LC(s
2
+
1
LC
)
Hence,
Z(s)
R s
2
+
1
RC
s +
1
LC


_
,
s
2
+
1
LC
SOLUTION 14.2.
(a)
Z
in
(s)
V
s
(s)
I
s
(s)

(10 + 0.2s)(
80
s
)
10 + 0.2s +
80
s

s(800 +16s)
s(0.2s
2
+10s + 80)

800s + 4000
s
2
+ 50s + 400
(b) If i
s
(t) 3e
20t
u(t) A then
I
s
(s)
3
s + 20
and
V
s
(s) Z
in
(s)I
s
(s)
800s + 4000
s
2
+ 50s + 400

3
s + 20

2400s +12,000
(s +10)(s + 20)(s + 40)

K
1
s +10
+
K
2
s + 20
+
K
3
s + 40
Here
K
1

2400s +12,000
(s + 20)(s + 40)
s10

+12,000 24,000
(10)(30)
40
K
2

2400s +12,000
(s +10)(s + 40)
s20

12,000 48,000
(10)(20)
180
2/23/02 page P14.2 R. A. DeCarlo, P. M. Lin
K
3

2400s +12,000
(s +10)(s + 20)
s40

12,000 96,000
(30)(20)
140
V
s
(s)
180
s + 20

40
s +10

140
s + 40
and for t > 0,
v
s
(t) 180e
20t
40e
10t
140e
40t
V
SOLUTION 14.3.
(a)
Y
p
(s) Cs +
1
R
2 10
3
s +
1
0.10
2 10
3
(s + 50)
Then
Z
p
(s)
500
s + 50
and
Z
in
(s) 1.25s +
500
s + 50

1.25s
2
+ 62.5s + 500
s + 50
and
Y
in
(s)
I
s
(s)
V
s
(s)

1
Z
in
(s)

s + 50
1.25s
2
+ 62.5s + 500

0.80s + 40
s
2
+ 50s + 400

With v
s
(t) 90e
40t
u(t) , then
V
s
(s)
90
s + 40
and
I
s
(s) V
s
(s)Y
in
(s)
90
s + 40

0.80s + 40
(s +10)(s + 40)

72s + 3600
(s +10)(s + 40)
2

K
1
s +10
+
C
1
s + 40
+
C
2
(s + 40)
2
Here
K
1

72s + 3600
(s + 40)
2
s10

3600 720
(30)
2

2880
900
3.20
and with
p(s)
72s + 3600
s +10
p(40)
720
30
24
p (s)
(s +10)(72) (72s + 3600)
(s +10)
2
p (40)
30(72) (2880 + 3600)
(30)
2

2160 720
900
3.20
Then
C
2

p(40)
0!
24 , C
1

p (40)
1!
3.20 ,
2/23/02 page P14.3 R. A. DeCarlo, P. M. Lin
I
s
(s)
3.20
s +10

3.20
s + 40
+
24
(s + 40)
2
and for t > 0
i
s
(t) 3.20e
10t
3.20e
40t
+ 24te
40t
A
SOLUTION 14.4.
(a) Find Z
in
(s) vis Y
in
(s)
Y
in
(s) Cs +
1
Ls + 20
+
1
10

200Cs +10LCs
2
+10 + 20 + Ls
10(Ls + 20)

10LCs
2
+ (200C + L)s + 30
10Ls + 200
and
Z
in
(s)
1
Y
in
(s)

10Ls + 200
10LCs
2
+ (200C + L)s + 30

With C 10
3
F and L = 0.05 H
Z
in
(s)
0.50s + 200
0.0005s
2
+ 0.25s + 30

1000s + 4 10
5
s
2
+ 500s + 60,000

(b) If i
s
(t) 0.3u(t) A, then
I
s
(s)
0.30
s
and
V
in
(s) Z
in
(s)I
s
(s) 300
s + 400
s(s
2
+ 500s + 60,000)




1
]
1
1
300
s + 400
s(s + 2000)(s + 300)



1
]
1
300
K
1
s
+
K
2
200
+
K
3
300



1
]
1
It follows that
K
1

s + 400
(s + 200)(s + 300)
s0

400
200(300)

1
150
K
2

s + 400
s(s + 300)
s200

200
(200)(100)

1
100
and
K
3

s + 400
s(s + 200)
s300

100
(300)(100)

1
300
Thus
2/23/02 page P14.4 R. A. DeCarlo, P. M. Lin
V
in
(s) 300
1
150
s

1
100
s + 200
+
1
300
s + 300





1
]
1
1
1

2
s

3
s + 200
+
1
s + 300
and for t > 0
v
in
(t) 2 3e
200t
+ e
300t
V
SOLUTION 14.5.
Z(s)
s + 20
s + 40
and the network is at rest
(a) If
v
in
(t) 20u(t) V
in
(s)
20
s
then
I
in
(s)
V
in
(s)
Z(s)

20
s
s + 40
s + 20


_
,
20
s + 40
s(s + 20)



1
]
1
Using a partial function expansion
I
in
(s) 20
s + 40
s(s + 20)



1
]
1
20
K
1
s
+
K
2
s + 20



1
]
1
in which case
K
1

s + 40
s + 20
s0

40
20
2, K
2

s + 40
s
s20

20
20
1
Thus
I
in
(s) 20
2
s

1
s + 20



1
]
1
and
i
in
(t) 20(2 e
20t
)u(t) A
(b) Note that
v
in
(t) 20e
40t
V
in
(s)
20
s + 40
Then
I
in
(s)
V
in
(s)
Z(s)

20
s + 40
s + 40
s + 20


_
,

20
s + 20
in which case
i
in
(t) 20e
20t
u(t) A
(c) Note that
v
in
(t) 20e
20t
V
in
(s)
20
s + 20
Then
2/23/02 page P14.5 R. A. DeCarlo, P. M. Lin
I
in
(s)
V
in
(s)
Z(s)

20
s + 20
s + 40
s + 20


_
,
20
s + 40
(s + 20)
2




1
]
1
1
Using a partial fraction expansion
I
in
(s) 20
s + 40
(s + 20)
2




1
]
1
1
20
C
1
s + 20
+
C
2
(s + 20)
2




1
]
1
1
Here
p(s) s + 40 p(20) 20
p'( s) 1 p'(20) 1
and
C
1

p'(20)
1!
1, C
2

p(20)
0!

20
1
20
in which case
I
in
(s) 20
1
s + 20
+
20
(s + 20)
2




1
]
1
1
so that
i
in
(t) 20e
20t
+ 400te
20t
( )
u(t) A
SOLUTION 14.6.
(a) Apply an arbitrary I
in
(s) to the upper terminal of Fig. P14.6a. Assuming branch currents I
a
(s) and
I
b
(s), it follows by KCL that
I
in
(s) I
a
(s) + I
b
(s) 0.020sV
a
(s) + 0.005s V
a(s)
sV
a
(s)
[ ]
(0.020 + 0.005 0.015)sV
a
(s) 0.010sV
a
(s)
Hence,
Z
in
(s)
V
a
(s)
I
in
(s)

1
0.010s

100
s

(b) Similarly apply an arbitrary I


in
(s) to Fig P14.6b to obtain, in the s-domain, by KCL
I
in
(s) 10sV
in
(s) +
1
50
V
in
(s) + 0.10s V
in
(s)
V
in
(s)
4



1
]
1
10s +
1
50
+
30
4s


_
,
V
in
(s)

2000s
2
+ 4s +1500
200s



_
,

V
in
(s)
Hence
Y
in
(s)
I
in
(s)
V
in
(s)

10s
2
+ 0.02s + 7.50
s
(c) Here we apply V
in
(s) to the input terminals of figure P14.6c. By KCL
V
in
(s) 10I
in
(s) + 0.2sI
in
(s) +
400
s
I
in
(s)
1
2
I
in
(s)



1
]
1
10 + 0.2s +
200
s


_
,
I
in
in which case
2/23/02 page P14.6 R. A. DeCarlo, P. M. Lin
Z
in
(s)
V
in
(s)
I
in
(s)

0.2s
2
+10s + 200
s

SOLUTION 14. 7. Writing two loop equations we obtain:


V
in
(s) 100I
1
(s) + 200I
2
(s)
and
100I
1
(s) + 100 +
100
s


_
,
I
2
(s) 0
In matrix form (dropping the s-dependence)
100 200
100 100 +
100
s




1
]
1
1
I
1
I
2



1
]
1

V
in
0



1
]
1
Using Cramer's rule,
I
1

det
V
in
200
0 100 +
100
s




1
]
1
1
det
100 200
100 100 +
100
s




1
]
1
1

100s +100
s

1
100
100s +100
s


_
,
200



1
]
1
V
in
Hence
Z
in
(s)
V
in
I
1
100
s 1
s +1
SOLUTION 14.8.
Working in the s-domain, apply KVL to the left side of the circuit to obtain
V
in
(s)
100
s
I
in
(s) +
s
100
I
in
(s) + V
2
(s)
Now apply KCL to the right side to obtain
I
in
(s)
s
100
V
2
(s) +
100
s
V
2
(s)
Thus
V
in
(s)
s
2
+10
4
100s



_
,

I
in
(s) +V
2
(s)
To find V
2
(s) note that
I
in
(s)
s
2
+10
4
100s
V
2
(s)
implying that
2/23/02 page P14.7 R. A. DeCarlo, P. M. Lin
V
2
(s)
100s
s
2
+10
4



_
,
I
in
(s)
Thus
V
in
(s)
s
2
+10
4
100s
+
100s
s
2
+10
4




1
]
1
1
I
in
(s)
(s
2
+10
4
)
2
+10
4
s
2
100s(s
2
+10
4
)
I
in
(s)
implying that
Z
in
(s)
V
in
(s)
I
in
(s)

s
4
+ 3 10
4
s
2
+10
8
100s(s
2
+10
4
)

and
Y
in
(s)
1
Z
in
(s)

100s(s
2
+10
4
)
s
4
+ 3 10
4
s
2
+10
8
S
SOLUTION 14.9.
Three mesh equations for the circuit
R + Z
2
(s) Z
2
(s) R
Z
2
(s) 2R + Z
2
(s) R
R R 2R + Z
1
(s)





1
]
1
1
1
I
1
(s)
I
2
(s)
I
3
(s)





1
]
1
1
1

V
in
(s)
0
0





1
]
1
1
1
Solve for I
1
(s) via Cramers rule
I
1

det
V
in
Z
2
(s) R
0 2R + Z
2
(s) R
0 R 2R + Z
1
(s)





1
]
1
1
1
det
R + Z
2
(s) Z
2
(s) R
Z
2
(s) 2R + Z
2
(s) R
R R 2R + Z
1
(s)





1
]
1
1
1

V
in
3R
2
+ 2R Z
1
+ Z
2
( ) + Z
1
Z
2 ( )
d(s)
where
d(s) R + Z
2
( ) 3R
2
+ 2R Z
1
+ Z
2
( ) + Z
1
Z
2 ( )
+ Z
2
Z
2
(2R + Z
1
) R
2
( )
2R
2
Z
2
+ 2R
3
( )
R
3
+ 2R
2
Z
1
+ 2R
2
Z
2
+ 3RZ
1
Z
2
Under the condition that Z
1
(s)Z
2
(s) = R
2
, we have
Z
in
(s)
V
in
I
1

R
3
+ 2R
2
Z
1
+ 2R
2
Z
2
+ 3RZ
1
Z
2
3R
2
+ 2R Z
1
+ Z
2
( ) + Z
1
Z
2

4R
3
+ 2R
2
Z
1
+ Z
2
( )
4R
2
+ 2R Z
1
+ Z
2
( )
R
SOLUTION 14.10.
(a) Y
in
(s) Cs +
1
R
implies a parallel RC circuit with values R and C respectively.
2/23/02 page P14.8 R. A. DeCarlo, P. M. Lin
(b) Y
in
(s)
1
Z
in
(s)
2s +
1
4
which is a parallel RC circuit of values 4 and 2 F respectively.
(c) Z
in
(s) 1 +
1
2s + 0.25
1 +
1
Y
b
(s)
. Using the result of part (b), this circuit is a 1 resistor in series
with the parallel RC of part (b).
(d) Z
in
(s)
2s + 8
s + 2
2 +
4
s + 2
2 +
1
0.25s +
1
2
. Using the results of parts (b) and (c), this circuit is a 2
resistor in series with a parallel combination of a 0.25 F capacitor and a 2 resistor.
(e) Z
in
(s)
s + 3
s +1
+
s + 6
s + 4
2 +
2
s +1
+
2
s + 4
2 +
1
0.5s +1 2
+
1
0.5s +1 0.5
. Using the above results, this
circuit is a 2 resistor in series with a parallel combination of a 0.5 F capacitor and a 2 resistor which is
in series with another parallel combination of a 0.5 F capacitor and a 0.5 resistor.
SOLUTION 14.11.
(a) Clearly this is an inductor of value L in series with a resistor of value R.
(b) Inverting the admittance we have Z
in
(s) of the form of part (a). Hence the circuit is a 0.5 H inductor
in series with a 10 resistor.
(c) Y
in
(s) 0.2 +
1
0.5s +10
0.2 +
1
Z
b
(s)
. Using the result of part (b), the circuit is 0.2 S resistor in
parallel with a series connection of a 0.5 H inductor and a 10 resistor.
(d) Y
in
(s)
10s + 50
s +1
10 +
40
s +1
10 +
1
0.025s + 0.025
. This is similar to part (c). Hence the circuit is a
10 S resistor in parallel with a series connection of a 25 mH inductor and a 0.025 resistor.
(e) Y
in
(s)
s + 3
s +1
+
s + 6
s + 4
2 +
2
s +1
+
2
s + 4
2 +
1
0.5s + 0.5
+
1
0.5s + 2
. Hence, the circuit is a 2 S
resistor in parallel with the series connection of a 0.5 H inductor and a 0.5 resistor which in turn is in
parallel with a 0.5 H inductor and 2 resistor.
SOLTUION 14.12.
(a) Z
in
(s) Ls +
1
Cs
represents a series connection of an inductance L and a capacitance C.
(b) Y
in
(s) Cs +
1
Ls
represents a parallel connection of an inductance L and a capacitance C.
(c) Z
in
(s)
0.125s
2
+1
0.25s
0.5s +
1
0.25s
which is a series connection of a 0.5 H inductor and a 0.25 F
capacitor.
(d) Y
in
(s)
0.125s
2
+1
0.25s
0.5s +
1
0.25s
which is a parallel connection of a 0.5 F capacitor and 0.25 H
inductor.
2/23/02 page P14.9 R. A. DeCarlo, P. M. Lin
(e) Z
in
(s)
s
2
+1
s
+
0.25s
2
+1
0.25s
2s +
1
s
+
4
s
2s +
1
0.2s
which is a 2 H inductor in series with a 0.2 F
capacitor.
(f) Z
in
(s)
s
2
+1
s
+
0.25s
0.25s
2
+1
s +
1
s
+
1
s +
1
0.25s
. This circuit is a 1 H inductor in series with a 1 F
capacitor which is in series with a parallel connection of a 1 F capacitor and a 0.25 H inductor.
(g) Y
in
(s)
s
2
+1
s
+
0.25s
0.25s
2
+1
s +
1
s
+
1
s +
1
0.25s
. This circuit is a 1 F capacitor in parallel with a 1 H
inductor which is in parallel with a series connection of a 1 H inductor and a 0.25 F capacitor.
SOLUTION 14.13.
With L[v
out
(t)] V
o
(s) and L[v
in
(t)] V
i
(s) and v
out
(0

) 0,
sV
0
(s) + 25V
0
(s) +
100
s
V
0
(s) 5V
i
(s)
10
s
V
i
(s)
which implies that
s
2
+ 25s +100
s




1
]
1
1
V
0
(s)
5s 10
s
V
i
(s)
The transfer function is
H(s)
V
0
(s)
V
i
(s)

5s10
s
2
+ 25s +100

5s 10
(s + 5)(s + 20)
(a) If v
in
(t) te
5t
u(t) V, then
V
i
(s)
1
(s + 5)
2
and
V
out
(s)
5s10
(s + 20)(s + 5)
3

K
1
s + 20
+
C
1
s + 5
+
C
2
(s + 5)
2
+
C
3
(s + 5)
3
K
1

5s 10
(s + 5)
3
s20

110
(15)
3

110
3375

22
675
p(s)
5s10
s + 20
p (s)
(s + 20)(5) (5s 10)
(s + 20)
2

110
(s + 20)
2
110(s + 20)
2
p (s)
220
(s + 20)
3
p(5)
25 10
15

35
15

7
3

1575
675
p (5)
110
(15)
2

110
225

330
675
2/23/02 page P14.10 R. A. DeCarlo, P. M. Lin
p (5)
220
(15)
3

220
3375

44
675
Then
C
3

p(5)
01

1575
675
, C
2

p (5)
1!

330
675
, C
1

p (5)
2!

1
2

44
675


_
,

22
675
and
V
out
(s)
1
675
22
5 + 20

22
s + 5
+
330
(s + 5)
2

1575
(s + 5)
3




1
]
1
1
This yields
v
out
(t)
1
675
22e
20t
22e
5t
+ 330te
5t

1575
2
t
2
e
5t


1
]
1
u(t) V
(b) If v
in
(t) u(t) V,
V
in
(s)
1
s
and
V
out
(s)
5s 10
s(s + 5)(s + 20)

K
1
s
+
K
2
s + 5
+
K
3
s + 20
K
1

5s 10
(s + 5)(s + 20)
s0

10
100

1
10
K
2

5s 10
s(s + 20)
s5

25 10
(5)(+15)

35
75

7
15
and
K
3

5s10
s(s + 5)
s20

100 10
(20)(15)

110
300

11
30
Thus
v
out
(t)
7
15
e
5t

11
30
e
20t

1
10



1
]
1
u(t) V
By virtue of linearity and time invariance, if v
in
(t) [u(t) u(t 0.5)] V,
v
out
(t)
7
15
e
5t

11
30
e
20t

1
10



1
]
1
u(t)

7
15
e
5(t 0.5)

11
30
e
20(t 0.5)

1
10



1
]
1
u(t - 0.5) V
SOLUTION 14.14.
Here v
in
(t) cos(t) u(t) V and i
out
(t) 2sin(t)u(t) A, in which case
2/23/02 page P14.11 R. A. DeCarlo, P. M. Lin
H(s)
I
out
(s)
V
in
(s)

2
s
2
+1
s
s
2
+1

2
s
SOLUTION 14.15.
Here v
in
(t) te
t
u(t) V which implies that V
in
(s)
1
(s +1)
2
. Further,
v
out
(t) (1+ t 0.5t
2
)e
t
u(t) + sin(t)u(t) cos(t)u(t) V in which case
V
out
(s)
1
s +1
+
1
(s +1)
2
+
1
(s +1)
3
+
1
s
2
+1

s
s
2
+1
(a) Hence
H(s)
V
out
(s)
V
in
(s)

1
s +1
+
1
(s +1)
2
+
1
(s +1)
3

s 1
s
2
+1




1
]
1
1
(s +1)
2
(s +1)+1 +
1
(s +1)

(s1)(s +1)
2
s
2
+1
Simplifying
H(s)
s
3
+ 2s
2
+ 5s + 2
(s +1)(s
2
+1)
(b) If v
in
(t) (1+ t)u(t) V, then V
in
(s)
1
s
+
1
s
2

s +1
s
2
. Hence
V
out
(s) H(s)V
in
(s)
s
3
+ 2s
2
+ 5s + 2
s
2
(s
2
+1)
=
5
s
+
2
s
2

4s
s
2
+1
implying that
v
out
(t) 5 + 2t 4cos(t) [ ]u(t) V
SOLTUION 14.16.
(a) By a voltage divider (Fig. P14.16a)
V
out
(s)
Z
4
(s)
Z
3
(s) + Z
4
(s)
V
in
(s)
and
H(s)
Z
4
(s)
Z
3
(s) + Z
4
(s)
(b) In Fig. P14.16b,
Y
in
(s) Y
1
(s) + Y
2
(s)
and
V
out
(s)
1
Y
in
(s)
I
in
(s)
1
Y
1
(s) + Y
2
(s)
I
in
(s)
2/23/02 page P14.12 R. A. DeCarlo, P. M. Lin
Hence
H(s)
V
out
(s)
I
in
(s)

1
Y
1
(s) +Y
2
(s)
(c) By current division,
I
out
(s)
1
Z
3
(s) + Z
4
(s)
Y
1
(s) + Y
2
(s) +
1
Z
3
(s) + Z
4
(s)
I
in
(s)
1
Y
1
(s) + Y
2
(s) [ ] Z
3
(s) + Z
4
(s) [ ] +1
I
in
(s)
Hence
V
out
(s) Z
4
(s)I
out
(s)
Z
4
(s)
Y
1
(s) +Y
2
(s) [ ] Z
3
(s) + Z
4
(s) [ ] +1
I
in
(s) .
and
H(s)
V
out
I
in

Z
4
(s)
[Y
1
(s) +Y
2
(s)] [Z
3
(s) + Z
4
(s)] +1
SOLUTION 14.17. With V
in
(s) V
i
and V
out
(s) V
0
, H(s)
V
0
V
i
. By voltage division,
H(s)
V
0
V
i

1
10
4
s +10
3
10
3
+
1
10
4
s +10
3

1
0.1s + 2

10
s + 20
(a) V
out
(s)
400
s(s + 20)

20
s

20
s + 20
v
out
(t) 20 20e
20t
( )
u(t) V. Plot omitted.
(b) If v
in
(t) 40 u(t) u(t 0.2) [ ]V, then by linearity and time invariance
v
out
(t) 20(1 e
20t
)u(t) 20 1 e
20(t 0.2)
[ ]
u(t 0.2) V
(c) If v
in
(t) 40 u(t) + u(t 0.2) [ ]V, then by linearity and time invariance
v
out
(t) 20(1 e
20t
)u(t) + 201 e
20(t 0.2)
[ ]
u(t 0.2) V
(d) If v
in
(t) 40e
20t
u(t) V, V
i
(s)
40
(s + 20)
. Hence,
V
0
(s) H(s)V
i
(s)
400
(s + 20)
2
v
out
(t) 400te
20t
u(t) V
(e) If v
in
40te
20t
u(t) V, then V
i
(s)
40
(s + 20)
2
. Hence,
V
0
(s) H(s)V
i
(s)
400
(s + 20)
3
v
out
(t) 200t
2
e
20t
u(t) V
2/23/02 page P14.13 R. A. DeCarlo, P. M. Lin
SOLUTION 14.18.
(a) By voltage division
V
out
(s)
2s +1
s
2 +
2
s
+
2s +1
s
V
in
(s)
2s +1
s
2s + 2 + 2s +1
s
V
in
(s)
2s +1
4s + 3
V
in
(s)
Hence
H(s)
V
out
(s)
V
in
(s)

2s +1
4s + 3
(b) With v
in
(t) 8u(t) then
V
out
(s) H(s)V
in
(s)
2s +1
4s + 3


_
,
8
s


_
,

16s + 8
s(4s + 3)
Using MATLAB
n = [16 8]; d = [4 3 0];
[r,p,k] = residue(n,d)
r =
1.3333e+00
2.6667e+00
p =
-7.5000e-01
0
k = []
Then
v
out
(t)
8
3
+
4
3
e
0.75t

_
,
u(t) V
(c) If v
in
(t) 8sin(2 t)u(t) , then
V
out
(s) H(s)V
in
(s)
2s +1
4s + 3


_
,
16
s
2
+ 4



_
,

Using MATLAB,
ilaplace( (32*s+16)/((4*s+3)*(s^2+4)) )
ans =
-32/73*exp(-3/4*t)+32/73*cos(2*t)+280/73*sin(2*t)
Hence,
v
out
(t) 0.43836e
0.75t
+ 0.43836cos(2t) + 3.8356sin(2t)
( )
u(t) V
(d) With v
in
(t) 8sin(8 t)u(t)
V
out
(s) H(s)V
in
(s)
2s +1
4s + 3


_
,
64
s
2
+ 64



_
,

128s + 64
(4s + 3)(s
2
+ 64)

0.12391s + 31.907
s
2
+ 64

0.12391
s + 0.75
Using MATLAB
ilaplace((128*s + 64)/((4*s+3)*(s^2+64)))
ans =
-128/1033*exp(-3/4*t)+128/1033*cos(8*t)+4120/1033*sin(8*t)
2/23/02 page P14.14 R. A. DeCarlo, P. M. Lin

128/1033
ans =
1.2391e-01
4120/1033
ans =
3.9884e+00
v
out
(t) 0.12391cos(8t) + 3.9884sin(8t) 0.12391e
0.75t
( )
u(t) V
SOLUTION 14.19.
With a source transformation I
in
(s)
V
in
(s)
R
.
(a) By current division,
I
C
(s)
Cs
1
R
+ Cs +
1
Ls
V
in
(s)
R

LCs
2
LCs
2
+
L
R
s +1
V
in
(s)
R

s
2
s
2
+
1
RC
s +
1
LC
V
in
(s)
R
Here
H(s)
I
C
(s)
V
in
(s)

1
R
s
2
s
2
+
1
RC
s +
1
LC
(b) With R
2
3
, C 0.5F and L 1H,
H(s)
3
2
s
2
s
2
+ 3s + 2




1
]
1
1
If v
in
(t) e
t
u(t) V, then V
in
(s)
1
s +1
. Hence
I
C
(s) H(s)I
in
(s)
3
2
s
2
s
2
+ 3s + 2



_
,

1
s +1


_
,

3s
2
2(s +1)
2
(s + 2)

K
1
s + 2
+
C
1
s +1
+
C
2
(s +1)
2
Using MATLAB,
n = [3 0 0]; d = conv([2 4],[1 2 1]);
[r,p,k] = residue(n,d)
r =
6.0000e+00
-4.5000e+00
1.5000e+00
p =
-2.0000e+00
-1.0000e+00
-1.0000e+00
k =
[]

2/23/02 page P14.15 R. A. DeCarlo, P. M. Lin


I
C
(s)
6
s + 2

4.5
s +1
+
1.5
(s +1)
2
and
i
C
(t) 6e
2t

9
2
e
t
+
3
2
te
t

_
,
u(t) A
SOLUTION 14.20.
(a) Make a source transformation:
V
in
(s)
1
Cs
I
in
(s)
250
s
I
in
(s)
By voltage division
V
out
(s)
10
250
s
+
250
s
+
1
20
s +10





_
,



250
s
I
in
(s)



1
]
1

2500
s
1
20
s
2
+10s + 500


_
,
I
in
(s)

50,000
s s
2
+ 200s +10,000
( )
I
in
(s)
and
H(s)
V
out
(s)
I
in
(s)

50,000
s(s
2
+ 200s +10,000)
(b) If i
in
(t) (t) implies I
in
(s) 1. Using MATLAB
n = 50e3; d = [1 200 10e3 0];
[r,p,k] = residue(n,d)
r =
-5
-500
5
p =
-100
-100
0
k =
[]
Hence
V
out
(s)
50,000
s(s +100)
2

5
s

5
s +100

500
(s +100)
2
and
v
out
(t) 5 5e
100t
500te
100t
( )
u(t) V
This is the impulse response
(c) If L
in
(t) 100u(t) mA so that I
in
(s)
0.1
s
. Therefore
V
out
(s)
5000
s
2
(s +100)
2
In MATLAB,
2/23/02 page P14.16 R. A. DeCarlo, P. M. Lin
n = 5000; d = conv([1 0 0],[1 200 1e4])
d =
1 200 10000 0 0
[r,p,k] = residue(n,d)
r =
1.0000e-02
5.0000e-01
-1.0000e-02
5.0000e-01
p =
-100
-100
0
0
k =
[]
Hence
V
out
(s)
0.01
s
+
0.5
s
2
+
0.01
s +100
+
0.5
(s +100)
2
and
v
out
(t) 0.01e
100t
+ 0.5te
100t
0.01 + 0.5t
[ ]
u(t)
(d) By superposition and time invariance, if
i
in
(t) 100 u(t) + u(t 1) [ ] mA
then the result of part (c) can be adjusted to
v
out
(t) 0.01e
100t
+ 0.5te
100t
0.01 + 0.5t
[ ]
u(t)
0.01e
100(t 1)
+ 0.5te
100(t 1)
0.01 + 0.5(t 1)
[ ]
u(t 1)

V
SOLUTION 14.21. For this problem change the 20 mH inductor to one of 0.3 H.
(a)
Y
in
=
1
15
+
1
0.3s + 90
+
1
0.1s + 10
=
( s + 200) (s + 400)
15( s+ 100) (s + 300)

and
H(s) =
I
out
I
in
=
1/15
Y
in
=
( s + 100) (s + 300)
( s+ 200) (s + 400)

(b) If i
in
(t) = (t), then I
in
(s) = 1 and
I
out
(s) = H(s) =
( s + 100) (s + 300)
( s+ 200) (s + 400)
= 1 -
50
s + 200
-
150
s + 400
Hence
i
out
(t) = (t) + (- 50 e
-200t
- 150 e
-400 t
) u(t) A
(c) We first find the response to i
in
(t) = 16u(t) mA. Here I
in
(s) = 0.016/s and
2/23/02 page P14.17 R. A. DeCarlo, P. M. Lin
I
out
(s) = H(s)I
in
(s) =
0.016( s + 100) (s + 300)
s( s+ 200) (s + 400)
=
0.006
s
+
0.004
s + 200
+
0.006
s + 400
Hence
i
out
(t) = (6 + 4 e
-200t
+ 6 e
-400 t
)u(t) mA
By linearity and time invariance, the response to i
in
(t) = 16[ut) u(t 0.01)] mA is
i
out
(t) = (6 + 4 e
-200t
+ 6 e
-400 t
)u(t) - (6 + 4 e
-200(t - 0.01)
+ 6 e
-400 (t - 0.01)
)u(t - 0.01) mA
A plot of i
out
(t) using MATLAB is given below.
t= 0: 0.0005: 0.05;
f1= (6 + 4*exp(-200*t) + 6*exp(-400*t)).*u(t);
f2= (6 + 4*exp(-200*(t-0.01)) + 6*exp(-400*(t-0.01))).*u(t-0.01);
iout= f1 - f2;
plot(t, iout)
grid
ylabel('iout in mA')
xlabel(' time in second')
-10
-5
0
5
10
15
20
0 0.01 0.02 0.03 0.04 0.05
i
o
u
t

i
n

m
A
time in second
REMARK: Notice that the resistor current is not continuous.
SOLUTION 14. 22. (a) First observe that the admittance of a parallel LC is
Y
LC
(s) Cs +
1
Ls
Using voltage division,
2/23/02 page P14.18 R. A. DeCarlo, P. M. Lin
V
out
(s)
C
1
s +
1
Ls
C
1
s +
1
Ls
+ C
2
s +
1
Ls
V
in
(s)
C
1
s +
1
Ls
C
1
+ C
2
( )s +
2
Ls
V
in
(s)
LC
1
s
2
+1
L C
1
+ C
2
( )s
2
+ 2
V
in
(s)
Finally
H(s)
V
out
(s)
V
in
(s)

C
1
C
1
+ C
2
( )
s
2
+
1
LC
1
s
2
+
2
L C
1
+ C
2
( )
0.2
s
2
+ 4 10
6
s
2
+1.6 10
6
(b) Using MATLAB,
syms s t
ilaplace(0.2*(s^2+4e6)/(s^2+1.6e6))
ans =
1/5*Dirac(t)+120*10^(1/2)*sin(400*10^(1/2)*t)
120*10^(1/2)
ans = 3.7947e+02
h(t) 0.2L
1
s
2
+ 4 10
6
s
2
+1.6 10
6




1
]
1
1
0.2 (t) + 379.47sin(1264.9t)u(t) V
SOLUTION 14.23.
Y
1
(s) C
1
s +
1
R
1

R
1
C
1
s +1
R
1
Y
2
(s) C
2
s +
1
R
2

R
2
C
2
s +1
R
2
Then Z
1
(s)
R
1
R
1
C
1
s +1
and Z
2
(s)
R
2
R
2
C
2
s +1
. By voltage division,
V
out
(s)
R
2
R
2
C
2
S +1
R
1
R
1
C
1
s +1
+
R
2
R
2
C
1
s +1
V
in
(s)
R
2
R
2
C
2
s +1



_
,

V
in
(s)
R
1
R
2
C
2
s + R
1
+ R
1
R
2
C
1
s + R
2
(R
1
C
1
s +1)(R
2
C
2
s +1)

R
2
(R
1
C
1
s +1)
(C
1
+ C
2
)R
1
R
2
s + R
1
+ R
2
V
in
(s)
Thus the transfer function is:
H(s)
V
out
(s)
V
in
(s)

R
2
(R
1
C
1
s +1)
(C
1
+ C
2
)R
1
R
2
s + R
1
+ R
2
(b) If C
1
0.5 F, C
2
1.0 F and v
in
(t) 10 u(t) V, then
2/23/02 page P14.19 R. A. DeCarlo, P. M. Lin
V
out
(s) H(s)V
in
(s)
0.5R
1
R
2
s + R
2
1.5R
1
R
2
s + R
1
+ R
2



_
,


10
s


_
,
Moreover, with
R
1
R
2
4 so that R
1
4R
2
V
out
(s)
2R
2
2
s + R
2
6R
2
2
s + 5R
2



_
,


10
s


_
,

20R
2
2
(s +
1
2R
2
)
6R
2
2
s(s +
5
6R
2
)



1
]
1

10
3
s +
1
2R
2
s(s +
5
6R
2
)






1
]
1
1
1
1
The partial fraction expansion is
V
out
(s)
10
3
s +
1
2R
2
s s +
5
6R
2



_
,








1
]
1
1
1
1
1

10
3
K
1
s
+
K
2
s +
5
6R
2






1
]
1
1
1
1
Observe that
K
2
s +
5
6R
2

L
1
K
2
e

5
6R
2
t
and that it is required that

5
6R
2

5
3
Thus
R
2
0.5 , R
1
4R
1
2
and
V
out
(s)
10
3
s +1
s s +
5
3


_
,






1
]
1
1
1
1

10
3
0.6
s
+
0.4
s +
5
3





1
]
1
1
1

2
s
+
4 / 3
s +
5
3
Thus,
v
out
(t) 2 +
4
3
e

5
3
t




1
]
1
1
u(t)
(c) If R
1
C
1
R
2
C
2
, then the transfer function is
H(s)
R
2
(R
1
C
1
s +1)
R
1
R
2
C
1
s + R
1
R
2
C
2
s + R
1
+ R
2

R
2
s + R
2
R
2
(R
1
C
1
) + R
1
(R
2
C
2
) [ ]s + R
1
+ R
2

R
2
( s +1)
(R
1
R
2
)( s +1)

R
2
R
2
+ R
1
The zero-state response is
2/23/02 page P14.20 R. A. DeCarlo, P. M. Lin
V
out
(s)
R
2
R
2
+ R
1
10u(t)
(d) Using H(s) from part (c) with the requirement that R
1
C
1
R
2
C
2
, then
H(s)
R
2
R
1
+ R
2

1
10
With R
2
10
6
, then 10R
2
R
1
+ R
2
R
1
9R
2
9 M. Since C
2
5 10
12
F, then
C
1

R
2
C
2
R
1
0.556 10
12
F
SOLUTION 14.24.
(a) H(s)
V
out
(s)
V
in
(s)
. Here I
b
(s)
V
in
(s)
2000
. The parallel admittance at the right is
Y
R
(s) Cs +
1
R

RCs +1
R
so that
Z
R
(s)
R
RCs +1
Then
V
out
(s) Z
R
(s) I
b
(s)
R
2000(RCs +1)
(b) With V
1
(s) V
in
(s) V
out
(s) , then
Cs V
out
(s) V
in
(s) ( ) +
1
s
V
out
(s) +
1
2
V
out
(s) 3V
in
(s) + 3V
out
(s) [ ] 0
Hence
Cs +
3
2


_
,
V
in
(s) Cs +
1
s
+ 2


_
,
V
out
(s)
and
H(s)
V
out
(s)
V
in
(s)

s Cs +1.5 ( )
Cs
2
+ 2s +1

s s + 0.75 ( )
s
2
+ s + 0.5
(c) Transform the current source i
in
(t) into a voltage source. In the s-domain with I
in
(s) 2V
in
(s)
Here Z
C
(s)
1
Cs

1
2s
which implies Y
C
(s) 2s. A single node equation yields
2
3
V
out
(s) V
in
(s) [ ]
2
3
V
out
(s) + 2sV
out
(s)
2
3
V
in
(s) +
2
3

2
3
+ 2s


_
,
V
out
(s) 0
2/23/02 page P14.21 R. A. DeCarlo, P. M. Lin
2
3
V
in
(s) 2sV
out
(s) implies V
out
(s)
V
in
(s)
3s
But V
in
(s)
I
in
(s)
2
in which case
V
out
(s)
I
in
(s)
6s
and
H(s)
V
out
(s)
I
in
(s)

1
6s
SOLUTION 14.25. Here in the s-domain
I
in
(s)
V
in
(s) V
C
(s)
0.5s +10
and with a node at V
C
(s)
I
in
(s) +
sV
C
(s)
500
+
1
2
I
in
(s) 0 implies
sV
C
(s)
50

1
2
I
in
(s)
V
in
(s) V
C
(s)
s + 20
Hence,
s
2
+ 20s + 500
500



_
,

V
C
(s) V
in
(s)
and the transfer function is
H(s)
V
C
(s)
V
in
(s)

500
s
2
+ 20s + 500
With v
in
(s) 4 5 u(t) V which implies V
in
(s)
4 5
s
, and
V
C
(s)
2000 5
s(s +10 j20)(s +10 + j20)

K
1
s
+
K
s +10 j20
+
K
*
s +10 + j20
where K
*
designates the complex conjugate of K
K
1

2000 5
s
2
+ 20s + 500
s0

2000 5
500
8.944
K
2000 5
s(s +10 + j20)
s(10 j 20)

2000 5
(10 + j20)( j40)

2000 5
(800 + j400)
4.472 + j2236 Je
ij
where 153.44
o
. Then with
K
*
4.472 j2.236 5e
j
V
C
(s)
8.944
s
+
A + jB
s +10 + j20
+
A jB
s +10 j20
Here
2/23/02 page P14.22 R. A. DeCarlo, P. M. Lin
A 4.472, B 2.236, A
2
+ B
2
5
and
arc tan
B
A
arc tan
2.236
4.472
arc tan
1
2
153.44
o
With the help of Table 13.1.
v
C
(t) 8.944 +10e
10t
cos(20t +153.44
o
)
[ ]
u(t) V
SOLUTION 14.26. In the s-domain we first find V
x
(s) in terms of V
in
(s) via voltage division:
V
x
(s)
Z
p
(s)
40 + Z
p
(s)
V
in
(s)
where
Z
p
(s)
(0.40s)(40)
0.40s + 40

40s
s +100
Hence
V
x
(s)
40s
s +100
40 +
40s
s +100
V
in
(s)
40s
80s + 4000

0.5s
s + 50
V
in
(s)
and
I
L
(s)
V
x
(s)
0.4s

2.5
s
V
x
(s)
Then from the right hand side by another voltage division
V
out
(s)
10
1000
s
+10
0.25V
x
(s)
10s
10s +1000
0.25V
x
(s)
0.25s
s +100
V
x
(s)
(a) If v
in
(s) 20(1 e
40t
)u(t), then V
in
(s)
20
s

20
s + 40
. Hence
V
x
(s)
0.5s
s + 50


_
,

20
s

20
s + 40



1
]
1

s
s + 50


_
,

10
s

10
s + 40


_
,
and
I
L
(s)
2.5
s
V
x
(s)
2.5
s + 50


_
,

1
s

1
s + 40



1
]
1

0.05
s
+
0.25
s + 40
+
0.2
s + 50
Hence
i
L
(t) 0.05 + 0.2e
50t
0.25e
40t
[ ]
u(t) A
(b) V
in
(s)
20
s

20
s + 40
. From, part (a), it was found that
V
out
(s)
0.25s
s +100
V
x
(s)
0.25s
s +100

0.5s
s + 50
V
in
(s)
s
s +100

s
s + 50

2.5
s

2.5
s + 40



1
]
1
2/23/02 page P14.23 R. A. DeCarlo, P. M. Lin

100s
s + 40 ( ) s + 50 ( ) s +100 ( )

20/ 3
s + 40 ( )
+
10
s + 50 ( )
+
10 /3
s +100 ( )
Thus,
V
out
(t) 10e
50t

10
3
e
100t

20
3
e
40t


1
]
1
u(t) V
SOLUTION 14.27. In both parts (a) and (b), the op-ampis ideal. It will not draw current and the
virtual groundprincipal requires that
v
+
v

0
(a) For a note at the inverting terminal with mode voltage v
1
(t) 0 , KCL gives in the s-domain
V
in
(s)
R
1
CsV
out
(s)
V
out
(s)
R
2
implies
V
in
(s)
R
1
Cs +
1
R
2



_
,

V
out
(s)
in which case
V
in
(s)
R
1

R
2
Cs +1
R
2



_
,

V
out
(s)
Then,
H(s)
V
out
(s)
V
in
(s)

R
2
R
1
1
R
2
Cs +1



_
,


1
R
1
C
1
s +
1
R
2
C





_
,



To make
H(s)
20
s + 4
make
1
R
1
C
20 and
1
R
2
C

1
4
If
C 1 F 10
6
F
1
R
2
C
4 or R
2

1
4C

10
6
4
250 k
and
1
R
1
C
20 or R
1

2
20C

106
20
50 k
(b) With Fig. 14.27b in the s-domain and v
1
(t) at the inverting terminal, KCL gives
C
1
sV
in
(s) +
V
in
(s)
R
1
C
2
sV
out
(s)
V
out
(s)
R
2
R
1
C
1
s +1
R
1



_
,

V
in
(s)
R
2
C
2
s +1
R
2



_
,

V
out
(s)
2/23/02 page P14.24 R. A. DeCarlo, P. M. Lin
Then
H(s)
V
out
(s)
V
in
(s)

R
2
R
1
R
1
C
1
s +1
R
2
C
2
s +1



_
,


R
1
R
2
C
1
R
1
RC
2
s +
1
R
1
C
1
s +
1
R
2
C
2





_
,




C
1
C
2
s +
1
R
1
C
1
s +
1
R
2
C
2






1
]
1
1
1
1
(c) If
H(s) 5, C
2
1 F and R
2
1M
then
C
1
C
2
= 5 and C
1
5 F
The bracketed term must cancel and with
R
2
C
2
10
6
(10
6
) 1
Then with C
1
5 F
R
1
C
1
1
R
1

1
C
1

1
5 10
6
200k
(d) Using H(s) in part (b)
H(s)
C
1
C
2
s +
1
R
1
C
1
s +
1
R
2
C
2





_
,



to obtain
H(s) 5
s +1
s + 2
with C
2
1 F
C
1
5C
2
5 10
6
F (5 F)
1
R
1
C
1
1 or R
1

1
C
1

10
6
5
200k
1
R
2
C
2
2 or R
2

1
2C
2

10
6
2
500k
SOLUTION 14.28. Here, the op-amp will not draw current at the non-inverting terminal and the
principal of the virtual ground demand that
v
1
v
2
V
in
(s)
For Fig. 14.28 in the s-domain with a node V
1
(s)
taken at the inverting terminal
V
in
(s)
R
1
+
V
in
(s) V
out
(s)
Z
p
(s)
0
Here
2/23/02 page P14.25 R. A. DeCarlo, P. M. Lin
Z
p
(s)
R
2
Cs
1
Cs
+ R
2

R
2
R
2
Cs +1

1
C(s +
1
R
2
C
)
and
1
R
1
+ C(s +
1
R
2
C
)



1
]
1
V
in
(s) C(s +
1
R
2
C
)V
out
(s)
R
1
C s +
1
R
2
C



_
,

+1




1
]
1
1
V
in
(s) R
1
C s +
1
R
2
C



_
,

V
out
(s)
and
H(s)
V
out
(s)
V
in
(s)

R
1
C(s +
1
R
2
C
) +1
R
1
C(s +
1
R
2
C
)

s +
1
R
2
C
+
1
R
1
C
s +
1
R
1
C
Here
1
R
2
C
+
1
R
1
C
4
and
1
R
1
C
2
If C 1 F then R
1

1
2C

10
6
2
500 k
and
1
R
2
C
4 2 2 implies R
2
500 k.
SOLUTION 14.29. For the non-inverting configuration in the s-domain, each of the two op-amps in
cascade have a transfer function
H(s)
Z
f
(s)
Z
in
(s)
Then for the two op-amps
H
0
(s)
Z
f 1
(s)
Z
in1
(s)



1
]
1

Z
f 2
(s)
Z
in,2




1
]
1
1

Z
f ,1
(s)Z
f ,2
(s)
Z
in,1
(s)Z
in,2
(s)
For Fig. P14.29a in the s-domain
Z
in,1
25k, Z
in,2
50k, Z
f ,1

1
C s +
1
RC


_
,

250,000
s + 5
, and Z
f ,2

1
C s +
1
RC


_
,

250,000
s + 2.5
Hence for Fig. P14.29a
2/23/02 page P14.26 R. A. DeCarlo, P. M. Lin
H
a
(s)
250,000
s + 2.5
25,000





_
,



250,000
s + 5.0
50,000





_
,


50
s + 2.5 ( )(s + 5)
If v
in
(t) u(t), then V
in
(s)
1
s
and
V
out
(s)
50
s s + 2.5 ( )(s + 5)

4
s

8
s + 2.5
+
4
s + 5
Hence,
v
out
(t) 4 8e
2.5t
+ 4e
5t
[ ]
u(t) V
(b) For Fig. P14.29b in the s-domain, Z
in,1
, Z
f 1
, and Z
f 2
are in part (a). However,
Z
in,2

1
Cs

250,000
s
Thus
H
b
(s)
250,000
s + 2.5
25,000





_
,




250,000
s + 5
250,000
s





_
,


10
s + 2.5


_
,

s
s + 5


_
,

10s
s +
5
2


_
,
(s + 5)
With V
in
(s)
1
s
,
V
out
(s)
10s
(s + 2.5)(s + 5)



1
]
1
1
s


_
,



1
]
1

20
(s + 2.5)(s + 5)

4
s + 2.5

4
s + 5
and
v
out
(t) 4e
2.5t
4e
5t
( )
u(t) V
SOLUTION 14.30.
(a) -(b). The subcircuit is an integrator, with
V
out
(t)
V
1
(s)
= -
1
s
(c) This subcircuit is again an integrator, with
V
1
(t)
V
2
(s)
= -
1
s
(d) Applying KCL to the inverting input terminal of the top left op amp, we have
G
2
V
out
(s) + V
3
(s) + V
2
(s) = 0
or
V
2
(s) = - G
2
V
out
(s) - V
3
(s)
2/23/02 page P14.27 R. A. DeCarlo, P. M. Lin
(e) Applying KCL to the inverting input terminal of the bottom op amp, we have
G
3
V
1
(s) + G
1
V
in
(s) + V
3
(s) = 0
or
V
3
(s) = - G
3
V
1
(s) - G
1
V
in
(s)
(f) The results of parts (b), (c) and (d) do not involve V
in
. Therefore, , we can solve for V
1
, V
2
and V
3
in terms of V
out
from these threes equations:
V
1
(s) = - sV
out
(s)
V
2
(s) = - sV
1
(s) = s
2
V
out
(s)
and
V
3
(s) = - V
2
(s) - G
2
V
out
(s) = - s
2
+ G
2
V
out
(s)
Substituting these relationships into the result of part (e), we obtain
- sG
3
V
out
(s) + G
1
V
in
(s) - s
2
+ G
2
V
out
(s) = 0
Therefore
H(s) =
V
out
(s)
V
in
(s)
=
G
1
s
2
+ G
3
s + G
2

SOLUTION 14.31. Use the parallel equivalent circuit model for the capacitor with the standard
directions for voltage and current as given in figure 14.16. For the single node with v
C
(0

) 20 V,
V
C
(s)
10
s
+
V
C
(s)
40 +10

1
10
v
C
(0

) 0 implies
s
10
+
1
50


_
,
V
C
(s)
1
10
(20) 2
Equivalently, (50s +10)V
C
(s) (s + 0.2)V
C
(s) 20 or V
C
(s)
20
s + 0.2
. Therefore,
v
C
(t) 20e
0.2t
u(t) V
SOLUTION 14.32. Using the equivalent model for the inductor in figure 14.19, we can compute the
total admittance as
Y(s)
5
2s
+
1
40
+
1
10

5
2s
+
1
8

2s + 40
16s

s + 20
8s
Using current division,
2/23/02 page P14.28 R. A. DeCarlo, P. M. Lin
I
out
(s)
0.1
s + 20
8s

i
L
(0)
s

0.8i
L
(0

)
s + 20

1.6
s + 20
Thus
i
out
(t) 1.6e
20t
u(t) A
SOLUTION 14.33. Using the equivalent model for the inductor in figure 14.19 and for the capacitor
using figure 14.16, we may combine the current sources to form an equivalent source (with C = 0.1 F) to
obtain
I
eq
(s)
1
10
v
C
(0

)
i
L
(0

)
s
0.2
1
s
Note that
Y(s) Cs +
1
LS

LCs
2
+1
Ls
C
s
2
+
1
LC
s





_
,



With C = 0.1 F and L = 0.4 H,
1
LC
25 and
Z(s)
1
C
s
s
2
+ 25



_
,

10s
s
2
+ 25
Thus
V
C
(s) Z(s)I
eq
(s)
10s
s
2
+ 25



_
,

s 5
5s


_
,

2(s 5)
s
2
+ 25

2s
s
2
+ 25

10
s
2
+ 25
and
v
C
(t) 2cos(5t) 2sin(5 t) ( ) u(t) V
SOLUTION 14.34. Consider the equivalent circuit below:
Writing a single node equation we have,
2/23/02 page P14.29 R. A. DeCarlo, P. M. Lin
0 0.5V
out
0.2V
R1
+
1
5 + 0.4s
V
out
+ 0.8 ( )
0.5V
out
+ 5 0.2
1
5 + 0.4s
V
out
+ 0.8 ( ) +
1
5 + 0.4s
V
out
+ 0.8 ( )
0.5V
out
+
2
5 + 0.4s
V
out
+ 0.8 ( )
Therefore
V
out

8
s + 22.5
and
v
out
(t) 8e
22.5t
u(t) V
SOLUTION 14.35. Redraw the circuit in the s-domain and use an equivalent circuit for the capacitor
(figure 14.16) that accounts for the initial condition. By KCL
V
C
(s)
R
+ CsV
C
(s) Cv(0

) + I
in
(s)
With R 50 and C 0.02 F,
V
C
(s)
50
+ 0.02sV
C
(s) 0.02v
C
(0

) + I
in
(s)
or
V
C
(s) + sV
C
(s) v
C
(0

) + 50I
in
(s)
which is equivalent to
(s +1)V
C
(s) v
C
(0

) + 50I
in
(s)
(a) With v
C
(0

) 8 V and i
in
(t) 40 (t) mA so that I
in
(s) 0.04 ,
(s +1)V
C
(s) 8 + 2 implies V
C
(s)
10
s +1
and
v
C
(t) 10e
10t
u(t) V
(b) With v
C
(0

) 1 V and i
in
(t) 200e
t
u(t) mA we have that I
in
(s)
0.2
s +1
. Thus
V
C
(s)
v
C
(0

)
(s +1)
+
50
(s +1)
I
in
(s)
1
(s +1)
+
10
(s +1)
2
and
v
C
(t) e
t
+10te
t
( )
u(t) V
SOLUTION 14.36.
2/23/02 page P14.30 R. A. DeCarlo, P. M. Lin
(a) By current division
H(s) =
I
L
(s)
I
in
(s)
=
1
Ls + 1
(b) The given data in Laplace transforms are:
I
in
(s) =
I
0
s
2
and I
L
(s) =
15
s
2
-
3
s
+
3
s+ 5
=
75
s
2
s+ 5
Under the assumption of zero initial inductor current,
I
L
(s) = H(s) I
in
(s) = H(s) =
I
L
(s)
I
in
(s)
=
I
0
Ls + 1 s
2

=
I
0
/L
s + 1/L s
2

=
75
s
2
s+ 5
Equating coefficients, we obtain the answers
L = 1/5 = 0.2 H and I
0
= 75L =15 A
(c) The s-domain equivalent is shown below.
Applying KVL to the right mesh, we have
LsI
L
(s) - Li
L
(0
-
) + 1[I
L
(s) - 1] = 0
Solving for I
L
(s) from this equation, and equating it to 10/(s+ 5), we have
I
L
(s) =
L i
L
(0
-
) + 1
L s + 1
=
i
L
(0
-
) + 1/L
s + 1/L
=
10
s + 5
from which L= 0.2 H and i
L
(0
-
) = 5 A.
SOLUTION 14.37.
(a) By inspection,
2/23/02 page P14.31 R. A. DeCarlo, P. M. Lin
H(s) =
I
L
(s)
V
in
(s)
=
1
2s + 200
=
0.5
s + 100
Given v
in
(t) = 2u(t) V, then V
in
(s) = 2/s , and
I
L
(s) =
1
s s + 100
=
0.01
s
-
0.01
s + 100
in which case
i
L
(t) = 0.01 ( 1 - e
-100t
) A
Plots are omitted.
(b) By linearity and time invariance,
i
L
(t) = 0.01 ( 1 - e
-100t
)u(t) - 0.01( 1 - e
-100(t- 0.05)
)u(t - 0.05) A
(c) Correction: (a) should be (c). With nonzero initial inducto current, the s-domain equivalent
becomes:
Given i
L
(0
-
) = 0.01 A and v
in
(t) = 2e
-200t
u(t) V, then V
in
(s) = 2/(s + 200) and
I
L
(s) =
2
s + 200
+ 0.02
2s + 200
=
0.01s + 3
( s + 100)( s + 200)
=
0.02
s + 100
-
0.01
s + 200
in which case
i
L
( t) = 0.02e
-100t
- 0.01e
-200t
u(t) A
(d) Correction . (b) should be (d).
Given i
L
(0
-
) = 0.01 A and v
in
(t)= 2cos(200t)u(t) V, then
V
in
(s) =
2s
s
2
+ 40000
and
2/23/02 page P14.32 R. A. DeCarlo, P. M. Lin
I
L
(s) =
2s
s
2
+ 40000
+ 0.02
2s + 200
=
0.01s
2
+ s + 400
( s + 100)( s
2
+ 40000)
We use MATLAB to do the partial fraction expansion.
n= [ 0.01 1 400];
d= conv([ 1 100], [ 1 0 40000]);
[ r p k ] = residue (n,d)
r =
0.0010 - 0.0020i
0.0010 + 0.0020i
0.0080
p =
1.0e+02 *
0.0000 + 2.0000i
0.0000 - 2.0000i
-1.0000
From the above MATLAB output,
I
L
(s) =
0.008
s + 100
+
0.001-j0.002
s - j200
+
0.001+j0.002
s + j200
=
0.008
s + 100
+
0.002s + 0.8
s
2
+ 200
2
From table 13.1, item 18,
i
L
( t) = 0.008e
-100t
+ 0.002 cos(200t) + 0.004 sin(200t) u(t) A
2/23/02 page P14.1 R. A. DeCarlo, P. M. Lin
SOLUTI ON 14.38. In the s-domain, we break the response up into the part due to the initial condition
and the part due to the source with the initial condition set to zero. The transfer function with the initial
condition set to zero is
H(s)
V
C
(s)
V
in
(s)

1 Cs
R +1 Cs

1 RC
s +1 RC

0.25
s + 0.25
Using the parallel equivalent circuit for the charged capacitor while setting the source voltage to zero,
the capacitor voltage due only the initial condition is:
V
C,IC
(s)
1
1
R
+ Cs
Cv
C
(0

)
[ ]

v
C
(0

)
s + 0.25
Hence,
V
C
(s)
0.25
s + 0.25
V
in
(s) +
v
C
(0

)
s + 0.25
and
I
C
(s)
V
in
(s) V
C
(s)
20
0.05 1
0.25
s + 0.25



1
]
1
V
in
(s)
0.05v
C
(0

)
s + 0.25

0.05s
s + 0.25
V
in
(s)
0.05v
C
(0

)
s + 0.25
for all inputs and initial conditions.
(a) If v
in
(t) 20u(t) and v
C
(0

) 10 V, then V
in
(s)
20
s
and
V
C
(s)
5
s(s + 0.25)
+
10
s + 0.25

20
s

10
s + 0.25
v
C
(t) 20 10e
0.25t
( )
u(t) V
and
I
C
(s)
1
s + 0.25

0.5
s + 0.25

0.5
s + 0.25
i
C
(t) 0.5e
0.25t
u(t) A
(b) If v
in
(t) 5e
0.25t
u(t) V, then V
in
(s)
5
s + 0.25
. Hence,
V
C
(s)
1.25
(s + 0.25)
2
+
10
s + 0.25
v
C
(t) 10 +1.25t ( )e
0.25t
u(t) V
and
I
C
(s)
0.25s
(s + 0.25)
2

0.5
s + 0.25

0.25
s + 0.25

0.0625
(s + 0.25)
2
Hence
i
C
(t) 0.25 + 0.0625t ( )e
0.25t
u(t) A
2/23/02 page P14.2 R. A. DeCarlo, P. M. Lin
0 5 10 15 20
0
1
2
3
4
5
6
7
8
9
10
Time in s
I
n
p
u
t

a
n
d

C
a
p
a
c
i
t
o
r

v
o
l
t
a
g
e
s
,

V
TextEnd
SOLUTI ON 14.39. The figure which accounts for the initial conditions is given below.
(a) For the zero-input response, the above circuit reduces to a parallel RLC driven by two current
sources.
Hence V
C
(s) equals the total current divided by the total admittance, i.e.,
2/23/02 page P14.3 R. A. DeCarlo, P. M. Lin
V
C
(s)
Cv
C
(0

) +
i
L
(0

)
s
Cs +
1
R
+
1
Ls

sv
C
(0

) +
i
L
(0

)
C
s
2
+
1
RC
s +
1
LC

20s +10
s
2
+ 250s +10
4

26.6
s + 200

6.6
s + 50
Hence
v
C
(t) =[26.6e
200t
6.6e
50t
]u(t) V
(b) For the zero-state response, the current sources disappear. Executing a source transformation on
the remaining voltage source, we obtain a current, I(s) =V
in
(s)/(Ls), driving a parallel RLC circuit.
Hence, the zero input response is
V
C
(s)
V
in
(s)
Ls
Cs +
1
R
+
1
Ls

1
LC


_
,
V
in
(s)
s
2
+
1
RC
s +
1
LC

20000
s
3
+ 250s
2
+10
4
s

2
s
+
0.6667
s + 200

2.6667
s + 50
Hence
v
C
(t) =[2 +0.6667e
200t
2.6667e
50t
]u(t) V
(c) By superposition, the complete response is the sum of the answers to (a) and (b). Hence
v
C
(t) =[2 +27.267e
200t
9.2667e
50t
]u(t) V
(d) By linearity and time-invariance,
v
C
(t) =[2 +0.6667e
200t
2.6667e
50t
]u(t)
+[4 +1.3334e
200(t0.01)
5.3334e
50(t0.01)
]u(t 0.01) V
2/23/02 page P14.4 R. A. DeCarlo, P. M. Lin
SOLUTI ON 14.40. f (t) L
in
(t) e
2t
u(t)A [part (c)]
for the zero-input response, f (t) L
in
(t)
L
2
(0

)
s
[part (d)]
for the zero-input response, f (t) L
in
(t) Cv
C
(0)A [part (e)]
(a) Y
in
(s) 1 +
1
s
+ s +1
s
2
+ 2s +1
s

(s +1)
2
s
(b) I
out
(s)
Y
1
(s)
Y
in
(s)
I
in
(s)
s +1
s
2
+ 2s +1
s

s(s +1)
(s +1)
2

s
s +1
I
in
(s)
and
H(s)
I
out
(s)
I
in
(s)

s
s +1
(c) If i
in
(t) e
2t
u(t) A, then I
in
(s)
1
s + 2
, then
I
out
(s) H(s)I
in
(s)
s
(s +1)(s + 2)

1
s +1
+
2
s + 2
which implies that the zero-state response is
i
out
(t) 2e
2t
e
t
( )
u(t) A
(d) If i
L
(0

) 2 A, v
C
(0

) 0, and i
in
(t) 0. Using the parallel equivalent circuit for the inductor,
figure 14.19, we have
I
out
(s) H(s)
i(0

)
s

s
s +1


_
,

2
s


_
,

2
s +1
i
out
(t) 2e
t
u(t) A
(e) Use the parallel equivalent circuit for the capacitor, figure 14.16, to obtain by current division,
I
out
(s)
1
s
+1
1
s
+1 + s +1
[Cv
C
(0

)] 4
s +1
s
2
+ 2s +1

4
s +1
i
out
(t) 4e
t
u(t) A
(f) By superposition, the complete response is the sum of the answers to parts (c), (d) and (e).
SOLUTI ON 14.41. With v
in
(t) 4u(t)V and v
C
(0

) 1 V, a single node equation at the front half of


the circuit yields with Cv
C
(0

) 11 1:
2/23/02 page P14.5 R. A. DeCarlo, P. M. Lin
2
4
5


_
,
+ 2 +
4
s
+ s


_
,
V
C1
(s) 1 0
s
2
+ 2s + 4
s



_
,

V
C1
(s)
8
s
+1
s + 8
s
or
V
C1
(s)
s + 8
s
2
+ 2s + 4
For the rear-half, represent the capacitor by a series equivalent circuit. Thus we can obtain an equivalent
voltage source with value:
V
eq
(s) 2V
C1
(s)
V
C
(0

)
s

2(s + 8)
s
2
+ 2s + 4

1
s

2s(s + 8) (s
2
+ 2s + 4)
s s
2
+ 2s + 4
( )
or equivalently
V
eq
(s)
s
2
+14s 4
s s
2
+ 2s + 4
( )
By a voltage division,
V
out
(s)
1
s
1
s
+ 0.5
V
eq
(s) +
v
C
(0

)
s

2
s + 2

s
2
+14s 4
s s
2
+ 2s + 4
( )
+
1
s

2s
2
+ 28s 8 + s + 2 ( ) s
2
+ 2s + 4
( )
s s + 2 ( ) s
2
+ 2s + 4
( )

s
2
+ 6s + 36
( )
s + 2 ( ) s
2
+ 2s + 4
( )
Using MATLAB
num =[1 6 36];
den =conv([1 2],[1 2 4])
den =
1 4 8 8
[r,p,k] =residue(num,den)
r =
7.0000e+00
-3.0000e+00 - 2.8868e+00i
-3.0000e+00 +2.8868e+00i
p =
-2.0000e+00
-1.0000e+00 +1.7321e+00i
-1.0000e+00 - 1.7321e+00i
k =
However, it would appear easier here to use ilaplace:
syms t s
ilaplace((s^2+6*s+36)/((s+2)*(s^2+2*s+4)))
ans =
7*exp(-2*t)-6*exp(-t)*cos(3^(1/2)*t)+10/3*exp(-t)*3^(1/2)*sin(3^(1/2)*t)
2/23/02 page P14.6 R. A. DeCarlo, P. M. Lin

Hence
v
out
(t) 7e
2t
+ e
t
3.334 3 sin( 3t ( ) 6cos( 3t)
[ ]
u(t) V
SOLUTI ON 14.42. Using the series equivalent circuit (figure 14.17) for C
1
, we have
I
1K
(s) =
v
C1
(0
-
)/s
R +
1
sC
1
=
- 0.25/s
1000 + 50/s
=
-0.25
1000s + 50
Next, since v
C2
(0

) 0, we have
V
C2
(s)
I
1k
(s)
C
2
s

0.25 500
s 1000s + 50 ( )

0 / 1 2 5
s(s + 0.05)
Finally,
V
out
(s) = -V
C2
(s) =
0.125
s(s + 0.05)
= 2.5(
1
s
-
1
s + 0.05
)
and
v
out
(t) = 2.5( 1 - e
-0.05t
)u(t) V
SOLUTI ON 14.43. (a) It is preferable to use the series equivalent circuit (figure 14.17) for C
1
, and the
parallel equivalent circuit (figure 14.16) for C
2.
(b) The current through the 2.5 k resistor is given by
I
2.5K
(s) =
v
C1
(0
-
)/s
R
1
+
1
sC
1
=
- 2/s
2500 + 5000/s
=
- 2
2500s + 5000
=
-810
-4
s + 2
Next,
V
out
(s) =V
C2
(s) = I
2.5K
(s)
1
sC
2
+
1
R
2
=
-810
-4
s + 2

1
0.0002s + 0.0002
=
- 4
(s + 2) (s + 1)


(c) Hence
V
out
(s) =
- 4
(s + 2) (s + 1)
= - 4
1
s + 1
-
1
s +2

and
v
out
(t) 4 e
2t
e
t
( )
u(t) V
(d) SPICE plot omitted.
SOLUTI ON 14.44. (a) From voltage division,
2/23/02 page P14.7 R. A. DeCarlo, P. M. Lin
H(s) =
V
C2
V
in
=
Z
2
Z
1
+ Z
2
=
R
2
R
2
C
2
s + 1
1
C
1
s
+ R
1
+
R
2
R
2
C
2
s + 1
=
R
2
C
1
s
R
1
C
1
R
2
C
2
s
2
+ ( R
1
C
1
+R
2
C
2
+ R
2
C
1
)s + 1
(b) If v
in
(t) =15u(t) V, then V
in
(s) = 15/s and
V
C2
= H(s)V
in
=
1.75s
s
2
+ 4.25 s + 1

15
s
=
7
s+ 0.25
-
7
s+ 4
Hence
v
C2
(t) 7 e
0.25t
e
4t
( )
u(t) V
0 1 2 3 4 5 6 7 8
0
5
10
15
Time in s
v
i
n

a
n
d

v
c
2
TextEnd
(c) Using the series equivalent for C
1
, we have
V
C2
= H(s)
v
C1
(0
-
)
s
=
1.75s
s
2
+ 4.25 s + 1

15
s
which is the same as result in part (b). Therefore
v
C2
(t) 7 e
0.25t
e
4t
( )
u(t) V
(d) Using the parallel equivalent for C
2, we have
2/23/02 page P14.8 R. A. DeCarlo, P. M. Lin
V
C2
(s) = C
2
v
C2
(0
-
)
1
2
7
s +
4
7
+
1
2 + 1/s
=
2
7
15
1
2
7
s +
4
7
+
s
2s + 1
=
1
s + 0.25
+
14
s + 4

Hence
v
C2
(t) e
0.25t
+14e
4t
( )
u(t) V
(e) By linearity, the answer is the sum of parts (b), (c) and (d).
SOLUTI ON 14.45. (a) Z
1
= R
1
+ L
1
s and Z
2
=
R
2
L
2
s
R
2
+ L
2
s
. From Ohm's law
I
L1
=
V
in
Z
1
+ Z
2
=
V
in
R
1
+ L
1
s +
R
2
L
2
s
R
2
+ L
2
s
=
(R
2
+ L
2
s)V
in
(R
1
+ L
1
s) (R
2
+ L
2
s) + + R
2
L
2
s
Using current division, we have
I
L2
=
R
2
R
2
+ L
2
s
I
L1
=
R
2
V
in
(R
1
+ L
1
s) (R
2
+ L
2
s) + + R
2
L
2
s
Therefore
H(s) =
I
L2
V
in
=
R
2
(R
1
+ L
1
s) (R
2
+ L
2
s) + + R
2
L
2
s
=
G
1
G
1
L
1
G
2
L
2
s
2
+ (G
1
L
1
+ G
2
L
2
+ G
1
L
2
)s + 1

With the given element values,
H(s) =
2
21
4
7

7
8
s
2
+ (21+
4
7

7
8
+ 2
7
8
)s + 1
=
2
s
2
+ 4.25s + 1

(b) If v
in
(t) =15u(t) V, then V
in
(s) = 15/s and
I
L2
= H(s)V
in
=
2
s
2
+ 4.25 s + 1

15
s
=
30
s
-
32
s+ 0.25
-
2
s+ 4
Hence
i
L2
(t) 30 32e
0.25t
+ e
4t
( )
u(t) A
Plot omitted.
(c) Using the series equivalent for L
1
, we have
I
L2
= H(s) L
1
i
L1
(0
-
) =
2
s
2
+ 4.25 s + 1
115 =
8
s + 0.25
-
8
s +4
Therefore
i
L2
(t) 8 e
0.25t
e
4t
( )
u(t) A
2/23/02 page P14.9 R. A. DeCarlo, P. M. Lin
(d) Using the parallel equivalent for L
2, we have
I
L2
(s) =
L
2
i
L2
(0
-
)
L
2
s +
R
2
(R
1
+ L
1
s)
R
2
+ (R
1
+ L
1
s)
=
7
8
15
7
8
s +
1.75(0.5 + s)
1.75 + (0.5 + s)
=
15( s + 2.25)
s
2
+4.25s + 1
=
8
s + 0.25
+
7
s + 4
Hence
i
L2
(t) = (8e
-0.25t
+ 7 e
-4t
) u(t) A
(e) By linearity, the answer is the sum of parts (b), (c) and (d).
SOLUTI ON 14.46. (a) Using the result of problem 14.44(a)

V
C2
2I
in
=
R
2
C
1
s
R
1
C
1
R
2
C
2
s
2
+ ( R
1
C
1
+R
2
C
2
+ R
2
C
1
)s + 1
=
1.75s
s
2
+ 4.25 s + 1

Therefore
H
1
(s) =
V
C2
I
in
= =
3.5s
s
2
+ 4.25 s + 1

(b) Using the result of problem 14.45(a),
H
2
(s) =
I
L2
V
C2
=
R
2
(R
1
+ L
1
s) (R
2
+ L
2
s) + + R
2
L
2
s
=
2
s
2
+ 4.25s + 1

(c)
H(s) =
I
L2
I
in
=H
1
(s)H
2
(s) =
3.5s
s
2
+ 4.25s + 1

2
s
2
+ 4.25s + 1
=
7s
(s
2
+ 4.25s + 1)
2
(d) We first represent the initialized capacitor by the series equivalent (figure 14.17), and then apply a
source transformation. From this circuit, by utilizing the expression derived in part (c), we have

I
L2
v
C1
(0)
2 s
=
sI
L2
7.5
H(s) =H
1
(s)H
2
(s) =
7s
(s
2
+ 4.25s + 1)
2
Therefore
I
L2
(s) =
52.5
(s
2
+ 4.25s + 1)
2
=
1.9911
s + 4
+
3.7333
( s + 4)
2
-
1.9911
s + 0.25
+
3.7333
( s + 0.25)
2
and
i
L2
(t) = [(1.9911 + 3.7333t) e
-4t
+(- 1.9911 + 3.7333t) e
-0.25t
] u(t) A
Note: the book answer for part (d) should be divided by 2.
(e) Since I
in
(s) =15/s, we have
2/23/02 page P14.10 R. A. DeCarlo, P. M. Lin

I
L2
(s) = H(s)I
in
(s) =
105
(s
2
+ 4.25s + 1)
2
=
3.9822
s + 4
+
7.4666
( s + 4)
2
-
3.9822
s + 0.25
+
7.4666
( s + 0.25)
2
and
i
L2
(t) = [(3.9822 + 7.4666t) e
-4t
+(- 3.9822 + 7.4666t) e
-0.25t
] u(t) A
SOLUTI ON 14.47. (a) For this passive circuit, we may write the nodal equations by inspection.
0.8s + 2 +
10
s
-
10
s
-
10
s
1 +
10
s

V
C
V
R
=
2V
s1
-I
s2
(b) V
s1
=3/s and I
s2
=3/s. We solve for V
R
by Cramer's rule to obtain
V
R
=
0.8s + 2 +
10
s
6
s
-
10
s
-
3
s
0.8s + 2 +
10
s
-
10
s
-
10
s
1 +
10
s
=
- 2.4s
2
- 6s + 30
s(0.8s
2
+ 10s + 30)
=
- 4
s + 7.5
+
1
s
and
v
R
(t) 1 4e
7.5t
( )
u(t) V
(c) We represent the initialized capacitor by the parallel equivalent circuit ( figures 14.16) In this case
the nodal equations becomes
0.8s + 2 +
10
s
-
10
s
-
10
s
1 +
10
s

V
C
V
R
=
6
s
+ 2.4
-
3
s
Solve for V
R
by Cramer's rule to obtain
V
R
=
0.8s + 2 +
10
s
6
s
+ 2.4
-
10
s
-
3
s
0.8s + 2 +
10
s
-
10
s
-
10
s
1 +
10
s
=
- 2.4s
2
+18s + 30
s(0.8s
2
+ 10s + 30)
=
1
s
-
16
s + 7.5
+
12
s + 5

2/23/02 page P14.11 R. A. DeCarlo, P. M. Lin
v
R
(t) = (1 - 16e
-7.5t
+ 12e
- 5t
) u(t) V
SOLUTI ON 14.48. (a) After performing the suggested source transformation, and representing the
initialized capacitor and inductor by their series equivalent circuits, we can write two mesh equations by
inspection:
0.5 +
1.25
s
-
1.25
s
-
1.25
s
1 + 0.1s +
1.25
s

I
s1
I
L
=
V
s1
-
v
C
(0)
s
v
C
(0)
s
+ Li
L
(0) + I
s2
(b) With V
s1
=3/s, I
s2
=3/s, v
C
(0) =0, and i
L
(0) =3 A, the above mesh equation becomes
0.5 +
1.25
s
-
1.25
s
-
1.25
s
1 + 0.1s +
1.25
s

I
s1
I
L
=
V
s1
-
v
C
(0)
s
v
C
(0)
s
+ Li
L
(0) + I
s2
=
3
s

0.3 +
3
s
Solve for I
L
(s) by Cramer's rule to obtain
I
L
(s) =
0.15s
2
+1.875s + 7.5
0.05s
3
+ 0.625s
2
+ 1.875s
=
2
s + 7.5
+
-3
s + 5
+
4
s

Therefore
i
L
(t) = (4 + 2e
-7.5t
-3 e
-5t
) u(t) A
SOLUTI ON 14.49. (a) Represent the initialized capacitors by their parallel equivalent circuits.
(b) Write two nodals equation by inspection
0.001s + 0.4 - 0.2
- 0.2 0.001s + 0.4

V
C1
V
C2
=
0.2V
in
+ 0.001v
C1
(0
-
)
0.001v
C2
(0
-
)
=
2.4
s
+ 0.006
0.002
(c) Solve for V
C2
(s) by Cramer's rule to obtain
V
C2
=
0.001s + 0.4 2.4/s + 0.006
- 0.2 0.002
0.001s + 0.4 - 0.2
- 0.2 0.001s + 0.4
=
2s
2
+ 210
3
s + 4810
4
s(s
2
+ 810
2
s + 1210
4
)
=
0
s + 600
+
-2
s + 200
+ +
4
s

(d)
2/23/02 page P14.12 R. A. DeCarlo, P. M. Lin
v
C2
(t) = (4 - 2e
-200t
) u(t) V
SOLUTI ON 14.50. (a) Let VC denote the node voltage across the capacitor. By inspection the nodal
equations in matrix form are:
1+1 /R + 4s 1 /R
1 /R 1 +1/ R +1 / ( 4 s)



1
]
1
V
C
V
out



1
]
1

V
in
V
in
/ ( 4s)



1
]
1
(b) By Cramer's rule,
H(s)
V
out
(s)
V
in
(s)

det
1 +1/ R + 4s 1
1 /R 1 / ( 4s)



1
]
1
det
1+1 /R + 4s 1 /R
1 /R 1 +1/ R +1 / ( 4 s)



1
]
1

( 4s +1)(1 +1/ R)
4s (1+1/ R + 4s)(1+ 4s / R + 4s) 1/ R
2
( )

(4s +1)(1 +1/ R)


(1 + 8s +16s
2
)(1+1 /R)

1
(1+ 4s)
Clearly, R does not affect the transfer function. The question is why? Note that the circuit can be
redrawn as a balanced Wheatstone bridge circuit in which there is no voltage across R and no current
through R. Hence R has no effect on the transfer function and on the impedance at the input. Hence R
can be removed in the analysis of the circuit. In this case, the transfer function follows trivially by
voltage division.
(c) In view of the answer to (b), the impedance can be calculated with R removed. Hence
Z
in
(s)
1+
1
4s


_
,
1+ 4s ( )
1+
1
4s


_
,
+ 1+ 4s ( )

1 + 4s ( )
2
1 + 4s ( )
2
1
Hence, the input impedance is a constant resistance and the network is called a constant resistance
network.
(d) The input is v
in
(t) =10e
at
u(t) V and R =5 . Find v
out
(t) for t 0 for the three cases, a =0,
0.5, 0.25.
(d) From part (b), for s 0.25,
V
out
(s)
0.25
(s + 0.25)

10
s + a

2. 5/ ( 0. 25 a)
s + a

2. 5/ ( 0. 25 a)
(s + 0.25)
which leads to
v
out
(t)
2.5
0.25 a


_
,
e
at
e
0.25t
( )
u(t) V
For a =0.25,
2/23/02 page P14.13 R. A. DeCarlo, P. M. Lin
V
out
(s)
2.5
(s + 0.25)
2
implying that v
out
(t) 2.5te
0.25t
u(t) V
SOLUTI ON 14.51.
(a) This bridged-T circuit was analyzed in problem 14.9. Here R =1 , Z
1
(s) =0.25s and Z
2
(s) =4/s.
Since the condition Z
1
(s) Z
2
(s) =R
2
is met, we have Z
in
(s)=1.
(b) The s-domain equivalent circuit accounting for initial conditions is given below.
(c) Two nodal equations at V
c
and V
out
are:
V
C
- V
in
+ V
C
- V
out
+ 0.25sV
C
= 0.25 v
C
(0
-
)
and
V
out
- V
C
+
V
out
- V
in
0.25s
+ V
out
=
i
L
(0
-
)
s

Writing these in matrix form, we have
0.25s + 2 -1
-1
4
s
+ 2

V
C
V
out
=
V
in
+ 0.25v
C
(0
-
)

4V
in
s
+
i
L
(0
-
)
s
Solving for V
out
by Cramer's rule yields
2/23/02 page P14.14 R. A. DeCarlo, P. M. Lin
V
out
(s) =
4
s+4
V
in
(s) +
0.5(s+ 8)
(s+4)
2

i
L
(0
-
) +
0.5s
(s+4)
2

v
C
(0
-
)
(d) Given v
in
(t) =4u(t) - 3e
-t
u(t) V, then
V
in
(s) =
4
s
-
3
s+1
=
s + 4
s( s + 1)
and
V
out
(s) =
4
s(s + 1)
+
0.25(s+ 8)
(s+4)
2

+
0.75s
(s+4)
2


Taking the inverse Laplace transform, we obtain, for t 0,
v
out
(t) = (4 - 4 e
-t
) + (0.25 e
-4t
+ te
-4t
) + (0.75 e
-4t
- 3 te
-4t
) = 4 - 4 e
-t
+ e
-4t
- 2te
-4t

V
SOLUTI ON 14.52. A supernode is defined by drawing a curve to enclose the controlled voltage source.
One node within the supernode has voltage V
out
and the other has voltage V
1
that is equal to
V
1
= -2I
1
- V
out
= -2
V
in
- V
C
2
= - V
in
+ V
C
- V
out
Next, we write nodal equations at V
C
and the supernode: At node V
C
0.5 (V
C
- V
in
) + 0.5sV
C
+
0.5
s
(V
C
- V
out
) = 0
At the supernode

1
2s
(V
out
- V
C
) +
1
2
V
out
+
- V
in
+ V
C
+ V
out
1
= 0
In matrix form, the nodal equations are:
0.5(s + 1 + 1/s) -0.5/s
1 - 0.5/s 1.5 +0.5/s

V
C
V
out
=
0.5 V
in
V
in
Solving by Cramer's rule yields
H(s) =
V
out
V
in
=
0.5s + 0.75/s
0.75s + 1+ 1.5/s
=
2( s
2
+ 1.5)
3s
2
+ 4s + 6
SOLUTI ON 14.53. A supernode is defined by drawing a curve to enclose the controlled voltage source.
One node within the supernode has voltage V
out
and the other has voltage V
1
which is equal to
2/23/02 page P14.15 R. A. DeCarlo, P. M. Lin
V
1
= -2I
1
- V
out
= -2
V
in
- V
C
2
= - V
in
+ V
C
- V
out
Next, write nodal equations at V
C
and the supernode: At node V
C
0.5 (V
C
- V
in
) + (0.25s +
4
s
)V
C
+
s
s
2
+ 16
(V
C
- V
out
) = 0
At the supernode

s
s
2
+ 16
(V
out
- V
C
) +
1
2
V
out
+
- V
in
+ V
C
+ V
out
1
= 0
In matrix form, the nodal equation are:
0.25s + 0.5 +s/( s
2
+ 16) -s/(s
2
+ 16)
1 - s/(s
2
+ 16) 1.5 + s/(s
2
+ 16)

V
C
V
out
=
0.5 V
in
V
in
Solving by Cramer's rule yields
H(s) =
V
out
V
in
=
0.25s
3
+5.5s
0.375s
3
+ s
2
+ 9s + 12

SOLUTI ON 14.54. Write nodal equations at V
1
and V
2
:
0.5 (V
1
- V
in
) + 0.125( V
1
+ 0.2V
2
) + 0.1s(V
1
- V
2
) = 0
and
0.1s(V
2
- V
1
) +
1
4
V
2
+
V
2
1
- 5V
1
= 0
In matrix form, the nodal equations are:
(0.1s + 0.625) (-0.1s + 0.025)
(-0.1s - 5) (0.1s + 1.25)

V
C
V
out
=
0.5 V
in
0
Solving by Cramer's rule yields
H(s) =
I
out
V
in
=
V
2
V
in
=
0.05s + 2.5
- 0.31s + 0.9062

SOLUTI ON 14.55.
(a) Simply replace each capacitor by the parallel form circuit model given in figure 14.16.
(b) For this passive circuit, we can write the nodal equation by inspection.
2/23/02 page P14.16 R. A. DeCarlo, P. M. Lin
0.5s +2 -1 0
-1 0.5s +2 -1
0 -1 0.5s +2

V
C1
V
C2
V
C3
=
V
in
+ 0.5v
C1
(0)
0.5v
C2
(0)
0.5v
C3
(0)
Solving for V
C3
by Cramer's rule yields
V
C3
(s) =
V
in
(s) + 0.5v
C1
(0) + 0.25s + 1 v
C2
(0) +( 0.125s
2
+ s +1.5 )v
C3
(0)
0.125s
3
+ 1.5s
2
+ 5s + 4

(c) Substituting V
in
(s) =12/s, v
C1
(0) =0, v
C2
(0)=6, and v
C3
(0) =2 into the above expression, we
obtain
V
C3
(s) =
12/s + 6 0.25s + 1 + 2( 0.125s
2
+ s +1.5 )
0.125s
3
+ 1.5s
2
+ 5s + 4
=
0.25s
3
+ 3.5s
2
+ 9s +12
s 0.125s
3
+ 1.5s
2
+ 5s + 4

Now use MATLAB to do the partial fraction expansion.
n=[ 0.25 3.5 9 12];
d=[ 0.125 1.5 5 4 0];
[ r p k ] =residue (n,d)
r =
-2.5000
4.0000
-2.5000
3.0000
p =
-6.8284
-4.0000
-1.1716
0
From the MATLAB output, we have, for t 0,
v
C3
(t) = 3 - 2.5 e
-6.828t
+ 4e
-4t
- 2. 5e
-1.1716t
V
SOLUTI ON 14.56. For this problem we utilize loop analysis with loops as indicated below.
2/23/02 page P14.17 R. A. DeCarlo, P. M. Lin
In doing the following loop analysis, note that we will use g
m
V
out
I
3
and that due to our judicious
choice of loops
V
out

1
Cs
+ Ls


_
,
I
1
or equivalently,
0
1
Cs
+ Ls


_
,
I
1
V
out
For loop 1,
V
1
2R +
1
Cs
+ Ls


_
,
I
1
+ 2R I
2
+ Rg
m
V
out
For loop 2,
V
1
V
2
2R I
1
+ 7R I
2
+ 5Rg
m
V
out
In Matrix form
1000
s
+ 0.016s 0 1
2 +
1000
s
+ 0.016s 2 2
2 7 10








1
]
1
1
1
1
1
1
I
1
I
2
V
out





1
]
1
1
1

0
V
1
V
1
V
2





1
]
1
1
1
By Cramer's rule
2/23/02 page P14.18 R. A. DeCarlo, P. M. Lin
V
out

det
1000
s
+ 0.016s 0 0
2 +
1000
s
+ 0.016s 2 V
1
2 7 V
1
V
2








1
]
1
1
1
1
1
1
det
1000
s
+ 0.016s 0 1
2 +
1000
s
+ 0.016s 2 2
2 7 10








1
]
1
1
1
1
1
1

1000
s
+ 0.016s


_
,
(5V
1
+ 2V
2
)
6000
s
+ 0.096s 10 +
7000
s
+ 0.112s


_
,
Hence
V
out

1000
s
+ 0.016s


_
,
(5V
1
+ 2V
2
)
10 +
1000
s
+ 0.016s

s
2
+ 62500
s
2
+ 625s + 62500
(5V
1
+ 2V
2
)
The answers to (a) and (b) are clear at this point.
(c) Using MATLAB
n =[21 0 21*62500];
d =[1 625 62500 0];
[r,p,k] =residue(n,d)
r =
35
-35
21
p =
-500
-125
0
k = []
Hence
v
out
(t) = 21 35e
125t
+ 35e
500t
( )
u t ( ) V.
SOLUTI ON 14.57. (a) Replace the LC combination by a 1 V source after setting V
1
and V
2
to zero.
We need to compute the current leaving the 1 V source which will be 1/R
th
. Let the left node be denoted
by V
a
and the right node by V
b
. Also let G =1/R. The nodal equations are by inspection
2G 0
0 1.25G



1
]
1
V
a
V
b



1
]
1

g
m
+ G
g
m
+ 0.25G



1
]
1

V
a
V
b



1
]
1

g
m
+ G ( ) 2G
g
m
+ 0.25G ( ) 1.25G




1
]
1
1
2/23/02 page P14.19 R. A. DeCarlo, P. M. Lin
Thus, the current leaving the 1 V source is
I
1V
G 1V
a
( ) + 0.25G 1 V
b
( ) G 1 g
m
+ G ( ) 2G ( ) + 0.25G 1 g
m
+ 0.25G ( ) 1.25G ( )
0.5G 0.5g
m
+ 0.25G + 0.2g
m
0.05G 0.7G 0.3g
m
Substituting G =1/R we obtain
R
th

1
I
1V

1
0.7G 0.3g
m

R
0.7 0.3g
m
R

1
0.7 0.3 2
10
(b) Replace the LC combination by a short circuit and compute Isc. This makes the controlled source
zero. By inspection
I
sc

V
1
2R
+
V
2
5R
Thus
V
oc
R
th
I
sc
R
th
V
1
2R
+
V
2
5R


_
,
10 0.5V
1
+ 0.2V
2
( )
(c) By voltage division
V
out

Z
LC
R
th
+ Z
LC
V
oc

1
Cs
+ Ls
1
Cs
+ Ls + R
th
V
oc

s
2
+1 LC
s
2
+
R
th
L
s +1 LC
5V
1
+ 2V
2
( )

s
2
+ 62500
s
2
+ 625s + 62500
5V
1
+ 2V
2
( )
SOLUTI ON 14.58. (a) The last equation is the constraint equation for the controlled floating voltage
source. Hence, we have
V
1
V
2
z
0
(s)I
0
0
(b) By Cramer's rule,
V
2

det
1
R
+ Cs


_
,
I
in
1
0 0 1
1 0 z
0
(s)







1
]
1
1
1
1
1
det
1
R
+ Cs


_
,
1 R 1
0 Cs 1
1 1 z
0
(s)







1
]
1
1
1
1
1

I
in
2
1
R
+ Cs


_
,
z
0
(s)Cs
1
R
+ Cs


_
,

I
in
1
R
+ Cs


_
,
2 + z
0
(s)Cs ( )
(c) Here
2/23/02 page P14.20 R. A. DeCarlo, P. M. Lin
V
2

I
in
1
R
+ Cs


_
,
2 + LCs
2
( )
in which case
2
LC
.
SOLUTI ON 14.59. (a) Since the switch has been at position A for a very long time, the inductor looks
like a short and i
L
(5
-
) =i
L
(5
+
) = 10/4 =2.5 A. For t >5, the switch moves to position B and the
inductor current decays according to
i
L
(t) i
L
(5
+
)e
(t 5)/
2.5e
(t 5)/0.1
2.5e
10(t 5)
A
(b) Note that i
L
(0
-
) =i
L
(0
+
) =0. Hence
I
L
(s)
1
10s + 4
V
in
(s)
0.1
s + 0.4
50
s

50
s + 0.5


_
,

12.5
s

62.5
s + 0.4
+
50
s + 0.5
Hence for 0 t 5s, i
L
(t) 12.5 62.5e
0.4t
+ 50e
0.5t
A. Here i
L
(5
-
) =i
L
(5
+
) = 8.1458 A.
For t >5, the inductor decays with a time constant of 0.1 s. Thus
i
L
(t) 8.1458e
10(t 5)
SOLUTI ON 14.60. (a) Since the switch has been at position A for a very long time, the capacitor
looks like an open and v
C
(5
-
) =v
C
(5
+
) = 40 V. For t >5, the switch moves to position B and the
capacitor voltage decays according to
v
C
(t) v
C
(5
+
)e
(t 5)/
40e
(t 5)/2
40e
0.5(t 5)
V
(b) Note that v
C
(0
-
) =v
C
(0
+
) =0. Hence
V
C
(s)
1 /Cs
1 /Cs + 40
V
in
(s)
12.5
s +12.5
50
s

50
s +12.5


_
,

7812.5
s s +12.5 ( )
2
In MATLAB,
syms t s
ilaplace(7.8125e3/(s*(s+12.5)^2))
ans =
50-625*t*exp(-25/2*t)-50*exp(-25/2*t)
Hence for 0 t 5s, v
C
(t) 50 625te
12.5t
50e
12.5t
V. Here v
C
(5
-
) =v
C
(5
+
) = 50 V.
For t >5, the capacitor voltage decays with a time constant of 0.08 s. Thus
v
C
(t) 50e
12.5(t 5)
V
2/23/02 page P14.21 R. A. DeCarlo, P. M. Lin
SOLUTI ON 14.61. (a) Since the switch has been closed for a very long time, the capacitor looks like
an open and v
C
(5
-
) =v
C
(5
+
) = 32 V. For t >5, the switch opens and the capacitor voltage decays
according to
v
C
(t) v
C
(5
+
)e
(t 5)/
32e
(t 5)/0.4
32e
2.5(t 5)
V
(b) Note that v
C
(0
-
) =v
C
(0
+
) =0 and v
out
=v
C
. Hence for 0 t 5,
V
C
(s)
1
1
50
+
1
200
+ 0.002s
V
in
(s)
50

10
s +12.5
V
in
(s)
10
s +12.5
50
s

50
s +12.5


_
,

6250
s(s +12.5)
2
In MATLAB,
syms t s
ilaplace(6250/(s*(s+12.5)^2))
ans =
40-500*t*exp(-25/2*t)-40*exp(-25/2*t)
Hence for 0 t 5s, v
C
(t) 40 500te
12.5t
40e
12.5t
V. Here v
C
(5
-
) =v
C
(5
+
) = 40 V.
For t >5, the capacitor voltage decays with a time constant of 0.4 s. Thus
v
C
(t) 40e
2.5(t 5)
V
SOLUTI ON 14.62. (a) At 0-, v
C
( 0
-
) =0 and i
L
(0
-
) =50/10 =5 A.
(b) For this part, consider the equivalent circuit below.
By inspection,
V
C

5
s 1+ 0.5s +
1
0.184s


_
,

5
0.5s
2
+ s +
1
0.184

10
s +1 ( )
2
+
2
From table 13.1,
2/23/02 page P14.22 R. A. DeCarlo, P. M. Lin
v
C
(t) 3.1831e
t
sin(t)u(t) V
(c) Using MATLAB,
SOLUTI ON 14.63. (a) Since the switch has been closed for a long time, i
L
(1
-
) =i
L
(1
+
) =30/0.8 =37.5 A
and v
C
(1
-
) =v
C
(1
+
) =0. Represent the initialized inductor by its parallel equivalent circuit. Then
V
C
(s)
37.5
s

1
Cs +
1
Ls

150
s
2
+ 4
Hence from table 13.1,
v
C
(t') 75sin(2 t') V v
C
(t) 75sin(2( t 1) ) V for t >1s.
(b) All initial conditions at t =0 are zero. For 0 t 1s,
V
C
(s)
1
1
0.8
+ 0.25s +
1
s
V
in
(s)
0.8

5s
s
2
+ 5s + 4
V
in
(s)
150s
s
2
+ 5s + 4
1
s

1
s + 2


_
,

300s
s
2
+ 5s + 4
1
s(s + 2)



_
,

100
s +1

150
s + 2
+
50
s + 4
Hence, for 0 t 1s,
v
C
(t) 100e
t
150e
2t
+ 50e
4t
V
Here, v
C
(1-) =v
C
(1+) =17.403 V. Note, v
in
(1-) =25.94 V. Next,
2/23/02 page P14.23 R. A. DeCarlo, P. M. Lin
i
C
(1

) C
dv
C
dt
1
]
1
t 1
0.25 100e
1
+ 300e
2
200e
4
[ ]
0.037378
Thus in MATLAB
vin1 =30*(1 - exp(-2))
vin1 = 2.5940e+01
vc1 =17.403
vc1 = 1.7403e+01
ic1 =0.25*(-100*exp(-1) +300*exp(-2)-200*exp(-4))
ic1 = 3.7378e-02
iL1 =(vin1 - vc1)/0.8 - ic1
iL1 = 1.0634e+01
Therefore, i
L
(1

) i
L
(1
+
) 10.634 A. For t 1, we use the parallel equivalent circuit for both the
inductor and the capacitor:
V
C
(s) e
s
4s
s
2
+ 4
i
L
(1
+
)
s
+ Cv
C
(1
+
)



_
,

e
s
4s
s
2
+ 4
10.634
s
+ 4.3507


_
,
Therefore from table 13.1, for t >1,
v
C
(t) 21.268sin 2(t 1) ( ) +17.403cos 2(t 1) ( ) V
Plots omitted.
SOLUTI ON 14.64. Here v
C
(0) =0 for both capacitors.
Part 1: 0 t 1s.
V
out
(s)
10
s
20 +
10
s
20
s

20
s + 2


_
,

20
s(s + 0.5)(s + 2)
From MATLAB
num =20;
den =[1 2.5 1 0];
[r,p,k] =residue(num,den)
r =
6.6667e+00
-2.6667e+01
2.0000e+01
p =
-2.0000e+00
-5.0000e-01
0
k = []
2/23/02 page P14.24 R. A. DeCarlo, P. M. Lin
Therefore for 0 t 1,
v
out
(t) 20 26.667e
0.5t
+ 6.667e
2t
( )
u(t) u(t 1) ( )
Part 2. 1 t. Here the initial condition for the right-most capacitor is v
out
(1

) v
out
(1
+
) 4.7281 V.
As above, the left-most capacitor has zero value at t =1s. Let us use the series equivalent circuit for the
right capacitor. Then,
I
C
(s) e
s
4.7281
s

1
10 +
20
s
e
s
0.47281
s + 2
Therefore,
V
out
(s)
10
s
I
C
(s) + e
s
4.7281
s



1
]
1
e
s
4.7281
s(s + 2)
+
4.7281
s



1
]
1
e
s
2.3641
s
+
2.3641
s + 2



1
]
1
and for t 1,
v
out
(t) 2.3641+ 2.3641e
2(t 1)
( )
u(t 1) V
SOLUTI ON 14.65. Assume the switch has been in position A for a long time. Both capacitors behave as
open circuits and both capacitors have initial voltages at t =0 of 10 V. For t 0, use the parallel
equivalent circuits for both capacitors and write nodal equations. Let the left capacitor have voltage
V
Ca
.
0.005s + 0.03 0.01
0.01 0.0025s + 0.01



1
]
1
V
Ca
V
out



1
]
1

0.005 10
0.0025 10



1
]
1

0.05
0.025



1
]
1
By Cramer's rule,
V
out
(s)
det
0.005s + 0.03 0.05
0.01 0.025



1
]
1
det
0.005s + 0.03 0.01
0.01 0.0025s + 0.01



1
]
1
10
s +10
s
2
+10s +16

4 0 /3
s + 2

10 / 3
s + 8
Therefore for t 0,
v
out
(t)
40
3
e
2t

10
3
e
8t
V
SOLUTI ON 14.66. (a) Z
in
(s) 2 +
1
0.5s +
2
s
2 +
2s
s
2
+ 4
.
(b) Here, the initial condition is zero and
2/23/02 page P14.25 R. A. DeCarlo, P. M. Lin
V
C
(s)
2s
s
2
+ 4
2 +
2s
s
2
+ 4

10
s

10
s
2
+ s + 4
syms t s
ilaplace(10/(s^2+s+4))
ans =
4/3*exp(-1/2*t)*15^(1/2)*sin(1/2*15^(1/2)*t)
Hence using MATLAB above or table 13.1 we have for 0 t 1.5s,
v
C
(t) 5.164e
0.5t
sin(1.9365t) V
and
v
C
(1.5

) v
C
(1.5
+
) 0.57237 V
(c) Use the parallel equivalent circuit for the left capacitor. The right capacitor has a zero initial voltage
at t =1.5. Hence, we do not use an equivalent circuit for the right capacitor.
(d) Therefore
e
1.5s
V
C
(s)
1
0.5s + 2 +
s
s + 4
0.5 0.57237 0.57237
s + 4
s
2
+10s +16
(e) In MATLAB
[r,p,k] =residue(0.57237*[1 4],[1 10 16])
r =V
3.8158e-01
1.9079e-01
p =
-8
-2
k =
[]
Hence
V
C
(s) e
1.5s
0.57237
s + 4
s
2
+10s +16
e
1.5s
0.19079
s + 2
+
0.38158
s + 8



1
]
1
(f) Finally
v
C
(t) 0.19079e
2(t 1.5)
+ 0.38158e
8(t 1.5)
[ ]
u(t 1.5) V
SOLUTI ON 14.67. (a) v
1
(0

) v
1
(0
+
) v
2
(0

) v
2
(0
+
)
R
2R
16 8 V.
(b) For 0 t 1, v
1
(t) 8 V and v
2
(t) 8e
t /RC
8e
0.6931t
V.
(c) v
1
(1

) 8 V and v
2
(1

) 8e
0.6931
4 V.
2/23/02 page P14.26 R. A. DeCarlo, P. M. Lin
(d) From KVL, v
1
(1
+
) v
2
(1
+
). From conservation of charge,
1 v
1
(0

) +1 v
2
(0

) 12 2 v
1
(0
+
) 2 v
2
(0
+
) . Therefore v
1
(1
+
) v
2
(1
+
) 6 V.
(e) This response represents a decay with time constant =2R =2.8854 s. Hence
v
1
(t) v
2
(t) 6e
0.34657( t 1)
u(t 1) V
It follows that
v
1
(3) v
2
(3) 6e
0.346572
3 V
(f) Both capacitor voltages change abruptly at t =1.
SOLUTI ON 14.68. Label the current down through the first inductor as i
1
(t).
(a) i
1
(0

) i
1
(0
+
) 1 A a n d i
out
(0

) i
out
(0
+
) 0.
(b) For 0 t we use a parallel equivalent for the first inductor. By current division
I
out
(s)
1
5 + 0.1s
0.057143 +
1
0.35s
+
1
5 + 0.1s

1
s

175
s
2
+ 275s + 2500
Use MATLAB to do the partial fraction expansion
num =-175; den =[ 1 275 2500]; [ r, p, k] =residue (num, den)
r =
0.6831
-0.6831
p =
-265.5869
-9.4131
From the MATLAB output
I
out
(s) = -
0.68313
s+ 9.4131
+
0.68313
s+ 265.59

Therefore ,
i
out
(t) 0.68313 e
265.59t
e
9.413t
( )
u(t) A
SOLUTI ON 14.69. (a) Here we use voltage division:
2/23/02 page P14.27 R. A. DeCarlo, P. M. Lin
V
1
(s)
1
4 10
6
s
1
4 10
6
s
+
1
4 10
6
s
+
1
110
6
s

30
s

30
6s

5
s
Therefore, v
1
(0
+
) 5 V.
(b) Again use voltage division:
V
1
(s)
1
5 10
6
s
1
5 10
6
s
+
1
110
6
s
+
1
2 10
6
s

40
s

80
17s
Therefore, v
1
(0
+
) 4.7059 V.
SOLUTI ON 14.70. (a) Consider a mesh current I(s) in the usual direction and use the series equivalent
circuit for each capacitor. Thus
I(s)
1
1
4 10
6
s
+
1
4 10
6
s
+
1
110
6
s
3
s

0.3
s

0.9
s

0.6
s


_
,

1.2
1.5
10
6
0.8 10
6
Therefore for t >0,
V
1
(s)
1
4 10
6
s
I(s) +
0.3
s

0.8 10
6
4 10
6
s
+
0.3
s

0.5
s
v
1
(t) 0.5 V
Similarly for t >0
V
2
(s)
0.8 10
6
4 10
6
s
+
0.9
s

1.1
s
v
2
(t) 1.1 V
and
V
3
(s)
0.8 10
6
110
6
s
+
0.6
s

1.4
s
v
3
(t) 1.4 V
(b) Consider a mesh current I(s) in the usual direction and use the series equivalent circuit for each
capacitor. Thus
I(s)
1
1
5 10
6
s
+
1
110
6
s
+
1
2 10
6
s
4
s

0.3
s

0.9
s

0.6
s


_
,

2.2
1.7
10
6
1.294110
6
Therefore for t >0,
V
1
(s)
1
5 10
6
s
I(s) +
0.3
s

1.294110
6
5 10
6
s
+
0.3
s

0.55882
s
v
1
(t) 0.55882 V
Similarly for t >0
2/23/02 page P14.28 R. A. DeCarlo, P. M. Lin
V
2
(s)
1.294110
6
110
6
s
+
0.9
s

2.1941
s
v
2
(t) 2.1941 V
and
V
3
(s)
1.294110
6
2 10
6
s
+
0.6
s

1.2471
s
v
3
(t) 1.2471 V
SOLUTI ON 14.71. (a) For 0 <t <2, the 150 mF capacitor is charged to 25 V. From conservation of
charge,
0.15 25 0.15v
C
(2
+
) + 0.1v
C
(2
+
) 0.25v
C
(2
+
)
Therefore
v
C
(2
+
)
0.15 25
0.25
15 V
This voltage remains constant for t >2 s.
(b) For 0 <t <2, the 150 mF capacitor is charged to 25 V. From conservation of charge,
0.15 25 + 0.110 0.15v
C
(2
+
) + 0.1v
C
(2
+
) 0.25v
C
(2
+
)
Therefore
v
C
(2
+
)
0.15 25 + 0.110
0.25
19 V
This voltage remains constant for t >2 s.
SOLUTI ON 14.72. (a) Let the middle node have voltage V
a
(s). Then writing node equations
8s 2s
2s 6s



1
]
1
V
a
V
out



1
]
1

1.144
0



1
]
1

V
a
V
out



1
]
1

1
44
6 2
2 8



1
]
1
1.144 s
0



1
]
1

0.156 / s
0.052 / s



1
]
1
V
Thus for t >0, v
out
(t) 0.052 V.
(b) Again define V
a
(s) as the middle node. Then
V
a
(s)
3
10s
1
10s
+
1
4s
+
3
10s

0.286
s

6
13

0.286
s

0.132
s
Hence for t >0,
V
out
(s)
1
4s
1
2s
+
1
4s

0.132
s

0.132
3s

0.044
s
v
out
(t) 0.044 V
2/23/02 page P14.29 R. A. DeCarlo, P. M. Lin
SOLUTI ON 14.73. With switches in position A, the equivalent capacitance to the right of v
2
is 4 mF.
Therefore at t =0+, by voltage division
V
1
(s) V
2
(s)
10
s
v
1
(t) v
2
(t) 10 V for 0 <t <1.
Hence with the switches in position B, let us write a single node equation using the parallel equivalent
circuit the initialized capacitors:
0.002sV
2
(s) 0.002 10 + 0.004sV
2
(s) + 0.004 10 0.004s
11
s
Equivalently
0.006sV
2
(s) 0.044 0.02 0. 024 V
2
(s) 4 / s
Hence, for t >1 s, v
2
(t) 4 V and v
1
(t) 7 V.
SOLUTI ON 14.74. (a) At t =0+, the frequency domain equivalent circuit is given below.
To compute C
eq
, we observe
C
eq
1+
1
1
3
+
1
2
+
1
6
2 mF
By voltage division
V
1

C
eq
s
C
eq
s + 0.002s

20
s

10
s
and V
2

0.002s
C
eq
s + 0.002s

20
s

10
s
Hence, for 0 <t <1, v
1
(t) =v
2
(t) =10 V.
(b) When the switch moves to B, the pertinent part of the equivalent frequency domain circuit is given
below.
2/23/02 page P14.30 R. A. DeCarlo, P. M. Lin
By superposition,
V
1

1
1 + 2

8
s
+
1
0.003s
0.02
1
0.003s
0.01
6
s
Hence, for 1 <t <2, v
1
(t) =6 V and by KVL, v
2
(t) =2 V.
SOLUTI ON 14.75. When the switch in position A, the 2 F capacitor is charged to 2 V. Hence, the
charge on the top plate is C*v
C
=4 C. When the switch is moved to position B, due to the virtual
ground, the 2 F capacitor voltage is zero meaning it cannot retain any charge. Hence, assuming an
ideal op amp, all charge moves to the 1 F capacitor with 4 C on the left plate. Hence, v
out
=(4
C)/1 F =4 V.
SOLUTI ON 14.76. (a) v
out
(t) =0 for 0 <t <1 ms. Every time the switch moves to position A, the
capacitor, C, charges to 8 V. When the switch moves to position B, because of the virtual ground, all
charges moves to the kC capacitor. Hence with k =1, v
out
(t) = 8 V for 1 ms <t <3 ms. For 3 ms <t <
5 ms, v
out
(t) = 16 V. Repeating the pattern implies that for 5 ms <t <7 ms, v
out
(t) = 24 V, etc. See
for example figure 14.51.
(b) With k =0.5, the voltages computed in part (a) double.
(c) With k =2, the voltages computed in part (a) are halved.
SOLUTI ON 14.77.
(a) For this part consider the circuit below.
2/23/02 page P14.31 R. A. DeCarlo, P. M. Lin
For the normalized design C = 1 F, G
1
= 0.5 S, G
2
= 2 S, and G
3
= 1.5 S. After magnitude scaling with
K
m
= 10
6
, then C 1 F, R
1
2 M, R
2
5 0 0 k , R
3
666. 7 k .
(b) For this part, consider the circuit below.
For the normalized design C = 1 F, G
1
= 1 S, G
3
= 2 S, G
2
= 0.5 S, and G
4
= 2.5 S. After magnitude
scaling with K
m
= 10
6
, then C 1 F, R
1
1 M, R
3
5 0 0 k , R
2
2 M, R
4
4 0 0 k .
(c) Now consider the circuit below.
2/23/02 page P14.32 R. A. DeCarlo, P. M. Lin
For the normalized design C = 1 F, G
1
= 0.25 S, G
3
= 0.5 S, G
2
= 0.75 S, G
4
= 1 S, and G
5
= 1 S. After
magnitude scaling with K
m
= 10
6
, then C 1 F, R
1
4 M, R
3
2 M, R
2
4 / 3 M,
R
4
1 M, R
5
1 M.
(d) Finally, we consider the following circuit.
For the normalized design C = 1 F, G
1
= 2 S, G
3
= 1.5 S, G
6
= 1 S, G
2
= 0.5 S, G
4
= 2 S, and G
5
= 2 S.
After magnitude scaling with K
m
= 10
6
, then C 1 F, R
1
0 . 5 M, R
3
2 /3 M,
R
6
1 M, R
2
2 M, R
4
0 . 5 M, R
5
0 . 5 M.
2/23/02 page P14.33 R. A. DeCarlo, P. M. Lin
SOLUTI ON 14.78. With V
1
=V
out
a prototype design is given by the topology below.
Using MATLAB
Km =1e7;
Gin =1; G1 =1.405; G2 =0.402; G3 =0.942;
DG =1.865;
Rinnew = Km/Gin
Rinnew =
10000000
R1new =Km/G1
R1new =
7.1174e+06
R2new =Km/G2
R2new =
2.4876e+07
R3new =Km/G3
R3new =
1.0616e+07
DRnew =Km/DG
DRnew =
5.3619e+06
Later, when we study frequency scaling, K
m
will be smaller and the filter will have a cutoff frequency in
a more reasonable range.
SOLUTI ON TO 14.79. Note corrections to problem statement. W(0+) should be W() in part (b) and in
part (c) one should calculate W(0-) W(). The frequency domain equivalent circuit is given by the
figure below.
2/23/02 page P14.34 R. A. DeCarlo, P. M. Lin
(a) To find the required time functions, we first find their Laplace equivalents.
I(s)
a
s

b
s
R +
1
C
1
s
+
1
C
2
s

a b
R
s +
1
RC
1
+
1
RC
2



_
,

K
1
s + p
1
in which case i(t) K
1
e
p
1
t
u(t) where
K
1

a b
R
and p
1

1
RC
1
+
1
RC
2



_
,

.
Further,
V
C1
(s)
I(s)
C
1
s
+
a
s

a
s

K
1
C
1
s s + p
1
( )
a
K
1
C
1
p
1



_
,

1
s
+
K
1
C
1
p
1

1
s + p
1
( )
in which case
v
C1
(t) a
K
1
C
1
p
1



_
,

+
K
1
C
1
p
1
e
p
1
t




1
]
1
1
u(t)
Also, by symmetry,
V
C2
(s)
I(s)
C
2
s
+
b
s

b
s
+
K
1
C
2
s s + p
1
( )
b +
K
1
C
2
p
1



_
,

1
s

K
1
C
2
p
1

1
s + p
1
( )
in which case
v
C2
(t) b +
K
1
C
2
p
1



_
,

K
1
C
2
p
1
e
p
1
t




1
]
1
1
u(t)
(b) The total energy stored in the capacitors at time 0- is
W(0

)
1
2
C
1
a
2
+
1
2
C
2
b
2
Also at t =,
v
C1
() a
K
1
C
1
p
1



_
,

and v
C2
() b +
K
1
C
2
p
1



_
,

.
Hence
2/23/02 page P14.35 R. A. DeCarlo, P. M. Lin
W()
1
2
C
1
a
K
1
C
1
p
1



_
,

2
+
1
2
C
2
b +
K
1
C
2
p
1



_
,

2
(c)
Ri
2
(t)dt
0

RK
1
2
e
2 p
1
t
dt
0

RK
1
2
2p
1

(a b)
2
2Rp
1

(a b)
2
2
1
C
1
+
1
C
2



_
,

Observe that
W(0

) W ()
1
2
C
1
a
2
+
1
2
C
2
b
2

1
2
C
1
a
K
1
C
1
p
1



_
,

2

1
2
C
2
b +
K
1
C
2
p
1



_
,

2

aK
1
p
1

bK
1
p
1

1
2
K
1
2
C
1
p
1
2

1
2
K
1
2
C
2
p
1
2

(a b)
2
Rp
1

1
2

(a b)
2
Rp
1

(a b)
2
2Rp
1

(a b)
2
2
1
C
1
+
1
C
2



_
,

This indicates that the total energy lost between 0- and infinity is the energy dissipated in the resistor
and the result is independent of the value of R.
(d) When R 0,
lim
R0
I(s)
a
s

b
s
1
C
1
s
+
1
C
2
s

a b
1
C
1
+
1
C
2

C
1
C
2
(C
1
+C
2
)
(a b)
Therefore
i(t)
C
1
C
2
(C
1
+C
2
)
(a b) (t)
Further
V
C1
(s)
C
2
s(C
1
+C
2
)
(a b) +
a
s

C
1
s(C
1
+C
2
)
a +
C
2
s(C
1
+C
2
)
b
and
V
C2
(s)
C
1
s(C
1
+C
2
)
(a b) +
b
s

C
1
s(C
1
+C
2
)
a +
C
2
s(C
1
+C
2
)
b
Therefore
v
C1
(t) v
C2
(t)
C
1
(C
1
+C
2
)
a +
C
2
(C
1
+C
2
)
b



_
,

u(t)
SOLUTI ON TO 14.80. (a) From conservation of charge
C
1
v
C1
(0

) + C
2
v
C2
(0

) C
1
v
C1
(0
+
) + C
2
v
C2
(0
+
) C
1
+ C
2
( )v
C1
(0
+
) C
1
+ C
2
( )v
C2
(0
+
)
Therefore
2/23/02 page P14.36 R. A. DeCarlo, P. M. Lin
v
C1
(0
+
) v
C2
(0
+
)
C
1
v
C1
(0

) + C
2
v
C2
(0

)
C
1
+ C
2
( )
(b) Inserting values into out answer for part (a) yields
v
C1
(0
+
) v
C2
(0
+
)
C
1
C
1
+ C
2
( )
0.5 V
and the voltage remains the same for t >0.
(c) Before the switch is closed, the energy in C
2
is zero and the energy in C
1
is the total stored energy:
W
tot
(0

) W
C1
(0

) 0.5C
1
v
C1
2
(0

) 0.5 J
After the switch is closed,
W
tot
(0
+
) W
C1
(0
+
) + W
C2
(0
+
) 0.5C
1
v
C1
2
(0
+
) + 0.5C
2
v
C2
2
(0
+
) 0.25 J
(d) (i) Using the series equivalent circuit for C
1
, we have
I(s)
1
R +
2
s

1
s

1 /R
s + 2 / R
i(t)
1
R
e
2t R
u(t) A
Thus
V
C2
(s)
I(s)
s

1/ R
s(s + 2 / R)

0.5
s

0.5
s + 2 / R
v
C2
(t) 0.5 1 e
2t R
( )
u(t) V
and
V
C1
(s)
I(s)
s
+
1
s

1/ R
s(s + 2 / R)
+
1
s

0.5
s
+
0.5
s + 2 / R
v
C1
(t) 0 . 5 1+ e
2t R
( )
u(t) V
(ii) The energy dissipated in the resistor is given by
W
R
(0,) R i
2
( )d
0

1
R
e
4t R
d
0


e
4t R
4
1
]
1
1
0

1
4
J
(iii) For all R,
Area under i(t) i( )d
0

1
R
e
2t R
d
0


e
2t R
2
1
]
1
1
0

1
2
Further, as R 0, i(t)
1
R
e
2t R
u(t) has a decay that becomes infinitely fast and its magnitude (1/R)
approaches . Thus we have infinite height, zero-width, but a finite area of 0.5. Thus as R 0,
i(t) 0.5 (t) A. (We have avoided a more rigorous explanation as the above argument is more
plausible to sophomores.). As R 0, the exponential terms in the expressions for v
C1
(t) and v
C2
(t)
have infinitely fast decays and hence disappear from the expressions yielding the stated result.
2/23/02 page P14.37 R. A. DeCarlo, P. M. Lin
SOLUTI ON TO 14.81.
(a)
Z
in
(s) =
2s + 4.5
(s+ 0.5)(s + 4)
=
1
s+ 0.5
+
1
s + 4
= Z
a
(s) + Z
b
(s)
Y
a
(s) =
1
Z
a
(s)
= s+ 0.5
Y
b
(s) =
1
Z
b
(s)
= s + 4
From the above expressions, the RC circuit consists of a series connection of (a 1 farad capacitor in
parallel with a 2 resistor) and (a 1 F capacitor in parallel with a 0.25 resistor).
(b)
Y
in
(s) =
12s + 440
(s+ 120)(s + 20)
=
10
s+ 120
+
2
s + 20
= Y
a
(s) + Y
b
(s)
Z
a
(s) =
1
Y
a
(s)
= 0.1s + 12
Z
b
(s) =
1
Y
b
(s)
= 0.5s + 10
From the above expressions, the RL circuit consists of a parallel connection of (0.1 H inductor in series
with a 12 resistor) and (a 0.5 H inductor in series with a 10 resistor).
(c) Following the hint, we have
Y
in
(s)
s
=
0.225s

+0.075
(s+ 0.2)(s + 0.5)
=
0.1
s+ 0.2
+
0.125
s + 0.5

Hence
Y
in
(s) =
0.1s
s+ 0.2
+
0.125s
s + 0.5
= Y
a
(s) + Y
b
(s)
Z
a
(s) =
1
Y
a
(s)
=
s + 0.2
0.1s
= 10 +
1
0.5s

Z
b
(s) =
1
Y
b
(s)
=
s + 0.5
0.125s
= 8 +
1
0.25s

From the above expressions, we see that each term in Y
in
(s) represents a series RC circuit. The RC
circuit for Y
in
(s) consists of a parallel connection of (a 0.5 F capacitor in series with a 10 resistor)
and (a 0.25 F capacitor in series with a 8 resistor).
(d) CORRECTI ON: for part (d),change the second term to 2s/(s
2
+2).
2/23/02 page P14.38 R. A. DeCarlo, P. M. Lin
Y
in
(s) =
0.5s
s
2
+ 1
+
2s
s
2
+ 2
= Y
a
(s) + Y
b
(s)
Z
a
(s) =
1
Y
a
(s)
=
s
2
+ 1
0.5s
= 2s +
1
0.5s

Z
b
(s) =
1
Y
b
(s)
=
s
2
+2
2s
= 0.5s +
1
s

From the above expressions, we see that each term in Y
in
(s) represents a series LC circuit. The LC
circuit for Y
in
(s) consists of a parallel connection of (a 0.5 F capacitor in series with a 2 H inductor)
and (a 1 F capacitor in series with a 0.5 H inductor).
SOLUTI ON 14.82. CORRECTI ONS TO PROBLEM STATEMENT: (i) v
0
(t), should read v
out
(t)
and (ii) there should be a connection from the circuit inside the shaded box to the bottom line or
reference node.
(a) (i) 0 t <1 ms. Since the capacitor voltage is initially zero and the switch is in position (a), a
simple source transformation yields a Norton equivalent (seen by the capacitor) consisting of a 20 mA
current source in parallel with 9.8039 k resistor. Hence
V
out
(s)
1
1
98039
+10
6
s

.02
s

20 10
3
s s +102 ( )
Using MATLAB
n =.02*1e6;
p1 =1e6/Rth
p1 = 102
d =[1 p1 0];
[r,p,k] =residue(n,d)
r =
-1.9608e+02
1.9608e+02
p =
-102
0
k = []
Hence, for 0 t <1 ms, v
out
(t) 196.08 1 e
102t
( )
u(t) V. It follows that v
out
(1 ms) 19.014 V.
(ii) 1 ms t <1.05 ms. The frequency domain equivalent circuit is given below.
2/23/02 page P14.39 R. A. DeCarlo, P. M. Lin
Writing a single node equation yields
19.014 10
6

V
out

200
s
10000
+
V
out

1
s
10
+10
6
sV
out
Equivalently 19.014 +
1.2 10
5
s
s +100100 ( )V
out
or

V
out

19.014s +1.2 10
5
s s +100100 ( )
Therefore, v
out
(t') 1.1988 +17.815e
100100t'
( )
u(t') , and for 1 ms t <1.05 ms,
v
out
(t) v
out
(t 0.001) in which case
v
out
(t) 1.1988 +17.815e
100100(t 0.001)
( )
u(t 0.001)
Again using MATLAB,
(b) Part 1: for t >0 up to t
1
which denotes the time when v
out
(t) reaches 80 V, i.e., the capacitor is
charging. The frequency domain equivalent circuit is
2/23/02 page P14.40 R. A. DeCarlo, P. M. Lin
Using our knowledge of part (a), this circuit simplifies to
Hence
V
out
(s)
5 10
6
+
0.02
s
10
6
s +
1
9803.9

5s + 20 10
3
s s +102 ( )
and from MATLAB
syms s t
ilaplace((5*s+20e3)/(s^2+102*s))
ans =
10000/51-9745/51*exp(-102*t)
in which case
v
out
(t) 196.08 191.08e
102t
( )
u(t) V
From this expression,
v
out
(t
1
) 80 196.08 191.08e
102t
1
V
and
t1 =log((80-10000/51)/(-9745/51))/(-102)
t1 = 4.8864e-03
This part of the problem considers t
1
t <t
2
, i.e., the capacitor is discharging where v
out
(t
2
) =5.
The equivalent frequency domain circuit is given below which is a slight modification of the circuit of
(a)-(ii):
2/23/02 page P14.41 R. A. DeCarlo, P. M. Lin
Making use of our knowledge of part (a)-(ii), we have

V
out

80s +1.2 10
5
s s +100100 ( )
in which case v
out
(t') 1.1988 + 78.801e
100100t '
( )
u(t') V, and
v
out
(t) 1.1988 + 78.801e
100100(t t
1
)
( )
u(t t
1
)
Here t
2
'
3.0286 10
5
s and t
2
t
1
+ t
2
'
4.9167 ms where t
2
'
is the duration of the discharge cycle.
As a final point, note that the frequency of the sawtooth is 1/t2 =203.39 Hz. Finally a plot is given
below.
2/23/02 page P14.42 R. A. DeCarlo, P. M. Lin
SOLUTI ON 14.83. CORRECTI ON: In example 14.10, page 560, delete the four minus signs in the
equation for V
C
(s) and one more for v
C
(t).
We use MATLAB instead of SPICE to solve this problem. Applying voltage division to the circuit of
figure P14.83, we have
V
C
(s) =
1
Cs
1
Cs
+ Ls + R
Li
L
(0
-
) =
i
L
(0
-
)
C

1
s
2
+
R
L
s +
1
LC
=
10
8
s
2
+ 125s + 1.2510
9

From table 13.1, item 18
v
C
(t) = 2828e
-62.5t
sin(35,355t) u(t) V
A plot of v
C
(t) is given below with the vertical axis in V and the horizontal axis in seconds.
-3000
-2000
-1000
0
1000
2000
3000
0 0.2 0.4 0.6 0.8 1
x10-3
The waveform for the first few cycles is essentially the same as the example 14.10. Thus for the firs few
cycles, the lossless circuit of example 14.10 is a good approximation to the more accurate circuit model
of this problem. The effect of the presence of 100 resistance is a slow decay (with respect to ms
intervals) of the peak values.
1/25/02 P16-1 R. A. DeCarlo, P. M. Lin
PROBLEM SOLUTIONS
Solution 16.1.
(a) By the definition of the convolution integral
f
2
(t) f
2
(t) f
2
(t ) f
2
( )d

2u(t )2u( )d 4 u(t )d


0

The integrand, u(t ) , is nonzero only when t. This suggests that there are two regions of
consideration: t < 0 and t 0.
Case 1: t < 0. Here u(t ) 0 since is restricted to the interval [0,). Hence
f
2
(t) f
2
(t) 0 , for t < 0.
Case 2: t 0.
f
2
(t) f
2
(t) 4 u(t )d 4 d
0
t

4t , for t 0.
In sum,
f
2
(t) f
2
(t)
0, t < 0
4t, t 0

'

(b) By the definition of the convolution integral
f
2
(t) f
3
(t) f
2
(t ) f
3
( )d

2u(t )4e
2
u( )d 8 e
2
u(t )d
0

The integrand, u(t ) , is nonzero only when t. This suggests that there are two cases to
consider: t < 0 and t 0.
Case 1: t < 0. Here u(t ) 0 since is restricted to the interval [0,). Hence
f
2
(t) f
3
(t) 0, for t < 0.
Case 2: t 0.
f
2
(t) f
3
(t) 8 e
2
d 4e
2
]
0
t
0
t

4(1 e
2t
)
In sum,
f
2
(t) f
2
(t)
0, t < 0
4(1 e
2t
), t 0

'

(c) By the definition of the convolution integral and the sifting property of the delta function
f
1
(t) f
3
(t) f
1
(t ) f
3
( )d

5 (t )4e
2
u( )d

20e
2
u( )
]
t
20e
2t
u(t)
(d) By the definition of the convolution integral
1/25/02 P16-2 R. A. DeCarlo, P. M. Lin
f
3
(t) f
3
(t) f
3
(t ) f
3
( )d

4e
2(t )
u(t )4e
2
u( )d 16e
2t
u(t )d
0

The integrand, u(t ) , is nonzero only when t. This suggests that there are two cases to
consider: t < 0 and t 0.
Case 1: t < 0. Here u(t ) 0 since is restricted to the interval [0,). Hence
f
3
(t) f
3
(t) 0 , for t < 0.
Case 2: t 0.
f
3
(t) f
3
(t) 16e
2t
d 16te
2t
0
t

In sum,
f
3
(t) f
3
(t)
0, t < 0
16te
2t
, t 0

'

(e) By the definition of the convolution integral and the sifting property of the delta function
f
1
(t + 2) f
2
(t + 4) f
1
(t + 2 ) f
2
( + 4)d 5 (t + 2 )2u( + 4)d

10u( + 4)]
t +2
10u(t + 6)
(f) By the distributive property of convolution
f
2
(t) f
2
(t) + f
3
(t) [ ] f
2
(t) f
2
(t) + f
2
(t) f
3
(t)
Using the results of parts (a) and (b) the result follows immediately
f
2
(t) f
2
(t) + f
3
(t) [ ]
0, t < 0
4(1+ t e
2t
), t 0

'

Solution 16.2.
(a) By definition
f
3
(t) f
1
( ) f
2
(t )d

K
1
u( T
1
)K
2
u(t T
2
)d K
1
K
2
u(t T
2
)d
T
1

Here observe that u(t T


2
) 0 for > t T
2
. Hence there are two cases to consider:
t T
2
< T
1
and t T
2
T
1
.
Case 1: t T
2
< T
1
. Here u(t T
2
) 0 , since is restricted to the interval [T
1
, ) .
f
3
(t) 0
Case 2: t T
2
T
1
.
1/25/02 P16-3 R. A. DeCarlo, P. M. Lin
f
3
(t) K
1
K
2
d K
1
K
2
(t T
2
T
1
)
T
1
t T
2

In sum,
f
3
(t)
0, t < T
1
+ T
2
K
1
K
2
(t T
2
T
1
), t T
1
+ T
2

'

(b) By definition
f
3
(t) f
1
(t ) f
2
( )d

K
1
u(t + T
1
)K
2
u( + T
2
)d K
1
K
2
u(t + T
1
)d
T
2

Here observe that u(t + T


1
) 0 for > t + T
1
. Hence there are two cases to consider:
t + T
1
< T
2
and t + T
1
T
2
.
Case 1: t + T
1
< T
2
. Here u(t + T
1
) 0 , since is restricted to the interval [T
2
,): f
3
(t) 0.
Case 2: t + T
1
T
2
.
f
3
(t) K
1
K
2
d K
1
K
2
(t + T
1
+ T
2
)
T
2
t +T
1

In sum,
f
3
(t)
0, t < T
1
T
2
K
1
K
2
(t T
2
T
1
), t T
1
T
2

'

(c) By definition
f
3
(t) f
1
(t ) f
2
( )d

K
1
u(t )K
2
e
a
u( )d K
1
K
2
e
a
u(t )d
0

The integrand is nonzero only when t. Hence, there are two cases to consider: t < 0 and t 0.
Case 1: t < 0. f
3
(t) 0, for t < 0.
Case 2: t 0.
f
3
(t) K
1
K
2
e
a
0
t

d K
1
K
2
e
a
a
1
]
1
1
0
t

K
1
K
2
a
(1 e
at
).
Therefore
f
3
(t)
K
1
K
2
a
(1 e
at
)u(t). for t 0.
(d) By definition
f
3
(t) f
1
(t ) f
2
( )d

K
1
u(t + T
1
)K
2
e
a
u( )d K
1
K
2
e
a
u(t + T
1
)d
0

1/25/02 P16-4 R. A. DeCarlo, P. M. Lin


The integrand is nonzero only when t + T
1
. Hence, there are two cases to consider: t + T
1
< 0
and t + T
1
0.
Case 1: t + T
1
< 0. Here u(t + T
1
) 0 , since is restricted to the interval [0,). Hence
f
3
(t) 0.
Case 2: t + T
1
0.
f
3
(t) K
1
K
2
e
a
0
t +T
1

d K
1
K
2
e
a
a
1
]
1
1
0
t +T
1

K
1
K
2
a
1 e
a(t +T
1
)
[ ]
.
Therefore
f
3
(t)
K
1
K
2
a
1 e
a(t +T
1
)
[ ]
u(t + T
1
).
(e) By definition
f
3
(t) f
1
(t ) f
2
( )d

K
1
u(t + )K
2
e
a
u( )d K
1
K
2
e
a
u(t + )d
0

The integrand is nonzero only when t. Hence, there are two cases to consider: t 0 and t > 0.
Case 1: t 0. Here u(t + ) 1, since 0. Hence
f
3
(t) K
1
K
2
e
a
0

d K
1
K
2
e
a
a
1
]
1
1
0

K
1
K
2
a
, for t 0.
Case 2: t > 0.
f
3
(t) K
1
K
2
e
a
t

d K
1
K
2
e
a
a
1
]
1
1
t

K
1
K
2
a
e
at
, for t > 0.
In sum,
f
3
(t)
K
1
K
2
a
, t 0
K
1
K
2
a
e
at
, t > 0

'



Solution 16.3.
(a) By definition
f
3
(t) f
1
( ) f
2
(t )d

K
1
e
a
u( )K
2
e
a(t )
u(t )d

K
1
K
2
e
at
u(t )d
0

The integrand, u(t ) , is nonzero only when t. Hence, there are two cases to consider: t < 0
and t 0.
Case 1: t < 0. Here u(t ) 0 , since 0. Hence f
3
(t) 0, for t < 0.
Case 2: t 0.
1/25/02 P16-5 R. A. DeCarlo, P. M. Lin
f
3
(t) K
1
K
2
e
at
d
0
t

K
1
K
2
e
at
t , for t 0.
In sum,
I
c
(s)
Cs
Cs +
1
R
I
in
(s)
(b) By definition
a 1
K 1
The integrand, u(t ) , is nonzero only when t. Hence, there are two cases to consider: t < 0
and t 0.
Case 1: t < 0. Here u(t ) 0 , since 0. Hence f
3
(t) 0, for t < 0.
Case 2: t 0.
f
3
(t) K
1
K
2
e
bt
e
(ba)
d
0
t


t if a b
1
b a
e
(ba)t
1
[ ]
if a b

'



Therefore, for t 0,
f
3
(t)
K
1
K
2
e
bt
tu(t) if a b
K
1
K
2
b a
e
at
e
bt
[ ]
if a b

'



(c) By replacing K
1
50, K
2
20 and a 10 in the formula of f
3
(t) in part (a) the answer for
part (i) is easily obtained as
f
3
(t)
0, t < 0
1000e
10t
t, t 0

'

For part (ii) the parameters have the following values: v
c
(t) e
(t )

d e
t
]

t
1,
K
2
0.2, a 10 and b 0.2. Using these values in the formula developed in part (b) for f
3
(t) the
answer follows immediately
f
3
(t) 0.102 e
0.2t
e
10t
( )
u(t)
SOLUTION 16.4.
(a) Using the impulse response theorem and the definition of the convolution integral the response
of the system, y(t), can be computed as follows
y(t) h(t )v( )d

2e
2(t )
u(t ) u( 1) u( 3) [ ]d

Observing that u( 1) u( 3) is nonzero only when 1 < 3 yields


1/25/02 P16-6 R. A. DeCarlo, P. M. Lin
y(t) 2 e
2( t)
u(t )d
1
3

The integrand in the above equation is nonzero only when t. This suggests three regions of
consideration: t <1, 1 t 3, and 3 < t.
Case 1: t <1. Here u(t ) 0 , since is restricted to the interval [1,3]. Hence y(t) 0, for t <1.
Case 2: 1 t 3.
y(t) 2 e
2( t)
1
t

d e
2( t)
]
1
t
1 e
2(1t )
, for 1 t 3
Case 3: 3 < t.
y(t) 2 e
2( t)
1
3

d e
2( t)
]
1
3
e
2(1t )
(e
4
1), for 3 < t
In sum,
y(t)
0, t < 1
1 e
2(1t )
, 1 t 3
e
2(1t )
(e
4
1), 3 < t

'



A picture of y(t) is sketched in the next figure.
(b) Using the impulse response theorem and the definition of the convolution integral the response
of the system, y(t), can be computed as follows
y(t) h(t )v( )d

2e
2(t )
u(t )u(2 t + ) u( 1) u( 3) [ ]d

Observing that u( 1) u( 3) is nonzero only when 1 < 3 yields


y(t) 2 e
2( t)
u(t )u(2 t + )d
1
3

The integrand in the above equation is nonzero only when t 2 t. This suggests four regions
of consideration: t <1, 1 t 3, 3 < t 5and 5 < t.
Case 1: t <1. Here u(t ) 0 , since is restricted to the interval [1,3]. Hence y(t) 0, for t <1.
Case 2: 1 t 3. Here u(t )u(2 t + ) is nonzero only when 1 t. Therefore,
1/25/02 P16-7 R. A. DeCarlo, P. M. Lin
y(t) 2 e
2( t)
1
t

d e
2( t)
]
1
t
1 e
2(1t )
, for 1 t 3.
Case 3: 3 < t 5. Here u(t )u(2 t + ) is nonzero only when t 2 3. Therefore,
y(t) 2 e
2( t)
t 2
3

d e
2( t)
]
t 2
3
e
2(3t)
e
4
, for 3 < t 5.
Case 4: 5 < t. Here u(t )u(2 t + ) 0 , since is restricted to the interval [1,3]. Therefore,
y(t) 0, for 5 < t.
A picture of y(t) is sketched in the next figure.
(c) By the impulse response theorem, the zero-state response of the circuit y(t) is
y(t) h(t) v(t)
Using the definition of the convolution integral and the sifting property of delta function it follows
that
y(t) 2h(t) 2h(t 1) + h(t 2)
0, t < 0
2, 0 t < 1
2, 1 t < 2
0, 2 t < 3
1, 3 t < 4
0, 4 t

'









Using the waveform of h(t) given in figure P16.4, y(t) is sketched in the next picture.
1/25/02 P16-8 R. A. DeCarlo, P. M. Lin
SOLUTION 16.5.
(a) By definition
f
4
(t) f
1
(t ) f
2
( )d (t 2) 2 u( +1)d

By the sifting property of delta function it follows that f


4
(t) 2u( +1)]
t 2
2u(t 1).
(b) By the definition of convolution and the sifting property of delta function
f
5
(t) f
1
(t ) f
3
( )d (t 2)e
2
u( )d

e
2
u( )
]
t 2
e
2(t 2)
u(t 2)
(c) By definition
f
6
(t) f
2
(t ) f
3
( )d 2u(t +1)e
2
u( )d

2 e
2
u(t +1)d
0

The integrand in the above equation is nonzero only when t +1. This suggests two regions of
consideration: t < 1and 1 t .
Case 1: t < 1. Here u(t +1) 0, since 0 . Hence f
6
(t) 0 , for t < 1.
Case 2: 1 t . Here u(t +1) is nonzero only when t +1. Therefore,
f
6
(t) 2 e
2
d e
2
]
0
t +1
1 e
2(t +1)
0
t +1

It follows that
f
6
(t) 1 e
2(t +1)
[ ]
u(t +1)
(d) By the definition of convolution and the sifting property of delta function we have
1/25/02 P16-9 R. A. DeCarlo, P. M. Lin
f
7
(t) f
1
(t ) f
3
( 2)d (t 2)e
2( 2)
u( 2)d

e
2( 2)
u( 2)
]
t 2
e
2(t 4)
u(t 4) .
SOLUTION 16.6.
(a) By definition
f
6
(t) 1 e
2(t +1)
[ ]
u(t +1)
Here observe that u(t ) = 0 for > t. Hence, there are two cases to consider: t 0 and t > 0.
Case 1: t 0

y ( t )
K e
a
d

t


K
e
a

a
1
]
1
1

t


K
e
at
a
Case 2: t > 0
y(t) K e
a
d

t

K e
a
d

0

K
e
a
a
1
]
1
1

K
a
(b) By definition
y(t) K u( t)

e
a
u()d K u()
t

e
a
d
Here observe that u( t) = 0 for < t; hence the lower limit of integration is t. Also, because of the
presence of u() in the integrand, there are two cases to consider: t < 0 and t 0.
Case 1: t < 0
y(t) K e
a
d
0

K
e
a
a
1
]
1
1
0

K
a
Case 2: t 0
y(t) K e
a
d
t

K
e
a
a
1
]
1
1
t

K
a
e
at
SOLUTION 16.7.
(a) Using the definition of the convolution integral and the sifting property of delta function,
f
5
(t)can be computed as below
f
5
(t) f
2
(t ) f
4
( )d e
a(t )
u(t ) ( 4)d

e
a(t )
u(t )
]
4
e
a(t 4)
u(t 4)
A picture of f
5
(t), for a 1, is sketched in the next figure.
1/25/02 P16-10 R. A. DeCarlo, P. M. Lin
(b) By definition
f
6
(t) f
1
(t ) f
1
( )d K
2
u(t )u( )d

K
2
u(t )d
0

Since u(t ) is nonzero only when t, there are two regions of consideration: t < 0and 0 t.
Case 1: t < 0. Here u(t ) 0 , since 0. Hence
f
6
(t) 0 , for t < 0.
Case 2: 0 t.
f
6
(t) K
2
d
0
t

K
2
t , for 0 t.
A picture of f
6
(t) , for K 1, is sketched in the next figure.
(c) By definition
f
7
(t) f
1
(t ) f
2
( )d Ku(t )e
a
u( )d

K e
a
u(t )d
0

Since u(t ) is nonzero only when t, there are two regions of consideration: t < 0and 0 t.
Case 1: t < 0. Here u(t ) 0 , since 0. Therefore, f
7
(t) 0 , for t < 0.
Case 2: 0 t.
f
7
(t) K e
a
d
0
t

K
a
e
a
]
0
t

K
a
1 e
at
( )
, for 0 t.
1/25/02 P16-11 R. A. DeCarlo, P. M. Lin
A picture of f
7
(t) , for t < 0 and a 1, is sketched in the next figure.
(d) By definition
f
8
(t) f
1
(t ) f
3
( )d Ku(t )e
a
u( )d

K e
a
u(t )d

The integrand, ) ( t u , is nonzero only when t . This suggests two regions of consideration:
0 t and t < 0 .
Case 1: 0 t .
f
8
(t) K e
a
u(t )d

K
a
e
a
]

K
a
e
at
, for t 0.
Case 2: 0 < t.
f
8
(t) K e
a
u(t )d

K
a
e
a
]

K
a
, for 0 < t.
In sum,
f
8
(t)
K
a
e
at
, t 0
K
a
, 0 < t

'



A picture of f
8
(t), for K 1 and a 1, is sketched in the next figure.
1/25/02 P16-12 R. A. DeCarlo, P. M. Lin
SOLUTION 16.8.
(a) Using the current division formula
I
c
(s)
Cs
Cs +
1
R
I
in
(s)
By Ohms law the Laplace transform of capacitors voltage
V
c
(s)
1
Cs
I
c
(s)
Therefore the transfer function of the circuit
H(s)
V
c
(s)
I
in
(s)

1
Cs +
1
R

1
s + 4
Taking the inverse Laplace transform of H(s) yields the impulse response h(t) e
4t
u(t) .
(b) By the impulse response theorem
v
c
(t) i
in
(t) h(t) i
in
(t

)h( )d 3e
(t )

u(t )e
4
u( )d
3 e
(t +3 )
0

u(t )d
The integrand is nonzero only when t. Therefore there are two regions of consideration:
t < 0and 0 t.
Case 1: t < 0. Here u(t ) 0 , since 0 . Hence v
c
(t) 0 , for t < 0.
Case 2: 0 t.
1/25/02 P16-13 R. A. DeCarlo, P. M. Lin
v
c
(t) 3 e
(t +3 )
0
t

d e
(t +3 )
]
0
t
e
t
e
4t
, for 0 t.
In sum,
v
c
(t) e
t
e
4t
( )
u(t)V.
SOLUTION 16.9.
(a) By voltage division formula
V
out
(s)
1
Cs
R +
1
Cs
V
in
(s)
Therefore the transfer function
H(s)
V
out
(s)
V
in
(s)

1
Cs
R +
1
Cs

1
s +1
Taking the inverse Laplace transform of H(s) yields yields the impulse response h(t) e
t
u(t).
By the impulse response theorem and the convolution definition
v
out
(t) h(t )v
in
( )

d e
(t )

u(t ) u( ) + 2e
2
u( )
[ ]
d
e
(t )

u(t )d + 2 e
(t + )
0

u(t )d
For both integrals the integrand is nonzero only when t. This suggests two regions of
consideration: t < 0and 0 t.
Case 1: t < 0. Here the second integral is zero since, for this integral, is restricted to [0,).
v
c
(t) e
(t )

d e
t
]

t
1, for t < 0.
Case 2: 0 t.
v
c
(t) e
(t )

d + 2 e
(t + )
0
t

d
e
t
]

0
2e
(t )
]
0
t
e
t
2 e
2t
e
t
( )
3e
t
2e
2t
, for 0 t
(c) By the impulse response theorem and the definition of convolution
1/25/02 P16-14 R. A. DeCarlo, P. M. Lin
v
out
(t) h(t )v
in
( )

d e
(t )

u(t )e
a| |
d
e
(t )

u(t )e
a
d + e
(t )
0

u(t )e
a
d
e
t
e
(a+1)

u(t )d + e
t
e
(1a)
0

u(t )d
For both integrals the integrand is nonzero only when t. This suggests two regions of
consideration: t < 0and 0 t.
Case 1: t < 0. Here the second integral is zero since, for this integral, is restricted to [0,).
v
out
(t) e
t
e
(a+1)

d e
t
e
(a+1)
a +1
1
]
1
1

e
at
a +1
, for t < 0.
Case 2: 0 t.
v
out
(t) e
t
e
(a+1)

d + e
t
e
(1a)
0
t

d
e
t
e
(a+1)
a +1
1
]
1
1

0
+ e
t
e
(1a)
0
t

d
e
t
a +1
+ e
t
e
(1a)
0
t

d
Here observe that a +1 is nonzero since a > 0.
For computing the second integral, in case 2, we need to distinguish two subcases: a 1
and a 1.
e
t
e
(1a)
0
t

d
e
t
t if a 1
1
1 a
e
at
e
t
( )
if a 1

'



Therefore, for 0 t,
v
out
(t)
e
t
a +1
+ e
t
t if a 1
e
t
a +1
+
1
1 a
e
at
e
t
( )
if a 1

'





SOLUTION 16.10.
(a) The impulse response is obtained by taking the inverse Laplace transform of the transfer
function
h(t) 2e
0.2t
u(t)
By the impulse response theorem the response y(t)equals
1/25/02 P16-15 R. A. DeCarlo, P. M. Lin
y(t) h(t) v(t) h(t )v( )d

Substituting v(t) u(t +1) u(t 1) in the above integral yields


y(t) h(t ) u( +1) u( 1) [ ]d

Here observe that u( +1) u( 1) is nonzero only when 1 1. Hence


y(t) h(t )d 2 e
0.2( t )
1
1

u(t )d
1
1

The integrand is nonzero only when t. This suggests three regions of consideration: t < 1,
1 t < 1 and 1 t.
Case 1: t < 1. y(t) 0
Case 2: 1 t < 1.
y(t) 2 e
0.2( t )
1
t

d 10 e
0.2(t +1)
1
[ ]
, for 1 t < 1.
Case 3: 1 t.
y(t) 2 e
0.2( t )
1
1

d 10e
0.2t
e
0.2
e
0.2
( )
, for 1 t.
(b) The transfer function of the leaky integrator (see equation 14.14 in the textbook) is given by
H(s)

1
R
1
Cs +
1
R
2
where R
2
is the leakage resistance of the capacitor C and R
1
is the resistance connected at the
inverting input of the op amp. Equating the two expressions of H(s) we obtain that

1
R
1
Cs +
1
R
2

2
s + 0.2
Matching the coefficients and taking into account that the smallest resistor is 10k the following
values are obtained: R
1
10k, R
2
100k and C 5 10
5
F .
(c) The impulse response is obtained by taking the inverse Laplace transform of the transfer
function
h(t) Ke
at
u(t)
By the impulse response theorem the response y(t)equals
1/25/02 P16-16 R. A. DeCarlo, P. M. Lin
y(t) h(t) v(t) h(t )v( )d

Substituting v(t) u(t + T) u(t T) in the above integral yields


y(t) h(t ) u( + T) u( T) [ ]d

Here observe that u( + T) u( T) is nonzero only when T T . Hence


y(t) h(t )d K e
a(t )
T
T

u(t )d
T
T

The integrand is nonzero only when t. This suggests three regions of consideration: t < T ,
T t < T and T t.
Case 1: t < T . y(t) 0
Case 2: T t < T .
y(t) K e
a(t )
T
t

d
K
a
1 e
a(t +T)
[ ]
, for T t < T .
Case 3: T t.
y(t) K e
a(t )
T
T

d
K
a
e
at
e
aT
e
aT
( )
, for T t.
In sum,
y(t)
0, t < T
K
a
1 e
a(t +T)
[ ]
, T t < T
K
a
e
at
e
aT
e
aT
( )
, T t

'







SOLUTION 16.11.
(a) First observe, from figure P16.11(a), that
f
2
(t) (2t + 4) u(t) u(t 2) [ ]
By definition
f
3
(t) f
1
(t ) f
2
( )d 4u(t )(2 + 4) u( ) u( 2) [ ]d 8 ( 2)u(t )d
0
2

The integrand is nonzero only when t. This suggests three regions of consideration: t < 0,
0 t < 2 and 2 t.
Case 1: t < 0. Here u(t ) 0 due to the fact that is restricted to the interval [0,2]. Hence
f
3
(t) 0, for t < 0.
1/25/02 P16-17 R. A. DeCarlo, P. M. Lin
Case 2: 0 t < 2.
f
3
(t) 8 ( 2)d 8
2
2
2



_
,

1
]
1
1
0
t

0
t
4 t
2
4t
( )
, for 0 t < 2.
Case 3: 2 t.
f
3
(t) 8 ( 2)d 8
2
2
2



_
,

1
]
1
1
0
2

0
2
16, for 2 t.
In sum,
f
3
(t)
0, t < 0
4 t
2
4t
( )
, 0 t < 2
16, 2 t

'



A picture of f
3
(t) is sketched in the next figure.
(b) First observe, from figure P16.11(b), that
f
2
(t) t u(t) u(t 2) [ ] + (4 t) u(t 2) u(t 4) [ ]
By definition
f
3
(t) f
1
(t ) f
2
( )d

By replacing f
1
(t) and f
2
(t) with their expressions we have
f
3
(t) 4u(t ) u( ) u( 2) [ ] + (4 ) u( 2) u( 4) [ ] { }

d
4u(t ) u( ) u( 2) [ ]d + 4u(t )

(4 ) u( 2) u( 4) [ ]

d
4 u(t )d + 4 (4 )u(t )
2
4

0
2

d
1/25/02 P16-18 R. A. DeCarlo, P. M. Lin
The integrands are nonzero only when t. This suggests four regions of consideration: t < 0,
0 t < 2, 2 t < 4, and 4 t.
Case 1: t < 0. Here u(t ) 0 due to the fact that is restricted to the interval [0,4]. Hence
f
3
(t) 0, for t < 0.
Case 2: 0 t < 2. here observe that the second integral is zero since, for this integral, is restricted
to the interval [2,4]. Therefore
f
3
(t) 4 d 2t
2
0
t

, for 0 t < 2.
Case 3: 2 t < 4.
f
3
(t) 4 d + 4 (4 )
2
t

0
2

d 8 + 4 4
2
2



_
,

1
]
1
1
2
t
2t
2
+16t 16, for 2 t < 4.
Case 4: 4 t.
f
3
(t) 4 d + 4 (4 )
2
4

0
2

d 8 + 4 4
2
2



_
,

1
]
1
1
2
4
16, for 4 t.
In sum,
f
3
(t)
0, t < 0
2t
2
, 0 t < 2
2t
2
+16t 16, 2 t < 4
16, 4 t

'





A picture of f
3
(t) is sketched in the next figure.
SOLUTION 16.12. (a) By voltage division,
1/25/02 P16-19 R. A. DeCarlo, P. M. Lin
H(s)
V
out
V
in

2
s
2s + 5 +
2
s

1
s
2
+ 2.5s +1

2 / 3
(s + 0.5)

2 / 3
(s + 2)
Hence, the impulse response is
h(t)
2
3
e
0.5t
u(t)
2
3
e
2t
u(t)
(b) By definition
h(t) * v
in
(t) h(t )v
in
()

d
20
3
e
0.5(t )
u(t )

u() d

20
3
e
2(t )
u(t )

u() d
Case 1: t 0.
v
out
(t)
20
3
e
0.5(t )

d
20
3
e
2(t)

d
20
3
e
0.5t

e
1.5
d
20
3
e
2t

e
3
d

20e
0.5t
4.5
e
1.5
[ ]

20e
2t
9
e
3
[ ]

t
4.444e
t
2.222e
t
2.222e
t
Case 2: t > 0.
v
out
(t)
20e
0.5t
4.5
e
1.5
[ ]

20e
2t
9
e
3
[ ]

20e
0.5t
4.5

20e
2t
9
SOLUTION 16.13.
(a) The impulse response of the circuit has been computed in problem 16.12
h(t)
2
3
e
0.5t
(t)
2
3
e
2t
u(t)
By the impulse response theorem and the convolution definition
v
out
(t) h(t) v
in
(t) h(t )v
in
( )d

1/25/02 P16-20 R. A. DeCarlo, P. M. Lin

20
3
e
0.5(t )
e
2(t )
[ ]
u(t )e
| |
d

The integrand is nonzero only when t. Hence


v
out
(t)
20
3
e
0.5(t )
e
2(t )
[ ]
e
| |
d

The existence of the function e


| |
under the integral suggests two regions of consideration: t 0
and 0 < t.
Case 1: t 0.
v
out
(t)
20
3
e
0.5(t )
e
2(t )
[ ]
e d

20
3
e
0.5t
e
1.5

d
20
3
e
2t
e
3
d

20
4.5
e
0.5t
e
1.5
[ ]

20
9
e
2t
e
3
[ ]

4.444e
t
2.222e
t
2.222e
t
, for t 0.
Case 2 : 0 < t.
v
out
(t)
20
3
e
0.5(t )
e
2(t )
[ ]
e d

+
20
3
e
0.5(t )
e
2(t )
[ ]
e

d
0
t

20
3
e
0.5t +1.5
e
2t +3
[ ]
d

+
20
3
e
0.5t 0.5
e
2t +
[ ]
d
0
t

20
3
e
0.5t +1.5
1.5

e
2t +3
3




1
]
1
1

0
+
20
3
e
0.5t 0.5
0.5
e
2t +




1
]
1
1
0
t

17.778e
0.5t
20e
t
+ 4.444e
2t
, for 0 < t.
In sum,
v
out
(t)
2.222e
t
, 0 t
17.778e
0.5t
20e
t
+ 4.444e
2t
, 0 < t

'



SOLUTION 16.14.
(a) The impulse response of the circuit is obtained by taking the inverse Laplace transform of H(s)
h(t) 2e
t
2e
2t
+ 4e
4t
( )
u(t)
(b) The result follows from the following MATLAB code:
1/25/02 P16-21 R. A. DeCarlo, P. M. Lin
>> p = [-1,-2,-4];
>> r = [2,-2,4];
>> k = 0;
>> [n,d] = residue(r,p,k)
n =
4 14 16
d =
1 7 14 8
Therefore,
H(s)
4s
2
+14s +16
s
3
+ 7s
2
+14s + 8
(c) By the impulse response theorem
y(t) u(t) h(t) u(t)* 2e
t
u(t) 2e
2t
u(t) + 4e
4t
u(t)
[ ]
Using the distributive property of convolution we have
y(t) u(t) 2e
t
u(t)
[ ]
+ u(t) 2e
2t
u(t)
[ ]
+ u(t) 4e
4t
u(t)
[ ]
In problem P16.2(c) the following equation has been obtained
K
1
u(t) [ ] K
2
e
at
u(t)
[ ]

K
1
K
2
a
(1 e
at
)u(t)
Using the above equation y(t)is immediately obtained
y(t) 2 1 e
t
( )
u(t) 1 e
2t
( )
u(t) + 1 e
4t
( )
u(t)
2 2e
t
+ e
2t
e
4t
( )
u(t)
(d) By the impulse response theorem
y(t) f (t) h(t) 8u(t) 8u(t T) [ ]* 2e
t
u(t) 2e
2t
u(t) + 4e
4t
u(t)
[ ]
Using the distributive property of convolution we have
y(t) 8u(t) 2e
t
u(t) 2e
2t
u(t) + 4e
4t
u(t)
[ ]

8u(t T)* 2e
t
u(t) 2e
2t
u(t) + 4e
4t
u(t)
[ ]
We denote
1/25/02 P16-22 R. A. DeCarlo, P. M. Lin
y
1
(t) 8u(t) 2e
t
u(t) 2e
2t
u(t) + 4e
4t
u(t)
[ ]
By the time invariance property it follows that
y(t) y
1
(t) y
1
(t T)
In order to compute y
1
(t) we will use the following equation which has been obtained in problem
16.2(e)
K
1
u(t) [ ] K
2
e
at
u(t)
[ ]

K
1
K
2
a
, t 0
K
1
K
2
a
e
at
, t > 0

'



Therefore,
y
1
(t)
16, t 0
16e
t
8e
2t
+ 8e
4t
, 0 < t

'

The zero-state response to the input f (t) can now be computed
y(t)
0, t 0
16e
t
8e
2t
+ 8e
4t
16, 0 < t T
16 e
t
e
(t T )
[ ]
8 e
2t
e
2(t T)
[ ]
+ 8 e
4t
e
4(t T)
[ ]
, T < t

'





SOLUTION 16.15.
(a) Using the convolution theorem the transfer function of the cascade is
H(s) L h(t) [ ] L h
1
(t) h
2
(t) h
3
(t) [ ] L h
1
(t) [ ] L h
2
(t) [ ] L h
3
(t) [ ] H
1
(s) H
2
(s) H
3
(s)
From table 13.1
H
1
(s)
1
s
H
2
(s)
10
s + 2
H
3
(s)
2
s
2
Therefore,
H(s)
20
s
3
(s + 2)
A partial fraction expansion of H(s) can be obtained using the residue command in MATLAB:
>> num = [20];
>> den = [1 2 0 0 0];
>> [r,p,k] = residue(num,den)
1/25/02 P16-23 R. A. DeCarlo, P. M. Lin
r =
-2.5000
2.5000
-5.0000
10.0000
p =
-2
0
0
0
k =
[]
Hence
H(s)
2.5
s + 2
+
2.5
s
+
5
s
2
+
10
s
3
Taking the inverse Laplace transform yields the impulse response of the cascade
h(t) 2.5e
2t
u(t) + 2.5u(t) 5tu(t) + 5t
2
u(t)
(b) By the impulse response theorem and the convolution theorem, the Laplace transform of the
step response of the cascade equals
Y(s) H(s) U(s)
20
s
3
(s + 2)

1
s

20
s
4
(s + 2)
A partial fraction expansion of H(s) can be obtained using the residue command in MATLAB:
>> num = [20];
>> den = [1 2 0 0 0 0];
>> [r,p,k] = residue(num,den)
r =
1.2500
-1.2500
2.5000
-5.0000
10.0000
p =
-2
0
0
1/25/02 P16-24 R. A. DeCarlo, P. M. Lin
0
0
k =
[]
Hence
Y(s)
1.25
s + 2
+
1.25
s
+
2.5
s
2
+
5
s
3
+
10
s
4
Taking the inverse Laplace transform yields the step response of the cascade
y(t) 1.25e
2t
u(t) 1.25u(t) + 2.5tu(t) 2.5t
2
u(t) +1.667t
3
u(t) .
SOLUTION 16.16.
(a) By the voltage division formula
V
out
(s)
1
Cs
R +
1
Cs
V
i
(s)
1
CRs +1
V
in
(s)
Therefore, the transfer function of the circuit is
H(s)
V
out
(s)
V
in
(s)

1
CRs +1

1
s +1
Taking the inverse Laplace transform yields the impulse response h(t) e
t
u(t).
(b) From table 13.1 the Laplace transform of v
in
(t) is
V
in
(s)
1
s
+
1
(s +1)
2
By the impulse response theorem and the convolution theorem it follows that
V
out
(s) H(s) V
in
(s)
1
s +1
1
s
+
1
(s +1)
2




1
]
1
1

1
s(s +1)
+
1
(s +1)
3
A partial fraction expansion of V
out
(s) is
V
out
(s)
1
s

1
s +1
+
1
(s +1)
3
Taking the inverse Laplace transform yields
1/25/02 P16-25 R. A. DeCarlo, P. M. Lin
v
out
(t) u(t) e
t
u(t) 0.5t
2
e
t
u(t).
Using the time domain convolution method v
out
(t) can be computed as follows
v
out
(t) h(t )v
in
( )d

e
(t )
u(t ) u( ) + e

u( )
[ ]
d

From the experience earned by computing convolution integrals we know that the computation of the
above integral requires more computational work than the Laplace transform method. More
computations imply, of course, more sources of errors.
From the solution of this problem we have seen that, in the case of the Laplace transform method,
the computational burden consists in computing Laplace and inverse Laplace transforms. For a
large class of functions these transforms can be found in tables(for example table 13.1). The only
computation that we did, in the solution of this problem, was the partial fraction expansion of
V
out
(s) .
(c) In this case the (unilateral) transform method cannot be used because v
in
(t) 0 for t < 0.
SOLUTION 16.17.
(a) The impulse response can be obtained by taking the inverse Laplace transform of H(s) .
Therefore
h(t) 8e
10t
u(t)
(b) From table 13.1
V
in
(s)
8
s
2
+16
By the impulse response theorem and the convolution theorem it follows that
V
out
(s) H(s) V
in
(s)
64
(s +10)(s
2
+16)
V
out
(s) can be further written as
V
out
(s)
0.5517s + 5.517
s
2
+16
+
0.5517
s +10
0.5517
s
s
2
+16
+1.379
4
s
2
+16
+
0.5517
s +10
The above expansion of V
out
(s) can be obtained by using the technique of example 13.14, page
514. Taking the inverse Laplace transform yields
v
out
(t) 0.5517cos(4t)u(t) +1.379sin(4 t)u(t) + 0.5517e
10t
u(t).
1/25/02 P16-26 R. A. DeCarlo, P. M. Lin
(c) In this case
V
in
(s) L 2e
2t
sin(4t)u(t)
[ ]

8
(s + 2)
2
+16
Therefore
V
out
(s) H(s) V
in
(s)
64
(s + 2)
2
+16
[ ]
(s +10)
Using again the technique of example 13.14, page 514, V
out
(s) can be written as
V
out
(s)
0.8s + 4.8
(s + 2)
2
+16
+
0.8
s +10
0.8
s + 2
(s + 2)
2
+16
+1.6
4
(s + 2)
2
+16
+
0.8
s +10
Taking the inverse Laplace transform yields
v
out
(t) 0.8e
2t
cos(4t)u(t) +1.6e
2t
sin(4 t)u(t) + 0.8e
10t
u(t) V.
In this context the Laplace transform method is faster than the time domain convolution. This is
due to the fact that v
in
(t) has a relatively complicated expression and the convolution integral will
require more computational work than the Laplace transform method.
(d) In this context the Laplace transform method cannot be used because v
in
(t) 0 for t < 0. The
time domain convolution method will be used to compute the response v
out
(t). By the impulse
response thorem
v
out
(t) h(t) v
in
(t) 8e
10t
u(t)
[ ]
u(t) [ ]
Using the result of problem 16.2, part (e), it follows that
v
out
(t)
0.8, t 0
0.8e
10t
, 0 < t

'

SOLUTION 16.18.
(a) Replacing R
1
, R
2
, C
1
and C
2
with their values the transfer function can be obtained
H(s)
s
s
2
+5s + 2
The only zero of H(s) is 0 and the poles of H(s) are 0.5 and 2. A partial fraction expansion of
H(s) is:
H(s)
0.167
s + 0.5
+
0.667
s + 2
The impulse response can be obtained by taking the inverse Laplace transform of H(s)
h(t) 0.167e
0.5t
+ 0.667e
2t
1/25/02 P16-27 R. A. DeCarlo, P. M. Lin
(b) v
out
(t) will be computed using the Laplace transform method. This approach is valid because
h(t) and v
in
(t) are zero for t < 0.
From table 13.1 the Lapace transform of v
in
(t) is
V
in
(s)
1
(s + 2)
2
By the impulse response theorem and the convolution theorem it follows that
V
out
(s) H(s) V(s)
s
2s
2
+ 5s + 2
1
(s + 2)
2

s
2s
4
+13s
3
+ 30s
2
+ 28s + 8
A partial fraction expansion of V
out
(s) can be obtained using the residue command in MATLAB:
>> a = [1 0];
>> b = [2 13 30 28 8];
>> [r,p,k] = residue(a,b)
r =
0.0741
0.1111
0.6667
-0.0741
p =
-2.0000
-2.0000
-2.0000
-0.5000
k =
[]
Therefore,
V
out
(s)
0.0741
s + 0.5
+
0.0741
s + 2
+
0.1111
(s + 2)
2
+
0.6667
(s + 2)
3
Taking the inverse Laplace transform yields
v
out
(t) 0.0741e
0.5t
+ 0.0741e
2t
+ 0.1111te
2t
+ 0.3333t
2
e
2t
[ ]
u(t) V.
One would prefer the time domain convolution method to compute v
out
(t), but the computations
may require more work relatively to the Laplace transform method.
(c) In this part v
in
(t) 0 for t < 0. Therefore the time domain convolution method will be used to
compute v
out
(t).
By the impulse response theorem
v
out
(t) h(t) v
in
(t)
0.1667e
0.5(t )
+ 0.6667e
2(t )
[ ]
u

(t )e
2
u( )d
1/25/02 P16-28 R. A. DeCarlo, P. M. Lin
0.1667e
0.5(t )
+ 0.6667e
2(t )
[ ]

u(t )e
2
d
The integrand of the previous integral is nonzero only when t. This suggests two regions of
consideration: t < 0 and 0 t.
Case 1: t < 0.
v
out
(t) 0.1667e
0.5t
e
2.5

d + 0.6667e
2t
e
4

d
0.1667e
0.5t
e
2.5
2.5




1
]
1
1

t
+ 0.6667e
2t
e
4
4




1
]
1
1

0.1e
2t
, for t < 0.
Case 2: 0 t.
v
out
(t) 0.1667e
0.5t
e
2.5

d + 0.6667e
2t
e
4

d
0.1667e
0.5t
e
2.5
2.5




1
]
1
1

0
+ 0.6667e
2t
e
4
4




1
]
1
1

0.1667e
0.5t
+ 0.6667e
2t
, for 0 t.
SOLUTION 16.19.
Replacing Rand C with their values
H(s)
s 5
s + 5
1
10
s + 5
The zero-state response v
out
(t) will be computed using the time domain convolution method
because v
in
(t) 0 for t < 0.
The impulse response of the circuit is
h(t) (t) 10e
5t
u(t)
By the impulse response theorem
v
out
(t) h(t) v
in
(t) h( )v
in
(t )d

10 ( ) 10e
5
u( )
[ ]
co

s10(t ) [ ]d
10 ( )co

s(t )d 10 10e
5
u( )co

s 10(t ) [ ]d
Using the sifting property of the delta function and expanding cos(t )it follows that
1/25/02 P16-29 R. A. DeCarlo, P. M. Lin
v
out
(t) 10cos(t ) 100cos(10t) e
5
co
0

s(10 )d 100sin(10t) e
5
0

sin(10 )d
Using the definition of the Laplace transform we observe that
e
5
cos
0

(10 )d L cos(10t)u(t) [ ]
s5

s
s
2
+100
1
]
1
s5
0.04
and
e
5
sin
0

(10 )d L sin(10t)u(t) [ ]
s5

10
s
2
+100
1
]
1
s5
0.08
Therefore
v
out
(t) 10cos(10t) 4cos(10t) 8sin(10t)
6cos(t) 8sin(10t) 10cos 10t + tan
1
(
4
3
)
[ ]
Notice that v
out
(t) and v
in
(t) have the same frequency and magnitude.
SOLUTION 16.20.
(a) First notice that v
in
(t T) u(t). Therefore w(t) u(t) and
W(s)
1
s
(b) Using the properties of the Laplace transform it follows that
V
out
w
(s) H(s) W (s)
2
s(s + 2)
A partial fraction expansion of V
out
w
(s) is
V
out
w
(s)
1
s

1
s + 2
Taking the inverse Laplace transform yields
v
out
w
(t) u(t) e
2t
u(t)
(b) Since
v
in
(t) w(t + T)
it follows, by the time invariance property, that
v
out
v
(t) v
out
w
(t + T).
Therefore,
v
out
v
(t) u(t + T) e
2(t +T )
u(t + T) V.
SOLUTION 16.21.
(a) First observe from figure P16.21 that
v
in
(t) u(t + T) u(t T)
From the definition of w(t) it follows that
1/25/02 P16-30 R. A. DeCarlo, P. M. Lin
w(t) v
in
(t T) u(t) u(t 2T)
Therefore
W(s)
1
s

1
s
e
2sT
(a) By the impulse response theorem and the convolution theorem it follows that
V
out
w
(s) H(s) W (s)
2
s(s + 2)
1 e
2sT
( )

1
s

1
s + 2


_
,
1 e
2sT
( )

1
s

1
s + 2

1
s

1
s + 2


_
,
e
2sT
Taking the inverse Laplace transform and using the time shift property of the Laplace transform
yields
v
out
w
(t) 1 e
2t
( )
u(t) 1 e
2(t 2T )
[ ]
u(t 2T)
Because
v
in
(t) w(t + T)
it follows, by the time invariance property, that
v
out
v
(t) v
out
w
(t + T)
1 e
2(t +T )
( )
u(t + T) 1 e
2(t T)
[ ]
u(t T) V.
SOLUTION 16.22. (a) The use of t t + T
1
in the problem statement means replace t by t + T
1
.
However, strictly speaking we should have used a statement of the form t t' +T
1
which is done in
the proof below. By definition of the convolution and the property of commutivity,
f (t T
1
) * g(t) f (t T
1
)g() d

f (t' )g() d





1
]
1
1
t' t T
1
f (t' ) * g(t' ) [ ]
t' t T
1
Observe that t t' +T
1
. Hence
f (t T
1
)* g(t) [ ]
t t' +T
1
f (t' ) * g(t' )
Realizing that t and t' are simply dummy variables, we immediately obtain the result. From a
systems perspective, this corresponds to the property of time-invariance where a shift of an input
function by T
1
yields a corresponding shift of the output function by T
1
.
(b) The steps in this part are similar to those of part (a).
1/25/02 P16-31 R. A. DeCarlo, P. M. Lin
f (t T
1
) * g(t T
2
) f (t T
1
)g( T
2
) d

Let T
2
in which case + T
2
and d d . Hence
f (t T
1
) * g(t T
2
) f (t T
1
T
2
)g()d

f (t' )g()d





1
]
1
1
t' t T
1
T
2
f (t' )* g(t' ) [ ]
t' t T
1
T
2
Since t t' +T
1
+ T
2
, and t and t' are dummy variables, we have
f (t T
1
)* g(t T
2
) [ ]
t t' +T
1
+T
2
f (t' )* g(t' )
and the result follows.
SOLUTION 16.23. From table 13.1, G(s)
1
s + 2
, H(s)
1
(s + 2)
2
.
(a) Define p(t) = f(t 2) = u(t).
Then, P(s)
1
s
. Consider
P(s)G(s)
1
s(s + 2)

0.5
s

0.5
s + 2
Hence
p(t) * g(t) 0.5u(t) 0.5e
2t
u(t)
From problem 16.22 part (a),
f (t)* g(t) p(t) * g(t) [ ]
t t +2
0.5u(t) 0.5e
2t
u(t)
[ ]
t t +2
0.5 0.5e
2(t +2)
[ ]
u(t + 2)
(b) Define p(t) = f(t 2) = u(t).
Then, P(s)
1
s
. Consider
P(s)H(s)
1
s(s + 2)
2

0.25
s

0.25
s + 2

0.5
(s + 2)
2
Hence
1/25/02 P16-32 R. A. DeCarlo, P. M. Lin
p(t) * h(t) 0.25 0.25e
2t
0.5te
2t
( )
u(t)
From problem 16.22 part (a),
f (t)* h(t) p(t)* h(t) [ ]
t t +2
0.25 0.25e
2t
0.5te
2t
( )
u(t)
[ ]
t t +2
0.25 0.25e
2(t +2)
0.5(t + 2)e
2(t +2)
[ ]
u(t + 2)
SOLUTION 16.24.
(a) The pictures of f (t) and g(t) are sketched in the next figures
(b) Using the convolution theorem it follows that
L f (t 2 ) g(t ) [ ] L f (t 2 ) [ ] L g(t ) [ ]
From table 13.1
L f (t 2 ) [ ] L u(t) [ ]
1
s
L g(t ) [ ] L sin(t)u(t) [ ]
1
s
2
+1
Therefore
L f (t 2 ) g(t ) [ ]
1
s(s
2
+1)

1
s

s
s
2
+1
Taking the inverse Laplace transform yields
f (t 2 ) g(t ) u(t) cos(t)u(t) 1 cos(t) [ ]u(t)
Using the property give in problem 16.22(b) it follows that
f (t) g(t) f (t 2 ) g(t ) [ ]
t t +2 +
1 cos(t + 3 ) [ ]u(t + 3 )
1/25/02 P16-33 R. A. DeCarlo, P. M. Lin
SOLUTION 16.25.
Define
w(t) v
in
(t 2)
Hence
w(t) u(t)
and, from table 13.1,
W(s)
1
s
From table 13.1 we also have that
H
1
(s)
1
s +1
and H
2
(s)
1
(s +1)
2
The impulse response of the cascade is
h(t) h
1
(t) h
2
(t)
Hence the transfer function of the cascade is
H(s) H
1
(s)H
2
(s)
1
(s +1)
3
We denote by v
out
w
(t)the zero state response due to the input w(t). Hence,
V
out
w
(s) H(s) W (s)
1
s(s +1)
3
A partial fraction expansion of V
out
w
(s) is obtained using the residue command in MATLAB:
>> a = [1];
>> b = [1 3 3 1 0];
>> [r,p,k] = residue(a,b)
r =
-1.0000
-1.0000
-1.0000
1.0000
p =
-1.0000
-1.0000
-1.0000
0
k =
[]
Therefore
V
out
w
(s)
1
s
+
1
s +1
+
1
(s +1)
2
+
1
(s +1)
3
Taking the inverse Laplace transform yields
1/25/02 P16-34 R. A. DeCarlo, P. M. Lin
v
out
w
(t) 1 e
t
te
t
0.5t
2
e
t
[ ]
u(t) V
Due to the fact that
v
in
(t) w(t + 2)
the time invariance property implies that
v
out
(t) v
out
w
(t + 2)
1 e
(t +2)
(t + 2)e
(t +2)
0.5(t + 2)
2
e
(t +2)
[ ]
u(t + 2) V.
SOLUTION 16.26.
(a) Using the sifting property of the delta function it follows that
f
4
(t) (t) + (t 4) [ ] f
2
(t) f
2
(t) + f
2
(t 4)
The right-hand side of the above equation interprets as a graphical sum of shifted pictures of f
2
(t) .
A picture of f
4
(t) is sketched in the next figure.
0 1 2 3 4 5 6 7
0
2
4
(b) In order to compute the area beneath f
2
(t ) f
2
( ) four regions will be considered: t < 0,
0 t <1, 1 t < 2 and 2 t.
Step 1: t < 0. In this case f
2
(t ) f
2
( ) 0 for all . Therefore
f
2
(t) f
2
(t) 0 for t < 0.
Step 2: 0 t <1. In this case f
2
(t ) f
2
( ) 16 for 0 t and is zero elsewhere. The area
beneath f
2
(t ) f
2
( ) equals 16t . Therefore
f
2
(t) f
2
(t) 16t for 0 t <1.
Step 3: 1 t < 2. In this case f
2
(t ) f
2
( ) 16 for t 1 < 1 and is zero elsewhere. Hence
the area beneath f
2
(t ) f
2
( )equals
f
2
(t) f
2
(t) 16(2 t) for 1 t < 2.
Step 4: 2 t. In this case f
2
(t ) f
2
( ) 0 for all . Therefore
f
2
(t) f
2
(t) 0 for2 t.
A picture of f
5
(t)is sketched in the next figure.
1/25/02 P16-35 R. A. DeCarlo, P. M. Lin
(c) In order to compute the area beneath f
2
(t ) f
3
( ) five regions will be considered: t < 0,
0 t <1, 1 t < 2, 2 t < 3 and 3 t.
Step 1: t < 0. In this case f
2
(t ) f
3
( ) 0 for all . Therefore f
2
(t) f
3
(t) 0 for t < 0.
Step 2 : 0 t <1. In this case f
2
(t ) f
3
( ) 8 for 0 t and is zero elsewhere. Therefore
the area beneath f
2
(t ) f
3
( ) equals
f
2
(t) f
3
( ) 8t for 0 t <1.
Step 3: 1 t < 2. Here
f
2
(t ) f
3
( )
8, t 1 < <1
24, 1 < t
0, otherwise

'



Hence, the area beneath f
2
(t ) f
3
( ) equals
f
2
(t) f
3
( ) 81 (t 1) [ ] + 24(t 1) 8(2t 1) for 1 t < 2.
Step 4: 2 t < 3. In this case f
2
(t ) f
3
( ) 24 for t 1 < < 2 and is zero otherwise. Hence,
the area beneath f
2
(t ) f
3
( ) equals
f
2
(t) f
3
(t) 24 2 (t 1) [ ] 24(3 t) for 2 t < 3.
Step 5: 3 t. Here f
2
(t ) f
3
( ) 0 for all . Therefore f
2
(t) f
3
(t) 0 for t < 0.
A picture of f
6
(t) is sketched in the next figure.
1/25/02 P16-36 R. A. DeCarlo, P. M. Lin
SOLUTION 16.27.
By the impulse response theorem, it follows that the response is
y(t) h(t) f (t)
h(t) (t) (t 1) [ ]
Using the distributive property of convolution and the sifting property of delta function y(t) can be
written as
y(t) h(t) h(t 1)
The right-hand side of the above equation interprets as a graphical sum of (shifted) pictures of h(t).
The pictures of h(t), h(t 1) and y(t) are sketched in the next figures.
SOLUTION 16.28.
(a) From the picture of f (t) and h(t) in figure P16.28 we observe that, in order to compute the
area beneath h(t ) f ( ) , we need to consider four cases: t < 0, 0 t < 4, 4 t < 8 and 8 t.
Step 1: t < 0. Here h(t ) f ( ) 0 for all . Therefore the area beneath h(t ) f ( ) equals zero
and
h(t) f (t) 0 for t < 0.
Step 2: 0 t < 4. In this case h(t ) f ( ) 1 for 0 t and is zero otherwise. Hence the area
beneath h(t ) f ( ) equals
h(t) f (t) t for 0 t < 4.
Step 3: 4 t < 8. In this case
h(t ) f ( )
1, t 4 < 4
2, 4 t
0, otherwise

'



Therefore the area beneath h(t ) f ( ) equals
h(t) f (t) 4 (t 4) [ ] + 2(t 4) t for 4 t < 8.
1/25/02 P16-37 R. A. DeCarlo, P. M. Lin
Step 4: 8 t. Here h(t ) f ( ) 2 for t 4 < t and is zero otherwise. Hence
h(t) f (t) 2 t (t 4) 8 [ ] for 8 t.
A picture of y(t) is sketched in the next figure.
(b) The impulse response is
h(t) u(t) u(t 4)
By the impulse response theorem
y(t) x(t) h(t) x( )h(t )d

x( ) u(t ) u(t 4) [ ]

d
Here observe that u(t ) u(t 4) is nonzero only when t < t 4.
Therefore
y(t) x( )d
t
t 4

which interprets as the running area under x(t) over the interval [t 4, t].
SOLUTION 16.29.
The response, y(t), is obtained as indicated in the statement of the problem, by using the following
MATLAB code:
>> tstep = 1;
>> vin = [1];
>> h = [0, 2, 3, 1, 1];
>> y = tstep*conv(vin, h);
>> y = [0 y 0];
>> t = 0:tstep:tstep*(length(vin)+length(h));
>> plot(t,y)
>> grid
The response is plotted in the next figure.
1/25/02 P16-38 R. A. DeCarlo, P. M. Lin
SOLUTION 16.30.
A picture of v
in
(t) sketched in the next figure.
In order to plot the response, y(t), the MATLAB code of problem 16.29 will be used with only one
modification. Namely
vin = [1, 1, 2, 2]
as it can be observed from the picture of v
in
(t) with the time step tstep = 1.
Therefore the MATLAB code is:
>> tstep = 1;
>> vin = [1, 1, 2, 2];
>> h = [0, 2, 3, 1, 1];
>> y = tstep*conv(vin,h);
>> y = [0 y 0];
>> t = 0:tstep:tstep*(length(vin)+length(h));
>> plot(t,y)
>> grid
The response is plotted in the next figure.
1/25/02 P16-39 R. A. DeCarlo, P. M. Lin
1/25/02 P16-1 R. A. DeCarlo, P. M. Lin
SOLUTION 16.31.
In order to compute the area beneath v(t )h( ) seven regions will be considered: t < 0, 0 t <1,
1 t < 2, 2 t < 3, 3 t < 4, 4 t < 5 and 5 t.
Step 1: t < 0. For t in this region v(t )h( ) 0 for all . Hence
y(t) v(t) h(t) 0 for t < 0.
Step 2: 0 t <1. In this case v(t )h( ) v0 h0 for 0 t and is zero otherwise. Therefore the
area beneath v(t )h( ) equals
y(t) v(t) h(t) v0 h0 t for 0 t <1.
Step 3: 1 t < 2. For t in this region we have
v(t )h( )
v1 h0, 0 t 1
v0 h0, t 1< < 1
v0 h1, 1 < t
0, otherwise

'





Therefore the area beneath v(t )h( ) equals
y(t) v(t) h(t) v1 h0 (t 1) 0 [ ] + v0 h0 1 (t 1) [ ] + v0 h1 (t 1)
t (v1 h0 v0 h0 + v0 h1) v1 h0 + 2 v0 h0 v0 h1, for 1 t < 2
Step 4: 2 t < 3. In this case
v(t )h( )
v1 h0, t 2 < < 1
v1 h1, 1 t 1
v0 h1, t 1< < 2
v0 h2, 2 < t
0, otherwise

'







Hence, for 2 t < 3,
y(t) v(t) h(t)
v1 h0 1 (t 2) [ ] + v1 h1 (t 1)1 [ ] + v0 h1 2 (t 1) [ ] + v0 h2 (t 2)
t (v1 h0 + v1 h1 v0 h1+ v0 h2) + 3 v1 h0 2 v1 h1 + 3 v0 h1 2 v0 h2
Step 5: 3 t < 4. In this case
v(t )h( )
v1 h1, t 2 < < 2
v1 h2, 2 t 1
v0 h2, t 1< < 3
0, otherwise

'





Hence, for 3 t < 4,
y(t) v(t) h(t)
v1 h1 2 (t 2) [ ] + v1 h2 (t 1) 2 [ ] + v0 h2 3 (t 1) [ ]
t (v1 h1 + v1 h2 v0 h2) + 4 v1 h1 3 v1 h2 + 4 v0 h2
Step 6: 4 t < 5. In this case v(t )h( ) v1 h2 for t 2 < < 3 and is zero otherwise. Therefore
y(t) v(t) h(t) v1 h2 3 (t 2) [ ] v1 h2 (5 t) for 4 t < 5.
1/25/02 P16-2 R. A. DeCarlo, P. M. Lin
Step 7: 5 t. For t in this region v(t )h( ) 0 for all . Hence
y(t) v(t) h(t) 0 for 5 t.
In sum,
y( t)
v0 h0 t , 0 t < 1
t (v1 h0 v0 h0+ v0 h1) v1 h0+ 2 v0 h0 v0 h1, 1 t < 2
t (v1 h0 + v1 h1 v0 h1+ v0 h2) + 3 v1 h0 2 v1 h1+ 3 v0 h1 2 v0 h2, 2 t < 3
t (v1 h1+ v1 h2 v0 h2) + 4 v1 h1 3 v1 h2+ 4 v0 h2, 3 t < 4
v1 h2 (5 t), 4 t < 5
0, otherwise

'









Hence,
y
1
y(1) v0 h0 6
y
2
y(2) v0 h1+ v1 h0 8
y
3
y(3) v0 h2 + v1 h1 6
y
4
y(4) v1 h2 4
(b) Using the expressions of p(x) and q(x) it follows that
p(x) q(x) x
3
(v0 h0) + x
2
(v0 h1+ v1 h0) + x (v0 h0 + v1 h1)+ v1 h2
We observe that the coefficients of p(x) q(x) are exactly y
1
, y
2
, y
3
and y
4
, respectively. Therefore
r(x) p(x) q(x) .
SOLUTION 16.32.
(a) This part will be solved using the techniques of convolution algebra. Therefore we can write f
3
(t) as
f
3
(t) f
1
(1)
(t) f
2
(1)
(t)
Where the superscript (-1) means integration and the superscript (1) means differentiation. From figure
P16.32 we observe that
f
1
(t) 4 u(t) u(t 4) [ ]
Hence
f
1
(1)
(t) 4 tu(t) (t 4)u(t 4) [ ]
4 r(t) r(t 4) [ ]
By inspection, from the same figure, we have
f
2
(1)
(t) 4 (t) 2 (t 2) + 2 (t 4) 2 (t 6) + (t 8) [ ]
Using the sifting property of the delta function f
3
(t) can be computed as follows
f
3
(t) 4 r(t) r(t 4) [ ] { } 4 (t) 2 (t 2) + 2 (t 4) 2 (t 6) + (t 8) [ ] { }
16 r(t) 2r(t 2) + r(t 4) r(t 8) + 2r(t 10) r(t 12) [ ]
A picture of f
3
(t) is sketched in the next figure.
1/25/02 P16-3 R. A. DeCarlo, P. M. Lin
(b) Using the techniques of problem 16.31 and considering the time step tstep = 2, the polynomials p(x) ,
q(x) and r(x) can be associated with the functions f
1
(t), f
2
(t) and
f
3
(t), respectively, as below:
p(x) 4x + 4
q(x) 4 x
3
4 x
2
+ 4 x 4
r(x) 32x
4
32
We need to verify that the equality
p(x) q(x) tstep r(x)
holds. The equality indeed holds because
p(x) q(x) tstep 32(x +1)(x
3
x
2
+ x 1) 32(x
4
1) r(x).
The results obtained in part (a) and part (b) coincide.
SOLUTION 16.33.
(a) Using the techniques of convolution algebra f
3
(t) can be written as
f
3
(t) f
1
(1)
(t) f
2
(1)
(t)
Where the superscript (-1) means integration and the superscript (1) means differentiation. From figure
P16.33 we observe that
f
1
(t) 2 u(t +1) u(t 4) [ ]
Therefore
f
1
(1)
(t) 4 (t +1)u(t +1) (t 4)u(t 4) [ ]
4 r(t +1) r(t 4) [ ] g(t)
By inspection, from the same figure, we have
f
2
(1)
(t) 4 (t) 8 (t 2) + 6 (t 4) 2 (t 6)
Using the sifting property of the delta function f
3
(t) can be computed as follows
f
3
(t) 4g(t) 8g(t 2) + 6g(t 4) 2g(t 6)
1/25/02 P16-4 R. A. DeCarlo, P. M. Lin
This is plotted in MATLAB as follows:
>> t = -2:0.01:14;
>> g = 2*(t+1).*u(t+1)-2*(t-4).*u(t-4);
>> g1 = 4*g;
>> g2 = -8*( 2*(t-1).*u(t-1)-2*(t-6).*u(t-6) );
>> g3 = 6*( 2*(t-3).*u(t-3)-2*(t-8).*u(t-8) );
>> g4 = -2*( 2*(t-5).*u(t-5)-2*(t-10).*u(t-10) );
>> f3 = g1+g2+g3+g4;
>> plot(t,f3);
>> grid;
A picture of f
3
(t) is sketched in the next figure.
(b) To account for the fact that f
1
(t)is nonzero for negative t the following formula (see problem 16.22)
f
1
(t) f
2
(t) f
1
(t 1) f
2
(t) [ ]
t t +1
will be used to compute f
3
(t). Using a slightly modified version of the code of problem 16.31, we have
>> f1 = [2, 2, 2, 2, 2];
>> f2 = [4, 4, -4, -4, 2, 2];
>> T = 1;
>> tstep = T;
>> f3 = tstep*conv(f1,f2);
>> f3 = [0 f3 0];
>> t = -1:tstep:tstep*(length(f1)+length(f2))-1;
>> plot(t,f3)
>> grid
1/25/02 P16-5 R. A. DeCarlo, P. M. Lin
The results of parts (a) and (b) coincide.
SOLUTION 16.34. This problem is solved using the techniques of the convolution algebra with the
graphical method left to the student.
f
3
(t) f
1
(t) * f
2
(t) f
1
(t) [ ]
(1)
* f
2
(t) [ ]
(1)
where the superscript (-1) means integration and the superscript (1) means differentiation. By inspection,
f
1
(t ) [ ]
(1)
4tu(t) 4(t 6)u(t 6) g(t)
and
f
2
(t) [ ]
(1)
4(t) 8(t 2) + 8(t 6) 4(t 8)
Hence the response say y(t) satisfies
f
3
(t) 4g(t) 8g(t 2) +8g(t 6) 4g(t 8)
This is plotted in MATLAB as follows:
t=0:.05:20;
g = 4*t .* u(t) - 4*(t-6) .*u(t-6);
g1=4*g;
g2 = -8*(4*(t-2) .* u(t-2) - 4*(t-8) .*u(t-8));
g3 = 8*(4*(t-6) .* u(t-6) - 4*(t-12) .*u(t-12));
g4 = -4*(4*(t-8) .* u(t-8) - 4*(t-14) .*u(t-14));
f3 = g1+g2+g3+g4;
plot(t,f3)
grid
1/25/02 P16-6 R. A. DeCarlo, P. M. Lin
0 2 4 6 8 10 12 14 16 18 20
-40
-30
-20
-10
0
10
20
30
40
(b) Using the code of problem 31, we have
f1 = [4 4 4];
f2 = [4 -4 -4 4];
T = 2;
tstep = T;
f3 = [0 conv(f1,f2)*tstep 0];
t = 0: tstep : tstep* (length(f1) + length(f2));
plot(t,f3)
grid
1/25/02 P16-7 R. A. DeCarlo, P. M. Lin
0 2 4 6 8 10 12 14
-40
-30
-20
-10
0
10
20
30
40
The results of parts (a) and (b) coincide.
SOLUTION 16.35.
(a) In order to compute the area beneath f
1
(t ) f
1
( ) four regions will be considered: t < 0, 0 t <1,
1 t < 2, and 2 t.
Step 1: t < 0. Here f
1
(t ) f
1
( ) 0 for all . Hence
f
3
(t) f
1
(t) f
1
(t) 0 for t < 0.
Step 2: 0 t <1. In this case f
1
(t ) f
1
( ) 1 for 0 t and is zero otherwise. Therefore the area
beneath f
1
(t ) f
1
( ) equals
f
3
(t) t for 0 t <1.
Step 3: 1 t < 2. In this case f
1
(t ) f
1
( ) 1 for t 1 < <1 and is zero otherwise. Therefore the
area beneath f
1
(t ) f
1
( ) equals
f
3
(t) 1 (t 1) 2 t for 1 t < 2.
Step 4: 2 t. Here f
1
(t ) f
1
( ) 0 for all . Hence
f
3
(t) f
1
(t) f
1
(t) 0 for2 t.
In sum,
f
3
(t)
t, 0 t <1
2 t, 1 t < 2
0, otherwise

'



1/25/02 P16-8 R. A. DeCarlo, P. M. Lin
(b) In order to compute the area beneath f
1
(t ) f
2
( ) four regions will be considered: t < 0, 0 t <1,
1 t < 2, and 2 t.
Step 1: t < 0. Here f
1
(t ) f
2
( ) 0 for all . Hence
f
4
(t) f
1
(t) f
2
(t) 0 for t < 0.
Step 2: 0 t <1. In this case f
1
(t ) f
2
( ) for 0 t and is zero otherwise. Therefore the area
beneath f
1
(t ) f
2
( ) equals
f
4
(t) 0.5t
2
for 0 t <1.
Step 3: 1 t < 2. For t in this region f
1
(t ) f
2
( ) for t 1 < <1 and is zero otherwise.
Therefore the area beneath f
1
(t ) f
2
( ) equals
f
4
(t) 0.5 0.5t
2
for 1 t < 2.
Step 4: 2 t. Here f
1
(t ) f
2
( ) 0 for all . Hence
f
4
(t) f
1
(t) f
1
(t) 0 for2 t.
In sum,
f
4
(t)
0.5t
2
, 0 t < 1
0.5 0.5t
2
, 1 t < 2
0, otherwise

'





(c) In order to compute the area beneath f
2
(t ) f
2
( ) four regions will be considered: t < 0, 0 t <1,
1 t < 2, and 2 t.
Step 1: t < 0. Here f
2
(t ) f
2
( ) 0 for all . Hence
f
5
(t) f
2
(t) f
2
(t) 0 for t < 0.
Step 2: 0 t <1. In this case f
2
(t ) f
2
( ) (t ) for 0 t and is zero otherwise. Therefore
the area beneath f
2
(t ) f
2
( ) equals
f
5
(t) (t ) d 0.3333
3
+ 0.5t
2
[ ]
0
t

0
t

0.1667t
3
for 0 t <1.
Step 3: 1 t < 2. For t in this region f
2
(t ) f
2
( ) (t ) for t 1 < <1 and is zero otherwise.
Therefore the area beneath f
2
(t ) f
2
( ) equals
f
5
(t) (t ) d
t 1
1

0.3333
3
+ 0.5t
2
[ ]
t 1
1
0.1667t
3
+ t 0.6667 for 1 t < 2.
Step 4: 2 t. Here f
2
(t ) f
2
( ) 0 for all . Hence
1/25/02 P16-9 R. A. DeCarlo, P. M. Lin
f
5
(t) f
2
(t) f
2
(t) 0 for2 t.
In sum,
f
5
(t)
0.1667t
3
, 0 t <1
0.1667t
3
+ t 0.6667, 1 t < 2
0, otherwise

'





SOLUTION 16.36.
In order to compute the area beneath f
1
(t ) f
2
( ) five regions will be considered: t < 0, 0 t <1,
1 t < 2, 2 t < 3and 3 t.
Step 1: t < 0. Here f
1
(t ) f
2
( ) 0 for all . Hence
f
3
(t) f
1
(t) f
2
(t) 0 for t < 0.
Step 2: 0 t <1. In this case f
1
(t ) f
2
( ) for 0 t and is zero otherwise. Therefore the area
beneath f
1
(t ) f
2
( ) equals
f
3
(t) 0.5t
2
for 0 t <1.
Step 3: 1 t < 2. For t in this region
f
1
(t ) f
2
( )
, t 1< < 1
2 , 1 t
0, otherwise

'



Therefore the area beneath f
1
(t ) f
2
( ) equals
f
3
(t) 0.5 0.5(t 1)
2
[ ]
+ 0.5 0.5(2 t)
2
[ ]
t
2
+ 3t 1.5 for 1 t < 2.
Step 4: 2 t < 3. For t in this region f
1
(t ) f
2
( ) 2 for all t 1 < < 2. Hence
f
3
(t) f
1
(t) f
2
(t) 0.5 0.5(2 t)
2
0.5t
2
+ 2t 1.5 for 2 t < 3.
Step 5: 3 t. Here f
1
(t ) f
2
( ) 0 for all . Hence
f
3
(t) f
1
(t) f
2
(t) 0 for 3 t.
In sum,
f
3
(t)
0.5t
2
, 0 t < 1
t
2
+ 3t 1.5, 1 t < 2
0.5t
2
+ 2t 1.5, 2 t < 3
0, otherwise

'





SOLUTION 16.37.
1/25/02 P16-10 R. A. DeCarlo, P. M. Lin
(a) In order to compute the area beneath f
1
( ) f
2
(t ) three regions will be considered: t < 0,
0 t < 2, and 2 t.
Step 1: t < 0. Here f
1
( ) f
2
(t ) 0 for all . Hence
f
3
(t) f
1
(t) f
2
(t) 0 for t < 0.
Step 2: 0 t < 2. In this case f
1
( ) f
2
(t ) 8 for 0 t and is zero otherwise. Therefore the
area beneath f
1
( ) f
2
(t ) equals
f
3
(t) 0.5(t 8t) 4t
2

for 0 t < 2.
Step 3: 2 t. For t in this region
f
1
( ) f
2
(t )
8 , 0 < 2
16, 2 < t
0, otherwise

'



Therefore the area beneath f
1
( ) f
2
(t ) equals
f
3
(t) 16 +16(t 2) 16(t 1) for 2 t.
In sum,
f
3
(t)
4t
2
, 0 t < 2
16(t 1), 2 t
0, otherwise

'





A picture of f
3
(t) is sketched in the next figure.
(b) From figure P16.37 we observe that
f
1
(t) 2tu(t) 2(t 2)u(t 2)
From table 13.1 and the time shift property of the Laplace transform it follows that
F
1
(s)
2
s
2
1 e
2s
( )
1/25/02 P16-11 R. A. DeCarlo, P. M. Lin
F
2
(s)
4
s
By the convolution theorem
F
3
(s) F
1
(s)F
2
(s)
Therefore
F
3
(s)
8
s
3
1 e
2s
( )
Taking the inverse Laplace transform yields
f
3
(t) 4t
2
u(t) 4(t 2)
2
u(t 2)
The results of parts (a) and (b) coincide.
SOLUTION 16.38.
(a) In order to compute the area beneath f
1
(t ) f
2
( ) six regions will be considered: t < 0, 0 t < 2,
2 t < 6, 6 t < 8, 8 t <10, and 10 t.
Step 1: t < 0. Here f
1
(t ) f
2
( ) 0 for all . Hence
f
3
(t) f
1
(t) f
2
(t) 0 for t < 0.
Step 2: 0 t < 2. In this case f
1
(t ) f
2
( ) 8(t ) for 0 t and is zero otherwise. Therefore
the area beneath f
1
( ) f
2
(t ) equals
f
3
(t) 0.5(t 8t) 4t
2

for 0 t < 2.
Step 3:2 t < 6. For t in this region
f
1
(t ) f
2
( )
16, 0 < t 2
8(t ), t 2 < t
0, otherwise

'



Therefore the area beneath f
1
(t ) f
2
( ) equals
f
3
(t) 16 +16(t 2) 16(t 1) for 2 t < 6.
Step 4: 6 t < 8. For t in this region
f
1
(t ) f
2
( )
16, 0 < t 2
8(t ), t 2 < 6
8(t ), 6 < t
0, otherwise

'





Therefore, for 6 t < 8, the area beneath f
1
(t ) f
2
( ) equals
f
3
(t) 16(t 2) + 16
8(t 6)
2
2




1
]
1
1

8(t 6)
2
2
1/25/02 P16-12 R. A. DeCarlo, P. M. Lin
8(t 6)
2
+16t 16 8t
2
+112t 304
Step 5: 8 t <10. For t in this region
f
1
(t ) f
2
( )
16, 0 < 6
16, 6 < t 2
8(t ), t 2 < 8
0, otherwise

'





Therefore, for 8 t <10, the area beneath f
1
(t ) f
2
( ) equals
f
3
(t) 96 16(t 8) 16
8(t 8)
2
2




1
]
1
1
4t
2
80t + 464
Step 6: 10 t. For t in this region
f
1
(t ) f
2
( )
16, 0 < 6
16, 6 < 8
0, otherwise

'



Therefore,
f
3
(t) 96 32 64 for 10 t.
In sum,
f
3
(t)
0, t < 0
4t
2
, 0 t < 2
16t 16, 2 t < 6
8t
2
+112t 304, 6 t < 8
4t
2
80t + 464, 8 t < 10
64, 10 t

'









A picture of f
3
(t) is sketched in the next figure.
(b) From figure P16.38 we observe that
1/25/02 P16-13 R. A. DeCarlo, P. M. Lin
f
2
(t) 4 u(t) u(t 6) [ ] 4 u(t 6)u (t 8) [ ]
4u(t) 8u(t 6) + 4u(t 8)
From table 13.1 and the time shift property of the Laplace transform it follows that
F
2
(s)
4
s

8
s
e
6s
+
4
s
e
8s
From problem 16.37 we have that
F
1
(s)
2
s
2
1 e
2s
( )
By the convolution theorem
F
3
(s) F
1
(s)F
2
(s)
Therefore
F
3
(s)
8
s
3

8
s
3
e
2s

16
s
3
e
6s
+
24
s
3
e
8s

8
s
3
e
10s
Taking the inverse Laplace transform yields
f
3
(t) 4t
2
u(t) 4(t 2)
2
u(t 2) 8(t 6)
2
u(t 6)
+12(t 8)
2
u(t 8) 4(t 10)
2
u(t 10)
The results of parts (a) and (b) coincide.
SOLUTION 16.39.
p(t) q(t) will be computed using the techniques of convolution algebra. Therefore we can write
p(t) q(t) p
(1)
(t) q
(1)
(t)
where the superscript (1)means integration and the superscript (1)means differentiation. By inspection
p(t) (t + 4) u(t + 4) u(t) [ ] + (t + 4) u(t) u(t 4) [ ] + 4 u(t 4) u(t 8) [ ]
(t + 4)u(t + 4) + (2t)u(t) + tu(t 4) 4u(t 8)
Therefore,
p
(1)
(t) 0.5(t + 4)
2
u(t + 4) t
2
u(t) + (0.5t
2
8)u(t 4) (4t 32)u(t 8)
By inspection we also have
q
(1)
(t) 4 (t)
By the sifting property of the delta function it follows that
p(t) q(t) 4 p
(1)
(t)
2(t + 4)
2
u(t + 4) 4t
2
u(t) + 2(t
2
16)u(t 4) 16(t 8)u(t 8).
SOLUTION 16.40.
(a) First observe that
1/25/02 P16-14 R. A. DeCarlo, P. M. Lin
h(t) 0.1u(t 0.1) + 0.2u(t 0.2) + 0.2u(t 0.3) + 0.2u(t 0.4) + 0.2u(t 0.5) +
+0.1u(t 0.6) 0.1u(t 1) 0.2u(t 1.3) 0.2u(t 1.5)
0.2u(t 1.7) 0.2u(t 2) 0.1u(t 2.2)
Due to the fact that h(t) is a linear combination of terms of the type Ku(t T), the convolution of h(t)
and v
in
(t) reduces to a linear combination of terms of the following type: K
1
u(t) [ ] K
2
u(t T) [ ]. Using
the definition of the convolution, the previous convolution product is computed below
K
1
u(t) [ ] K
2
u(t T) [ ] K
1
K
2
u( )u(t T)d

K
1
K
2
u(t T)d
0

0, t < T
K
1
K
2
d
0
t T

, T t

'


0, t < T
K
1
K
2
(t T), T t

'

K
1
K
2
(t T)u(t T)
Therefore v
out
(t) is a linear combination of functions of type K
1
K
2
(t T)u(t T),
v
out
(t) h(t) v
in
(t)
10(t 0.1)u(t 0.1)+ 20(t 0.2)u(t 0.2) + 20(t 0.3)u(t 0.3) + 20(t 0.4)u(t 0.4) +
+20(t 0.5)u(t 0.5) +10(t 0.6)u(t 0.6) 10(t 1)u(t 1) 20(t 1.3)u(t 1.3)
20(t 1.5)u(t 1.5) 20(t 1.7)u(t 1.7) 20(t 2)u(t 2) 10(t 2.2)u(t 2.2)
At t 0s
v
out
(0) 0 V.
At t 0.5s
v
out
(0.5) 16 V.
At t 1s
v
out
(1) 65 V.
At t 1.5s
v
out
(1.5) 106 V.
(b) In this case v
out
(t) will be computed using the techniques of the convolution algebra. Hence we have
v
out
(t) v
in
(t) h(t) v
in
(1)
(t) h
(1)
(t)
50t
2
u(t)
[ ]
0.1 (t 0.1) + 0.2 (t 0.2) + 0.2 (t 0.3) + 0.2 (t 0.4) + [
+0.2 (t 0.5) + 0.1 (t 0.6) 0.1 (t 1) 0.2 (t 1.3)
0.2 (t 1.5) 0.2 (t 1.7) 0.2 (t 2) 0.1 (t 2.2)]
Using the sifting property of delta function it follows that
+ + + ) 3 . 0 ( ) 3 . 0 ( 10 ) 2 . 0 ( ) 2 . 0 ( 10 ) 1 . 0 ( ) 1 . 0 ( 5 ) (
2 2 2
t u t t u t t u t t v
out
+10(t 0.4)
2
u(t 0.4) +10(t 0.5)
2
u(t 0.5) + 5(t 0.6)
2
u(t 0.6)
5(t 1)
2
u(t 1)10(t 1.3)
2
u(t 1.3) 10(t 1.5)
2
u(t 1.5)
10(t 1.7)
2
u(t 1.7) 10(t 2)
2
u(t 2) 5(t 2.2)
2
u(t 2.2) V.
1/25/02 P16-15 R. A. DeCarlo, P. M. Lin
At t 1s
v
out
(1) 22.25 V.
(c) From the expression of v
out
(t)obtained in part (a) we observe that v
out
(t)does not change after
t 2.2s. Therefore it is sufficient to compute v
out
(t)for t 2.2s. Hence v
in
(t) can be considered to be
equal to
v
in
(t) 100 u(t) u(t 2.2) [ ] V.
Using the code of problem 16.31 we have
>> vin = 100*ones(1,22);
>> h = [0, 0.1, 0.3, 0.5, 0.7, 0.9, 1, 1, 1, 1, 0.9, 0.9, 0.9, 0.7, 0.7, 0.5, 0.5, 0.3, 0.3,
0.3, 0.1, 0.1];
>> T = 0.1;
>> tstep = T;
>> y = tstep*conv(vin,h);
>> y = [0 y 0];
>> t = 0:tstep:tstep*(length(h)+length(vin));
% After t = 2.2s vout(t) does not change
>> t = t(1:length(h)+1);
>> y = y(1:length(h)+1);
>> plot(t,y)
>> grid
A picture of y(t) is sketched in the next figure.
Using the previous MATLAB code we the values of y(t) at the specified instants of time are:
At t 0s
y(0) 0 V
1/25/02 P16-16 R. A. DeCarlo, P. M. Lin
At t 0.5s
y(0.5) 16 V
At t 1s
y(1) 65 V
At t 1.5s
y(1.5) 106 V
The results of parts (a) and (c) coincide.
SOLUTION 16.41.
Using the MATLAB code of problem 16.31 we have:
>> vin = [1];
>> h = [9, -6, 3, -2];
>> T = 1;
>> tstep = T;
>> y = tstep*conv(vin,h);
>> y = [0 y 0];
>> t = 0:tstep:tstep*(length(h)+length(vin));
>> plot(t,y)
>> grid
The breakpoints in y(t) of the above figure are [9, -6, 3 2] as expected because the polynomial
associated with v
in
(t) is the constant 1 and the polynomial associated with h(t) is the polynomial
9x
3
6x
2
+ 3x 2, as it can be observed from figure P16.41.
SOLUTION 16.42.
(a) Let v
out ,40
(t) denote the response that has been obtained in problem 16.40, part (a), to the input
100u(t) .
The expression of v
out ,40
(t) is (see problem 16.40, part (a)):
v
out ,40
(t) h(t) 100u(t) [ ]
10(t 0.1)u(t 0.1)+ 20(t 0.2)u(t 0.2) + 20(t 0.3)u(t 0.3) + 20(t 0.4)u(t 0.4) +
1/25/02 P16-17 R. A. DeCarlo, P. M. Lin
+20(t 0.5)u(t 0.5) +10(t 0.6)u(t 0.6) 10(t 1)u(t 1) 20(t 1.3)u(t 1.3)
20(t 1.5)u(t 1.5) 20(t 1.7)u(t 1.7) 20(t 2)u(t 2) 10(t 2.2)u(t 2.2)
Using the distributive property of the convolution product and the time invariance property it follows that
v
out
(t) h(t) v
in
(t) h(t) 100u(t) 100u(t 0.2) [ ]
h(t) 100u(t) [ ] h(t) 100u(t 0.2) [ ]
v
out,40
(t) v
out,40
(t 0.2)
Using the above expression of v
out ,40
(t) we have:
v
out ,40
(0) 0 V and v
out ,40
(0.2) 0V,
v
out ,40
(0.5) 16 V and v
out ,40
(0.3) 4 V,
v
out ,40
(1) 65 V and v
out ,40
(0.8) 45V,
v
out ,40
(1.5) 106 V and v
out ,40
(1.3) 92V.
At t 0s
v
out
(0) v
out ,40
(0) v
out ,40
(0.2) 0 V.
At t 0.5s
v
out
(0.5) v
out ,40
(0.5) v
out ,40
(0.3) 12 V.
At t 1s
v
out
(1) v
out ,40
(1) v
out ,40
(0.8) 20 V.
At t 1.5s
v
out
(1.5) v
out ,40
(1.5) v
out ,40
(1.3) 14 V.
(b) In this case v
out
(t) will be computed using the techniques of convolution algebra.
We have
v
out
(t) v
in
(t) h(t) v
in
(1)
(t) h
(1)
(t)
where the superscript (-1) means integration and the superscript (1) means differentiation.
From figure P16.42 observe that
v
in
(t) 100t u(t) u(t 0.5) [ ] +100(1 t) u(t 0.5) u(t 1) [ ]
100tu(t) +100(1 2t)u(t 0.5) +100(t 1)u(t 1)
Therefore
v
in
(1)
(t) 50t
2
u(t) + (100t
2
+100t 25)u(t 0.5) + (50t
2
100t + 50)u(t 1) g(t)
By the sifting property of the delta function we have
v
out
(t) v
in
(1)
(t) h
(1)
(t) g(t) h
(1)
(t)
g(t) 0.1 (t 0.1) + 0.2 (t 0.2) + 0.2 (t 0.3) + 0.2 (t 0.4) + [
+0.2 (t 0.5) + 0.1 (t 0.6) 0.1 (t 1) 0.2 (t 1.3)
0.2 (t 1.5) 0.2 (t 1.7) 0.2 (t 2) 0.1 (t 2.2)]=
0.1g(t 0.1)+ 0.2g(t 0.2) + 0.2g(t 0.3) + 0.2g(t 0.4) +
+0.2g(t 0.5) + 0.1g(t 0.6) 0.1g(t 1) 0.2g(t 1.3)
1/25/02 P16-18 R. A. DeCarlo, P. M. Lin
0.2g(t 1.5) 0.2g(t 1.7) 0.2g(t 2) 0.1g(t 2.2)
The values ofv
out
(t) at the specified instants of time can be computed using MATLAB. The results are:
v
out
(0) 0V
v
out
(0.5) 2.2V
v
out
(1) 17.85V
v
out
(1.5) 23.3V.
SOLUTION 16.43.
Using the techniques of convolution algebra we have
v
out
(t) h(t) v
in
(t) h
(1)
(t) v
in
(1)
(t)
where the superscript (-1) means integration and the superscript (1) means differentiation.
We have
h
(1)
(t) 2(1 e
2t
)u(t)
and
v
in
(1)
(t) (t) (t 1)
Using the sifting property of the delta function it follows that
v
out
(t) 2(1 e
2t
)u(t)
[ ]
(t) (t 1) [ ]
2(1 e
2t
)u(t) 2(1 e
2(t 1)
)u(t 1)
A picture of v
out
(t)is sketched in the next figure.
SOLUTION 16.44.
(a) From table 13.1 it follows that the Laplace transform of v
in
(t) is
V
in
(s)
1
s

1
s +1

1
s + 2

s
2
+ 2
s(s +1)(s + 2)
1/25/02 P16-19 R. A. DeCarlo, P. M. Lin
And the Laplace transform of v
out
(t) is
V
out
(s)
1
s

2
s +1

1
(s +1)
2
+
1
s + 2

s
2
+ 2
s(s +1)
2
(s + 2)
Therefore the transfer function of the circuit is
H(s)
V
out
(s)
V
in
(s)

1
s +1
A simple RC circuit that represents this transfer function is a series RC circuit with R 1 and C 1F .
v
out
(t)is represented by the capacitor voltage and v
in
(t) is the source voltage.
(b) The impulse response of the circuit is
h(t) L
1
H(s) [ ] L
1
1
s +1



1
]
1
e
t
u(t).
(c) Assuming zero initial conditions we have
v
out
(t) v
in
(t) h(t) v
in
(1)
(t) h
(1)
(t) (t) (1 e
t
)u(t)
[ ]
(1 e
t
)u(t) V.
(d) Using the techniques of convolution algebra the zero-state response can be computed as
v
out
(t) v
in
(t) h(t) v
in
(2)
(t) h
(2)
(t)
where the superscript (2) means double differentiation and the superscript (-2) means double integration.
First, from figure P16.44, observe that
v
in
(1)
(t) u(t 1) u(t 2) [ ] + u(t 3) u(t 4) [ ]
Therefore
v
in
(2)
(t) (t 1) (t 2) + (t 3) (t 4)
h
(2)
(t) is computed as the integral of h
(1)
(t) .
h
(2)
(t) h
(1)
( )d

(1 e

)u( )d

(1 e

)d
0

(t + e
t
1)u(t)
The zero-state response can now be computed
v
out
(t) v
in
(2)
(t) h
(2)
(t)
(t 1) (t 2) + (t 3) (t 4) [ ] (t + e
t
1)u(t)
[ ]
By the sifting property of the delta function it follows that
v
out
(t) t 2 + e
(t 1)
( )
u(t 1) t 3 + e
(t 2)
( )
u(t 2)
+ t 4 + e
(t 3)
( )
u(t 3) t 5 + e
(t 4)
( )
u(t 4) V.
1/25/02 P16-20 R. A. DeCarlo, P. M. Lin
SOLUTION 16.45.
Using the techniques of the convolution algebra we have
y(t) f (t) g(t) f
(2)
(t) g
(2)
(t)
where
g
(1)
(t)
2
cos( )d sin( t)u(t)
0

and
g
(2)
(t)
2
sin( )d 1 cos( t) [ ]u(t)
0

Differentiating f (t) twice leads to


f
(2)
(t) (t) + 2 (t 1) 2 (t 3)+ (t 4)
Therefore
y(t) f
(2)
(t) g
(2)
(t)
(t) + 2 (t 1) 2 (t 3) + (t 4) [ ] 1 cos( t) [ ]u(t) { }
Using the sifting property of the delta function it follows that
y(t) 1 cos( t) [ ]u(t) + 2 1 cos[ (t 1)] { }u(t 1)
2 1 cos[ (t 3)] { }u(t 3) + 1 cos[ (t 4)] { }u(t 4)
Simplifying the expression of y(t) yields
y(t) 1 cos( t) [ ] u(t) u(t 4) [ ] + 2 1 + cos( t) [ ] u(t 1) u(t 3) [ ].
A picture of y(t) is sketched in the next figure.
1/25/02 P16-21 R. A. DeCarlo, P. M. Lin
SOLUTION 16.46.
(a) By the current division formula
I
C
(s)
Cs
Cs +
1
Ls
I
in
(s)
s
2
s
2
+1
I
in
(s)
The transfer function of the circuit can be computed as below
H(s)
V
C
(s)
I
in
(s)

I
C
(s) Z
C
(s)
I
in
(s)

s
2
s
2
+1

1
s

s
s
2
+1
(b) The impulse response is computed as the inverse Laplace transform of the transfer function
h(t) L
1
H(s) [ ] cos(t)u(t).
(c) Assuming zero initial conditions it follows, by the impulse response theorem, that
v
out
(t) i
in
(t) h(t)
Using the techniques of the convolution algebra we have
v
out
(t) i
in
(2)
(t) h
(2)
(t)
By inspection
i
in
(1)
(t) u(t) u(t 2 ) [ ] + u(t 4 ) u(t 6 ) [ ]
Therefore
i
in
(2)
(t) (t) (t 2 ) + (t 4 ) (t 6 )
And
h
(1)
(t) cos( )d sin(t)u(t)
0

Hence
h
(2)
(t) sin( )d 1 cos(t) [ ]u(t)
0

Using the sifting property of the delta function we have


v
out
(t) (t) (t 2 ) + (t 4 ) (t 6 ) [ ] 1 cos(t) [ ]u(t) { }
1 cos(t) [ ]u(t) 1 cos(t 2 ) [ ]u(t 2 )
+ 1 cos(t 4 ) [ ]u(t 4 ) 1 cos(t 6 ) [ ]u(t 6 )
1 cos(t) [ ] u(t) u(t 2 ) + u(t 4 ) u(t 6 ) [ ]
1/25/02 P16-22 R. A. DeCarlo, P. M. Lin
V.
(d) A picture of v
out
(t) is sketched in the next figure.
SOLUTION 16.47.
(a) The step response, v
out
(t), is computed using the convolution algebra techniques.
We have
v
out
(t) h(t) v
in
(t) h
(1)
(t) v
in
(1)
(t)
From figure P16.47 observe that
h(t) 2u(t) u(t 1) 2u(t 2) u(t 3)+ u(t 5) + 2u(t 6) + u(t 7) 2u(t 8)
Differentiating we have
h
(1)
(t) 2 (t) (t 1) 2 (t 2) (t 3) + (t 5) + 2 (t 6) + (t 7) 2 (t 8)
Since
v
in
(t) u(t),
by integration it follows that
v
in
(1)
(t) tu(t).
Using the sifting property of the delta function it follows that
v
out
(t) h
(1)
(t) v
in
(1)
(t)
2 (t) (t 1) 2 (t 2) (t 3) + (t 5) + 2 (t 6) + (t 7) 2 (t 8) [ ] tu(t) [ ]
2tu(t) (t 1)u(t 1) 2(t 2)u(t 2) (t 3)u(t 3) +
+(t 5)u(t 5) + 2(t 6)u(t 6) + (t 7)u(t 7) 2(t 8)u(t 8)V.
A picture of the step response is sketched in the next figure.
1/25/02 P16-23 R. A. DeCarlo, P. M. Lin
(b) Using the convolution algebra techniques we have
v
out
(t) h(t) v
in
(t) h
(1)
(t) v
in
(1)
(t)
For computing v
in
(1)
(t) we have
v
in
(1)
(t) e u( )d
e d

, t < 0
e d

, 0 t

'






e
t
, t < 0
1, 0 t

'



Using the sifting property of the delta function it follows that
v
out
(t) h
(1)
(t) v
in
(1)
(t)
2 (t) (t 1) 2 (t 2) (t 3) + (t 5) + 2 (t 6) + (t 7) 2 (t 8) [ ] v
in
(1)
(t)
[ ]

2v
in
(1)
(t) v
in
(1)
(t 1) 2v
in
(1)
(t 2) v
in
(1)
(t 3) +
+v
in
(1)
(t 5) + 2v
in
(1)
(t 6) + v
in
(1)
(t 7) 2v
in
(1)
(t 8) V.
Using the expression of v
in
(1)
(t) computed above it follows that
v
out
(7.5) 0.7869 V,
v
out
(6.5) 1.1603V,
v
out
(5.5) 0.2720V,
v
out
(0.5) 0.8848V.
SOLUTION 16.48.
First observe, from figure P16.48(a), that
h(t) (1 t) u(t) u(t 1) [ ]
1/25/02 P16-24 R. A. DeCarlo, P. M. Lin
(1 t)u(t) (1 t)u(t 1).
Using the convolution algebra techniques it follows that the step response y(t) is
y(t) v(t) h(t) v
(1)
(t) h
(1)
(t)
(t) h
(1)
(t) h
(1)
(t)
where
h
(1)
(t) (1 )u( )d (1 )u( 1)d

0.5
2
[ ]
0
t
u(t) 0.5
2
[ ]
1
t
u(t 1)
(t 0.5t
2
)u(t) (t 0.5t
2
0.5)u(t 1).
Hence the step response is
y(t) (t 0.5t
2
)u(t) (t 0.5t
2
0.5)u(t 1).
Let y
v
(t) denote the zero-state response to the input v(t).
From figure P16.48(b) we observe that
v(t) u(t) + u(t 1) 2u(t 2) .
Using the distributive property of the convolution it follows that
y
v
(t) h(t) v(t)
h(t) u(t) + u(t 1) 2u(t 2) [ ]
h(t) u(t) + h(t) u(t 1) 2 h(t) u(t 2).
Due to the fact that
y(t) h(t) u(t)
by the linearity and time invariance properties it follows that
y
v
(t) y(t) + y(t 1) 2y(t 2)
(t 0.5t
2
)u(t) + (t 1)u(t 1)+
+(1.5t
2
8t +12)u(t 2) + (0.5t
2
3t + 4)u(t 3).
SOLUTION 16.49.
(a) By the voltage division formula it follows that
V
out
(s)
1
Cs
1
Cs
+ Ls
V
in
(s)
1
LCs
2
+1
V
in
(s)
Therefore the transfer function is
H(s)
V
out
(s)
V
in
(s)

1
LCs
2
+1
Taking the inverse Laplace transform yields
h(t) L
1
1
LCs
2
+1



1
]
1

1
LC
L
1
1
LC
s
2
+
1
LC






1
]
1
1
1
1

1
LC
sin
1
LC
t



_
,
u(t) .
(b) The step response is computed as the convolution of the impulse response and the step function.
v
out
(t) h(t) u(t)
Using the techniques of the convolution algebra it follows that
1/25/02 P16-25 R. A. DeCarlo, P. M. Lin
v
out
(t) h
(1)
(t) u
(1)
(t)
where the superscript (-1) means integration and the superscript (1) means differentiation.
Taking the integral of h(t) we have
h
(1)
(t)
1
LC
sin
1
LC



_
,
u( )d

u(t) cos
1
LC



_
,




1
]
1
0
t
1 cos
1
LC
t



_
,




1
]
1
u(t) .
Therefore
v
out
(t) h
(1)
(t) u
(1)
(t)
1 cos
1
LC
t



_
,




1
]
1
u(t)

'


;

(t)
1 cos
1
LC
t



_
,




1
]
1
u(t) V.
(c) We denote by v
out
T
(t) the output to the rectangular pulse in figure P16.49(b).
Observe, from figure P16.49(b), that
v
in
(t)
1
T
u(t) u(t T) [ ] V.
By linearity and time invariance it follows that
v
out
T
(t)
1
T
v
out
(t) v
out
(t T) [ ]
where v
out
(t)is the step response obtained in part (b). Therefore
v
out
T
(t)
1
2 LC
1 cos
1
LC
t



_
,




1
]
1
u(t)
1
2 LC
1 cos
1
LC
t 2



_
,




1
]
1
u(t 2 LC)

1
2 LC
1 cos
1
LC
t



_
,




1
]
1
u(t) u(t 2 LC
[ ]
A picture of v
out
T
(t), for L 1Hand C 1F , is sketched in the next figure.
1/25/02 P16-26 R. A. DeCarlo, P. M. Lin
SOLUTION 16.50.
The impulse response of the configuration in the figure P16.50 is
h(t) h
1
(t) h
2
(t) + h
3
(t) [ ] h
4
(t)
Due to the fact that h
4
(t) 2 (t) , the sifting property of the delta function can be applied and it follows
that
h(t) 2 h
1
(t) h
2
(t) + h
3
(t) [ ]
Using the techniques of the convolution algebra we can further write
h(t) 2 h
1
(1)
(t) h
2
(t) + h
3
(t) [ ]
(1)

h(t) 2 h
1
(1)
(t) h
2
(1)
(t) + h
3
(1)
(t)
[ ]
We have
h
1
(1)
(t) u
(1)
(t) (t)
h
2
(1)
(t) 2e
2
u( )d u(t) e
2
[ ]
0
t
1 e
2t
( )
u(t)

h
3
(1)
(t) 8e
4
u( )d u(t) 2e
4
[ ]
0
t
2 1 e
4t
( )
u(t)

Substituting the expressions of h


1
(1)
(t) , h
2
(1)
(t) and h
3
(1)
(t) in the expression of h(t), and using the
sifting property of the delta function we have
h(t) 2 1 e
2t
( )
u(t) + 4 1 e
4t
( )
u(t).
A picture of h(t) is sketched in the next figure.
1/25/02 P16-27 R. A. DeCarlo, P. M. Lin
SOLUTION 16.51.
Observe first that h
2
(t) , h
3
(t) and h
4
(t) have the same expressions as in the problem 16.50. In problem
16.50 the following convolution has been computed
u(t) h
2
(t) + h
3
(t) [ ] h
4
(t) 2 1 e
2t
( )
u(t) + 4 1 e
4t
( )
u(t)
Using the time shift property of the convolution (see problem 16.22, part (a)) it follows that
u(t 2) h
2
(t) + h
3
(t) [ ] h
4
(t) 2 1 e
2(t 2)
( )
u(t 2) + 4 1 e
4( t 2)
( )
u(t 2)
Using the above expressions and the distributive property of the convolution it follows that the overall
impulse response can be computed as below
h(t) h
1
(t) h
2
(t) + h
3
(t) [ ] h
4
(t) u(t) u(t 2) [ ] h
2
(t) + h
3
(t) [ ] h
4
(t)
u(t) h
2
(t) + h
3
(t) [ ] h
4
(t) u(t 2) h
2
(t) + h
3
(t) [ ] h
4
(t)
21 e
2t
( )
u(t) + 4 1 e
4t
( )
u(t) 2 1 e
2(t 2)
( )
u(t 2) + 4 1 e
4(t 2)
( )
u(t 2).
A picture of h(t) is sketched in the next figure.
1/25/02 P16-28 R. A. DeCarlo, P. M. Lin
SOLUTION 16.52.
The overall impulse response of the configuration is
h(t) h
1
(t) h
2
(t) + h
3
(t) [ ] h
4
(t)
Using the distributive property of the convolution we have
h(t) h
1
(t) h
2
(t) h
4
(t) + h
1
(t) h
3
(t) h
4
(t)
Replacing the expressions for h
2
(t) and h
3
(t), and using the sifting property of the delta function it
follows that
h(t) 2 h
1
(t) (t) h
4
(t) 2 h
1
(t) (t 2) h
4
(t)
2 h
1
(t) h
4
(t) 2 h
1
(t) h
4
(t) [ ]
t t 2
Using the techniques of the convolution algebra it follows that
h
1
(t) h
4
(t) h
1
(1)
(t) h
4
(1)
(t)
where the superscript (1) means differentiation and the superscript (-1) means integration.
We have
h
1
(1)
(t) u
(1)
(t) (t)
and
h
4
(1)
(t) 2e

u( )d

2u(t) e

d
0
t

2u(t) e

[ ]
0
t
2(1 e
t
)u(t)
Therefore
h
1
(t) h
4
(t) (t) 2(1 e
t
)u(t)
[ ]
2(1 e
t
)u(t)
1/25/02 P16-29 R. A. DeCarlo, P. M. Lin
Replacing the above expression into the expression of h(t) it follows that
h(t) 4(1 e
t
)u(t) 4 (1 e
t
)u(t)
[ ]
t t 2
4(1 e
t
)u(t) 41 e
(t 2)
[ ]
u(t 2).
A picture of h(t) is sketched in the next picture.
SOLUTION 16.53.
(a) The overall impulse response of the configuration in figure P16.53 is
h(t) h
1
(t) + h
2
(t) [ ] h
3
(t)
By the distributive property of convolution it follows that
h(t) h
1
(t) h
3
(t) + h
2
(t) h
3
(t) (t) cos( t)u(t) + (t 1)cos( t)u(t)
By the sifting property of the delta function it follows that
h(t) cos( t)u(t) + cos (t 1) [ ]u(t 1).
A picture of h(t) is sketched in the next figure.
1/25/02 P16-30 R. A. DeCarlo, P. M. Lin
(b) The response y(t) is computed as
y(t) h(t) u(t) cos( t)u(t) + cos (t 1) [ ]u(t 1) { } u(t)
Using the distributive property and the time shift property of convolution(see problem 16.22, part (a) ) we
have
y(t) cos( t)u(t) [ ] u(t) + cos (t 1) [ ]u(t 1) { } u(t)
cos( t)u(t) [ ] u(t) + cos( t)u(t) [ ] u(t) { }
t t 1
Using the techniques of convolution algebra, the convolution cos( t)u(t) u(t) is computed as
cos( t)u(t) [ ] u(t) cos( t)u(t) [ ]
(1)
u
(1)
(t)
where the superscript (-1) means integration and the superscript (1) means differentiation.
We have
cos( t)u(t) [ ]
(1)
cos( )u( )d

u(t) cos( )d
0
t

u(t)
1
sin( t) [ ]
0
t

sin( t)
u(t) .
Therefore
cos( t)u(t) [ ] u(t)
sin( t)
u(t)



1
]
1
(t)
sin( t)
u(t)
Hence the step response is
y(t)
sin( t)
u(t) +
sin (t 1) [ ]
u(t 1)
sin( t)
u(t) u(t 1) [ ].
SOLUTION 16.54.
(a) The overall impulse response of the configuration in figure P16.53 is
1/25/02 P16-31 R. A. DeCarlo, P. M. Lin
h(t) h
1
(t) + h
2
(t) [ ] h
3
(t)
By the distributive property of convolution it follows that
h(t) h
1
(t) h
3
(t) + h
2
(t) h
3
(t) (t) cos( t)u(t) + (t 3) cos( t)u(t)
By the sifting property of the delta function it follows that
h(t) cos( t)u(t) + cos (t 3) [ ]u(t 3).
A picture of h(t) is sketched in the next figure.
(b) The response y(t) is computed as
y(t) h(t) u(t) cos( t)u(t) + cos (t 3) [ ]u(t 3) { } u(t)
Using the distributive property and the time shift property of convolution(see problem 16.22, part (a) we
have
y(t) cos( t)u(t) [ ] u(t) + cos (t 3) [ ]u(t 3) { } u(t)
cos( t)u(t) [ ] u(t) + cos( t)u(t) [ ] u(t) { }
t t 3
Using the techniques of convolution algebra, the convolution cos( t)u(t) u(t) is computed as
cos( t)u(t) [ ] u(t) cos( t)u(t) [ ]
(1)
u
(1)
(t)
where the superscript (-1) means integration and the superscript (1) means differentiation. We have
cos( t)u(t) [ ]
(1)
cos( )u( )d

u(t) cos( )d
0
t

u(t)
1
sin( t) [ ]
0
t

sin( t)
u(t) .
Therefore
1/25/02 P16-32 R. A. DeCarlo, P. M. Lin
cos( t)u(t) [ ] u(t)
sin( t)
u(t)



1
]
1
(t)
sin( t)
u(t)
Hence the step response is
y(t)
sin( t)
u(t) +
sin (t 3) [ ]
u(t 3)
sin( t)
u(t) u(t 3) [ ].
SOLUTION 16.55.
(a) The overall impulse response of the configuration in figure P16.55 is
h(t) h
1
(t) + h
2
(t) + h
3
(t) + h
4
(t) [ ] h
5
(t)
By the distributive property of convolution it follows that
h(t) h
1
(t) h
5
(t) + h
2
(t) h
5
(t) + h
3
(t) h
5
(t) + h
4
(t) h
5
(t)
(t) cos( t)u(t) + (t 1)cos( t)u(t)
(t 3) cos( t)u(t) (t 4) cos( t)u(t)
By the sifting property of the delta function it follows that
h(t) cos( t)u(t) + cos (t 1) [ ]u(t 1)
cos (t 3) [ ]u(t 3) cos (t 4) [ ]u(t 4) .
A picture of h(t) is sketched in the next figure.
(b) The response y(t) is computed as
y(t) h(t) u(t) cos( t)u(t) + cos (t 1) [ ]u(t 1) {
cos (t 3) [ ]u(t 3) cos (t 4) [ ]u(t 4)} u(t)
1/25/02 P16-33 R. A. DeCarlo, P. M. Lin
Using the distributive property and the time shift property of convolution (see problem 16.22, part (a)) we
have
y(t) cos( t)u(t) [ ] u(t) + cos (t 1) [ ]u(t 1) { } u(t)
cos (t 3) [ ]u(t 3) { } u(t) cos (t 4) [ ]u(t 4) { } u(t)
cos( t)u(t) [ ] u(t) + cos( t)u(t) [ ] u(t) { }
t t 1

cos( t)u(t) [ ] u(t) { }


t t 3
cos( t)u(t) [ ] u(t) { }
t t 4
Using the techniques of convolution algebra, the convolution cos( t)u(t) u(t) is computed as
cos( t)u(t) [ ] u(t) cos( t)u(t) [ ]
(1)
u
(1)
(t)
where the superscript (-1) means integration and the superscript (1) means differentiation. We have
cos( t)u(t) [ ]
(1)
cos( )u( )d

u(t) cos( )d
0
t

u(t)
1
sin( t) [ ]
0
t

sin( t)
u(t) .
Therefore
cos( t)u(t) [ ] u(t)
sin( t)
u(t)



1
]
1
(t)
sin( t)
u(t)
Hence the step response is
y(t)
sin( t)
u(t) +
sin (t 1) [ ]
u(t 1)
sin (t 3) [ ]
u(t 3)
sin (t 4) [ ]
u(t 4)

sin( t)
u(t) u(t 1)+ u(t 3) u(t 4) [ ].
SOLUTION 16.56.
(a) By definition
h(t) f (t) h(t ) f ( )d

By making a change of variable


1
t
we have
h(t) f (t) h(
1
) f (t
1
)d
1

f (t
1
)h(
1
)d
1

By definition
f (t) h(t) f (t )h( )d

From the above expressions we observe that


h(t) f (t) f (t) h(t)
because and
1
are only variables of integration.
1/25/02 P16-34 R. A. DeCarlo, P. M. Lin
(b) Using the definition of convolution we have
h(t) f (t) [ ] g(t) h( ) f (t )d





1
]
1
1
g(t) h( ) f (t
1
)d





1
]
1
1
g(
1
)

'




;



d
1

h( ) f (t
1
)g(
1
) [ ]d


'




;



d
1

Changing the order of integration we have


h(t) f (t) [ ] g(t) h( ) f (t
1
)g(
1
)d
1





1
]
1
1

'




;



d

By the definition of the convolution we have


f (t) g(t) f (t )g( )d

Therefore
f (t) g(t) [ ]
t t
f (t )g( )d

Hence
h(t) f (t) [ ] g(t) h( ) f (t) g(t) [ ]
t t
{ }d

By the definition of the convolution product we have


h(t) f (t) g(t) [ ] h( ) f (t) g(t) [ ]
t t
{ }d

Therefore
h(t) f (t) [ ] g(t) h(t) f (t) g(t) [ ].
Thus the associative property of convolution is proved.
SOLUTION 16.57.
We have
f (t) h(t) f (t) (t kT)
k0

For some nonnegative k , the Laplace transform of f (t) (t kT) is


L f (t) (t kT) [ ] f (kT)e
skT
1/25/02 P16-35 R. A. DeCarlo, P. M. Lin
by the sifting property of the delta function. Therefore we have
L f (t)h(t) [ ] L f (t) (t kT)
k 0





1
]
1
1
f (kT)e
skT
k 0

Using the notation z e


sT
we have
L f (t)h(t) [ ] f (kT)z
k
k 0

.
SOLUTION 16.58.
(a) By the voltage division formula we have
V
out
(s)
1
Cs
R +
1
Cs
V
in
(s)
Therefore the transfer function of the circuit is
H(s)
V
out
(s)
V
in
(s)

1
Cs
R +
1
Cs

1
CRs +1

1
2s +1
.
Taking the inverse Laplace transform yields
h(t) L
-1
1
2s +1



1
]
1
L
-1
1
2
s +
1
2





1
]
1
1
1
0.5e
0.5t
u(t).
(b) By the impulse response theorem it follows that
v
out
(t) h(t) v
in
(t)

0.5e
0.5t
u(t)
[ ]
(t) + (t 1) + (t 2) +K [ ]
Using the sifting property of the delta function it follows that

v
out
(t) 0.5e
0.5t
u(t) + 0.5e
0.5(t 1)
u(t 1) + 0.5e
0.5(t 2)
u(t 2) +K V.
Therefore for 0 < t <1
v
out
(t) 0.5e
0.5t
V
because only u(t) is nonzero for 0 < t <1.
(c) From the expression of v
out
(t)obtained in the part (b) it follows that
1/25/02 P16-36 R. A. DeCarlo, P. M. Lin
v
out
(t) 0.5e
0.5t
+ 0.5e
0.5(t 1)
V, for 1 < t < 2
because only u(t) and u(t 1) are nonzero for 1 < t < 2.
(d) For t in the interval (4,5) , only u(t) , u(t 1), u(t 2) , u(t 3) and u(t 4) are nonzero. Therefore
v
out
(t) 0.5e
0.5t
+ 0.5e
0.5(t 1)
+ 0.5e
0.5(t 2)
+ 0.5e
0.5(t 3)
+ 0.5e
0.5(t 4)
V.
The above expression can be written as
v
out
(t) 0.5e
0.5(t 4)
1+ e
0.5
( )
+ e
0.5
( )
2
+ e
0.5
( )
3
+ e
0.5
( )
4



1
]
1
V.
(e) Using the sum formula for geometric series with e
0.5
we have
1 + e
0.5
( )
+ e
0.5
( )
2
+ e
0.5
( )
3
+ e
0.5
( )
4

1 e
0.5
( )
5
1 e
0.5
v
out
(t) 0.5e
0.5(t 4)

1 e
0.5
( )
5
1 e
0.5
V.
Therefore
v
out
(t) 8.6189 e
0.5t
V.
(f) Using the expression of v
out
(t)obtained in part (b) it follows that, for n < t < n +1,

v
out
(t) 0.5e
0.5t
+ 0.5e
0.5(t 1)
+ 0.5e
0.5(t 2)
+K+ 0.5e
0.5(t n)


0.5e
0.5(t n)
1+ e
0.5
( )
+ e
0.5
( )
2
+K+ e
0.5
( )
n



1
]
1
V.
Using the sum formula for geometric series we have
v
out
(t) 0.5e
0.5(t n)

1 e
0.5
( )
n+1
1 e
0.5
V.
(g) For large n, e
0.5
( )
n+1
0. Therefore, for large n we have
v
out
(t) e
0.5(t n)

0.5
1 e
0.5
V for n < t < n +1.
A picture of v
out
(t), for large n, is sketched in the next figure.
1/25/02 P16-37 R. A. DeCarlo, P. M. Lin
SOLUTION 16.59.
By the impulse response theorem we have
v
out
(t) h(t) v
in
(t)

0.5e
0.5t
u(t)
[ ]
(t) + (t +1)+ (t + 2) +K [ ] V.
Using the sifting property of the delta function it follows that

v
out
(t) 0.5e
0.5t
u(t) + 0.5e
0.5(t +1)
u(t +1) + 0.5e
0.5(t +2)
u(t + 2) +K V.
For 0 < t we have

v
out
(t) 0.5e
0.5t
+ 0.5e
0.5(t +1)
+ 0.5e
0.5(t +2)
+K

0.5e
0.5t
1+ e
0.5
( )
+ e
0.5
( )
2
+K



1
]
1
V.
Using the sum formula for geometric series( for n ) we have
v
out
(t) 0.5e
0.5t

1
1 e
0.5
V for 0 < t.
Simplifying the expression of v
out
(t) it follows that
v
out
(t) 1.2707 e
0.5t
V for 0 < t.
SOLUTION 16.60. First we plot for reader convenience v
in
(t) and its staircase approximation.
1/25/02 P16-38 R. A. DeCarlo, P. M. Lin
0 0.1 0.2 0.3 0.4 0.5 0.6 0.7 0.8 0.9 1
1
1.2
1.4
1.6
1.8
2
2.2
2.4
2.6
2.8
vin and its Staircase Approximation
The transfer function of the circuit of figure 16.58 is H(s)
1
2s + 1
in which case
h(t) 0.5e
0.5t
u(t). Because we only want the output for 0 t 2, we only need h(t) for 0 t 2 s.
Hence we need to generate staircase approximations to both v
in
(t) and h(t) as follows:
t = 0:0.05:2;
vin = exp(t .^2) .* (u(t) - u(t - 1));
h = 0.5*exp( - 0.5*t) .* (u(t) - u(t - 2));
T = 0.05;
tstep = T;
y = [0 conv(vin,h)*tstep 0];
t = 0:tstep:tstep*(length(vin)+length(h));
% For plotting through time 2 s we set
t=t(1:41);
y = y(1:41);
plot(t,y)
grid
1/25/02 P16-39 R. A. DeCarlo, P. M. Lin
0 0.2 0.4 0.6 0.8 1 1.2 1.4 1.6 1.8 2
0
0.1
0.2
0.3
0.4
0.5
0.6
0.7
Convolution of vin and h
5/15/01 P17-1 R.A. DeCarlo & P.M. Lin
CHAPTER 17 PROBLEM SOLUTIONS
SOLUTION PROBLEM 17.11. Case 1: suppose R
1
> R
2
. From example 17.3, page 696, if L
and C are connected as indicated in part (a), then Z
1
can be made real and larger than R
2
. This
means we can solve the problem at least for Z
1
. Specifically, consider the figure below
From example 17.3, at a specified frequency,
r
, for which Z
1
is real, then L, C, and R
2
must
satisfy,

1
LC

R
2
2
L
2
(1)
Further from example 17,3, at
r
,
Z
1
( j
r
)
L
R
2
C
We require that Z
1
( j
r
) R
1
in which case
R
1
R
2

L
C
(2)
It is necessary to solve equations (1) and (2) simultaneously for L and C. From (2), L R
1
R
2
C.
Substituting into the square of (1) yields

r
2

1
R
1
R
2
C
2

R
2
2
R
1
2
R
2
2
C
2
Hence
C
1

r
R
1
R
1
R
2
R
2
It follows that
5/15/01 P17-2 R.A. DeCarlo & P.M. Lin
L R
1
R
2
1

r
R
1
R
1
R
2
R
2



_
,

r
R
2
R
1
R
2
( )
Observe that since R
1
> R
2
, both C and L are real, i.e., exist. Please note that this connection would
not result in real values of C and L had R
1
< R
2
. If we can now show that Z
2
= R
2
, then parts (a)
and (b) are valid for this case.
By direct computation
Z
2
(j
r
) j
r
L +
1
j
r
C+
1
R
1
j
r
L +
R
1
j
r
CR
1
+1
j R
2
R
1
R
2
( ) +
R
1
j
R
1
R
2
R
2
+1
j R
2
R
1
R
2
( ) +
R
1
jR
1
R
1
R
2
R
2
1+
R
1
R
2
R
2
j R
2
R
1
R
2
( ) + R
2
jR
2
R
1
R
2
R
2
R
2
Thus, (a) and (b) are true for the case R
1
> R
2
.
We can also arrive at the conclusion that Z
2
= R
2
using maximum power transfer concepts.
Since Z
1
is constructed so that Z
1
= R
1
, we have set up the conditions for maximum power transfer
of a V-source in series with R
1
to the "load" Z
1
. Since the LC coupling network is lossless,
whatever average power is received by the network to the right of R
1
, will be dissipated by R
2
.
Therefore maximum power is transferred to the load R
2
. Looking back from R
2
, it must be that R
2
sees a Thevenin resistance Z
2
= R
2
since it is known that there is a non-zero R
1
.
Case 2, R
1
< R
2
. Now consider the configuration
5/15/01 P17-3 R.A. DeCarlo & P.M. Lin
Interchanging the subscripts of 1 and 2 in case 1 produces the derivation for this case.
Combining cases 1 and 2 using the text notation produces
C
1
R
large
R
large
R
small
R
small
L
1

R
small
R
large
R
small ( )
SOLUTION PROBLEM 17.16.
(a)
C = 0.1e-6;
R = 510;
wr = 2*pi*1.56e3;
%From equation 17.4 in example 17.3 which analyzes
%this particular circuit,
%L must satisfy
% wr^2*L^2 - L/C + R^2 = 0.
% Therefore

L = roots([wr^2 -1/C R^2])


L =
5.3133e-02
5.0952e-02
(b) From HW problem 17.9,

r
( )
2

1
LC

1
R
2
C
2
Hence, in MATLAB,
Linv = C*(wr^2 + 1/(R^2*C^2))
Linv =
4.8054e+01
L = 1/Linv
L = 2.0810e-02
5/15/01 P17-4 R.A. DeCarlo & P.M. Lin
SOLUTION PROBLEM 17.22. (Correction: change R
L
in figure P17.22 to R.) We solve this
problem using phasors rather than the Laplace transform approach as it is simpler. Consider
As such, since by definition
0
1 LC, the resonant frequency,
V
C
V
L
Z
in
( j
0
)I
in
RI
in
RI
m
Hence
v
C
(t) v
L
(t) RI
m
cos(
0
t) and i
L
(t)
RI
m

0
L
sin(
0
t).
(a) By direct computation
w
C
(t)
1
2
Cv
C
2
(t)
1
2
CR
2
I
m
2
cos
2
(
0
t)
(b) By direct computation
w
L
(t)
1
2
Li
L
2
(t)
R
2
I
m
2
2
0
2
L
sin
2
(
0
t)
(c) Also by direct computation, since
0
2
1 LC,
w
C
(t) + w
L
(t)
1
2
CR
2
I
m
2
cos
2
(
0
t) +
R
2
I
m
2
2
0
2
L
sin
2
(
0
t)

1
2
R
2
I
m
2
C cos
2
(
0
t) +
1

0
2
LC
sin
2
(
0
t)



_
,

1
2
R
2
I
m
2
C
(d) The energy dissipated in the resistance in one period is
w
R
(0, T) R i
R
2
(t) dt
0
T

R i
in
2
(t) dt
0
T

RI
m
2
cos
2
(
0
t)dt
0
T

5/15/01 P17-5 R.A. DeCarlo & P.M. Lin

RI
m
2
2
dt
0
T

+
RI
m
2
2
cos(2
0
t) dt
0
T

RI
m
2
2
T
RI
m
2

0
(e) Finally
2
maximum energy stored
total energy lost per period
2
0.5R
2
I
m
2
C
RI
m
2

0

0
RC Q
by equation 17.13.
SOLUTION PROBLEM 17.27.
(a) H
1
(s)
I
L
V
s

1
Z
in
(s)

1
R + Ls +
1
Cs

1
L
s
s
2
+
R
L
s +
1
LC
H
2
(s) H
1
(s)
1
Cs

1
V
s
I
L

1
Cs


_
,

V
C
V
s

s
L

1
Cs
s
2
+
R
L
s +
1
LC

1
LC
s
2
+
R
L
s +
1
LC
(b) H
1
(s) is precisely of the form of equation 17.18 with a single zero at the origin. It follows
that
p
2

1
LC
and 2
p

R
L
. Hence, from equation 17.19,
m

p

1
LC
, H
1
( j
m
)
1
R
,
B

2
p

R
L
, and Q
cir
Q
p

p
2
p

1
LC

L
R

1
R
L
C
.
(c) With s = j,
H
2
( j)
2

1
LC


_
,
2
1
LC

2

_
,
2
+
R
L



_
,
2

1
1 LC
2
( )
2
+ RC ( )
2
Instead of maximizing H
2
( j)
2
we minimize its reciprocal, through differentiation. Let
5/15/01 P17-6 R.A. DeCarlo & P.M. Lin
f (C)
1
H
2
( j)
2
1 LC
2
( )
2
+ RC ( )
2
Then
f ' (C) 2C(R)
2
+ 2
2
LC 1
( )

2
L 0
implies that
C
L
R
2
(L)
2

1
L
2
1
R
2
L
2

2
+1






1
]
1
1
1
1

1
L
2
1
1
Q
coil
2
+1






1
]
1
1
1
1
If the coil has high Q, then
C
1
L
2
in which case
H
2
( j)
V
C
V
s

L
R
Q
coil
Therefore
V
C
max
Q
coil
V
s
SOLUTION PROBLEM 17.31. Here
H( j)
2
K
a + j
(
p
2

2
) + j2
p

2
K
2
a
2
+
2
(
p
2

2
)
2
+ 4
p
2

2
K
2
f
2
( )
(a) Differentiating with respect to
2
and setting the derivative equal to zero yields
0
d f
2
( )
d
2

d
d
2
a
2
+
2
(
p
2

2
)
2
+ 4
p
2

2



_
,

5/15/01 P17-7 R.A. DeCarlo & P.M. Lin

1
(
p
2

2
)
2
+ 4
p
2

a
2
+
2
( )
2(
p
2

2
) + 4
p
2
( )
(
p
2

2
)
2
+ 4
p
2

2
[ ]
2
Given that the denominator of the first term is non-zero, this is equivalent to
0 (
p
2

2
)
2
+ 4
p
2

2
a
2
+
2
( )
2(
p
2

2
) + 4
p
2
( )

2
( )
2
+
p
4
2
p
2

2
+ 4
p
2

2
+ 2 a
2
+
2
( )
(
p
2

2
) 4 a
2
+
2
( )

p
2

2
( )
2
+
p
4
2
p
2

2
+ 4
p
2

2
+ 2a
2

p
2
2a
2

2
+ 2
p
2

2
2
2
( )
2
4a
2

p
2
4
p
2

2

2
( )
2
2a
2

2
+
p
4
+ 2a
2

p
2
4a
2

p
2
Hence

2
( )
2
+ 2a
2

2
+ a
4

p
4
+ 2a
2

p
2
4a
2

p
2
+ a
4
where we have added a
4
to produce perfect squares, i.e.,

2
+ a
2
( )
2

p
2
+ a
2
( )
2
2a
p
( )
2
This implies that

2
a
2
t
p
2
+ a
2
( )
2
2a
p
( )
2
Thus, to achieve a real positive solution we obtain,

m
a
2
+
p
2
+ a
2
( )
2
2a
p
( )
2
(b) Here, as in part (a), the arithmetic is simpler if we deal with 1/H(s) rather than H(s).
Specifically
1
H(s)

s
2
+ 2
p
s +
p
2
K(s + a)

s
2
+ 2
p
(s + a) 2
p
a +
p
2
K(s + a)

2
p
K
+
s
2
2
p
a +
p
2
K(s + a)
5/15/01 P17-8 R.A. DeCarlo & P.M. Lin
The problem asks for a non-zero value of . For zero phase shift of H(j), Im[H(j)] = 0
Im[H(j)] > 0. This necessarily requires that the imaginary part of the above expression at s = j
must be zero. Thus
Im
1
H( j)



1
]
1
Im

2
2
p
a +
p
2
K( j + a)




1
]
1
1
Im

2
2
p
a +
p
2
( )
(a j)
K(
2
+ a
2
)




1
]
1
1

2
2
p
a +
p
2
( )
()
K(
2
+ a
2
)
0
The solutions to this are dc, i.e., = 0, and
(for zero phase shift)
p
2
2
p
a
Note that such a frequency may not exist if the quantity under the radical is negative.
SOLUTION PROBLEM 17.40. (a) Compute the transfer function:
H(s)
V
out
I
in

1
Y
in
(s)

1
1
Ls + R
s
+ Cs +
1
R
p

1
C

s +
R
s
L
s
2
+
R
s
L
+
1
R
p
C



_
,

s +
R
s
R
p
+1



_
,

1
LC
8 10
5
s +1.333 10
3
s
2
+ 2000s +1.0006
0
2
where
0
2
1.6 10
9
and
p
2
1.0006
0
2
1.60089 10
9
and
p
4.00110
4
rad/sec.
(b) Q
p

p
2
p
20 which is high Q
p
. Further a
R
s
L
1.333 10
3
<<
p
40.011109 10
3
.
(c) Equation 17.28,
5/15/01 P17-9 R.A. DeCarlo & P.M. Lin

m
a
2
+
p
2
+ a
2
( )
2
2a
p
( )
2
gives the exact value of
m
. In MATLAB,
wmsqrd = -a^2 + sqrt((wp^2+a^2)^2 - (twosig*a)^2)
wmsqrd = 1.600886670362673e+09
wm = sqrt(wmsqrd)
wm = 4.001108184444246e+04
Clearly,
m
approximates
p
. Finally,
H( j
m
) H( j
p
) 8 10
5
j
p
+1.333 10
3

p
2
+ j2000
p
+
p
2

8 10
5
2000
1 j
1.333 10
3

p
400
(d) From equation 17.30,
B

2
p
2000 rad/sec,
1

m
0.5B

39.011 10
3
rad/sec, and

m
+ 0.5B

41.011 10
3
rad/sec.
(e) From equation 17.30,
Q
cir
Q
p

p
2
p

R
s
R
p
+ 1



_
,

1
LC
R
s
L
+
1
R
p
C



_
,

0
R
s
L
+
1
R
p
C



_
,

1
Q
cir

R
s
L
+
1
R
p
C

R
s
L
0
+
1
R
p
C
0

1
Q
coil
+
1
Q
cap

Q
coil
+ Q
cap
Q
coil
Q
cap
Hence,
5/15/01 P17-10 R.A. DeCarlo & P.M. Lin
Q
cir

Q
coil
Q
cap
Q
coil
+ Q
cap
SOLUTION PROBLEM 17.65. Here we consider the equivalent circuit valid for t > 0. Note that
i
L
(0
-
) = i
L
(0
+
) = E/Rs and v
C
(0
-
) = v
C
(0
+
) = 0. Hence
It follows that
V
C
(s)
1
1
R
+
1
Ls
+ Cs

E
R
s
s

1
s
2
+
1
RC
s +
1
LC

E
CR
s
Complex roots occur when
1
RC


_
,
2

4
LC

1
RC


_
,
2
4
0
2
< 0
Equivalently,

0
RC >
1
2
as was to be shown. Further, since v
C
(0
+
) = 0, the general form of the capacitor voltage for constant
excitation is (as per chapter 10)
v
C
(t) e
t
Acos(
d
t) + Bsin(
d
t) ( ) Be
t
sin(
d
t)
From the characteristic equation, the complex roots are
t j
d

1
2RC
t j
1
LC

1
2RC


_
,
2

0
2Q
t j
0
1
1
4Q
2
Here
5/15/01 P17-11 R.A. DeCarlo & P.M. Lin
a

0
2Q

d

0
1
1
4Q
2
Letting B = V
m
, for the appropriate value of V
m
, we obtain the desired result.
(b) If Q is large,
d

0
. The V
m
e
at
will drop to 1/e of its peak value in t
1
a

2Q

2Q

d
seconds. The period of oscillation of the damped sinusoid is
2

d
. Therefore the number of cycles
contained in this interval is
2Q

d
2

5/31/01 Mag Crt Probs P18-1 R. A. DeCarlo & P.M. Lin


CHAPTER 18 PROBLEM SOLUTIONS
SOLUTION PROBLEM 18.34. There is a correction to this problem: set M = 3 H.
(a) The stored energy at t = 0 is:
W(0) 0.5L
1
i
1
2
(0) + 0.5L
2
i
2
2
(0) + Mi
1
(0)i
2
(0) 8 J
L1 = 10; L2 = 2; M = 3; i10 = 1; i20 = -3;
W0 = 0.5*L1*i10^2 + 0.5*L2*i20^2 + M*i10*i20
W0 = 5
(b) Writing two differential mesh equations we obtain
L
1
di
1
dt
+ M
di
2
dt
+ R
1
i
1
10
di
1
dt
+ 3
di
2
dt
+ i
1
0
and
L
2
di
2
dt
+ M
di
1
dt
+ R
2
i
2
2
di
2
dt
+ 3
di
1
dt
+ i
2
0
Taking the Laplace transform of these equations yields
10sI
1
10i
1
(0) + 3sI
2
3i
2
(0) + I
1
(10s +1)I
1
+ 3sI
2
10i
1
(0) 3i
2
(0) 0
and
2sI
2
2i
2
(0) + 3sI
1
3i
1
(0) + I
2
(2s +1)I
2
+ 3sI
1
3i
1
(0) 2i
2
(0) 0
Putting these equations in matrix form yields
(10s +1) 3s
3s (2s +1)



1
]
1
I
1
I
2



1
]
1

10i
1
(0) + 3i
2
(0)
3i
1
(0) + 2i
2
(0)



1
]
1

1
3



1
]
1
Solving yields
I
1
I
2



1
]
1

(10s +1) 3s
3s (2s + 1)



1
]
1
1
1
3



1
]
1

1
11s
2
+12s +1
2s +1 3s
3s 10s +1



1
]
1
1
3



1
]
1

1
(11s +1)(s +1)
11s +1
33s 3



1
]
1

1
s +1
1
3



1
]
1
Therefore, by inspection,
i
1
(t) e
t
u(t) A and i
2
(t) 3e
t
u(t) A
Remark: normally, i1(t) and i2(t) would have two exponential terms present. Because of the special
choice of initial conditions, a pole cancelled out.
(c) From equation 18.24 with the lower limit changed to zero and the upper limit changed to , we have
5/31/01 Mag Crt Probs P18-2 R. A. DeCarlo & P.M. Lin
W(0, ) v
1
i
1
+ v
2
i
2
( )
0

dt 0.5L
1
i
1
2
() + 0.5L
2
i
2
2
() + Mi
1
()i
2
()
0.5L
1
i
1
2
(0) + 0.5L
2
i
2
2
(0) + Mi
1
(0)i
2
(0)
From part (b) all currents at t = are zero, hence
W(0, ) 0.5L
1
i
1
2
(0) + 0.5L
2
i
2
2
(0) + Mi
1
(0)i
2
(0) 5 J
The result of part (a) indicates that the initial store energy is 5 J. The result of part (c) indicates that the
energy returned to the circuit is also 5 J, i.e., the total energy accumulated in the inductors over [0,) is
5 J. Hence 5 J is dissipated in the resistors.
Remark: the interested student might computer the integral R
1
i
1
2
(t) + R
2
i
2
2
(t)
( )
0

dt , the actual energy


dissipated in the resistors over [0,), and show that this is 5 J.
SOLUTION PROBLEM 18.35.
(a)
L1 = 4; L2 = 9; M = 3;
I1 = 2; I2 = -3;
W = 0.5*L1*I1^2 + 0.5*L2*I2^2 - M*I1*I2
W =
6.6500e+01
(b)
K = 0.5*L1*I1^2
K = 8
% Minimize (over I2) K + 0.5*9*I2^2 - 3*2*I2
% Take Derivative and set to zero; then solve for I2.
% Derivative is: 9*I2 6 = 0
% The result is I2 = 2/3 A.
I2 = 2/3;
Wmin = 0.5*L1*I1^2 + 0.5*L2*I2^2 - M*I1*I2
Wmin = 6
(c)
5/31/01 Mag Crt Probs P18-3 R. A. DeCarlo & P.M. Lin
(d)
L1 = 4; L2 = 9; M = 3;
k = M/sqrt(L1*L2)
k = 5.0e-01
SOLUTION PROBLEM 18.37.
k = 0.5;
L1 = 9; L2 = 4;L3 = 1;
M = 0.5*sqrt(L1*L2)
M = 3
Lcpld = L1 + L2 + 2*M
Lcpld = 19
Leq = Lcpld + L3
Leq = 20
Imax = 2;
Wmax = 0.5*Leq*Imax^2
Wmax = 40 J
SOLUTION PROBLEM 18.41.
RL = 100; Rs = 300e3; R = 10e3;
m = 20; n = 5;
(a)
Z2 = RL*m^2
Z2 = 40000
Req1 = Z2*R/(Z2+R)
Req1 = 8000
Z1 = Req1*n^2
Z1 = 200000
5/31/01 Mag Crt Probs P18-4 R. A. DeCarlo & P.M. Lin
(b)
%Gv1 = v1/vin
%v1 = [Z1/(Rs + Z1)]vin
Gv1 = Z1/(Rs+Z1)
Gv1 = 4.0000e-01
% Gv2 = v2/vin
% Gv2 = v2/v1 * v1/vin = (1/n) * G1
Gv2 = G1/n
Gv2 = 8.0000e-02
% Gv3 = v3/vin
Gv3 = -Gv2/m
Gv3 = -4.0000e-03
(c)
% Gi2 = i2/iin
Gi2 = n*R/(R+Z2)
Gi2 = 1
% Gi3 = i3/iin = i3/i2 * i2/iin = m*Gi2
Gi3 = -m*Gi2
Gi3 = -20
SOLUTION PROBLEM 18.55. (a) The parameters in the circuit of figure P18.55b are given by
equations in figure 18.22b. Specifically, since k M L
1
L
2
0.16 3.5 0.008 0.95618
M = 0.16; L1 = 3.5; L2 = 0.008;
k=M/sqrt(L1*L2)
k =
9.5618e-01
La = (1 - k^2)*L1
La =
3.0000e-01
Lb = k^2 * L1
Lb =
3.2000e+00
N =M/L2
N =
20
(b)
R = 500;
w = 2*pi*60;
Vseff = 110;
Zin = R + j*La*w + j*Lb*w
Zin =
5.0000e+02 + 1.3195e+03i
Iseff = Vseff/Zin
Iseff =
2.7624e-02 - 7.2899e-02i
Pave = R*abs(Iseff)^2
5/31/01 Mag Crt Probs P18-5 R. A. DeCarlo & P.M. Lin
Pave =
3.0387e+00
(c)
Zin2 = R + j*La*w
Zin2 =
5.0000e+02 + 1.1310e+02i
Iseff2 = Vseff/Zin2
Iseff2 =
2.0929e-01 - 4.7341e-02i
Is2mag = abs(Iseff2)
Is2mag =
2.1458e-01
% The current in the secondary is (in A):
Isecmag = Ismag*N
Isecmag =
4.2916e+00
(d)
% Our first step is to compute the reflected impedance:
Zrefl = 100*N^2
Zrefl =
40000
% We now compute the impedance of the parallel combination
% of Lb and Zrefl denoted Zpar
Zpar = 1/(1/Zrefl + 1/(j*w*Lb))
Zpar =
3.6350e+01 + 1.2053e+03i
% We now compute the input impedance:
Zin = R+j*w*La + Zpar
Zin =
5.3635e+02 + 1.3184e+03i
% Now we compute the voltage across the primary of the
% ideal transformer, by voltage division:
Vpar = Vseff*Zpar/Zin
Vpar =
8.7342e+01 + 3.2500e+01i
% Now we compute the voltage across the load:
Vload = Vpar/N
Vload =
4.3671e+00 + 1.6250e+00i
Vloadmag = abs(Vload)
Vloadmag =
4.6596e+00
Iloadmag = Vloadmag/100
Iloadmag =
4.6596e-02
5/31/01 Mag Crt Probs P18-6 R. A. DeCarlo & P.M. Lin
SOLUTION PROBLEM 18.65. (a) The equivalent circuit accounting for initial conditions is given
below:
(b) From the definition of coupled inductors
V
1
V
2



1
]
1

0.6s 0.1472s
0.1472s 0.1472s



1
]
1
I
1
'
I
2
'




1
]
1
1
(c) Hence
I
1
'
I
2
'




1
]
1
1

0.6s 0.1472s
0.1472s 0.1472s



1
]
1
1
V
1
V
2



1
]
1

15
s
0.1472 0.1472
0.1472 0.6



1
]
1
V
1
V
2



1
]
1
(d) Writing nodal equations we obtain,
V
s
0



1
]
1

2 1
1 2



1
]
1
V
1
V
2



1
]
1
+
I
1
I
2



1
]
1

2 1
1 2



1
]
1
V
1
V
2



1
]
1
+
1
s
i
1
(0

)
i
2
(0

)




1
]
1
1
+
I
1
'
I
2
'




1
]
1
1
(e) Now we substitute our result of part (c):
V
s
0



1
]
1

2 1
1 2



1
]
1
V
1
V
2



1
]
1
+
1
s
2
2



1
]
1
+
1
s
2.208 2.208
2.208 9



1
]
1
V
1
V
2



1
]
1
which simplifies to
V
s
0



1
]
1

1
s
2
2



1
]
1

1
s
2s + 2.208 (s + 2.208)
(s + 2.208) 2s + 9



1
]
1
V
1
V
2



1
]
1
or equivalently
2s + 2.208 (s + 2.208)
(s + 2.208) 2s + 9



1
]
1
V
1
V
2



1
]
1

sV
s
+ 2
2



1
]
1
(f) Solving these equations we obtain:
5/31/01 Mag Crt Probs P18-7 R. A. DeCarlo & P.M. Lin
V
1
V
2



1
]
1

2s + 2.208 (s + 2.208)
(s + 2.208) 2s + 9



1
]
1
1
sV
s
+ 2
2



1
]
1

1
3s
2
+18s + 15
2s + 9 (s + 2.208)
(s + 2.208) 2s + 2.208



1
]
1
sV
s
+ 2
2



1
]
1
(g) If v
s
(t) = 10u(t) V, then
V
1
V
2



1
]
1

1
3s
2
+18s +15
2s + 9 (s + 2.208)
(s + 2.208) 2s + 2.208



1
]
1
12
2



1
]
1

1
3s
2
+18s + 15
26s +112.42
16s + 30.912



1
]
1
From MATLAB
[r,p,k]=residue([16 30.912], [3 18 15])
r =
4.0907e+00
1.2427e+00
p =
-5
-1
k =
[]
Therefore,
v
2
(t) 4.0907e
5t
+1.2427e
t
( )
u(t) V
SOLUTION PROBLEM 18.67. The solution to this problem is based upon the following: (i) L
eq
= L
1
+
L
2
+ 2M for series aiding connection (see example 18.4) and (ii) k M L
1
L
2
(a definition), and (iii) k
= 1 (an assumption).
(a) Given L
1
= L
2
= L and k = 1, L
eq
L
1
+ L
2
+ 2M L + L + 2k L
2
4L. Hence, when the
number of turns is doubled, the inductance is quadrupled.
(b) For this part, let us first consider L
2
which has 2N turns. We can view L
2
as two coils of N turns
each connected in series aiding with coupling coefficient k = 1. Hence, according to part (a), the
inductance of L
2
is four times that of L
1
which only has N turns. Hence,
L
eq
L
1
+ L
2
+ 2M L + 4L + 2k 4L
2
9L
Observe that the coil has 3N turns yielding an inductance of 9L = 3
2
L.
(c) Suppose coil 1 and coil 2 consist of one turn each. Here the total number of turns is 2N where N =
1 turn. Suppose further that L
1
= L. From part (a), L
eq
4L (2)
2
L . Now suppose coil one consists
of one turn and coil 2 consists of M turns. We assume here as an induction hypothesis that
5/31/01 Mag Crt Probs P18-8 R. A. DeCarlo & P.M. Lin
L
eq
M +1 ( )
2
L
We must show that if coil 2 has (M+1)N turns then,
L
eq
M + 2 ( )
2
L
Our first step is to compute the equivalent inductance of coil 2. However, coil 2 consists of a single turn
coupled to an M-turn coil, which by the induction hypothesis means that
L
2
M +1 ( )
2
L
Thus coil 1 in a series aiding connection with L2 leads to
L
eq
L
1
+ L
2
+ 2M L + M + 1 ( )
2
L + 2 M + 1 ( )
2
L
2
L + M +1 ( )
2
L + 2 M +1 ( )L
L M +1 ( )
2
+ 2 M +1 ( ) +1
[ ]
L M +1 ( ) +1 [ ]
2
M + 2 ( )
2
L
Given this relationship, if coil 1 consists of N1 turns, and one turn has an inductance L, then
L
1
N
1
( )
2
L. Similarly, L
2
N
2
( )
2
L, and M k L
1
L
2
L
1
L
2
N
1
( )
2
N
2
( )
2
L
2
N
1
N
2
L. It
immediately follows that
L
1
: L
2
: M = N
1
2
: N
2
2
: (N
1
N
2
)
SOLUTION PROBLEM 18.70. (a) k M / L
1
L
2
M = 1.5; L1 = 1.5; L2 = 6;
k = M/sqrt(L1*L2)
k = 5.0000e-01
(b) For this part and the remaining parts consider the following equivalent circuit where the coupled
coils have been replaced by the model of figure 18.22(b).
La = (1 - k^2)*L1
La =
1.1250e+00
Lb = k^2 * L1
Lb =
3.7500e-01
5/31/01 Mag Crt Probs P18-9 R. A. DeCarlo & P.M. Lin
% The turns ratio is M:L2, i.e.,
1.5/6
ans = 2.5000e-01
% Therefore the turns ratio is 1:4. It follows that
Rb = 200*(1/4)^2
Rb =
1.2500e+01
To compute Z(s) we have,
Z(s) 20 +
1
Cs
+ L
a
s +
R
b
L
b
s
L
b
s + R
b
1.125s + 20 +
12.5s
s +
100
3
+
1
Cs
Hence,
Z( j) j1.125 + 20 +
j12.5
j +
100
3

j
C
20 +
12.5
2

2
+
10
4
9
+ j 1.125
1
C
+
416.67

2
+
10
4
9






_
,




(c) For this part we need to make the imaginary part of Z(jw) real. To this end:
K1 = 12.5*100/3
K1 = 4.1667e+02
w = 1333;
K2 = 1.125*w + K1*w/(w^2 + 1e4/9)
K2 = 1.4999e+03
C = 1/(K2*w)
C = 5.0015e-07
Hence, we take C = 5 F.
(d) At resonance, we have
Z( j
r
) Z( j1333) 20 +
12.5
r
2

r
2
+
10
4
9
32.5
and
Z
b
(j
r
)
j12.5
r
j
r
+
100
3

j16,662
33.33 + j1333
12.492 + j0.31238
By voltage division
V
out
( j
r
)
V
s
( j
r
)

V
out
V
b

V
b
V
s

4
1

Z
b
( j
r
)
Z( j
r
)
1.5379 + j0.038456
Zb = j*12.5*w/(j*w + 100/3)
Zb =
1.2492e+01 + 3.1238e-01i
Zwr=20 + 12.5*w^2/(w^2 + 1e4/9)
Zwr =
5/31/01 Mag Crt Probs P18-10 R. A. DeCarlo & P.M. Lin
3.2492e+01
Gv = 4*Zb/Zwr
Gv =
1.5379e+00 + 3.8456e-02i
MagGv = abs(Gv)
MagGv =
1.5384e+00
AngGv = angle(Gv)*180/pi
AngGv =
1.4325e+00
SOLUTION PROBLEM 18.71.
(a) Following the hint we apply a source transformation to obtain
Writing two mesh equations we obtain the following matrix form of the mesh equations:
s ++ 1 s ks
ks s ++ 1 s



1
]
1
I
1
I
2



1
]
1

I
in
s
0



1
]
1
Solving for I
2
yields
I
2

det
s ++ 1 s I
in
s
ks 0



1
]
1
det
s ++ 1 s ks
ks s ++ 1 s



1
]
1

kI
in
s ++ 1 s ( )
2
ks ( )
2
To find H(s) we have
H(s)
V
out
I
in

I
2
s
I
in

k s
s ++ 1 s ( )
2
ks ( )
2

k s
s + ks ++ 1 s ( ) s ks ++ 1 s ( )

ks
(1+ k)s
2
+s +1
[ ]
(1 k)s
2
+s +1
[ ]
5/31/01 Mag Crt Probs P18-11 R. A. DeCarlo & P.M. Lin
(b) R = 0.02 , b = R = 0.02 , Q = 50, and k = 0.01, 0.02, and 0.04.
beta = 0.02; k1 = 0.01; k2 = 0.02; k3 = 0.04;
p11 = roots([(1+k1) beta 1]);
p12 = roots([(1-k1) beta 1]);
p1 = [p11;p12]
p1 =
-9.9010e-03 + 9.9499e-01i
-9.9010e-03 - 9.9499e-01i
-1.0101e-02 + 1.0050e+00i
-1.0101e-02 - 1.0050e+00i
p21 = roots([(1+k2) beta 1]);
p22 = roots([(1-k2) beta 1]);
p2 = [p21;p22]
p2 =
-9.8039e-03 + 9.9010e-01i
-9.8039e-03 - 9.9010e-01i
-1.0204e-02 + 1.0101e+00i
-1.0204e-02 - 1.0101e+00i
p31 = roots([(1+k3) beta 1]);
p32 = roots([(1-k3) beta 1]);
p3 = [p31;p32]
p3 =
-9.6154e-03 + 9.8053e-01i
-9.6154e-03 - 9.8053e-01i
-1.0417e-02 + 1.0206e+00i
-1.0417e-02 - 1.0206e+00i
(c)
f = 0.14:.0001:.18;
n1 = [k1/(1-k1)^2 0];
n2 = [k2/(1-k2)^2 0];
n3 = [k3/(1-k3)^2 0];
h1 = freqs(n1, poly(p1), 2*pi*f);
h2 = freqs(n2, poly(p2), 2*pi*f);
h3 = freqs(n3, poly(p3), 2*pi*f);
plot(f, abs(h1), f, abs(h2), f, abs(h3))
grid
5/31/01 Mag Crt Probs P18-12 R. A. DeCarlo & P.M. Lin
(d) An inspection of [f, abs(h2)] (i.e., a tabulation of the values) in part (c) indicates that f
peak
= 0.15865
Hz and H
peak
= 26.148 . The frequency scale factor Kf is defined according to:
Kf = 455e3/fpeak
Kf = 2.8679e+06
Further,
Km = Kf*2.35e-3
Km = 6.7397e+03
Lnew = Km*1/Kf
Lnew =
2.3500e-03
Cnew = 1/(Km*Kf)
Cnew =
5.1736e-11
Rnew = Km*0.02
Rnew =
1.3479e+02
The 3 dB down value of h2 is Hpeak/sqrt(2). Hence
5/31/01 Mag Crt Probs P18-13 R. A. DeCarlo & P.M. Lin
Hmax = max(abs(h2))
Hmax =
2.6148e+01
H3db = Hmax/sqrt(2)
H3db =
1.8490e+01
Again, inspecting the tabulated values indicates that the 3 dB frequencies are: f
1
= 0.1569 Hz and
f
2
= 0.1614 Hz. Finally
Bf = f2 - f1
Bf = 4.5000e-03 Hz
Bfnew = Kf*Bf
Bfnew = 1.2906e+04 Hz
(e) For this part we redo part (a) with R, L, and C as literals.
Writing two mesh equations we obtain the following matrix form of the mesh equations:
Ls + R+1 Cs kLs
kLs Ls + R +1 Cs



1
]
1
I
1
I
2



1
]
1

I
in
Cs
0



1
]
1
Solving for I
2
yields
I
2

det
Ls + R+ 1 Cs I
in
Cs
kLs 0



1
]
1
det
Ls + R +1 Cs kLs
kLs Ls + R+1 Cs



1
]
1

kL C
Ls + R +1 Cs ( )
2
kLs ( )
2
I
in
To find H(s) we have
H(s)
V
out
I
in

I
2
Cs
I
in

kL C
2
s
Ls + R+ 1 Cs ( )
2
kLs ( )
2

kL C
2
s
Ls + kLs + R +1 Cs ( ) Ls kLs + R+1 Cs ( )

kLs
LC(1+ k)s
2
+ CRs +1
( )
LC(1 k)s
2
+ CRs +1
( )

ks
LC
2
(1+ k)s
2
+
R
L
s +
1
LC


_
,
(1 k)s
2
+
R
L
s +
1
LC


_
,

k
0
2
s
C
(1+ k)s
2
+

0
Q
s +
0
2



_
,

(1 k)s
2
+

0
Q
s +
0
2



_
,

Evaluating this expression at s = j
0
, yields
5/31/01 Mag Crt Probs P18-14 R. A. DeCarlo & P.M. Lin
H( j
0
)
jk
0
3
C
(1+ k)
0
2
+
j
0
2
Q
+
0
2



_
,

(1 k)
0
2
+
j
0
2
Q
+
0
2



_
,

jk
C
0
k
j
Q



_
,

k +
j1
Q



_
,

jk
C
0
k
2
+
1
Q
2
Therefore
H( j
0
)
1
C
0

k
k
2
+
1
Q
2
(f) To solve this part we differentiate and set equal to zero as follows:
d H(j
0
)
dk

1
C
0

d
dk
k
k
2
+
1
Q
2





_
,


1
C
0

1
k
2
+
1
Q
2

2k
2
k
2
+
1
Q
2



_
,

2







1
]
1
1
1
1
1
0
It follows that
2k
2
k
2
+
1
Q
2
1
Hence
k
2

1
Q
2
or k = 1/Q. With this value of k,
H( j
0
)
max

1
C
0

1/ Q
2
Q
2

Q
2C
0
At
0
, the magnitude of the transfer function increases with increasing k, reaching a peak at k = 1/Q and
then decreases with a further increase in k as born out in the plots of part (c).
Prbs Chap 19, 1/7/02 P19-1 R. A. DeCarlo, P. M. Lin
CHAPTER 19 PROBLEM SOLUTIONS
SOLUTION PROBLEM 19.1. Refer to figure 19.3.
Vs = 100; ZL = 20; Rs = 1e3;
beta = 149;
Zinbox = (beta + 1)*ZL
Zinbox =
3000
% By voltage division
V1 = Vs*Zinbox/(Zinbox + Rs)
V1 = 75
% To obtain the power delivered by the source
I1 = Vs/(Zinbox +Rs)
I1 = 2.5000e-02
Psource = I1*Vs
Psource = 2.5000e+00
SOLUTION PROBLEM 19.2. Refer to figure 19.4.
Vs = 100; Z1 = 30e3; Rs = 50; beta = 149;
Zboxin = Z1/(beta+1)
Zboxin = 200
V1 = Vs*Zboxin/(Zboxin + Rs)
V1 = 80
Psource = Vs^2/(Rs + Zboxin)
Psource = 40
SOLUTION PROBLEM 19.3. Refer to figure 19.5.
(a) C = 0.1e-3; vc0 = 10; Z1 = 300; Z2 = 1e3;
Z3 = 1e3; gm = 9e-3;

Zin = Z1 + (1 + gm*Z1)*Z2
Zin = 4.0000e+03
(b)
tau = Zin*C
Prbs Chap 19, 1/7/02 P19-2 R. A. DeCarlo, P. M. Lin
tau = 4.0000e-01
Hence, v
C
(t) v
C
(0)e
t /
10e
2.5t
V.
SOLUTION PROBLEM 19.4. (a) First observe that since no current can flow into the secondary
we have
V
oc
aV
pri
aRI
in
800 Vrms
Now
Z
th

1
j C
+ a
2
R 640 j 360
(b) Z
L
Z
th
( )
*
640 + j360 .
Voc = 800; Rth = 640;
Pmax = Voc^2/(4*Rth)
Pmax = 250
(c) By inspection the circuit is a 640 resistor in series with a 3.6 H inductor.
SOLUTION PROBLEM 19.5. Because the output is open circuited, no current flows into the
secondary of the transformer, hence
v
oc
v
sec
+ sin(3t)u(t) 2v
pri
+ sin(3t)u(t) cos(3t) + sin(3t) [ ]u(t).
Additionally
Z
th
(s)
10
s
+ s + 4 + 4
2.5
s
+ 0.25s + 9


_
,

20
s
+ 2s + 40
SOLUTION PROBLEM 19.6. Using Cramer's rule,
Prbs Chap 19, 1/7/02 P19-3 R. A. DeCarlo, P. M. Lin
I
1

det
V
1
1 a
0 0.5 0
0 0 0.5





1
]
1
1
1
1.5a

0.25V
1
1.5a

1
6a
V
1
Therefore R
in
6a .
To compute the average power, V
1,eff
10 V. Hence P
ave

V
1,eff
2
R
in

100
6a
watts.
SOLUTION PROBLEM 19.7. As per the hint, we write loop equations as follows:
V
out
10 s
40 s





1
]
1
1
1

s +1 1 0
1 6 2
0 2 4





1
]
1
1
1
I
out
I
2
I
3





1
]
1
1
1
Using equation 19.6,
V
out
s +1 1 0 [ ]
0.2 0.1
0.1 0.3



1
]
1
1
0



1
]
1



_
,

I
out
+ 1 0 [ ]
0.2 0.1
0.1 0.3



1
]
1
10 s
40 s



1
]
1
(s + 0.8)I
out
+
2
s
Therefore
V
out
(s + 0.8) I
out
( )
2
s
Z
th
I
out
( ) +V
oc
i.e., Z
th
s + 0.8, and V
oc

2
s
.
SOLUTION PROBLEM 19.8. (a) Let the node voltages from left to right be V
1
, V
2
, and V
out
.
Also inject a current I
3
into node 3. Writing nodal equations by inspection we have:
Prbs Chap 19, 1/7/02 P19-4 R. A. DeCarlo, P. M. Lin
I
in
0
I
3





1
]
1
1
1

1.5 1 0.25
1 2 0.5
0.25 0.5 0.0625s + 0.75





1
]
1
1
1
V
1
V
2
V
out





1
]
1
1
1
Using equation 19.11, we have
I
3
W
22
W
21
W
11
1
W
12 ( )
V
out
+ W
21
W
11
1
I
in
0



1
]
1
0.0625s + 0.75 0.25 0.5 [ ]
1 0.5
0.5 0.75



1
]
1
0.25
0.5



1
]
1



_
,

V
out
0.25 0.5 [ ]
1 0.5
0.5 0.75



1
]
1
I
in
0



1
]
1
Thus
I
3
0.0625s + 0.375 ( )V
out
0.5I
in
Therefore I
sc
I
3
]
V
out
0
0.5I
in
. Further Z
th
(s)
1
0.0625s + 0.375

16
s + 6
.
(b) V
out
(s) Z
th
(s)I
sc

16I
sc
s + 6

8I
in
s + 6
. By inspection, the impulse response is
v
out ,imp
(t) 8e
6t
u(t) V. Further, from MATLAB
n = 8; d = [1 6 0];
[r,p,k] = residue(n,d)
r =
-1.3334e+00
1.3334e+00
p =
-6
0
k =
[]
Hence the step response is:
v
out ,step
(t)
4
3
1 e
6t
( )
u(t) V
SOLUTION PROBLEM 19.9. (a) Consider the following figure:
Prbs Chap 19, 1/7/02 P19-5 R. A. DeCarlo, P. M. Lin
Let I
out
enter the output terminal and I
1
and I
2
be the currents entering the primary and secondary
of the transformer respectively. It follows that
I
in

bV
out
R
+ I
1
+
(b 1)V
out
R

(2b 1)V
out
R
+ I
1
which implies that
I
1
I
in

(2b 1)V
out
R
Further,
I
out

(1 b)V
out
R
+ I
2

(1 b)V
out
R
bI
1
Therefore
I
out

(1 b)V
out
R
+ I
2

(1 b)V
out
R
bI
1

(1 b)V
out
R
b I
in

(2b 1)V
out
R


_
,

2b
2
2b +1
R
V
out
bI
in
Equivalently
V
out

R
2b
2
2b +1
I
out
+
bR
2b
2
2b +1
I
in
Therefore
Z
th

R
2b
2
2b +1
, V
oc

bR
2b
2
2b +1
I
in
, I
sc
bI
in
(b) I
sc
b45
o
, Z
th

R
2b
2
2b +1
.
Prbs Chap 19, 1/7/02 P19-6 R. A. DeCarlo, P. M. Lin
(c) B 10
1
Z
th
C

1
R
2b
2
2b +1
C

2b
2
2b +1
RC

5
25C
. Hence C 0.02 F. Further,
0
2
25
1
LC

1
0.02L
. Therefore L 2 H.
(d) H(s)
V
out
I
in

V
out
0.5I
sc
2
1
2b
2
2b +1
R
+
1
Ls
+ Cs

2
0.2 +
1
2s
+ 0.02s

100s
s
2
+10s + 25
and I
in
(s)
2
2

s10
s
2
+100
. Hence,
V
out
(s)
50 2 s(s10)
(s
2
+10s + 25)(s
2
+100)
To compute v
out
(t) in MATLAB,
%partial fraction expansion of Vout/2 = = n(s)/d(s)
n =50*[ 1 -10 0];
d = conv([ 1 0 100], [ 1 10 25]);
[r ,p, k] = residue(n,d)
%numerator polynomial of combined complex pole terms
num = [1 -p(1)]*r(2) +[1 -p(2)]*r(1)
Output from MATLAB
r =
2.8000 + 0.4000i
2.8000 - 0.4000i
-5.6000
30.0000
p =
-0.0000 +10.0000i
-0.0000 -10.0000i
-5.0000
-5.0000
num =
5.6000 -8.0000
Laplace transform of ouput
V
out
(s)
5.6 2
s + 5
+
30 2
(s + 5)
2
+
(5.6s 8) 2
s
2
+100
Prbs Chap 19, 1/7/02 P19-7 R. A. DeCarlo, P. M. Lin
Taking the inverse Laplace transform using table 13.1 on page 515:
v
out
(t) 30 2te
5t
5.6 2e
5t
+ 5.6 2 cos(10t) 0.8 2 sin(10t)
[ ]
u(t) V
The steady state part consists of the cosine and sine terms only. Since the parallel RLC acts like
a band pass circuit and the peak value occurs at 5 rad/s, one expects the magnitude of the steady
state output to be much smaller at 100 rad/s.
SOLUTION PROBLEM 19.10. (a) Write two nodal equations by inspection:
I
1
I
2



1
]
1

Y
1
+ Y
3
Y
3
+ g
m
Y
3
Y
2
+ Y
3



1
]
1
V
1
V
2



1
]
1

y
11
y
12
y
21
y
22



1
]
1
V
1
V
2



1
]
1
(b) When port-2 is shorted, y
11
is the input admittance. Therefore
Z
in

1
y
11

1
Y
1
+ Y
3
and since V
2
= 0,
I
2
y
21
V
1

KY
3
s
SOLUTION PROBLEM 19.11. (a) By inspection
I
1
I
2



1
]
1

Y
1
+ Y
3
Y
3
Y
3
+ g
m
Y
2
+ Y
3



1
]
1
V
1
V
2



1
]
1

y
11
y
12
y
21
y
22



1
]
1
V
1
V
2



1
]
1
Clearly, Y
3
y
12
. Then, Y
1
y
11
Y
3
y
11
+ y
12
and Y
2
y
22
Y
3
y
22
+ y
12
. Finally,
g
m
y
21
+Y
3
y
21
y
12
.
(b) Recall that
Prbs Chap 19, 1/7/02 P19-8 R. A. DeCarlo, P. M. Lin
I
1
I
2



1
]
1

Y
1
+ Y
3
Y
3
+ g
m
Y
3
Y
2
+ Y
3



1
]
1
V
1
V
2



1
]
1

y
11
y
12
y
21
y
22



1
]
1
V
1
V
2



1
]
1
Clearly, Y
3
y
21
. Then, Y
1
y
11
Y
3
y
11
+ y
21
and Y
2
y
22
Y
3
y
22
+ y
21
. Finally,
g
m
y
12
+ Y
3
y
12
y
21
.
SOLUTION PROBLEM 19.12. (a) By definition of coupled inductors
V
1
V
2



1
]
1

L
1
s Ms
Ms L
2
s



1
]
1
I
1
I
2



1
]
1

I
1
I
2



1
]
1

1
L
1
L
2
M
2
( )
s
L
2
M
M L
1



1
]
1
V
1
V
2



1
]
1
Hence, the y-parameters are:
1
L
1
L
2
M
2
( )
s
L
2
M
M L
1



1
]
1
(b) By definition of coupled inductors
V
1
V
2



1
]
1

L
1
s Ms
Ms L
2
s



1
]
1
I
1
I
2



1
]
1

I
1
I
2



1
]
1

1
L
1
L
2
M
2
( )
s
L
2
M
M L
1



1
]
1
V
1
V
2



1
]
1
Hence, the y-parameters are:
1
L
1
L
2
M
2
( )
s
L
2
M
M L
1



1
]
1
If the coupling coefficient is 1, L
1
L
2
M
2
and the y-parameters do not exist since the
determinant of the z-parameter matrix is zero.
SOLUTION PROBLEM 19.13. Let I
2
'
denote the current entering the dotted terminal of the
secondary of the coupled inductors. Then using the result of problem 12a,
Prbs Chap 19, 1/7/02 P19-9 R. A. DeCarlo, P. M. Lin
I
1
I
2
'



1
]
1

1
L
1
L
2
M
2
( )
s
L
2
M
M L
1



1
]
1
V
1
V
2



1
]
1

1
6s
4 1
1 1



1
]
1
V
1
V
2



1
]
1
From the given circuit I
2
I
2
'
+ 2I
1

1
6s
V
1
+
1
6s
V
2
+
8
6s
V
1

2
6s
V
2

7
6s
V
1

1
6s
V
2
. Therefore
the y-parameter matrix is:
1
6s
4 1
7 1



1
]
1
SOLUTION PROBLEM 19.14.
(a) By definition and the properties of the ideal transformer y
11

I
1
V
1
1
]
1
V
2
0
G
1
and
y
22

I
2
V
2
1
]
1
V
1
0

G
1
+ G
2
a
2
. Additionally, since the circuit is obviously reciprocal,
y
12
( y
21
)
I
1
V
2
1
]
1
V
1
0

G
1
a
.
(b) V
2
reflected to the primary side, denoted by

V
1
, is

V
1

2K a
s
2
+ 4
Hence
I
1

2G
1
K a
s
2
+ 4
To compute I
2
, we reflect the parallel of G
1
and G
2
to the secondary of the ideal transformer.
Hence the impedance in parallel with V
2
, denoted Z
sec
, is
Z
sec

a
2
G
1
+ G
2
Therefore,
Prbs Chap 19, 1/7/02 P19-1 0 R. A. DeCarlo, P. M. Lin
I
2

V
2
Z
sec

2K G
1
+ G
2
( ) a
2
s
2
+ 4
SOLUTION PROBLEM 19.15. (a) By definition
y
11

I
1
V
1
1
]
1
V
2
0

1
Z
in
1
]
1
V
2
0

1
9
S
where
Z
in
6 +12/ /
1
16
(320/ /80)


_
,
9
Similarly, by definition
y
22

I
2
V
2
1
]
1
V
1
0

1
Z
out
1
]
1
V
1
0

3
400
S
where
Z
out
80 + 320/ /16 (6 //12) ( )
400
3

SOLUTION PROBLEM 19.16. Write nodal equations:
I
1
I
2
0





1
]
1
1
1

2s + 2 1 2s
1 s + 2 s
2s s 5s





1
]
1
1
1
V
1
V
2
V
3





1
]
1
1
1
Using the matrix partitioning method, we obtain the 2-port y parameters
I
1
I
2



1
]
1

2s + 2 1
1 s + 2



1
]
1

1
5s
2s
s



1
]
1
2s s [ ]



_
,

V
1
V
2



1
]
1

1.2s + 2 0.4s1
0.4s1 0.8s + 2



1
]
1
V
1
V
2



1
]
1

y
11
y
12
y
21
y
22



1
]
1
V
1
V
2



1
]
1
Prbs Chap 19, 1/7/02 P19-1 1 R. A. DeCarlo, P. M. Lin
SOLUTION PROBLEM 19.17. (a) The ideal transformer yields the constraints
I
1
a

I
2
and

V
2
aV
1
The three resistors have nodal equations


I
2
I
2




1
]
1
1

2 1
1 2



1
]
1

V
2
V
2




1
]
1
1
Substituting the first two equations into the last one, we obtain
I
1
I
2



1
]
1

2a
2
a
a 2




1
]
1
1
V
1
V
2



1
]
1

y
11
y
12
y
21
y
22



1
]
1
V
1
V
2



1
]
1
(b)
Y
in
y
11

y
12
y
21
y
22
+ Y
L
2a
2

a
2
4
1.75a
2
S
Z
in

1
Y
in

4
7a
2

and
G
v
=
V
2
V
1
=
-y
21
y
22
+ y
L1
=
a
4
SOLUTION PROBLEM 19.18. This problem is solved in MATLAB.
Part (a)
% Parameter specification
Ys = 1e-3;YL = 1e-3;
y11=4e-3; y12 = -0.1e-3;
y21 = 50e-3; y22 = 1e-3;
% Calculation of input admittance and impedance
Yin = y11 - y12*y21/(y22 + YL)
Yin = 6.5000e-03
Zin = 1/Yin
Zin = 1.5385e+02
% Calculation of output admittance
Prbs Chap 19, 1/7/02 P19-1 2 R. A. DeCarlo, P. M. Lin
Yout = y22 - y12*y21/(y11 + Ys)
Yout = 2.0000e-03
Part (b)
% Calculation of voltage gain
Gv = (Ys/(Ys + Yin))*(-y21/(y22 + YL))
Gv = -3.3333e+00
Part (c)
V2 = Gv*10
V2 = -3.3333e+01
Therefore, v
2
(t) 33.333u(t) V. Finally,
PL = V2^2/1e3
PL = 1.1111e+00
SOLUTION PROBLEM 19.19.
(a)
V
1
V
s
=
Z
in
Z
in
+ Z
s
=
Z
in
Z
in
+ 10
= 0.5 Z
in
= 10 or 0.1 S
Now
Y
in
= y
11
-
y
12
y
21
y
22
+Y
L
= y
11
-
0.022
0.2 + 0.1
= 0.1
Solving for y
11
yields y
11
0.2333 S.
(b)
v
2
v
1
=
- y
21
y
22
+Y
L

=
- 2
0.2 +0.1
= - 6.667
Hence
v
2
(t) = - 6.667 v
1
(t) = - 3.333 v
s
(t) = - 33.33 u(t) V
and
P
L
=
V
2
2
R
L
=
33.33
2
10
= 111.11 W
SOLUTION PROBLEM 19.20.
(a) Writing a node equation at port 1 and mesh equation at port 2, we obtain by inspection
I
1
= 2V
1
+ 3I
2
V
2
= 2V
1
+ 2I
2
Rearranging in matrix form , we have
Prbs Chap 19, 1/7/02 P19-1 3 R. A. DeCarlo, P. M. Lin
1 -3
0 2
I
1
I
2
=
2 0
-2 1

V
1
V
2
Therefore
I
1
I
2
=
-1 1.5
-1 0.5

V
1
V
2
=
y
11
y
12
y
21
y
22

V
1
V
2

(b) Y
in
= y
11
-
y
12
y
21
y
22
+Y
L
= -1 +
1.51
0.5 + 0.25
= 1 S
(c) Here we compute Y
out
seen looking into port-2, i.e.,
Y
out
= y
22
-
y
12
y
21
y
11
+Y
s
= 0.5 +
1.51
-1 + 0
= -1 S
From current division,
i
2
(t) =
Y
out
Y
out
+Y
L
i
s
(t) =
-1
-1 + 0.25
5u(t) = 6.667u(t) A
Finally from Ohm's law
v
1
(t) = - 0.5 3i
2
(t) = - 0.5 36.667u(t) = - 10u(t) V
REMARK: Because the current source sees a negative resistance, the circuit is unstable as it
stands.
SOLUTION PROBLEM 19.21. (a) With port 2 shorted, the Laplace transform of the given data
are:
I
1
(s)
1
s
, V
1
(s)
1
s

1
s + 4

4
s(s + 4)
, I
2
(s)
1
s + 3
Hence
y
11

I
1
V
1

1 s
4
s(s + 4)

s + 4
4
and
y
21

I
2
V
1

1 (s + 3)
4
s(s + 4)

s(s + 4)
4(s + 3)
Next, with port-2 terminated in a 1- reistor, the Laplace transform of the given data are:
Prbs Chap 19, 1/7/02 P19-1 4 R. A. DeCarlo, P. M. Lin
Z
L
=Y
L
= 1
I
1
(s) = 1/s,
V
1
(s) =
1
s
-
1
s+4
+
1
(s+4)
2

=
5s + 16
s(s + 4)
2
I
2
=
- 1
s +7

Now
I
2
= -
V
2
Z
L
= - V
2
=
y
21
V
1
y
22
+Y
L


Solving for y
22
,
and using y
21

expression found earlier,

we obtain
y
22
= y
21
V
1
/I
2
- Y
L
=
- s(s + 4)
4 (s + 3)

5s + 16
s(s + 4)
2

s + 7
-1
- - 1
=
s
2
+ 23s + 64
4(s + 3)(s+ 4)
Finally, to compute y
12
, we use the defining equation for y-parameters, i.e.,
I
1
= y
11
V
1
+ y
12
V
2
from which we obtain
y
12
=
I
1
- y
11
V
1
V
2
=
1
s
-
s + 4
4

5s + 16
s(s + 4 )
2
1
s + 7
=
- (s + 7)
4 ( s + 4)

(b) Given Y
L
=1 S, the input admittance is
Y
in
= y
11
-
y
12
y
21
y
22
+Y
L
=
s + 4
4
-
- (s + 7)
4 ( s + 4)

- s(s + 4)
4(s + 3)
s
2
+ 23s + 64
4(s + 3)(s+ 4)
+1
=
(s + 4)
2
5(s+3.2)

and
Z
in
=
1
Y
in
=
5(s+3.2)
(s + 4)
2

Prbs Chap 19, 1/7/02 P19-1 5 R. A. DeCarlo, P. M. Lin
(c) For this part, we use phasors to do the sinusoidal steady state analysis: = 10 rad/s and I
1
=
1. Also,

V
1
I
1
= Z
in
=
5(j10+3.2)
(j10 + 4)
2
= 0.1973 - j0.4072
and

V
2
V
1
=
-y
21
y
22
+Y
L

=
- j10(j10 + 4)
4(j10 + 3)
-100 + 230j + 64
4(j + 3)(j+ 4)
+1
= -0.2873+ j0.1787
Thus

V
2
I
1
=
V
2
V
1

V
1
I
1
= (-0.2873+ j0.1787)(0.1973 - j0.4072)
from which we obtain in Ohms,

V
2
I
1
= -0.2873+ j0.1787 0.1973 - j0.4072 = 0.8192
SOLUTION PROBLEM 19.22. Looking into port-1, the admittance is:
Y
port 1
(s) = y
11
-
y
12
y
21
y
22
+Y
L
= 0 +
22
0 + 0.1s
=
40
s

and
Z
port 1
(s) =
s
40

Therefore
Z
in
(s) =
1000
s
+
s
40
=
s
2
+ 200
2
40s
Hence
Z
in
( j )
4 10
4

2
j40
j
4 10
4

2
40
The imaginary part is zero when 200 rad/s. Hence, the resonant frequency is 200 rad/s.
SOLUTION PROBLEM 19.23. (a) By writing nodal equations for the boxed 2-port, we have by
inspection (note passive circuit in which y
21
= y
12
):
Prbs Chap 19, 1/7/02 P19-1 6 R. A. DeCarlo, P. M. Lin
I
1
I
2
=
3s + 2 -2s -2
-2s -2 3s + 3

V
1
V
2
=
y
11
y
12
y
21
y
22

V
1
V
2
(b) In Ohms, Z
L
= s+1; Z
s
= 2. Therefore,
Y
in
= y
11
-
y
12
y
21
y
22
+Y
L
= 3s+2 -
(2s +2)
2
3s + 3 + s + 1
=
2s
2
+ 3s + 1
s + 1
= 2s + 1

V
1
V
s
=
Z
in
Z
in
+ Z
s
=
1
1 + Z
s
Y
in
=
1
1 + 2(2s + 1)
=
1
4s + 3
and
V
2
V
1
=
-y
21
y
22
+ y
L
=
2s +2
3s + 3 + s + 1
= 0.5
Thus,
V
2
V
s
=
V
1
V
s

V
2
V
1
=
1
4s + 3
0.5 =
1
8s
2
+ 14s + 6
=
1
8s+ 6

(c) The impulse reponse is
h(t) L
1
H(s) { } L
1
1
8s + 6

'


;

L
1
0.125
s + 0.75

'


;

0.125e
0.75t
u(t)
For the step response,
v
2
(t) L
1
H(s)
s

'


;

L
1
0.125
s(s + 0.75)

'


;

1
6
1 e
0.75t
( )
u(t) V
(d) We must compute the complete Laplace transform and invert. Here
V
s
(s) = 12.75
2
s
2
+ 4
and
V
2
(s) = H(s)V
2
(s) =
0.125
s+ 0.75
12.75
2
s
2
+ 4
=
3.1875
(s + 0.75) (s
2
+ 4)
We use MATLAB to compute the partial fraction expansion
n=3.1875;
d= conv([1 0.75], [ 1 0 4]);
[r p k ] = residue (n,d)
r =
-0.3493 - 0.1310i
-0.3493 + 0.1310i
0.6986
p =
-0.0000 + 2.0000i
Prbs Chap 19, 1/7/02 P19-1 7 R. A. DeCarlo, P. M. Lin
-0.0000 - 2.0000i
-0.7500
Hence, after combining the two complex terms, we obtain
V
2
(s) =
0.6986
s + 0.75
+
- 0.6986s + 0.524
s
2
+ 4
From Table 13.1, the steady state response is
v
2,ss
(t) = [-0.6986cos(2t) + 0.262sin(2t)]u(t) V
and the transient response is
v
2,tran
(t) = 0.6986e
-0.75t
u(t) V
SOLUTION PROBLEM 19.24. (a) Using the y-parameters of stage 2,
Y
in2
= y
11
-
y
12
y
21
y
22
+Y
L
= 500 +
0.175000
185 +4000
= 501.8 S
The load for stage 1 is the parallel combination of Z
in2
and the 2 k resistor. Hence, using the y-
parameters of stage 1, we obtain
Y
in1
= y
11
-
y
12
y
21
y
22
+Y
L
= 2000 +
0.52410
4
100 +501.8 + 500
= 2108.9 S
(b) We compute the following voltage gains:

V
1
V
s
=
Z
in1
Z
in1
+ Z
s
=
1
1 + Z
s
Y
in1
=
1
1 + 252108.910
-6
= 0.9499
V
2
V
1



_
,

stage1

y
21
y
22
+ Y
L,stage1

0.24
(100 + 500 + 501.8) 10
6
217.8
and
V
2
V
1



_
,

stage2

y
21
y
22
+Y
L,stage2

0.075
(185 + 4000) 10
6
17.92
Finally G
v
is the product of the three gains calculated above
G
v
= 0.9499(- 217.8) (-17.92) = 3708.2
Prbs Chap 19, 1/7/02 P19-1 8 R. A. DeCarlo, P. M. Lin
SOLUTION PROBLEM 19.25. (a) With the switch in position A, the load to the 2-port is Y
L
=
Cs = 0.25s. Hence,
V
2
V
1

y
21
y
22
+Y
L

y
21
y
22
+ 0.25s
(b) From the given data, V
1
(s) = 2/s and hence,
V
2
(s)
y
21
y
22
+ 0.25s
V
1
(s)
1 s
1 s + 0.25s

2
s

8
s(s
2
4)

2
s
+
1
s + 2
+
1
s 2
Hence, for t 0,
v
2
(t) = ( -2 + e
-2t
+ e
2t
) u(t) V
(c) The circuit is not stable in the time interval 0 to 1 s, because the transfer function has a pole
in the right half plane.
(d) v
2
(1
-
) = -2 + e
-2
+ e
2
= 5.524 V
(e) Replace the charged capacitor by the parallel combination of an admittance of 0.25s and a
current source of value 0.258.524 (in accordance with figure 14.16
(f) Z
1
(s) = b
2
1 = 4 .
(g) For t 1s, the capacitor is discharging through a 4- equivalent resistance, with a time
constant 0.254 = 1 s, and an initial voltage v
2
(1
-
) = 5.542 V.
Hence
v
2
(t) 5.524e
(t 1)
u(t 1)V
and by the ideal transformer voltage ratio property,
v
3
(t) 2.762e
(t 1)
u(t 1)V
SOLUTION PROBLEM 19.26. (a) By definition of coupled inductors
V
1
V
2



1
]
1

L
1
s Ms
Ms L
2
s



1
]
1
I
1
I
2



1
]
1
Hence, the z-parameters are:
Prbs Chap 19, 1/7/02 P19-1 9 R. A. DeCarlo, P. M. Lin
L
1
s Ms
Ms L
2
s



1
]
1
(b) By definition of coupled inductors
V
1
V
2



1
]
1

L
1
s Ms
Ms L
2
s



1
]
1
I
1
I
2



1
]
1
Hence, the y-parameters are:
L
1
s Ms
Ms L
2
s



1
]
1
The z-parameters exist independent of the values of M, L
1
, and L
2
.
SOLUTION PROBLEM 19.27. (a) By definition and the properties of the ideal transformer
z
11

V
1
I
1
1
]
1
I
2
0
R
1
+ R
2
and z
22

V
2
I
2
1
]
1
I
1
0
a
2
R
2
. Additionally, since the circuit is obviously
reciprocal, z
21
( z
12
)
V
2
I
1
1
]
1
I
2
0
aR
2
.
(b) The input impedance is given by the formula
Z
in
z
11

z
12
z
21
z
22
+ Z
L
R
1
+ R
2

a
2
R
2
2
a
2
R
2
+ R
2
R
1
+
1
a
2
+1
R
2
(c) If port-1 is open circuited, I
1
= 0. Hence,
V
1
z
12
I
2

2aKR
2
s
2
+ 4
and V
2
z
22
I
2

2a
2
KR
2
s
2
+ 4
SOLUTION PROBLEM 19.28. (a) For this part consider the figure below:
Prbs Chap 19, 1/7/02 P19-2 0 R. A. DeCarlo, P. M. Lin
With port 2 open, it follows that Z
pr
= 320/16 = 20 and
z
11

V
1
I
1
1
]
1
I
2
0
6 +
20 12
20 + 12
13.5
With port 1 open and I
2
injected into port 2, we have Z
sec
= 12*16 = 192 and
z
12

V
1
I
2
1
]
1
I
1
0

V
pr
I
2
1
]
1
I
1
0

V
sec
4I
2
1
]
1
I
1
0

I
2
320 / /192 ( )
4I
2
1
]
1
I
1
0
30
With port 2 open and I
1
injected into port 1, we have Z
pr
= 20 and
z
21

V
2
I
1
1
]
1
I
2
0

V
sec
I
1
1
]
1
I
2
0

4V
pr
I
1
1
]
1
I
2
0

4I
1
12 //20 ( )
I
1
1
]
1
I
2
0
30
With port 1 open, it follows that Z
sec
= 12*16 = 192 and
z
22

V
2
I
2
1
]
1
I
1
0
80 +
320 192
320 + 192
200
(b) V
1
(s) z
12
I
2
(s)
300
s
2
+ 4
and V
2
(s) z
22
I
2
(s)
2000
s
2
+ 4
.
Prbs Chap 19, 1/7/02 P19-2 1 R. A. DeCarlo, P. M. Lin
SOLUTION PROBLEM 19.29. (a) Writing two mesh equations we have by inspection,
V
1
V
2



1
]
1

Z
1
+ Z
3
Z
3
Z
3
+ r
m
Z
2
+ Z
3



1
]
1
I
1
I
2



1
]
1

z
11
z
12
z
21
z
22



1
]
1
I
1
I
2



1
]
1
(b) V
1
z
11
I
1

K Z
1
+ Z
3
( )
s
and V
2
z
21
I
1

K Z
3
+ r
m
( )
s
SOLUTION PROBLEM 19.30. From the result of problem 29,
Z
1
+ Z
3
Z
3
Z
3
+ r
m
Z
2
+ Z
3



1
]
1

z
11
z
12
z
21
z
22



1
]
1
Therefore, Z
3
z
12
, Z
1
z
11
Z
3
z
11
z
12
, Z
2
z
22
Z
3
z
22
z
12
, and
r
m
z
21
Z
3
z
21
z
12
.
SOLUTION PROBLEM 19.31. (a) The z-parameters can be computed by inspection (first write
the z-parameters of the passive part of the network, i.e., with the dependent source ignored; then
add the effect of the dependent source to the resulting equations.) As such, using loop equations,
Z
5 +
10
s
10 +
10
s
10
s
10 +
10
s





1
]
1
1
1

10
s
0.5s +1 s +1
1 s +1



1
]
1

(b) Z
in

5s +10
s

10
s

10
s
(s +1)
10(s +1)
s
+10


_
,

5s +10
s

10(s +1)
s(2s +1)

(5s +10)(2s +1)10(s +1)


2s(s + 0.5)

5s
2
+ 7.5s
s(s + 0.5)

5s + 7.5
s + 0.5
(c) I
1
(s)
V
1
(s)
Z
in
(s)

10
s

s + 0.5
5s + 7.5

0.6667
s
+
1.3334
s +1.5
. Hence
Prbs Chap 19, 1/7/02 P19-2 2 R. A. DeCarlo, P. M. Lin
i
1
(t)
2
3
+
4
3
e
1.5t

_
,
u(t) A
SOLUTION PROBLEM 19.32. (a) To find the resonant frequency we first find
Z
in
(s) z
11

z
12
z
21
z
22
+ Z
L
z
11

z
11
z
11
z
11
+ 10

10z
11
z
11
+10

10
1 +
10
z
11

10
1 +
s
2
+ 25
s

10s
s
2
+ s + 25
This is of the form of equation 17.18 with K = 10. Here according to equation 17.19f,

r

p
25 5 rad/s
(b) To find Q we use equation 17.19e, i.e.,
Q Q
p

p
2
p

5
1
5
(c) Using MATLAB we obtain the frequency response plots below:
Prbs Chap 19, 1/7/02 P19-2 3 R. A. DeCarlo, P. M. Lin
0 2 4 6 8 10 12 14 16 18 20
0
1
2
3
4
5
6
7
8
9
10
Frequency in rad/s
M
a
g
n
i
t
u
d
e

o
f

Z
i
n

TextEnd
Prbs Chap 19, 1/7/02 P19-2 4 R. A. DeCarlo, P. M. Lin
0 2 4 6 8 10 12 14 16 18 20
-100
-80
-60
-40
-20
0
20
40
60
80
100
Frequency in rad/s
P
h
a
s
e

o
f

Z
i
n

d
e
g
r
e
e
s
TextEnd
SOLUTION PROBLEM 19.33. (a) Since Z
z
11
z
12
z
21
z
22



1
]
1

2 3
30 4



1
]
1
. Assuming that Z
in
does
not include Z
s
, it follows that
Z
in
z
11

z
12
z
21
z
22
+ Z
L
2 +
90
24
5.75
Assuming that Zout does not include the parallel connection of Z
L
, then
Z
out
z
22

z
12
z
21
z
11
+ Z
s
4 +
90
3
34
(b) From equation 19.27
G
V
G
v2
G
v1

Z
L
z
22
+ Z
L



_
,

z
21
Z
in
+ Z
S



_
,

.
Prbs Chap 19, 1/7/02 P19-2 5 R. A. DeCarlo, P. M. Lin
Thus in MATLAB
Gv = (20/(4 + 20))*(30/(5.75+1))
Gv = 3.7037e+00
% Therefore
v2 = Gv*30
v2 = 1.1111e+02
Hence, v
2
(t) 111.11u(t) V. The power absorbed by Z
L
is therefore (in watts):
PZL = v2^2/20
PZL = 6.1728e+02
SOLUTION PROBLEM 19.34. (a) Before determining b, it is necessary to compute the
impedance seen at the secondary of the 2-port. Here,
Z
out
z
22

z
12
z
21
z
11
+ Z
s
33
2
R
0

2
3
R
0
2
R
0
+ R
0
33
2
R
0

2
R
0
32
2
R
0
Thus b must be chosen so that the impedance reflected to the secondary of the transformer is 2
, i.e.,
2
32
2
R
0
b
2
b 4 R
0
(b) First do a source transformation on the front end of the two port to obtain
v
s
(t) R
0
2 cos(2t) 32 2 cos(2t) V
Therefore, V
s,eff
32 V. Also note that the impedance looking into the primary of the two port
is
Z
in
z
11

z
12
z
21
z
22
+ Z
L
R
0

2
3
R
0
2
33
2
R
0
+ 32
2
R
0
31.015
Hence
G
V
G
v2
G
v1

Z
L
z
22
+ Z
L



_
,

z
21
Z
in
+ Z
S



_
,

0.5
Consider the following MATLAB calculations:
a=2; R0 = 16;
Zin = a*R0-2*a^3 * R0^2/(65*a^2 *R0)
Prbs Chap 19, 1/7/02 P19-2 6 R. A. DeCarlo, P. M. Lin
Zin = 3.1015e+01
ZL = 32*a^2 * R0
ZL = 2048
Zs = a*R0
Zs = 32
z22 = 33*a^2*R0
z22 = 2112
z21 = 2*a*R0
z21 = 64
Gv = (ZL/(z22+ZL))*(z21/(Zin+Zs))
Gv = 5.0000e-01
V1eff = 32;
V2eff = Gv*V1eff
V2eff = 16
b = 4*a*sqrt(R0)
b = 32
VLoadeff=V2eff/b
VLoadeff = 5.0000e-01
Pmax = VLoadeff^2/2
Pmax = 1.2500e-01
Hence, max power transferred to the load is 125 mW.
SOLUTION PROBLEM 19.35. (a) Using the usual formula
Z(s) z
11

z
12
z
21
z
22
+ Z
L
0
1000 (1000)
0 +10
8
s
0.01s
Yes, the input to the 2-port looks like a 0.01 H inductor.
(b) Under the given conditions, the circuit reduces to parallel RLC with R
eq
= 50 k, L = 0.01
H, and C = 100 pF. Therefore,
m

1
LC
10
6
rad/s and B
1
R
eq
C
2 10
5
rad/s.
(c) For this circuit
Prbs Chap 19, 1/7/02 P19-2 7 R. A. DeCarlo, P. M. Lin
V
1
(s)
1
1
R
eq
+
1
Ls
+ Cs

V
in
R
1

s C
s
2
+
1
R
eq
C
s +
1
LC

V
in
R
1

10
6
s
s
2
+ 2 10
6
s +10
12
V
in
For the impulse response, V
in
= 1 and from MATLAB
n = [1e6 0];d = [1 2e6 1e12];
[r,p,k] = residue(n,d)
r =
1.0000e+06
-1.0000e+12
p =
-1000000
-1000000
k =
[]
Therefore, the impulse response is:
h(t) 10
6
10
12
t
( )
e
10
6
t
u(t) V
(d) y
0 1000
1000 0



1
]
1
1

0 0.001
0.001 0



1
]
1
S
(e) V
2
(s)
y
21
y
22
+Y
L
V
1
(s)
0.001
0 +10
8
s

10
6
s
s
2
+ 2 10
6
s +10
12

10
11
(s +10
6
)
2
Hence
v
2
(t) 10
11
te
10
6
t
u(t) V.
SOLUTION PROBLEM 19.36.
Prbs Chap 19, 1/7/02 P19-2 8 R. A. DeCarlo, P. M. Lin
(a) Z
in2
z
11

z
12
z
21
z
22
+ Z
L
62.582
1.2075 63.75
1.25 + 0.016
1.7778 k
Because z
12
for stage 1 is zero,
Z
in
z
11

z
12
z
21
z
22
+ Z
in2
/ /2
z
11
2 k
(b) G
v1

V
1
V
s

Z
in
Z
in
+ 75
0.96386 . Let Z
L1
Z
in2
//2 0.94118 k. Then
G
v2

V
2
V
1

Z
L1
Z
L1
+ z
22

z
21
Z
in
22.472. Finally G
v3

V
out
V
2

Z
L
Z
L
+ z
22

z
21
Z
in2
0.45319.
Thus
G
v
G
v1
G
v 2
G
v 3
9.816
SOLUTION PROBLEM 19.37. (a) By inspection via mesh standard equations
z
L
1
s Ms
Ms L
2
s



1
]
1
(b) Utilizing the properties of an ideal transformer,
z
11

V
1
I
1
1
]
1
I
2
0
L
1
s z
22

V
2
I
2
1
]
1
I
1
0

k
2
L
2
L
1
L
1
s + (1 k
2
)L
2
s L
2
s
z
12

V
1
I
2
1
]
1
I
1
0

k L
2
L
1
L
1
s k L
1
L
2
s Ms
Finally,
z
21

V
2
I
1
1
]
1
I
2
0

k L
2
L
1
L
1
s k L
1
L
2
s Ms
(c) Utilizing the properties of an ideal transformer,
Prbs Chap 19, 1/7/02 P19-2 9 R. A. DeCarlo, P. M. Lin
z
22

V
2
I
2
1
]
1
I
1
0
L
2
s z
11

V
1
I
1
1
]
1
I
2
0

k
2
L
1
L
2
L
2
s + (1 k
2
)L
1
s L
1
s
z
12

V
1
I
2
1
]
1
I
1
0

k L
1
L
2
L
2
s k L
1
L
2
s Ms
Finally,
z
21

V
2
I
1
1
]
1
I
2
0

k L
1
L
2
L
2
s k L
1
L
2
s Ms
(d) For this circuit k = 1 and the turns ratio a
10
2
1
0.1. Under this condition the given
circuit reduces to a current source of value I
s
V
in
5000 driving a parallel RLC with R =
5000 , L = L
1
= 0.01 H, the capacitance reflected to the primary of value C 10
8
F.
Therefore
m

1
LC
10
5
rad/s and B
1
R
eq
C
2 10
4
rad/s.
Finally, at w = wm, the circuit is resonant and Vin appears across the primary of the transformer.
This voltage is then stepped up by a factor of 10. Therefore
V
out
V
in
max
10.
SOLUTION PROBLEM 19.38. (a) By inspection
V
1
I
2



1
]
1

Z
1
1
1 Y
2



1
]
1
I
1
V
2



1
]
1

h
11
h
12
h
21
h
22



1
]
1
I
1
V
2



1
]
1
(b) By inspection
V
1
V
2
Z
1
I
2
V
1
+ Z
1
I
2
V
2
and
I
1
Y
2
V
1
I
2
Y
2
V
1
I
2
I
1
Prbs Chap 19, 1/7/02 P19-3 0 R. A. DeCarlo, P. M. Lin
In matrix form,
V
1
I
2



1
]
1

0 1
1 0



1
]
1
I
1
V
2



1
]
1

V
1
I
2



1
]
1

1 Z
1
Y
2
1



1
]
1
1
0 1
1 0



1
]
1
I
1
V
2



1
]
1
Thus
V
1
I
2



1
]
1

1
Z
1
Y
2
+1
Z
1
1
1 Y
2



1
]
1
I
1
V
2



1
]
1
SOLUTION PROBLEM 19.39. (a) From problem 19.38b we have
V
1
I
2



1
]
1

1
Z
1
Y
2
+1
Z
1
1
1 Y
2



1
]
1
I
1
V
2



1
]
1

1
1
RCs
+1
1
Cs
1
1
1
R





1
]
1
1
1
I
1
V
2



1
]
1

1
RCs +1
R RCs
RCs Cs



1
]
1
I
1
V
2



1
]
1
(b) This part is a cascade of part (a) and an ideal transformer. Label the voltage and current at
the port 2 of N
1
as

V
2
and

I
2
. From the properties of the ideal transformer, V
2
n

V
2
and
I
2


I
2
n . Hence
V
1

I
2



1
]
1

1
RCs +1
R RCs
RCs Cs



1
]
1
I
1

V
2



1
]
1

V
1
nI
2



1
]
1

1
RCs +1
R RCs
RCs Cs



1
]
1
I
1
V
2
n



1
]
1
Therefore
V
1
I
2



1
]
1

1
RCs +1
R RCs n
RCs n Cs n
2



1
]
1
I
1
V
2



1
]
1
From table 19.1, if h
22
=
Cs
n
2
(RCs +1)
0, then the z-parameters exist and if h
11
=
R
RCs +1
0,
the y-parameters exist, i.e., if C 0 and R 0 respectively.
Prbs Chap 19, 1/13/02 P19-1 R. A. DeCarlo, P. M. Lin
CHAPTER 19 PROBLEM SOLUTIONS
SOLUTION PROBLEM 19.40. (a) Let Z
1
= R and Z
2
= 1/Cs or Y
2
= Cs. From problem 38,
h
Z
1
1
1 Y
2



1
]
1

R 1
1 Cs



1
]
1
(b) This part is a cascade of an ideal transformer and part (a). Label the voltage and current at
the port 1 of N
1
as

V
1
and

I
1
. From the properties of the ideal transformer, V
1
b

V
1
and
I
1


I
1
b . Hence

V
1
I
2




1
]
1
1

R 1
1 Cs



1
]
1

I
1
V
2




1
]
1
1

V
1
b
I
2



1
]
1

R 1
1 Cs



1
]
1
bI
1
V
2



1
]
1
Therefore
V
1
I
2



1
]
1

b
2
R b
b Cs




1
]
1
1
I
1
V
2



1
]
1
From table 19.1, if h
22
= Cs 0, then the z-parameters exist and if h
11
= b
2
R 0, the y-
parameters exist, i.e., if C 0 and R 0 (assuming reasonably that b 0) respectively.
SOLUTION PROBLEM 19.41. For this solution we apply the definition of h-parameters: by
inspection
h
11

V
1
I
1
V
2
0

1
2 + 2s
h
21

I
2
I
1
V
2
0

2V
1
2sV
1
I
1
V
2
0

(2 2s)V
1
(2 + 2s)V
1
V
2
0

1 s
s +1
When I
1
= 0, then I
2
2V
1
+ 2V
1
4V
1
and V
2
0.5(2V
1
) +
1
2s
(2V
1
)
s +1
s
V
1
.
Prbs Chap 19, 1/13/02 P19-2 R. A. DeCarlo, P. M. Lin
Therefore, h
12

V
1
V
2
I
1
0

s
s +1
and h
22

I
2
V
2
I
1
0

4V
1
V
2
I
1
0

4s
s +1
.
SOLUTION PROBLEM 19.42. (a) In MATLAB
h11 = 250; h12 = 0.025; h21 = 12.5; h22 = 2.25e-3;
Zs = 1e3; ZL = 500;
YL = 1/ZL
YL = 2.0000e-03
Zin = h11 - h12*h21/(h22 + (1/ZL))
Zin = 1.7647e+02
Yout = h22 - h12*h21/(h11 + Zs)
Yout = 2.0000e-03
Zout = 1/Yout
Zout = 500
(b)
% Gv1 = V1/Vs
Gv1 = Zin/(Zin + Zs)
Gv1 = 1.5000e-01
% Gv2 = V2/V1
Gv2 = -h21/(Zin*(h22 + YL))
Gv2 = -1.6667e+01
Gv = Gv1*Gv2
Gv = -2.5000e+00
(c) Given the above, the Thevenin equivalent seen by the capacitor is V
oc
2.5V
in
and
R
th
500 .
In MATLAB
Zth = ZL*Zout/(ZL + Zout)
Zth = 250
Vin = 10;
Voc = -2.5*Vin;
w = 400;
Prbs Chap 19, 1/13/02 P19-3 R. A. DeCarlo, P. M. Lin
Zc = 1/(j*w*10e-6)
Zc = 0 - 2.5000e+02i
Vc = Voc*Zc/(Zth + Zc)
Vc = -1.2500e+01 + 1.2500e+01iV2mag = abs(Vc)
V2mag = abs(Vc)
V2mag =
1.7678e+01
V2ang = angle(Vc)*180/pi
V2ang =
135
From above,
v
2
(t) 17.678 2 cos(400t +135
o
) V
Therefore
Pave = V2mag^2/500
Pave =
6.2500e-01
SOLUTION PROBLEM 19.43.
(a) Using the h-parameters of stage 2
Z
in2
= h
11
-
h
12
h
21
h
22
+Y
L
= 1000+
0.96651
0.0008 + 1/64
= 4000
The load for stage 1 is the parallel combination of Z
in2
and the 3 k resistance. However,
because h
12
= 0, the input impedance is unaffected by the load, and hence for stage 1,
Z
in1
= h
11
= 2000
(b) For stage 1, because h
12
= 0, the output impedance is unaffected by the source impedance.
Thus,
Y
out1
= h
22
= 0.0510
-3
S,
Z
out1
=1/Y
out1
= 2010
3

Prbs Chap 19, 1/13/02 P19-4 R. A. DeCarlo, P. M. Lin
For stage 2, the source impedance is the parallel combination of Z
out1
and the 3 k resistance.
Thus
Z
s2
=
200003000
20000 + 3000
=2608.7
and
Y
out2
= h
22
-
h
12
h
21
h
11
+Z
s2
= 0.0008+
0.96651
1000 + 2608.7
= 0.0145 S
Z
out2
=1/Y
out2
= 69.19
(c)

V
1
V
s
=
Z
in1
Z
in1
+ Z
s
=
2000
2000 + 2000
= 0.5
The load of stage 1 is the parallel combination of Z
in2
and Z
m
. Thus
Y
L1
= Y
in2
+ 1/3000 = 5.834 10
-4
S
Hence
Z
L1
= 1/ Y
L1
= 1714
and
V
2
V
1
stage 1
=
1
Z
in1

-h
21
h
22
+ Y
L1
=
1
2000

-50
(0.05 + 0.5834) 10
-3
= - 39.46
For stage 2, the load is 64 . Hence
V
2
V
1
stage 2
=
1
Z
in2

-h
21
h
22
+ Y
L2
=
1
4000

51
(0.8 + 1000/64) 10
-3
= 0.7762
Finally, the overall voltage gain is the product of the three gains calculated above
V
out
V
s
= 0.5(- 39.46) (0.7762) = - 15.32
(d) The input circuit consists of a series connection of V
s
, Z
s
, C and Z
in1
. The remainder of the
circuit is resistive and has no effect on the frequency response. The magnitude response is of the
high pass type with
f
3dB

1
2(R
s
+ R
in1
)C

1
2(2000 + 2000)10
6
39.789 Hz
SOLUTION PROBLEM 19.44. To meet the required matching, we must have
Prbs Chap 19, 1/13/02 P19-5 R. A. DeCarlo, P. M. Lin
Z
out
= Z
out2
= Z
L
= 64 =
1
Y
out2
This requires that
Y
out2
= h
22
-
h
12
h
21
h
11
+Z
s2
= 0.0008+
0.96651
1000 +Z
s2
=
1
64

Solving for Z
s2
, we obtain Z
s2
2323.2 . Now for stage 2, Z
s2
is the parallel combination of
Z
out1
= 20 k and Z
m
:
Z
s2
=
20000Z
m
20000 +Z
m
= 2323.2
from which Z
m
2628.5 . With this new value of Z
m
, we repeat the calculations of problem
19.43 to obtain Z
in
= 2000 and V
out/
V
s
= -14 .26. Details follow.
Using the h parameters of stage 2
Z
in2
= h
11
-
h
12
h
21
h
22
+Y
L
= 1000+
0.96651
0.0008 + 1/64
= 4000
For stage 1, because h
12
= 0, the input impedance is not affected by the load and
Z
in1
= h
11
= 2000 . The voltage gains of the various stages are:

V
1
V
s
=
Z
in1
Z
in1
+ Z
s
=
2000
2000 + 2000
= 0.5
The load of stage 1 is the parallel combination of Z
in2
and Z
m
. Thus
Y
L1
= Y
in2
+ 1/2628.5 = 6.3044 10
-4
S
Hence
V
2
V
1
stage 1
=
1
Z
in1

-h
21
h
22
+ Y
L1
=
1
2000

-50
(0.05 + 0.63044) 10
-3
= - 36.74
For stage 2, the load is 64 . Hence
V
2
V
1
stage 2
=
1
Z
in2

-h
21
h
22
+ Y
L2
=
1
4000

51
(0.8 + 1000/64) 10
-3
= 0.7762
Finally, the overall voltage gain is the product of the three gains calculated above
V
out
V
s
= 0.5(- 36.74) (0.7762) = - 14.26
Prbs Chap 19, 1/13/02 P19-6 R. A. DeCarlo, P. M. Lin
SOLUTION PROBLEM 19.45.
(a) Using the h-parameters of stage 2
Z
in2
= h
11
-
h
12
h
21
h
22
+Y
L
= 500 +
0.96651
0.0016 + 1/32
= 2000
The load for stage 1 is the parallel combination of Z
in2
and the 1.5 k resistance. However,
because h
12
= 0, the input impedance is unaffected by the load, and Z
in1
= h
11
= 1000 .
(b) For stage 1, because h
12
=0, the output impedance is unaffected by the source
impedance, and
Y
out1
= h
22
= 0.110
-3
S,
Z
out1
=1/Y
out1
=10
4

For stage 2, the source impedance is the parallel combination of Z
out1
and the 1.5 k resistor
Z
s2

10
4
1.5 10
3
10
4
+1.5 10
3
1304.3
and
Y
out2
= h
22
-
h
12
h
21
h
11
+Z
s2
= 0.0016+
0.96651
500 + 1304.3
= 0.0289 S
Z
out2
=1/Y
out2
= 34.6
(c)

V
1
V
s
=
Z
in1
Z
in1
+ Z
s
=
1000
1000 + 1000
= 0.5
The load of stage 1 is the parallel combination of Z
in2
and Z
m
. Thus
Y
L1
= Y
in2
+ 1/1500 = 1.1667 10
-3
S
Hence
Z
L1
= 1/ Y
L1
= 857.1
V
2
V
1
stage 1
=
1
Z
in1

-h
21
h
22
+ Y
L1
=
1
1000

-50
(0.1 + 1.1667) 10
-3
= - 39.47
For stage 2, the load is 32 . Hence
Prbs Chap 19, 1/13/02 P19-7 R. A. DeCarlo, P. M. Lin
V
2
V
1
stage 2
=
1
Z
in2

-h
21
h
22
+ Y
L2
=
1
2000

51
0.0016 + 1/32
= 0.7762
Finally, the overall voltage gain is the product of the three gains calculated above
V
out
V
s
= 0.5(- 39.47) (0.7762) = - 15.32
(d) The input circuit consists of a series connection of V
s
, Z
s
, C and Z
in1
. The remainder of the
circuit is resistive and has no effect on the frequency response. The magnitude response is of the
highpass type with
f
3dB

1
2(R
s
+ R
in1
)C

1
2(1000 +1000)10
6
79.58 Hz
SOLUTION PROBLEM 19.46. (a) Since the currents through Y
L
and h
22
are the same, h
22
= Y
L
.
(b) From current division, I
2

Y
L
Y
L
+ h
22
h
21
I
1

I
2
I
1

Y
L
h
21
Y
L
+ h
22
.
(c) 150
I
2
I
1

Y
L
h
21
Y
L
+ h
22
0.5h
21
h
21
300.
(d) h
12

V
1
V
2
1
]
1
I
1
0

1
2
0.5 .
(e)
I
1
I
s

Z
s
Z
s
+ Z
in

9 10
3
9 10
3
+ Z
in
0.9 Z
in
1000 . Given this quantity,
h
11
Z
in
+
h
12
h
21
h
22
+Y
L
1000
150
0.25
400 .
SOLUTION PROBLEM 19.47. Recall that
V
1
I
2



1
]
1

h
11
h
12
h
21
h
22



1
]
1
I
1
V
2



1
]
1
Prbs Chap 19, 1/13/02 P19-8 R. A. DeCarlo, P. M. Lin
(a) From this expression and specification 1, h
12

V
1
V
2
1
]
1
I
1
0
0 .
(b) From the formula for Y
out
(equation 19.50), specification 2, and the result of part (a), we
have
Y
out

1
Z
out

1
800
h
22

h
12
h
21
h
11
+ Z
s
h
22
Hence, h
22

1
800
1.25 mS.
For maximum power transfer from amplifier to the load,
Z
out
800 b
2
Z
L
8b
2
Therefore, b = 10.
(c) and (d) Observe that
Z
in
h
11

h
12
h
21
h
22
+ Y
L
b
2
h
11
From specification 3 and voltage division,
V
1
V
s

24
25

Z
in
Z
in
+ 40
Equivalently,
V
s
V
1

25
24
1+
40
Z
in
Hence h
11
Z
in
40 24 960 .
(e) From equation 19.51,
G
v2

V
2
V
1
100
h
21
Z
in
h
22
+ Y
L
b
2
( )

h
21
960 1.25 10
3
+1.25 10
3
( )

h
21
2.4
Hence, h
21
240 .
Prbs Chap 19, 1/13/02 P19-9 R. A. DeCarlo, P. M. Lin
(f) The power delivered to the load is P
L

V
2
2
800
and the power delivered to the amplifier is
P
amp

V
1
2
Z
in

V
1
2
960
. Therefore the power gain is
P
L
P
amp

96
80
V
2
V
1



_
,

2
1.2 10
4
SOLUTION PROBLEM 19.48. Recall that
V
1
I
2



1
]
1

h
11
h
12
h
21
h
22



1
]
1
I
1
V
2



1
]
1
(a) From this expression and specification 1, h
12

V
1
V
2
1
]
1
I
1
0
0.01.
(b) For maximum power transfer from amplifier to the load,
Z
out
800 b
2
Z
L
8b
2
Therefore, b = 10.
Now we find Z
in
. From specification 3 and voltage division,
V
1
V
s

24
25

Z
in
Z
in
+ 40
(1)
Equivalently,
V
s
V
1

25
24
1 +
40
Z
in
Z
in
40 24 960
Using the formula for Z
in
we have the following equation
Z
in
960 h
11

0.01h
21
h
22
+1.25 10
3
h
11
960 + 0.01
h
21
h
22
+1.25 10
3
(2)
Prbs Chap 19, 1/13/02 P19-10 R. A. DeCarlo, P. M. Lin
But from the given specs,
V
2
V
1
100
h
21
Z
in
h
22
+ Y
L
b
2
( )

h
21
960 h
22
+1.25 10
3
( )
which implies that
h
21
h
22
+1.25 10
3
( )
960 10
2
(3)
Substituting (3) into (2) allows us to solve for h
11
:
h
11
960 + 0.01 960 10
2
1920
Now let us rewrite equation (3) as:
h
21
960 10
2
h
22
960 10
2
1.25 10
3
120 (4)
Also
Y
out
1.25 10
3
h
22

h
12
h
21
h
11
+ Z
s
h
22

0.01h
21
1960

Equivalently,
1960 1.25 10
3
2.45 1960h
22
0.01h
21
(5)
Solving equations (4) and (5) simultaneously in MATLAB yields
A = [1 -960e2;-0.01 1960]
A =
1.0000e+00 -9.6000e+04
-1.0000e-02 1.9600e+03
b = [120; 2.45]
b =
1.2000e+02
2.4500e+00
x = A\b
x =
Prbs Chap 19, 1/13/02 P19-11 R. A. DeCarlo, P. M. Lin
4.7040e+02
3.6500e-03
h21 = x(1); h22 = x(2);
h = [1920 0.01;h21 h22]
h =
1.9200e+03 1.0000e-02
4.7040e+02 3.6500e-03
We can verify these results as well as compute the overall amplifier gain using the following m-
file:
% two-port analysis in terms of h-parameters
function [zin, zout] =twoport(h, zL, zs)
['twoport analysis using h-parameters']
h11= h(1,1); h12=h(1,2); h21=h(2,1); h22=h(2,2);
zin = h11 - h12*h21/(h22+ 1/zL)
yout= h22 - h12*h21/(h11+zs);
zout= 1/yout
v1tovs= zin/(zin+zs)
v2tov1= -h21/(zin*(h22+1/zL))
v2tovs= v1tovs*v2tov1
twoporth(h,ZL,Zs)
ans =
twoport analysis using h-parameters
zin =
960
zout =
8.0000e+02
v1tovs =
9.6000e-01
v2tov1 =
-100
v2tovs =
-96
Prbs Chap 19, 1/13/02 P19-12 R. A. DeCarlo, P. M. Lin
Hence the overall voltage gain is V
L
/V
s
= 96/10 = 96 because of the transformer.
Finally to compute power gains,
Vs = 1; Vin = 24/25;
VL = -9.6;
Pin = Vin^2/960
Pin =
9.6000e-04
Pload = VL^2/8
Pload =
1.1520e+01
Pgain = Pload/Pin
Pgain =
12000
SOLUTION PROBLEM 19.49.
(a) h
21

I
2
I
1
V
2
0

C sV

+ g
m
V

1
R

+ C

+ C
( )
s



_
,

V

C s + g
m
C

+ C
( )
s +
1
R

(b) h
11

V
1
I
1
V
2
0

R
x
+
1
1
R

+ C

+ C
( )
s





_
,



I
1
I
1
R
x
+
1
1
R

+ C

+ C
( )
s
(c) Under the condition that I
1
= 0, V
1
= V. Using voltage division from V
2
to V

:
Prbs Chap 19, 1/13/02 P19-13 R. A. DeCarlo, P. M. Lin
h
12

V
1
V
2
I
1
0

V
2

1
1
R

+ C

s





_
,



1
C s
+
1
1
R

+ C

C s
1
R

+ C

s + C s

C s
C

+ C
( )
s +
1
R

(d) h
22

I
2
V
2
I
1
0

g
m
V

+
1
R

+ C

s



_
,

V

+ C
( )
s +
1
R

C s
V

C s C

s + g
m
+
1
R




_
,

C

+ C
( )
s +
1
R

SOLUTION PROBLEM 19.50: (a) Recall t-parameter relationship:


V
1
I
1



1
]
1

t
11
t
12
t
21
t
22



1
]
1
V
2
I
2



1
]
1
For the given network,V
2
Z
L
(I
2
) and
V
1
t
11
V
2
+ t
12
(I
2
) t
11
Z
L
(I
2
) + t
12
(I
2
) (t
11
Z
L
+ t
12
)(I
2
)
Further,
I
1
t
21
V
2
+ t
22
(I
2
) (t
21
Z
L
+ t
22
)(I
2
)
Hence,
Z
in

V
1
I
1

(t
11
Z
L
+ t
12
)(I
2
)
(t
21
Z
L
+ t
22
)(I
2
)

t
11
Z
L
+ t
12
t
21
Z
L
+ t
22
(b) For the output impedance relationship, from the t-parameter relationships
V
1
Z
s
(I
1
) t
11
V
2
+ t
12
(I
2
) Z
s
(t
21
V
2
+ t
22
(I
2
))
Grouping V2 and I2 terms together on separate sides of the equation implies
(t
21
Z
s
+ t
11
)V
2
(t
22
Z
s
+ t
12
)I
2
Thus
Prbs Chap 19, 1/13/02 P19-14 R. A. DeCarlo, P. M. Lin
Z
out

V
2
I
2

t
22
Z
s
+ t
12
t
21
Z
s
+ t
11
SOLUTION PROBLEM 19.51:
45. From the z-parameter relationships
V
1
z
11
I
1
+ z
12
I
2
V
1
z
11
I
1
z
12
I
2
z
12
(I
2
)
and
V
2
z
21
I
1
+ z
22
I
2
z
21
I
1
V
2
+ z
22
(I
2
)
These two equations in matrix form are:
1 z
11
0 z
21



1
]
1
V
1
I
1



1
]
1

0 z
12
1 z
22



1
]
1
V
2
I
2



1
]
1
Solving for V
1
I
1
[ ]
T
yields
V
1
I
1



1
]
1

1
z
21
z
21
z
11
0 1



1
]
1
0 z
12
1 z
22



1
]
1
V
2
I
2



1
]
1

1
z
21
z
11
z
1 z
22



1
]
1
V
2
I
2



1
]
1
SOLUTION PROBLEM 19.52: For figure 19.52a, by inspection
V
1
I
1



1
]
1

1 Z
1
0 1



1
]
1
V
2
I
2



1
]
1
Therefore T is as indicated in the problem.
For figure 19.52b, by inspection
V
1
I
1



1
]
1

1 0
Y
2
1



1
]
1
V
2
I
2



1
]
1
Therefore T is as indicated in the problem.
SOLUTION PROBLEM 19.53: Here we use the results of problem 19.52:
(a) For figure (a)
T
new

1 Z
1
0 1



1
]
1
1 0
Y
2
1



1
]
1

1+ Z
1
Y
2
Z
1
Y
2
1



1
]
1
(b) For figure (b)
Prbs Chap 19, 1/13/02 P19-15 R. A. DeCarlo, P. M. Lin
T
new

1 0
Y
2
1



1
]
1
1 Z
1
0 1



1
]
1

1 Z
1
Y
2
1 + Z
1
Y
2



1
]
1
SOLUTION PROBLEM 19.54: By the properties of an ideal transformer,
V
1
nV
2
I
1

1
n
I
2

1
n


_
,
(I
2
)
Therefore
V
1
I
1



1
]
1

n 0
0 1 n



1
]
1
V
2
I
2



1
]
1
with the t-parameters given by the 2x2 matrix.
SOLUTION PROBLEM 19.55: This problem uses the results of the previous two problems.
(a)
T
1+ Z
1
Y
2
Z
1
Y
2
1



1
]
1
n 0
0 1 n



1
]
1

n 1+ Z
1
Y
2
( ) Z
1
n
nY
2
1 n



1
]
1
(b)
T
n 0
0 1 n



1
]
1
1 + Z
1
Y
2
Z
1
Y
2
1



1
]
1

n 1+ Z
1
Y
2
( ) nZ
1
Y
2
n 1 n



1
]
1
SOLUTION PROBLEM 19.56:
(a)
T
0.25 0
0 4



1
]
1
8 4
2 5



1
]
1

2 1
8 20



1
]
1
(b)
T
1 0
1 R 1



1
]
1
8 4
2 5



1
]
1

8 4
2 + 8 R 5 + 4 R



1
]
1
(c)
T
1+ Z
1
Y
2
Z
1
Y
2
1



1
]
1
8 4
2 5



1
]
1

1+ 0.25s 0.5s
0.5 1



1
]
1
8 4
2 5



1
]
1

8 + 3s 4 + 3.5s
6 7



1
]
1
SOLUTION PROBLEM 19.57: For each 2-port of the form of figure P19.53a, we have that the t-
parameters are given by
Prbs Chap 19, 1/13/02 P19-16 R. A. DeCarlo, P. M. Lin
T
1+ Z
1
Y
2
Z
1
Y
2
1



1
]
1
The given network consists of three such sections in cascade whose t-parameters are respectiely,
T
1

1 + s
2
s
s 1




1
]
1
1
, T
2

1+1 0.5s 1
0.5s 1



1
]
1

1 + 0.5s 1
0.5s 1



1
]
1
, and
T
3

1 + 2s
1
4
s 2s
1
4
s 1





1
]
1
1
1

1+
1
2
s
2
2s
1
4
s 1





1
]
1
1
1
Observe that
T
1
T
2

1
2
s
3
+
3
2
s
2
+
1
2
s +1 s
2
+ s +1
1
2
s
2
+
3
2
s s +1





1
]
1
1
1

and the overall t-parameters are
T T
1
T
2
T
3

1
4
s
5
+
3
4
s
4
+ s
3
+
9
4
s
2
+
3
4
s +1 s
4
+ 3s
3
+ 2s
2
+ 3s +1
1
4
s
4
+
3
4
s
3
+
3
4
s
2
+
7
4
s s
3
+ 3s
2
+ s +1





1
]
1
1
1
SOLUTION PROBLEM 19.58: This poblem is done primarily in MATLAB.
Part (a)
% The following code solves part (a) of the problem.
% Parameter Specification
t11= 0.895+j*0.022;
t22= t11;
t12= 40 + j*180;
% t21= (t11*t22 -1)/t12;
% The above formula follows because it is a reciprocal network.
% The actual value is specified.
t21= -2.6175e-05+j*1.1023e-03;
t=[t11 t12; t21 t22]
% Part (a) calculations
vr= 115200
ir = 361
vs= t11*vr +t12*ir
magvs=abs(vs)
angvs= angle(vs)*180/pi
Prbs Chap 19, 1/13/02 P19-17 R. A. DeCarlo, P. M. Lin
is=t21*vr+t22*ir
magis= abs(is)
angis= angle(is)*180/pi
pscomp=vs*conj(is)
ps=real(pscomp)
pr=real(vr*conj(ir))
eff= pr/ps
pf= ps/abs(pscomp)
ploss= ps- pr
The MATLAB output is as follows:
T =
8.9500e-01 + 2.2000e-02i 4.0000e+01 + 1.8000e+02i
-2.6175e-05 + 1.1023e-03i 8.9500e-01 + 2.2000e-02i
vr = 115200
ir = 361
vs = 1.1754e+05 + 6.7514e+04i
magvs = 1.3555e+05
angvs = 2.9872e+01
is = 3.2008e+02 + 1.3493e+02i
magis = 3.4736e+02
angis = 2.2857e+01
pscomp = 4.6733e+07 + 5.7501e+06i
ps = 4.6733e+07
pr = 41587200
eff = 8.8989e-01
pf = 9.9252e-01
ploss = 5.1458e+06
Part (b)
% The following code solves part (b) of the problem.
zL=500;
zin= (t11*zL + t12)/( t21*zL + t22)
yin = 1/zin
vsnew=134000;
iin= yin*vsnew
psnew= vsnew^2*real(yin)
m= inv(t)
v2=m(1,1)*vsnew +m(1,2) *iin
magv2=abs(v2)
iload= m(2,1) *vsnew + m(2,2)* iin
magild = abs(iload)
% Check value of rload
rload=abs(v2)/abs(iload)
The MATLAB output for part (b) is:
Prbs Chap 19, 1/13/02 P19-18 R. A. DeCarlo, P. M. Lin
zin = 4.8759e+02 - 1.0031e+02i
yin = 1.9676e-03 + 4.0478e-04i
iin = 2.6366e+02 + 5.4241e+01i
psnew = 3.5331e+07
% m is the inverse of T-matrix.
m =
8.9500e-01 + 2.2000e-02i -4.0000e+01 - 1.8000e+02i
2.6175e-05 - 1.1023e-03i 8.9500e-01 + 2.2000e-02i
v2 = 1.1915e+05 4.6681e+04i
magv2 = 1.2796e+05
iload = 2.3829e+02 - 9.3362e+01i
magild = 2.5593e+02
rload = 5.0000e+02
SOLUTION PROBLEM 19.59. From the given information, the circuit is linear and reciprocal.
(a) Here i
2
(t) is the integral of i
1
(t). Therefore, the new v
1
(t) is the integral of the old v
2
(t).
The result for t 0 is:
v
1
(t) 3.005 3e
t
+ e
t
[0.00865sin(500t) 0.005cos(500t)]
3.005 3e
t
+ e
t
2 10
5
cos(500t /6) + 0.01sin(500t /6)
[ ]
(b) From the problem statement
z
21
(s)
3
s +1
+ 5
cos

6


_
,
s +1 ( ) + 250
s +1 ( )
2
+ 500 ( )
2
From reciprocity, z
12
(s) z
21
(s). For steady state analysis, we use phasors to obtain
V
1
z
12
( j500)I
2

3
1+ j500
+ 5
cos

6


_
,
(1+ j500) + 250
(1+ j500)
2
+ (500)
2

v
1ss
(t) 2.505cos(500t 30.15
o
) V
SOLUTION PROBLEM 19.60. Writing loop equations we have:
(i) For the left loop,
V
1
aV
2
3(I
1
+ I
3
) 0 V
1
aV
2
3I
1
+ 3I
3
(ii) For the right loop,
Prbs Chap 19, 1/13/02 P19-19 R. A. DeCarlo, P. M. Lin
V
2
+ bI
1
(I
2
I
3
) 0 V
2
bI
1
+ I
2
I
3
(iii) For the middle loop,
bI
1
+ (I
3
I
2
) + I
3
+ 3(I
1
+ I
3
) 0 0 (b + 3)I
1
I
2
+ 5I
3
Writing the first two equations in matrix form yields
1 a
0 1



1
]
1
V
1
V
2



1
]
1

3 0 3
b 1 1



1
]
1
I
1
I
2
I
3





1
]
1
1
1
whose solution is
V
1
V
2



1
]
1

1 a
0 1



1
]
1
3 0 3
b 1 1



1
]
1
I
1
I
2
I
3





1
]
1
1
1

3 ab a 3 a
b 1 1



1
]
1
I
1
I
2
I
3





1
]
1
1
1
Hence
V
1
V
2



1
]
1

3 ab a
b 1



1
]
1
I
1
I
2



1
]
1
+
3 a
1



1
]
1
I
3
From the third equation
I
3
0.2b 0.6 0.2 [ ]
I
1
I
2



1
]
1
Thus
V
1
V
2



1
]
1

3 ab a
b 1



1
]
1
+
3 a
1



1
]
1
0.2b 0.6 0.2 [ ]



_
,

I
1
I
2



1
]
1
V
1
V
2



1
]
1

0.6b 0.8ab + 0.6a +1.2 0.6 + 0.8a


0.8b + 0.6 0.8



1
]
1
I
1
I
2



1
]
1
For reciprocity,
z
12
z
21
a b
SOLUTION PROBLEM 19.61. (a) and (b) together: Observe that
V
1
0.5V
1
I
1
V
2
I
1
0.5V
1
V
2
V
1
2I
1
+ 2V
2
and
I
1
+ I
2
0.5(V
2
aI
1
) I
2
(0.5a +1)I
1
+ 0.5V
2
The h-parameters are
H
2 2
(0.5a +1) 0.5



1
]
1
Prbs Chap 19, 1/13/02 P19-20 R. A. DeCarlo, P. M. Lin
Reciprocity requires that
h
12
h
21
2 0.5a +1 a 2
Thus
H
2 2
2 0.5



1
]
1
SOLUTION PROBLEM 19.62. There are 2 corrections in the problem statement concerning the
second set of expressions:
(1) v
1
(t) = 2e
-t
- 1.5e
-1.5t
V
(2) i
2
(t) = 0.5 e
-1.5t
A
For both parts, recall, the y-parameters:
I
1
I
2



1
]
1

y
11
y
21
y
12
y
22



1
]
1
V
1
V
2



1
]
1
(a) Part-1: From the first set of given data (V
2
= 0)
I
1

1
s
, I
2

0.5
s +1
, V
1

0.5
s +1
+
0.5
(s +1)
2

0.5(s + 2)
(s +1)
2
Hence,
y
11

I
1
V
1
1
]
1
V
2
0

2(s +1)
2
s(s + 2)
, y
21

I
2
V
1
1
]
1
V
2
0

s +1
s + 2
(a) Part-2: From the second set of given data (Z
L
= 1 , I
1
1 s, etc.), we have
I
2

0.5
s +1.5
, V
1

2
s +1

1.5
s +1.5

0.5(s + 3)
(s +1)(s +1.5)
For a terminated 2-port,
I
2

V
2
Z
L
V
2

y
21
y
22
+ Y
L
V
1

y
21
y
22
+1
V
1
Therefore
y
22
y
21
V
1
I
2
1
s +1
s + 2

0.5(s + 3)
(s +1)(s +1.5)

s +1.5
0.5
1
(s + 3)
(s + 2)
1
1
s + 2
Also, I
1
y
11
V
1
+ y
12
V
2
y
12

I
1
y
11
V
1
V
2

I
1
y
11
V
1
I
2
. Hence
y
12

s +1.5
0.5

1
s

2(s +1)
2
s(s + 2)
0.5(s + 3)
(s +1)(s +1.5)



_
,


(2s + 3)(s + 2)
s(s + 2)
+
2(s +1)(s + 3)
s(s + 2)
Prbs Chap 19, 1/13/02 P19-21 R. A. DeCarlo, P. M. Lin

1
(s + 2)
In conclusion
y
11
y
21
y
12
y
22



1
]
1

2(s +1)
2
s(s + 2)
1
(s + 2)
s +1
s + 2
1
(s + 2)






1
]
1
1
1
1

1
(s + 2)
2(s +1)
2
s
1
s +1 1





1
]
1
1
1
(b) This is a straightforward application of the conversion table 19.1.
SOLUTION PROBLEM 19.63. (a) Consider figure (a). Write two mesh equations:
V
1
16I
1
+
2
s
(I
1
+ I
2
) + 4I
1
20 +
2
s


_
,
I
1
+
2
s
I
2
V
2
sI
2
+
2
s
(I
1
+ I
2
) + 4I
1
4 +
2
s


_
,
I
1
+ s +
2
s


_
,
I
2
Therefore
Z
1
s
20s + 2
4s + 2
2
s
2
+ 2



1
]
1
Taking the inverse yields the y-parameters
Y
1
Z
z
22
z
12
z
21
z
11



1
]
1

1
20s
2
+ 2s + 32
s
2
+ 2 2
(4s + 2) 20s + 2




1
]
1
1
where
Z
(20s + 2)(s
2
+ 2) 2(4s + 2)
s

20s
3
+ 2s
2
+ 32s
s
20s
2
+ 2s + 32
Finally, the h-parameters are given as
H
1 y
11
z
12
z
22
z
21
z
22
1 z
22



1
]
1

1
s
2
+ 2
20s
2
+ 2s + 32 2
(4s + 2) s




1
]
1
1
(b) Now consider figure (b). z
11
is V
1
when I
2
= 0. But if I
2
= 0, then because of the ideal
transformer I
1
= 0, meaning that the ratio is not defined. Hence the z-parameters do not exist.
To find the h-parameters, observe that because of the ideal transformer, I
2
0.5I
1
,
I
1
2I
2
, and V
pri
0.5V
sec
. Writing a mesh equation at the right mesh first we obtain
V
2
RI
2
+ V
sec
+ (I
1
+ I
2
)R V
sec
Prbs Chap 19, 1/13/02 P19-22 R. A. DeCarlo, P. M. Lin
Now writing a mesh equation on the left we have
V
1
RI
1
+ V
pri
+ (I
1
+ I
2
)R 1.5RI
1
+V
pri
1.5RI
1
+ 0.5V
2
Therefore
H
1.5R 0.5
0.5 0



1
]
1
To obtain the y-parameters we use table 19.1:
Y
1
h
11
1 h
12
h
21
h



1
]
1

1
6R
4 2
2 1



1
]
1
Note: the det[Y] = 0 implying again that the z-parameters do not exist.
(c) For this network we consider it as a cascade (left to right) of an ideal transformer, the middle
network of a transformer and an inductor across the top, and finally another ideal transformer.
The t-parameters of these two ports are respectively:
T
1

a 0
0 1 a



1
]
1
, T
2

A B
C D



1
]
1
, T
3

1 b 0
0 b



1
]
1
To find T
2
, we replace the mutually coupled inductors by the pi-equivalent circuit of figure
18.25c where L
1
4 H, L
2
9 H, M k L
1
L
2
3 H, and 27. Thus L
left

L
2
M
4.5
H, L
right

L
1
M
27 H, and L
top

M
9 H. Notice that the external 9 H inductor is in
parallel with L
top
leading to L
par
= 4.5 H. The y-matrix of this new pi-network is by inspection:
Y
mid

1
s
2 4.5 1 4.5
1 4.5 1 27 +1 4.5



1
]
1

1
27s
12 6
6 7



1
]
1
To compute the t-parameters we have
T
2

y
22
y
21
1
y
21
y
y
21
y
11
y
21






1
]
1
1
1
1

7 6 4.5s
8 27s 2



1
]
1
Therefore
T T
1
T
2
T
3

a 0
0 1 a



1
]
1
7 6 4.5s
8 27s 2



1
]
1
1 b 0
0 b



1
]
1

7a 6b 4.5abs
8 27abs 2b a



1
]
1
From table 19.1, we obtain
Prbs Chap 19, 1/13/02 P19-23 R. A. DeCarlo, P. M. Lin
H
a
2
2.25s 0.5a / b
0.5a / b 4 27b
2




1
]
1
1
, Z s
63a
2
16
27ab
8
27ab
8
27b
2
4






1
]
1
1
1
1
, Y
1
27a
2
b
2
s
12b
2
6ab
6ab 7a
2




1
]
1
1
SOLUTION PROBLEM 19.64.
(a) The defining equation for the g-parameters is:
I
1
V
2



1
]
1

g
11
g
12
g
21
g
12



1
]
1
V
1
I
2



1
]
1
Because of the load impedance Z
L
, we have V
2
Z
L
I
2
. Hence substituting for V
2
in the
second equation yields
V
2
g
21
V
1
+ g
22
I
2
Z
L
I
2
I
2

g
21
V
1
g
22
+ Z
L
Substituting this equation into I
1
g
11
V
1
+ g
12
I
2
we obtain
I
1
g
11
V
1
g
12
g
21
V
1
g
22
+ Z
L



_
,

g
11

g
12
g
21
g
22
+ Z
L



_
,

V
1
Therefore,
Y
in
g
11

g
12
g
21
g
22
+ Z
L
(b) Because of the source impedance Z
s
, we have V
1
Z
s
I
1
or I
1
Y
s
V
1
. Hence substituting
for V
1
in the first equation yields
I
1
g
11
V
1
+ g
12
I
2
Y
s
V
1
V
1

g
12
I
2
g
11
+ Y
s
Substituting this equation into V
2
g
21
V
1
+ g
22
I
2
we obtain
V
2
g
21
g
12
I
2
g
11
+ Y
s



_
,

+ g
22
I
2
g
22

g
12
g
21
g
11
+ Y
s



_
,

I
2
Z
out
g
22

g
12
g
21
g
11
+ Y
s
(c)
G
1

V
1
V
s

Z
in
Z
s
+ Z
in

Y
s
Y
s
+Y
in

Y
s
Y
s
+ g
11

g
12
g
21
g
22
+ Z
L

Y
s
g
22
+ Z
L
( )
g
22
+ Z
L
( ) g
11
+Y
s
( ) g
12
g
21
(d) Refer to figure P19.64, where V
2

Z
L
g
22
+ Z
L
g
21
V
1
G
2

V
2
V
1

g
21
Z
L
g
22
+ Z
L
.
Prbs Chap 19, 1/13/02 P19-24 R. A. DeCarlo, P. M. Lin
(e) G
v
G
1
G
2

Y
s
g
22
+ Z
L
( )
g
22
+ Z
L
( ) g
11
+Y
s
( ) g
12
g
21

g
21
Z
L
g
22
+ Z
L

g
21
Y
s
Z
L
g
22
+ Z
L
( ) g
11
+ Y
s
( ) g
12
g
21
SOLUTION PROBLEM 19.65. Recall
I
1
V
2



1
]
1

g
11
g
12
g
21
g
22



1
]
1
V
1
I
2



1
]
1
For the given circuit
V
2

1
4
(I
2
4V
2
) + V
1
V
2

1
2
V
1
+
1
8
I
2
Also
V
2
V
1

1
4
I
1
sV
1
( )
This implies that
I
1
(4 + s)V
1
4V
2
(4 + s)V
1
4
1
2
V
1
+
1
8
I
2


_
,
(2 + s)V
1

1
2
I
2
Thus the g-parameter matrix is:
G
2 + s
1
2
1
2
1
8





1
]
1
1
1
SOLUTION PROBLEM 19.66. We first convert the y-parameters to t-parameters using table 19.1:
Y
N1
T
N1

y
22
y
21
1
y
21
y
y
21
y
11
y
21






1
]
1
1
1
1

0.1 0.5
3.82 25.1



1
]
1
where y 50.2 0.2 2 1.2 7.64 . Now we convert the h-parameters to t-parameters using
table 19.1:
H
N2
T
N 2

h
h
21
h
11
h
21
h
21
h
21
1
h
21






1
]
1
1
1
1

1
5
56 13
2 1



1
]
1

11.2 2.6
0.4 0.2



1
]
1
where h 13 2 + 6 5 56 . To obtain the cascaded t-parameters we compute
Prbs Chap 19, 1/13/02 P19-25 R. A. DeCarlo, P. M. Lin
T
cas
T
N1
T
N2

1
5
6.6 1.8
264.12 74.76



1
]
1

1.32 0.36
52.824 14.952



1
]
1
Thus,
Y
in

1
Z
in

t
21
Z
L
+ t
22
t
11
Z
L
+ t
12

603
5
15
5
40.2 S
To obtain the voltage gain, we first convert the t-parameters back to y-parameters (table
19.1) and then use the derived voltage gain formula:
T
cas

1.32 0.36
52.824 14.952



1
]
1
Y
cas

41.533 2
2.7778 3.6667



1
]
1
S
Hence,
G
v

V
L
V
1

y
21
y
22
+ Y
L

2.7778
3.6667 + 0.5
0.66667
Alternately, one could consider the load as a 2-port, compute its t-parameters, construct
the overall t-parameters as a cascade of three networks, and then use G
v

1
t
11
.
SOLUTION PROBLEM 19.67.
(a) Since only one C or L, weh have in general: H(s)
as + b
cs + d
. Since H() 0
a
c
, we
have that a = 0. Therefore,
H(s)
b
c
s +
d
c

K
s +
c
(b) To prove that
c

1
R
th
C
or
R
th
L
where R
th
is the Thevenin resistance seen by the energy
storage element, we refer the reader to problem 19.69 which provides a general derivation with
H() arbitrary; hence this problem is the special case of H() = 0.
(c) HereR
th
2000/ /(50 +1000/ /200)) 195.49 . Hence
c

1
R
th
C
25.58 10
6
rad/s
Prbs Chap 19, 1/13/02 P19-26 R. A. DeCarlo, P. M. Lin
SOLUTION PROBLEM 19.68.
(a) Because there is only one energy storage element, it follows that the most general form of
the transfer function is: H(s)
as + b
cs + d
. Since H(0) 0
b
d
, we have b = 0. Further, since the
transfer function is high pass, c 0, and
H(s)
as
cs + d

a
c


_
,
s
s +
d
c


_
,

Ks
s +
c
(b) To prove that
c

1
R
th
C
or
R
th
L
where R
th
is the Thevenin resistance seen by the energy
storage element, we refer the reader to problem 19.69 which provides a general derivation with
H(0) arbitrary; hence this problem is the special case of H(0) = 0.
(c) Let us apply the result of example 19.1 to that part of the circuit to the right of the 1 k
resistor and call the associated resistance Z
in
. Here Z
1
= , Z
2
= 2 k, Z
L
= 100 , and beta =
50. Hence
Z
in
beta +1 ( )Z
L
5.1 k
Thus
R
th
200 +1000 //(2000 + 5100) 1076.5
Hence,
c

1
R
th
C

1
1076.54 2 10
6
464.45 rad/s
SOLUTION PROBLEM 19.69. In this problem we assume (i) a single input single output system
and that linear circuit seen by the energy storage element has a Thevenin equivalent or a
Norton equivalent. For simplicity we will presume the existence of a Thevenin equivalent.
(a) The Thevenin equivalent seen by the dynamic element L or C consists of Z
th
(s) in series with
V
oc
(s). Since the remainder network seen by L or C is non-dynamic (resistive, resistive
with dependent sources and ideal op amps, etc), we have Z
th
(s) = R
th
and V
oc
(s) =
K
o
Input(s), R
th
and

K
o
being real constants.
Prbs Chap 19, 1/13/02 P19-27 R. A. DeCarlo, P. M. Lin
By voltage division,
V
L

Ls
Ls + R
th
V
oc

K
0
s
s +
R
th
L
Input(s) (1)
and
V
C

1
Cs
1
Cs
+ R
th
V
oc

K
0
R
th
C
s +
1
R
th
C
Input(s) (2)
After V
L
(s) or V
C
(s) has been determined, we can find the Laplace transform of any other output
(voltage or current) using the voltage source substitution theorem (chapter 6) and linearity
(chapter 5):
Ouput(s) K
1
Input(s) + K
2
V
L
(s) or V
C
(s) ( ) (3)
For the case of V
L
(s),
Ouput(s) K
1
+ K
2
K
0
s
s +
R
th
L





_
,



Input(s) (4)
and for the case of V
C
(s),
Ouput(s) K
1
+ K
2
K
0
R
th
C
s +
1
R
th
C





_
,



Input(s) (5)
For either case, the transfer function H(s) has the form
H(s)
Ouput(s)
Input(s)

K
3
s + K
4
s +
c
(6)
Prbs Chap 19, 1/13/02 P19-28 R. A. DeCarlo, P. M. Lin
where
c

R
th
L
or
1
R
th
C
with K
3
and K
4
real constants.
It remains to give K
3
and K
4
some physical interpretations. In (6), let s = , we have
H()
K
3
s + K
4
s +
c
1
]
1
s
K
3
On the other hand, letting s = 0 in (6) produces
H(0)
K
3
s + K
4
s +
c
1
]
1
s0

K
4

c
Substituting K
3
and K
4
into (6), we obtain the desired result
H(s)
Ouput(s)
Input(s)

H()s +
c
H(0)
s +
c
(7)
(b) When s = , the impedance of C is
Z
C
()
1
Cs
1
]
1
s
0
and the impedance of L is
Z
L
() Ls]
s

Therefore in calculating H(), we may replace C by a short circuit and L by an open circuit. On
the other hand, when s = 0, the impedance of C is
Z
C
(0)
1
Cs
1
]
1
s0

and the iimpedance of L is
Z
L
(0) Ls]
s0
0
Therefore in calculating H(0), we may replace C by an open circuit and L by a short circuit.
Prbs Chap 19, 1/13/02 P19-29 R. A. DeCarlo, P. M. Lin
(c) For figure P19.69a, by inspection
R
th
= 3//(2 + 4) = 3//6 = 2

c = 1/(Rth C) = 1/(2 0.5) = 1


H() = 4/(2 + 4) = 2/3
H(0) = 4/(2 + 3 + 4) = 4/9
Therefore
H(s)
H()s +
c
H(0)
s +
c

2
3
s +
4
9
s + 1

2
3
s +
2
3


_
,
s +1
For figure P19.69b
R
th
= 3//(2 + 4) = 3//6 = 2

c
= R
th
/ L = 2/2 =1
H() = 4/(2 + 4) = 2/3
H(0) = 4/(2 + 3 + 4) = 4/9
Therefore
H(s)
H()s +
c
H(0)
s +
c

4
9
s +
2
3
s + 1

4
9
s + 1.5 ( )
s +1
SOLUTION PROBLEM 19.70. According to problem 19.68, the transfer function of the circuit is
H(s)
Ks
s +
c
where
c

1
R
th
C
and R
th
is the Thevenin resistance seen by the storage element C. To find Rth
we make use of figure 19.4 and the associated formula. The details are in the MATLAB code
below:
R1 = 200*1e3/1200
R1 =
1.6667e+02
Z1 = R1+2e3
Z1 =
2.1667e+03
Z3 = 100;
Prbs Chap 19, 1/13/02 P19-30 R. A. DeCarlo, P. M. Lin
beta = -50;
Zout = Z1/(1+beta)
Zout =
-4.4218e+01
Rth = 2000 + Zout
Rth =
1.9558e+03
wc = 1/(Rth*2e-6)
wc =
2.5565e+02
Hence
c

1
1955.8 2 10
6
255.65 rad/s
SOLUTION PROBLEM 19.71. (a) Except for the terminating resistor, let the other element
branches of the circuit be given by

1
3s
1
, Y
2

1
2s +
1
2s

2s
4s
2
+1, and Z
3

1
s
Consider the circuit
Here from problem 19.53a and from 19.52 a we obtain
Prbs Chap 19, 1/13/02 P19-31 R. A. DeCarlo, P. M. Lin
V
1
I
1



1
]
1

1+
2
3
4s
2
+1
1
3s
2s
4s
2
+1
1







1
]
1
1
1
1
1
V
3
I
3



1
]
1
and
V
3
I
3



1
]
1

1
1
s
0 1




1
]
1
1
V
2
I
2



1
]
1
which implies that
V
1
I
1



1
]
1

1
4s
2
+1
4s
2
+ 5/ 3
16s
2
+ 6
( )
3s
2s 4s
2
+ 3





1
]
1
1
1
V
2
I
2



1
]
1
(b)
G
V
(s)
y
21
y
22
+ y
L
(c)
y
21

1
t
12

3s(4s
2
+1)
16s
2
+ 6
, y
22

t
11
t
12

3s 4s
2
+
5
3


_
,
16s
2
+ 6

s(12s
2
+ 5)
16s
2
+ 6
, and Y
L
1
(d) Hence,
G
V
(s)
y
21
y
22
+ y
L

3s(4s
2
+1)
16s
2
+ 6
s(12s
2
+ s)
16s
2
+ 6
+1

3s(4s
2
+1)
12s
3
+16s
2
+ 5s + 6
SOLUTION PROBLEM 19.72. For part (a), treating each capacitor as a short circuit yields the
equivalent circuit below.
Prbs Chap 19, 1/13/02 P19-32 R. A. DeCarlo, P. M. Lin
h11T = 4.2e3; h12T = 0; h21T = 150; h22T = 0.1e-3;
hT = [h11T, h12T;h21T h22T];
yT = htoy(hT)
yT =
2.3810e-04 0
3.5714e-02 1.0000e-04
By inspection, the y11 parameter of the overall two-port consists of the sum of y11T plus the
conductances of the two front end resistors. Also, the y22 parameter of the overall two port
is y22T plus the conductance of the 4.7k resistor. Hence,
y = yT + [1/1e4+1/1e5 0;0 1/4700]
y =
3.4810e-04 0
3.5714e-02 3.1277e-04
Hence, the overall h-parameters are:
h = ytoh(y)
h =
2.8728e+03 0
1.0260e+02 3.1277e-04
(b)
yout = h(2,2) - (h(1,2)*h(2,1)/(h(1,1)+50))
yout =
Prbs Chap 19, 1/13/02 P19-33 R. A. DeCarlo, P. M. Lin
3.1277e-04
Zout = 1/yout
Zout =
3.1973e+03
a = sqrt(8/Zout)
a =
5.0021e-02
(c) To compute the gain we first need Z
in
. From equation 19.49, since h
12
= 0, Z
in
= h
11
.
Zin = h(1,1)
Zin =
2.8728e+03
From equation 19.51,
Gv2 = -h(2,1)/(Zin*(h(2,2)+h(2,2)))
Gv2 =
-5.7094e+01
From equation 19.52,
Gv1 = Zin/(Zin + 50)
Gv1 =
9.8289e-01
To compute V
L
/V
s
we use:
Gv = Gv1*Gv2*a
Gv =
-2.8071e+00
To compute V
L
/V
1
we use:
Gvv = Gv2*a
Gvv =
Prbs Chap 19, 1/13/02 P19-34 R. A. DeCarlo, P. M. Lin
-2.8559e+00
(d) For these calculations, we assume Vs is normalized to 1 V. Since we are computing gains,
we may do this without loss of generality.
Vs = 1;
Is = 1/(50 + Zin)
Is =
3.4214e-04
Now we compute the normalized power delivered by the voltage source.
Psnorm = Vs*Is
Psnorm =
3.4214e-04
Next we compute the normalized power absorbed by the load.
VL = Gv*1
VL =
-2.8071e+00
PLnorm = VL^2/8
PLnorm =
9.8496e-01
Next, the power gain from source to load is:
GpLs = PLnorm/Psnorm
GpLs =
2.8788e+03
Further, we compute the power gain from input to the two port to the load as follows:
V1 = Vs*Zin/(Zin + 50)
V1 =
Prbs Chap 19, 1/13/02 P19-35 R. A. DeCarlo, P. M. Lin
9.8289e-01
P1 = V1*Is
P1 =
3.3629e-04
GpL1 = PLnorm/P1
GpL1 =
2.9289e+03
(e) SPICE Simulation Because the frequency response is flat for freqency above 800 Hz, we
only plotted up to 1.6k Hz. The circuit diagram reflects the load back to the primary of the
ideal transformer. In general, this is not possible. Hence for a SPICE simulation, it is
necessary to use one of the models given in Figure 18.15 consisting of two controlled
sources. For this example, this is not necessary. Note however that the actual output
voltage is 0.05 times the values on the graph given below. This simulation assumes a 1 V
source voltage and the parameter of GVCCS is 0.035714. Notice that in this problem
Prbs Chap 19, 1/13/02 P19-36 R. A. DeCarlo, P. M. Lin
MAG(V(IVM2))
Frequency (Hz) Prb 19.72-Small Signal AC-3
+0.000e+000
+10.000
+20.000
+30.000
+40.000
+50.000
+60.000
+200.000 +400.000 +600.000 +800.000 +1.000k +1.200k +1.400k +1.600k
(f) For this part, we change 100 F to 10 F. The resulting plot shows degradation of the low
end frequency response.
MAG(V(IVM2))
Frequency (Hz) Prb 19.72-Small Signal AC-4
+0.000e+000
+10.000
+20.000
+30.000
+40.000
+50.000
+60.000
+200.000 +400.000 +600.000 +800.000 +1.000k +1.200k +1.400k +1.600k
SOLUTIONS PROBLEMS CHAPTER 20
USEFUL MATLAB M-FILES FOR USE IN THE SOLUTION TO PROBLEMS IN THIS CHAPTER.
Program 1: converts y-parameters to t-parameters
% convert y parameters to t parameters
function [t,t11,t12,t21,t22] = ytot(y)
y11=y(1,1);
y12=y(1,2);
y21=y(2,1);
y22=y(2,2);
deltay= y11*y22-y12*y21;
t11=-y22/y21;
t12 = -1/y21;
t21= -deltay/y21;
t22= -y11/y21;
t= [ t11 t12; t21 t22];
Program 2: Computes Zin, Zout, and gains using two port h-parameters.
% two-port analysis in terms of h-parameters
function [zin, zout] =twoport(h, zL, zs)
['twoport analysis using h-parameters']
h11= h(1,1); h12=h(1,2); h21=h(2,1); h22=h(2,2);
zin = h11 - h12*h21/(h22+ 1/zL)
yout= h22 - h12*h21/(h11+zs);
zout= 1/yout
v1tovs= zin/(zin+zs)
v2tov1= -h21/(zin*(h22+1/zL))
v2tovs= v1tovs*v2tov1
Program 3: Computes Zin, Zout, and gains using two port y-parameters.
% two-port analysis in terms of y-parameters
function [zin, zout] =twoporty(y, zL, zs)
['twoport analysis using y-parameters']
y11= y(1,1); y12=y(1,2); y21=y(2,1); y22=y(2,2);
yin = y11 - y12*y21/(y22+ 1/zL)
zin= 1/yin
yout= y22 - y12*y21/(y11+1/zs)
zout= 1/yout
v1tovs= zin/(zin+zs)
v2tov1= -y21/(y22+1/zL)
v2tovs= v1tovs*v2tov1
Program 4: Computes Zin, Zout, and gains using two port t-parameters.
% two-port analysis in terms of t-parameters
function [zin, zout] =twoportt(t, zL, zs)
['analysis of terminated twoport using t-parameters']
t11= t(1,1); t12=t(1,2); t21=t(2,1); t22=t(2,2);
zin= (t11*zL + t12)/(t21*zL + t22)
zout= (t22*zs + t12)/(t21*zs + t11)
v2tov1= zL/(t11*zL + t12)
v1tovs= zin/(zin+zs)
v2tovs= v2tov1*v1tovs
Program 5: converts z-parameters to t-parameters
%converting z to t paramters (same sormulas as
%converting t to z parameters)
function [t,t11,t12,t21,t22] = ztot(z)
z11=z(1,1); z12=z(1,2); z21=z(2,1); z22=z(2,2);
deltaz= z11*z22 - z12*z21;
t11= z11/z21;
t12= deltaz/z21;
t21= 1/z21;
t22= z22/z21;
t= [ t11 t12; t21 t22];
SOLUTION 20.1.
For network a, the z-parameters are by inspection
Z
a

R
1
+ R
3
R
3
R
3
R
2
+ R
3



1
]
1
Similarly, for network b, the z-parameters are the same:
Z
b

R
1
+ R
3
R
3
R
3
R
2
+ R
3



1
]
1
The interconnection of networks a and b conforms to figure 20.2b, which is a series
interconnection. Hence, the new overall z-parameters are
Z
new
Z
a
+ Z
b
2
R
1
+ R
3
R
3
R
3
R
2
+ R
3



1
]
1
SOLUTION 20.2. For networks a and b, the y-parameters are by inspection
Y
a
Y
b

3 2
2 4



1
]
1
S
Hence, their z-parameters are the inverse of the y-parameter matrix:
Z
a
Z
b

1
8
4 2
2 3



1
]
1

The interconnection of networks a and b conforms to figure 20.2b, which is a series
interconnection. Hence, the new overall z-parameters are
Z
new
Z
a
+ Z
b

1
4
4 2
2 3



1
]
1

SOLUTION 20.3. For network a consisting of the single inductor,
Z
a

s s
s s



1
]
1

For network b, we have
Z
b
Y
b
[ ]
1

1
2
1 1
2 4



1
]
1

0.5 0.5
1 2



1
]
1

The interconnection of networks a and b conforms to figure 20.2b, which is a series
interconnection. Hence, the new overall z-parameters are
Z
new
Z
a
+ Z
b

s + 0.5 s + 0.5
s +1 s + 2



1
]
1

SOLUTION 20.4. For network a consisting of the single capacitor,
Z
a

1
s
1 1
1 1



1
]
1

The interconnection of networks a and b conforms to figure 20.2b, which is a series
interconnection. Hence, the new overall z-parameters are
Z
new
Z
a
+ Z
b

1
s
1 + 0.5s 1+ 0.5s
1+ s 1 + 2s



1
]
1

SOLUTION 20.5. For network a, the y-parameters are by inspection
Y
a

3 2
2 3



1
]
1
S
Hence, their z-parameters are the inverse of the y-parameter matrix:
Z
a

1
5
3 2
2 3



1
]
1

0.6 0.4
0.4 0.6



1
]
1

The interconnection of networks a and b conforms to figure 20.2b, which is a series
interconnection. Hence, the new overall z-parameters are
Z
new
Z
a
+ Z
b

1.6 0.9
0.4 0.4



1
]
1

SOLUTION 20.6. For networks Na and Nb, the y-parameters are:
Y
Na
Y
Nb

0.7 0.2
0.2 0.7



1
]
1
S
Hence, their z-parameters are the inverse of the y-parameter matrix:
Z
Na
Z
Nb

20
9
0.7 0.2
0.2 0.7



1
]
1

1
9
14 4
4 14



1
]
1

The network Na* has the same z-parameters as Na and continues to act as a two when series
interconnected to another 2-port because of the transformer. Hence, the interconnection of
networks Na* and Nb forms a valid series interconnection in which cas
Z
new
Z
Na*
+ Z
Nb

2
9
14 4
4 14



1
]
1

SOLUTION 20.7. For network Nb, the y-parameters are the same as in problem 20.6, i.e.,
Y
Nb

0.7 0.2
0.2 0.7



1
]
1
S and Z
Nb

1
9
14 4
4 14



1
]
1

For network Na, consider the purely resistive part without the transformer. The y-parameters of
this part are half the y-parameters of Nb, i.e.,
Y
R

0.35 0.1
0.1 0.35



1
]
1
S
In MATLAB we use the m-file which converts y-parameters to t-parameters:
y = [0.35 -0.1;-0.1 0.35];
t = ytot(y)
t =
3.5000e+00 1.0000e+01
1.1250e+00 3.5000e+00
From figure 19.34, the transformer has to parameters with a = 2 given by
t
trans

1 a 0
0 a



1
]
1

0.5 0
0 2



1
]
1
ttrans = [0.5 0;0 2];
tNa = ttrans*t
tNa =
1.7500e+00 5.0000e+00
2.2500e+00 7.0000e+00

zNa = ttoz(tNa)
zNa =
7.7778e-01 4.4444e-01
4.4444e-01 3.1111e+00

zNb = [14 4;4 14]/9;


znew = zNa + zNb
znew =
2.3333e+00 8.8889e-01
8.8889e-01 4.6667e+00

SOLUTION 20.8. This problem is identical numerically to problem 20.6. Here however the
isolation transformer is on the right hand side which makes no difference to the interconnection.
Therefore,
Z
new
Z
Na*
+ Z
Nb

2
9
14 4
4 14



1
]
1

SOLUTION 20.9. Figure 20.4 is
Using the values in figure 20.3, we obtain the following mesh equation matrix
V
1
V
2
0





1
]
1
1
1

4 + R
1
2 R
1
2 4 + R
2
R
2
R
1
R
2
R
1
+ R
2





1
]
1
1
1
I
1
I
2
I
3





1
]
1
1
1
For I3 to be zero, the third equation implies that R
1
I
1
= R
2
I
2
. Therefore,
V
1
4 + R
1
( )I
1
+ 2
R
1
R
2
I
1
4 + R
1
+
2R
1
R
2



_
,

I
1
Similarly
V
2
2I
1
+ 4 + R
2
( )
R
1
R
2
I
1
+ 2 +
(4 + R
2
)R
1
R
2



_
,

I
1
Therefore
V
1
V
2

4 + R
1
+
2R
1
R
2



_
,

2 +
( 4 + R
2
)R
1
R
2



_
,

4R
2
+ R
1
R
2
+ 2R
1
( )
2R
2
+ R
1
R
2
+ 4R
1
( )

42
48

7
8
as was to be shown.
Now suppose,
V
1
V
2

7
8
or equivalently V
1

7
8
V
2
. With specific values
V
1
V
2
0





1
]
1
1
1
V
2
7 8
1
0





1
]
1
1
1

10 2 6
2 7 3
6 3 9





1
]
1
1
1
I
1
I
2
I
3





1
]
1
1
1
In MATLAB
Z = [10 2 -6;2 7 3;-6 3 9];
b = [7/8 1 0]';
I = inv(Z)*b
I =
6.2500e-02
1.2500e-01
2.7756e-17
Thus
I
1
I
2
I
3





1
]
1
1
1
V
2
0.0625
0.124
0





1
]
1
1
1
SOLUTION 20.10
(a) Write two loop equations assuming I
1
and I
2
are the usual port currents. Here
V
1
Z
1
I
1
+ V
13
+ Z
3
I
1
+ I
2
( )
and
V
2
Z
2
I
2
+V
23
+ Z
3
I
1
+ I
2
( )
which in matrix form are
V
1
V
2



1
]
1

Z
1
+ Z
3
Z
3
Z
3
Z
2
+ Z
3



1
]
1
I
1
I
2



1
]
1
+
V
13
V
23



1
]
1
However,
V
13
V
23



1
]
1
Z
A
I
1
I
2



1
]
1
Therefore
V
1
V
2



1
]
1

Z
1
+ Z
3
Z
3
Z
3
Z
2
+ Z
3



1
]
1
I
1
I
2



1
]
1
+ Z
A
I
1
I
2



1
]
1

Z
1
+ Z
3
Z
3
Z
3
Z
2
+ Z
3



1
]
1
+ Z
A



_
,

I
1
I
2



1
]
1
(b) The procedure of part (a) is repeated to produce the same result.
REMARK: this problem states that the two networks are equivalent two ports. Thus the
configurations can be used interchangeably.
SOLUTION 20.11. Because of the isolation transformers, the overall Z-parameters are the sum of
the three component Z-parameters. For N
a
,
Z
a

2 1
1 2



1
]
1

For N
b
,
Z
b

8 1
1 5



1
]
1

For N
c
,
Z
c
Z
a

2 1
1 2



1
]
1

Therefore
Z Z
a
+ Z
b
+ Z
c

12 3
3 9



1
]
1

SOLUTION 20.12. (a) Given the Z-parameters of N, then
Y
N
Z
N
1

7 2
10 3



1
]
1
1

3 2
10 7



1
]
1
S
The 1-F capacitor considered as a two port has y-parameters
Y
C

s s
s s



1
]
1
S
Therefore, the parallel connection of the two ports has y-parameters
Y Y
N
+Y
C

s s
s s



1
]
1
+
3 2
10 7



1
]
1

s + 3 (s + 2)
(s +10) s + 7



1
]
1
S
(b) For this part, the same reasoning applies with s replaced by 1/s.
SOLUTION 20.13. The isolation transformer allows for the valid parallel connection of N
a
*
and
N
b
in the sense that the overall y-parameters are the sum of the individual y-parameters. Further,
because the ideal transformer is 1:1 with the standard dot locations, the y-parameters of N
a
*
are
those of N
a
. Further, the y-parameters of N
b
are the same as those of N
a
as the circuits are
simple vertical flips of each other. Therefore
Y
a
*
Y
a

0.7 0.2
0.2 0.7



1
]
1
Y
b
Hence, the overall y-parameters are:
Y 2
0.7 0.2
0.2 0.7



1
]
1

1.4 0.4
0.4 1.4



1
]
1
S
True-False: FALSE because the connection does not conform to figure 20.2a.
SOLUTION 20.14. From the previous example, the y-parameters of N
b
are
Y
b

0.7 0.2
0.2 0.7



1
]
1
S
The resistance values of the resistive part of N
a
are twice those of N
b
. Hence, the y-parameters
of the resistive part are half those of N
b
, i.e.,
I
1
'
I
2




1
]
1
1
Y
a,R
V
1
'
V
2




1
]
1
1

0.35 0.1
0.1 0.35



1
]
1
V
1
'
V
2




1
]
1
1
S
We obtain the y-parameters of N
a
by considering the effect of the transformer on these y-
parameters. Observe that
I
1
2I
1
'
and V
1
0.5V
1
'
Hence
I
1
I
2



1
]
1

0.7 0.2
0.1 0.35



1
]
1
V
1
'
V
2




1
]
1
1

0.7 0.2
0.1 0.35



1
]
1
2V
1
V
2



1
]
1

1.4 0.2
0.2 0.35



1
]
1
V
1
V
2



1
]
1
Thus
Y
a

1.4 0.2
0.2 0.35



1
]
1
S
It follows that
Y Y
a
+ Y
b

2.1 0.4
0.4 1.05



1
]
1
S
SOLUTION 20.15. (a)
MAG(V(IVM))
Frequency (Hz) Prob 20.15-Small Signal AC-10
+0.000e+000
+200.000m
+400.000m
+600.000m
+800.000m
+1.000
+100.000m +200.000m +300.000m +400.000m +500.000m
(b)
C
31.8n
R
1k
R0
1k
C0
15.9n
C1
15.9n
R1
500
IVm
V
0
MAG(V(IVM))
Frequency (Hz) Prob 20.15-Small Signal AC-11
+0.000e+000
+200.000m
+400.000m
+600.000m
+800.000m
+1.000
+10.000k +20.000k +30.000k
SOLUTION 20.16. (a) For network N
a
, the y-parameters by inspection are:
Y
a

G
1
+ j C
1
0
g
m
G
0



1
]
1

2 + j10.21 0
95 0.07143



1
]
1
mS
For network N
b
, the y-parameters by inspection are:
Y
b
G
f
+ j C
2
( )
1 1
1 1



1
]
1
0.8333 + j1.021 ( )
1 1
1 1



1
]
1
mS
Therefore
Y Y
a
+ Y
b

2.8333 + j11.23 0.8333 j1.021


94.167 j1.021 0.9048 + j1.021



1
]
1
mS
(b), (c), and (d). Here we use the MATLAB m-file for two port analysis in terms of y-
parameters:
zs = 50; zL = 50;
twoporty(Y, zL, zs)
ans =
twoport analysis using y-parameters
yin =
6.8501e-03 + 1.5594e-02i
zin =
2.3614e+01 - 5.3756e+01i
yout =
5.3617e-03 + 3.0023e-03i
zout =
1.4199e+02 - 7.9506e+01i
v1tovs =
5.5701e-01 - 3.2349e-01i
v2tov1 =
-4.4915e+00 + 2.6821e-01i
v2tovs =
-2.4150e+00 + 1.6023e+00i

SOLUTION 20.17. The t-parameters of the LR circuit follow from problem 19.53 with Z
1
= Ls =
s and Z
2
= 0.5 :
T
LR

1+ 2s s
2 1



1
]
1
Therefore
T
cascade
T * T
LR

5 + 2s 2 + s
5 + 2s 2 + s



1
]
1
SOLUTION 20.18. This problem can be solved in many ways. Here we emphasize the cascade
nature of the two ports.
% The y-parameters of Nb are:
Yb = [8 2;20 6];
% The z-parameters of Na are:
Za = [0.75 -0.25;-2.5 1];
% The t-parameters of Na are:
Ta = ztot(Za)
Ta =
-3.0000e-01 -5.0000e-02
-4.0000e-01 -4.0000e-01
% The t-parameters of Nb are:
Tb = ytot(Yb)
Tb =
-3.0000e-01 -5.0000e-02
-4.0000e-01 -4.0000e-01
% The t-parameters of the cascaded two port are:

Tab = Ta*Tb
Tab =
1.1000e-01 3.5000e-02
2.8000e-01 1.8000e-01
% Doing a t-parameter analysis we obtain:
twoportt(Tab,0.25,0.5)
ans =
analysis of terminated twoport using t-parameters
zin =
2.5000e-01
zout =
5.0000e-01
v2tov1 =
4
v1tovs =
3.3333e-01
v2tovs =
1.3333e+00
ans =
2.5000e-01

SOLUTION 20.19. Use the h-parameter analysis m-file.


H1 = [1e3 0.001;50 6e-5]; H2 = [1e3 0.99;-5 8e-4];
zL = 100; zs = 2e3; zm = 10e3;
% For part (a), the common collector stage:
twoporth(H2,zL,0)
ans =
twoport analysis using h-parameters
zin =
1.4583e+03
zout =
1.7391e+02
v1tovs =
1
v2tov1 =
3.1746e-01
v2tovs =
3.1746e-01
Remark: in the above, v2 is Vout and v1 is vm; zout has no signifigance.
% We now compute the load on the first stage.
zin2 = 1.4583e+03;
zLm = zm*zin2/(zin2 + zm)
zLm =
1.2727e+03
% zLm is the load impedance on stage 1.
% For part (a), the common emitter stage:
twoporth(H1,zLm,zs)
ans =
twoport analysis using h-parameters
zin =
9.4088e+02
zout =
2.3077e+04
v1tovs =
3.1993e-01
v2tov1 =
-6.2835e+01
v2tovs =
-2.0103e+01
Remark: in the above, v2 is vm and v1 is Vin; zout is the output impedance of stage 1.
Conclusion: the input impedance to stage 1 is 940.88 and the input impedance to stage 2 is
1.4583 k.
(b) From the above output and remark, Vm/Vin = -62.835. Further Vout/Vm = 0.31746.
(c) From above Vm/Vs = -20.103. Therefore Vout/Vs = Vout/Vm * Vm/Vs = -6.38.
(d)
zout1 = 2.3077e+04;
zs2 = zm*zout1/(zm + zout1)
zs2 =
6.9768e+03
twoporth(H2,zL,zs2)
ans =
twoport analysis using h-parameters
zin =
1.4583e+03
zout =
7.0395e+02
v1tovs =
1.7289e-01
v2tov1 =
3.1746e-01
v2tovs =
5.4885e-02
Conclusion: Zout of amplifier is 704 .
SOLUTION 20.20
(a) Using MATLAB
na = 1.1514;
nb = 3.4012;
Zlprime = nb^2*75 + j*1042.94
ZLprime =
8.6761e+02 + 1.0429e+03i
Zsprime = 75/na^2 + j*30
Zsprime =
5.6573e+01 + 3.0000e+01i
(b) Since the h-parameters of the transistors are given, we can again use MATLAB and the m-
file twoporth defined earlier. Hence:
h = [ 60-j*50 0.01; -j*2 0.0005+j*0.0004];
[Zin, Zout] = twoporth(h,ZLprime, Zsprime)
Zin =
5.6569e+01 - 3.0000e+01i
Zout =
8.6763e+02 - 1.0429e+03i
(c) Observe that Zin = 5.6569e+01 - 3.0000e+01i and Zsprime = 5.6573e+01 + 3.0000e+01i,
which are clearly conjugates of each other. Further, Zout = 8.6763e+02 - 1.0429e+03i
and ZLprime = 8.6761e+02 + 1.0429e+03i, which are also conjugates of each other.
Hence maximum power is transferred into and out of the transistor.
(d) For this part, we change all cascaded two ports to t-parameters. Specifically,
t0 = [1 75;0 1];
t1 = [na 0; 0 1/na];
t2 = [ 1 j*30; 0 1];
t3 = htot(h);
t4 = [ 1 j*1042.9; 0 1];
t5 = [nb 0; 0 1/nb];
t6 = [1 0;1/75 1];
t = t0*t1*t2*t3*t4*t5*t6
t =
2.2176e-01 - 2.5959e-01i 8.3160e+00 - 9.7351e+00i
1.4785e-03 - 1.7307e-03i 6.6577e-02 - 7.4415e-02i
gain = 1/t(1,1)
gain = 1.9024e+00 + 2.2270e+00i
gainmag = abs(gain)
gainmag = 2.9289e+00
gainangle = angle(gain)*180/pi
gainangle = 4.9494e+01
In this case, Vout/Vs = gain = 1.9024e+00 + 2.2270e+00i = 2.928949.494
o
SOLUTION 20.21.
(a)
Y2N = [1 0;20.1 0]*1e-3;
Y10k = [1 -1;-1 1]*1e-4;
Yshade = Y2N + Y10k
Yshade =
1.1000e-03 -1.0000e-04
2.0000e-02 1.0000e-04

(b) This is a series connection of two 2-ports. Hence we first convert the answer of part (a) to z-
parameters.
Y2N = [1 0;20.1 0]*1e-3;
Y10k = [1 -1;-1 1]*1e-4;
Yshade = Y2N + Y10k
Yshade =
1.1000e-03 -1.0000e-04
2.0000e-02 1.0000e-04
Zshade = inv(Yshade)
Zshade =
4.7393e+01 4.7393e+01
-9.4787e+03 5.2133e+02
Z1k = [1 1;1 1]*1e3;
Zdashed = Zshade + Z1k
Zdashed =
1.0474e+03 1.0474e+03
-8.4787e+03 1.5213e+03

(c)
twoportz(Zdashed,1e12,1e3)
ans =
twoport analysis using z-parameters
zin =
1.0474e+03
zout =
5.8588e+03
v1tovs =
5.1157e-01
v2tov1 =
-8.0950e+00
v2tovs =
-4.1412e+00
Conclusion: Vout/Vs = -4.1412.
SOLUTION 20.22.
Z = [3 1;5 2]*1e3;
Y = inv(Z)
Y =
2.0000e-03 -1.0000e-03
-5.0000e-03 3.0000e-03
% Consider the parallel connection of Y with the 1 k resistor
Y1 = Y + [1 -1;-1 1]*1e-3
Y1 =
3.0000e-03 -2.0000e-03
-6.0000e-03 4.0000e-03

% Now consider Y/Z in parallel with 1 k connected between B and C


Y2 = Y + [0 0;0 1]*1e-3
Y2 =
2.0000e-03 -1.0000e-03
-5.0000e-03 4.0000e-03

% This combination is in series with another 1 k resistor


% Hence we need to compute Z2 first
Z2 = inv(Y2)
Z2 =
1.3333e+03 3.3333e+02
1.6667e+03 6.6667e+02
% Now we compute the series combo of Z2 and the 1 k resistor
Z3 = Z2 + [1 1;1 1]*1e3
Z3 =
2.3333e+03 1.3333e+03
2.6667e+03 1.6667e+03

% We now convert Y1 and Z3 to t-parameters and then multiply


% together to obtain the overall t-parameters

T1 = ytot(Y1)
T1 =
6.6667e-01 1.6667e+02
0 5.0000e-01
T3 = ztot(Z3)
T3 =
8.7500e-01 1.2500e+02
3.7500e-04 6.2500e-01
T = T1*T3
T =
6.4583e-01 1.8750e+02
1.8750e-04 3.1250e-01
Y = ttoy(T)
Y =
1.6667e-03 -8.8889e-04
-5.3333e-03 3.4444e-03

SOLUTION 20.23. Refer to figure 20.13 for all problems.


(a) By inspection,
I
1
I
2
I
3





1
]
1
1
1

6 4 2
4 4 + 5s 5s
2 5s 2 + 5s





1
]
1
1
1
V
1
V
2
V
3





1
]
1
1
1
The 3x3 coefficient matrix is the desired Y
ind
(s).
(b) Writing the nodal equation by inspection yields
I
1
I
2
I
3
0







1
]
1
1
1
1
1

0.5 0 0.25 0.25


0 1 0.5 0.5
0.25 0.5 1.25 0.5
0.25 0.5 0.5 1.25







1
]
1
1
1
1
1
V
1
V
2
V
3
V
4







1
]
1
1
1
1
1
Using MATLAB,
W11=[0.5, 0 -0.25;0 1 -0.5;-0.25 -0.5 1.25]
W11 =
5.0000e-01 0 -2.5000e-01
0 1.0000e+00 -5.0000e-01
-2.5000e-01 -5.0000e-01 1.2500e+00
W12 = [-0.25 -0.5 -0.5]'
W12 =
-2.5000e-01
-5.0000e-01
-5.0000e-01
W21 = [-0.25 -0.5 -0.5]
W21 =
-2.5000e-01 -5.0000e-01 -5.0000e-01
W22 = 1.25
W22 =
1.2500e+00
Yind = W11 - W12*inv(W22)*W21
Yind =
4.5000e-01 -1.0000e-01 -3.5000e-01
-1.0000e-01 8.0000e-01 -7.0000e-01
-3.5000e-01 -7.0000e-01 1.0500e+00

(c) Again, writing the nodal equation by inspection yields


I
1
I
2
I
3
0







1
]
1
1
1
1
1

5 0 0 5
0 6 7 1
0 4 8 4
5 2 1 8







1
]
1
1
1
1
1
V
1
V
2
V
3
V
4







1
]
1
1
1
1
1
Again, using MATLAB,
W11 = [5 0 0;0 6 -7;0 -4 8]
W11 =
5 0 0
0 6 -7
0 -4 8
W12 = [-5 1 -4]'
W12 =
-5
1
-4
W21 = [-5 -2 -1]
W21 =
-5 -2 -1
W22 = 8;
Yind = W11 - W12*inv(W22)*W21
Yind =
1.8750e+00 -1.2500e+00 -6.2500e-01
6.2500e-01 6.2500e+00 -6.8750e+00
-2.5000e+00 -5.0000e+00 7.5000e+00

(d) This part is similar to part (a) as it does not require the method of matrix partitioning. By
inspection,
I
G
I
D
I
S





1
]
1
1
1

(C
GD
+ C
GS
)s C
GD
s C
GS
s
C
GD
s + g
m
C
GD
s g
m
C
GS
s g
m
0 C
GS
s + g
m





1
]
1
1
1
V
G
V
D
V
S





1
]
1
1
1
The 3x3 coefficient matrix is the desired Y
ind
(s).
SOLUTION 20.24. (a) With regard to the given information, the associated indefinite admittance
matrix is the coefficient matrix in the following nodal equation given reference to figure 20.13:
I
G
I
D
I
S





1
]
1
1
1

0 0 0
g
m
0 g
m
g
m
0 g
m





1
]
1
1
1
V
G
V
D
V
S





1
]
1
1
1
We use property 5 to compute the remaining answers:
(a)-(a): Y
GD
is as given.
(a)-(b): Y
SD

g
m
0
g
m
0



1
]
1
S
(a)-(c): Y
GS

0 0
g
m
g
m



1
]
1
S
(a)-(d): Y
DG

0 g
m
0 0



1
]
1
S
(a)-(e): Y
DS

0 g
m
0 g
m



1
]
1
S
(a)-(e): Y
SG

g
m
g
m
0 0



1
]
1
S
(b) Transmission from port-1 to port-2 occurs when the 2-1 entry of the 2-port y-matrix is
nonzero. Hence, the following all have the desired transmission: Y
GD
, Y
SD
, Y
GS
.
SOLUTION 20.25. Using the zero-sum properties of the rows and columns, we have by
inspection:
Y
ind

40 2 42
? ? 50
? 22 92





1
]
1
1
1

40 2 42
30 20 50
70 22 92





1
]
1
1
1
S
SOLUTION 20.26. (a) Y
ind

8 ? ?
2 9 7
6 ? ?





1
]
1
1
1

8 2 6
2 9 7
6 7 13





1
]
1
1
1
S, where the second equality arises
because a purely
resistive network has a symmetric indefinite admittance matrix.
(b) Construct a common ground 2-port with input terminal C, common terminal A, and output
terminal B:
Y
CB

13 7
7 9



1
]
1
S
Therefore, since the output terminal B is shorted, Y
L
= , i.e.,
Y
in
y
11

y
12
y
21
y
22
+ Y
L
13 S
Hence
Z
in

1
13

(c) Construct a common ground 2-port with input terminal A, common terminal B, and output
terminal C:
Y
AC

8 6
6 13



1
]
1
S
Hence, the required voltage gain is
G
V

V
2
V
1

y
21
y
22
+ Y
L

y
21
y
22

6
13
Therefore, V
2

6
13
V.
SOLUTION 20.27.
(a) Using the zero-sum property of the rows and columns of an indefinite admittance matrix we
can write down by inspection (in mS)
I
B
I
E
I
C





1
]
1
1
1

1 1 0
100 100.1 0.1
99 99.1 0.1





1
]
1
1
1
V
B
V
E
V
C





1
]
1
1
1
(b) The y-parameters (also in mS) of the common emitter configuration are easily computed as
I
B
I
C



1
]
1

1 0
99 0.1



1
]
1
V
BE
V
CE



1
]
1
Hence in MATLAB
Y = [1 0;99 0.1];
Z = inv(Y)
Z =
1.0000e+00 0
-9.9000e+02 1.0000e+01
where Z is in k. It follows that V
CE
990I
B
+ 10I
C
where I
B
and I
C
are in mA and V
CE
is
in volts.
SOLUTION 20.28. Expanding the given y-parameter matrix into an indefinite admittance matrix
yields
I
G
I
D
I
S





1
]
1
1
1
Y
ind
V
G
V
D
V
S





1
]
1
1
1

0.2 + j2.5 0.01 j0.65 0.19 j1.85


3.1 j0.65 0.05 + j0.8 3.15 j0.15
3.3 j1.85 0.04 j0.15 3.34 + j2





1
]
1
1
1
V
G
V
D
V
S





1
]
1
1
1
By inspection, with G as the common terminal, S as the input terminal, and D as the output
terminal, we obtain,
Y
new

3.34 + j2 0.04 j0.15


3.15 j0.15 0.05 + j0.8



1
]
1
mS
SOLUTION 20.29. Here we use the zero-sum properties of the columns and rows to complete the
indefinite admittance matrix:
Y
ind

s ? ?
s 1 2 ?
1 2s 1 2s





1
]
1
1
1

s 2s 1 3s +1
s1 2 s1
1 2s1 2s





1
]
1
1
1
S
In figure (b), the top 2-port has y
top

s 2s1
s 1 2



1
]
1
S and the bottom 2-port has y-parameters
y
bot

2s 2s 1
s 1 2



1
]
1
S. Since these 2-ports are connected in parallel, the overall 2-port y-
parameters are
y y
top
+ y
bot

s 2s1
s 1 2



1
]
1
+
2s 2s 1
s 1 2



1
]
1

3s 2
2 4



1
]
1
S
SOLUTION 20.30. From problem 28,
I
G
I
D
I
S





1
]
1
1
1
Y
ind
V
G
V
D
V
S





1
]
1
1
1

0.2 + j2.5 0.01 j0.65 0.19 j1.85


3.1 j0.65 0.05 + j0.8 3.15 j0.15
3.3 j1.85 0.04 j0.15 3.34 + j2





1
]
1
1
1
V
G
V
D
V
S





1
]
1
1
1
Hence
y
GS

0.2 + j2.5 0.19 j1.85


3.3 j1.85 3.34 + j2



1
]
1
S
SOLUTION 20.31. (a) Here, by inspection we can compute the indefinite admittance matrix as
the coefficient matrix of the following nodal equations:
I
A
I
B
I
C





1
]
1
1
1
Y
ind
V
A
V
B
V
C





1
]
1
1
1

Y
1
+ Y
2
+ 2 Y
2
Y
1
2
Y
2
2 Y
2
2
Y
1
2Y
2
2Y
2
Y
1





1
]
1
1
1
V
A
V
B
V
C





1
]
1
1
1
(b)
y
AB

Y
1
+ Y
2
+ 2 Y
2
Y
2
2 Y
2



1
]
1
(c)
y
AC

Y
1
+ Y
2
+ 2 Y
1
2
Y
1
2Y
2
Y
1



1
]
1
SOLUTION 20.32. (a) Here, by inspection we can compute the indefinite admittance matrix as
the coefficient matrix of the following nodal equations:
I
A
I
B
I
C





1
]
1
1
1
Y
ind
V
A
V
B
V
C





1
]
1
1
1

Y
1
+ Y
2
Y
2
Y
1
Y
2
+ g
m
Y
2
+Y
3
Y
3
g
m
Y
1
g
m
Y
3
Y
1
+ Y
3
+ g
m





1
]
1
1
1
V
A
V
B
V
C





1
]
1
1
1
(b)
y
AB

Y
1
+ Y
2
Y
2
Y
2
+ g
m
Y
2
+Y
3



1
]
1
(c)
y
AC

Y
1
+ Y
2
Y
1
Y
1
g
m
Y
1
+ Y
3
+ g
m



1
]
1
SOLUTION 20.33. (a) Here the nodal equation matrix is:
I
A
I
B
I
C
0







1
]
1
1
1
1
1

W
11
W
12
W
21
W
22



1
]
1
V
A
V
B
V
C
V
D







1
]
1
1
1
1
1

8 0 0 8
0 2 + 2s 2s 10 8
0 2s 16 + 2s 16
8 2 6 16







1
]
1
1
1
1
1
V
A
V
B
V
C
V
D







1
]
1
1
1
1
1
where V
D
is the internal node voltage. Using the method of matrix partitioning,
Y
ind
W
11
W
12
W
22
1
W
21

8 0 0
0 2 + 2s 2s10
0 2s 16 + 2s





1
]
1
1
1

1
16
8
8
16





1
]
1
1
1
8 2 6 [ ]

8 0 0
0 2 + 2s 2s10
0 2s 16 + 2s





1
]
1
1
1

0.5
0.5
1





1
]
1
1
1
8 2 6 [ ]
4 1 3
4 3 + 2s 2s 7
8 2s 2 10 + 2s





1
]
1
1
1
S
(b) When C is grounded,
y
AB

4 1
4 3 + 2s



1
]
1
S and z
AB

1
8s +16
3 + 2s 1
4 4



1
]
1

(c) When B is grounded
y
AC

4 3
8 10 + 2s



1
]
1
S
SOLUTION 20.34. (a) Writing the usual node equations we have,
I
A
I
B
I
C
0







1
]
1
1
1
1
1

W
11
W
12
W
21
W
22



1
]
1
V
A
V
B
V
C
V
D







1
]
1
1
1
1
1

s + 0.5 s 0 0.5
s s + 0.5 0 0.5
0 0 0.5s 0.5s
0.5 0.5 0.5s 0.5s +1







1
]
1
1
1
1
1
V
A
V
B
V
C
V
D







1
]
1
1
1
1
1
Using the method of matrix partitioning,
Y
ind
W
11
W
12
W
22
1
W
21

s + 0.5 s 0
s s + 0.5 0
0 0 0.5s





1
]
1
1
1

0.5
s + 2
1
1
s





1
]
1
1
1
1 1 s [ ]

s + 0.5 s 0
s s + 0.5 0
0 0 0.5s





1
]
1
1
1

0.5
s + 2
1 1 s
1 1 s
s s s
2





1
]
1
1
1

1
s + 2
s
2
+ 2.5s + 0.5 s
2
2s 0.5 0.5s
s
2
2s 0.5 s
2
+ 2.5s + 0.5 0.5s
0.5s 0.5s s






1
]
1
1
1
1
S
(b) Here
y
AB

1
s + 2
s
2
+ 2.5s + 0.5 s
2
2s 0.5
s
2
2s 0.5 s
2
+ 2.5s + 0.5




1
]
1
1
S
SOLUTION 20.35. (a) Rule 1: Consider the two networks N
A
(3 external nodes) and N
B
(4
external nodes) given below:
The two indefinite admittance matrices are
Y
indN
A

G
1
+ G
2
G
2
G
1
G
2
G
2
+ G
3
G
3
G
1
G
3
G
1
+ G
3





1
]
1
1
1
and Y
indN
B

G
1
+ G
2
G
2
G
1
0
G
2
G
2
+ G
3
G
3
0
G
1
G
3
G
1
+ G
3
0
0 0 0 0







1
]
1
1
1
1
1
Observe that Y
indN
B
can be obtained from Y
indN
A
by adding a column of zeros and a row of
zeros to form a 4x4 matrix.
(b) Rule 2: Consider two networks N
A
and N
B
and a third network N
C
which combines
elements of N
A
and N
B
as given below:
The corresponding indefinite admittance matrices are:
Y
indN
A

6 2 4
2 3 1
4 1 5





1
]
1
1
1
, Y
indN
B

5 3 2
3 5 2
2 2 4





1
]
1
1
1
, and Y
indN
C

11 5 6
5 8 3
6 3 9





1
]
1
1
1
.
Clearly,
Y
indN
C
Y
indN
A
+Y
indN
B
(c) Rule 3: Consider the 3-terminal network
with indefinite admittance matrix
Y
indN
A

5 3 2
3 5 2
2 2 4





1
]
1
1
1
S
If we move node 3 inside to form a 2-terminal network and labeled as N
B
,
then from nodal analysis we have
I
1
I
2
0





1
]
1
1
1

5 3 2
3 5 2
2 2 4





1
]
1
1
1
V
1
V
2
V
3





1
]
1
1
1
Using the method of matrix partitioning,
Y
indN
B

5 3
3 5



1
]
1

1
4
2
2



1
]
1
2 2 [ ]
4 4
4 4



1
]
1
S
This computation is the one given by the formula in the problem. To see that this is correct, we
observe that the internal simplification of NB leads to the following:
SOLUTION 20.36.
Part (a)
Yinda= [1/2 -1/4 0 -1/4 0; -1/4 1/2 0 0 -1/4; ...
0 0 0 0 0; -1/4 0 0 1/2 -1/4; 0 -1/4 0 -1/4 1/2]
Yinda =
5.0000e-01 -2.5000e-01 0 -2.5000e-01 0
-2.5000e-01 5.0000e-01 0 0 -2.5000e-01
0 0 0 0 0
-2.5000e-01 0 0 5.0000e-01 -2.5000e-01
0 -2.5000e-01 0 -2.5000e-01 5.0000e-01
Yindb= [0 0 0 0 0; 0 0 0 0 0; 0 0 3/4 -1/2 -1/4; ...
0 0 -1/2 5/8 -1/8; 0 0 -1/4 -1/8 3/8]
Yindb =
0 0 0 0 0
0 0 0 0 0
0 0 7.5000e-01 -5.0000e-01 -2.5000e-01
0 0 -5.0000e-01 6.2500e-01 -1.2500e-01
0 0 -2.5000e-01 -1.2500e-01 3.7500e-01
Part (b)
Yind = Yinda + Yindb
Yind =
5.0000e-01 -2.5000e-01 0 -2.5000e-01 0
-2.5000e-01 5.0000e-01 0 0 -2.5000e-01
0 0 7.5000e-01 -5.0000e-01 -2.5000e-01
-2.5000e-01 0 -5.0000e-01 1.1250e+00 -3.7500e-01
0 -2.5000e-01 -2.5000e-01 -3.7500e-01 8.7500e-01
% To suppress nodes 4 and 5 we use the partitioned matrix formula as follows:
W11= [Yind(1:3, 1:3)]
W11 =
5.0000e-01 -2.5000e-01 0
-2.5000e-01 5.0000e-01 0
0 0 7.5000e-01
W12=[Yind(1:3, 4:5)]
W12 =
-2.5000e-01 0
0 -2.5000e-01
-5.0000e-01 -2.5000e-01
W21= [Yind(4:5, 1:3)]
W21 =
-2.5000e-01 0 -5.0000e-01
0 -2.5000e-01 -2.5000e-01
W22= [Yind(4:5, 4:5)]
W22 =
1.1250e+00 -3.7500e-01
-3.7500e-01 8.7500e-01
Yind123 = W11 - W12*inv(W22)*W21
Yind123 =
4.3519e-01 -2.7778e-01 -1.5741e-01
-2.7778e-01 4.1667e-01 -1.3889e-01
-1.5741e-01 -1.3889e-01 2.9630e-01
(d) For the required Y-matrix we delete row and column 3 to obtain
Ysc = Yind123(1:2,1:2)
Ysc =
4.3519e-01 -2.7778e-01
-2.7778e-01 4.1667e-01
Zoc = inv(Ysc)
Zoc =
4.0000e+00 2.6667e+00
2.6667e+00 4.1778e+00
SOLUTION 20.37. Since complex roots must occur in conjugate pairs, we will only check j
0
.
0 p( j
0
) ja
3 0
3
a
2 0
2
+ ja
1 0
+ a
0
a
0
a
2 0
2
+ j a
1 0
a
3 0
3
( )
Both real and imaginary parts must be zero, i.e.,
a
0
a
2 0
2
0 and a
1 0
a
3 0
3

0
a
1
a
3 0
2
( )
0
From the first equation, a
2 0
2
a
0
. The second equation above must be true for arbitrary
0
which implies that a
1
a
3 0
2
. Equivalently a
1
a
2
a
3
a
2 0
2
a
3
a
0
. Conclusion: this condition
leads to imaginary complex roots.
(b) Given the above condition, what are the resulting imaginary roots of the polynomial? Since
the polynomial is cubic, we can assume a
3
0. In this case,
0 p(s) s
3
+
a
2
a
3
s
2
+
a
1
a
3
s +
a
0
a
3
s s
2
+
a
1
a
3



_
,

+
a
2
a
3
s
2
+
a
1
a
2
a
3
2
s s
2
+
a
1
a
3



_
,

+
a
2
a
3
s
2
+
a
1
a
3



_
,

s
2
+
a
1
a
3



_
,

s +
a
2
a
3



_
,

Therefore, the roots are: s tj
a
1
a
3
,
a
2
a
3
.
SOLUTION 20.38.
(a) The four 2-port equations arising from the interconnection are:
V
1
V
1a
V
1b
I
1
I
1a
I
1b
V
2
V
2a
V
2b
I
2
I
2a
+ I
2b
Thus
V
1
V
1a
V
1b
h
11a
I
1a
+ h
12a
V
2a
( ) h
11b
I
1b
+ h
12b
V
2b
( ) h
11a
+ h
11b
( )I
1
+ h
12a
h
12b
( )V
2
and
I
2
I
2a
+ I
2b
h
21a
I
1a
+ h
22a
V
2a
( ) + h
21b
I
1b
+ h
22b
V
2b
( ) h
21a
h
21b
( )I
1
+ h
22a
+ h
22b
( )V
2
This proves that the series-parallel connection has the required h-parameters.
(b) The four 2-port equations arising from the interconnection are:
V
2
V
2a
V
2b
I
2
I
2a
I
2b
V
1
V
1a
V
1b
I
1
I
1a
+ I
1b
Thus,
I
1
I
1a
+ I
1b
g
11a
V
1a
+ g
12a
I
2a
( ) + g
11b
V
1b
+ g
12b
I
2b
( ) g
11a
+ g
11b
( )V
1
+ g
12a
g
12b
( )I
2
and
V
2
V
2a
V
2b
g
21a
V
1a
+ g
22a
I
2a
( ) g
21b
V
1b
+ g
22b
I
2b
( ) g
21a
g
21b
( )V
1
+ g
22a
+ g
22b
( )I
2
This proves that the parallel-series connection has the required g-parameters.
SOLUTION 20.39.
(a) Refer here to N
a
in figure P20.39b. With reference to figure 19.28b, h
11
= 11 k, h
12
= 0,
h
21
= 95.9, and h
22
= 1/10
5
= 10
-5
S. Similarly, by inspection with reference to equation 19.33,
h
11
= 90||10 = 9 k, h
12
= 0.1 (reverse voltage division), h
21
= 0.1, and h
22
= 10
-5
S.
(b) By problem 20.38, part (a),
h
11
= h
11a
+ h
11b
= 20 k h
12
= h
12a
h
12b
= 0.1
h
21
= h
21a
h
21b
= 96 h
22
= h
22a
+ h
22b
= 210
-5
S
(c) Recall from chapter 19 that Z
in

V
1
I
1
h
11

h
12
h
21
h
22
+ Y
L
and Y
out

I
2
V
2
h
22

h
12
h
21
h
11
+ Z
s
in
which case Z
out
is the reciprocal. Using our MATLAB script, we have
h = [20e3 -0.1; 96 0.02e-3 ];
zL= 1e8;
zs= 5e3;
twoporth(h,zL,zs)
ans =
twoport analysis using h-parameters
zin =
4.9976e+05
zout =
2.4752e+03
v1tovs =
9.9009e-01
v2tov1 =
-9.5998e+00
v2tovs =
-9.5047e+00
REMARK: We have used the following m-file code for "twoporth":
% two-port analysis in terms of h-parameters
function [zin, zout] =twoport(h, zL, zs)
['twoport analysis using h-parameters']
h11= h(1,1); h12=h(1,2); h21=h(2,1); h22=h(2,2);
zin = h11 - h12*h21/(h22+ 1/zL)
yout= h22 - h12*h21/(h11+zs);
zout= 1/yout
v1tovs= zin/(zin+zs)
v2tov1= -h21/(zin*(h22+1/zL))
v2tovs= v1tovs*v2tov1
SOLUTION 20.40.
(a) The y-parameters for N
a
are:
y
AB

7 4
4 7



1
]
1
1

1
33
7 4
4 7



1
]
1
S
Hence the indefinite admittance matrix for N
a
is:
Y
indN
a

1
33
7 4 3
4 7 3
3 3 6





1
]
1
1
1
S
Let us consider the associate 2-port with port A grounded and B as the new port 1 input. The
problem is then solved by computing the input impedance with port 2 open circuited. Hence, the
new y-parameters are:
y
BC

1
33
7 3
3 6



1
]
1
S
Thus Y
in
y
11

y
12
y
21
y
22

1
33
7
9
6


_
,

5.5
33

1
6
S. Hence, Z
in
6 is the unique reading.
(b) The answer is not unique as demonstrated in part (c).
(c) The following two networks have the given Z-parameters, but the meter reading for N
1
is 4
but for N
2
it is 2 .
SOLUTIONS CHAPTER 21 PROBLEMS
SOLUTION TO 21.1.
(a) Low pass
(b) High pass
SOLUTION TO 21.2.
SOLUTION TO 21.3.
(a)
n = 0.65378;
d = [1 0.80381643 0.82306043];
w = 0:0.005:2;
h = freqs(n,d,w);
plot(w, 20*log10(abs(h)))
grid
xlabel('Frequency rads/s')
ylabel('dB Gain')

0 0.5 1 1.5 2
-15
-10
-5
0
Frequency rads/s
d
B

G
a
i
n
TextEnd
(b) poles = roots(d)
poles =
-4.0191e-01 + 8.1335e-01i
-4.0191e-01 - 8.1335e-01i
(c)
% Poles of new transfer function
wp = 2*pi*750'
wp =
4.7124e+03
wp = 2*pi*750;
polesnew = poles*wp
polesnew =
-1.8939e+03 + 3.8328e+03i
-1.8939e+03 - 3.8328e+03i
% All zeros remain at infinity.
Further
H(s) = H
NLP
(s
p
) =
(
p
)
2
s
2
+ 0.80381643
p
s + 0.82306043(
p
)
2
=
2.2207 10
7
s
2
+ 3.7879 10
3
s +1.8277 10
7
SOLUTION TO 21.4. (a) The 2
nd
order normalized LP transfer function is H
NLP
(s) =
1
s
2
+ 2s +1
. This
must be frequency scaled by K
f
= 1000. Hence,
H(s) = H
NLP
(s K
f
) =
(K
f
)
2
s
2
+ K
f
2s + (K
f
)
2
=
9.8696 10
6
s
2
+ 4.4429 10
3
s + 9.8696 10
6
(b) Using MATLAB,
n = (1000*pi)^2;
d = [1 sqrt(2)*pi*1e3 (1000*pi)^2];
w = 0:1:2*pi*1500;
h = freqs(n,d,w);
plot(w/(2*pi),abs(h))
grid
xlabel('Frequency in Hz')
ylabel('Magnitude')
plot(w/(2*pi),20*log10(abs(h)))
grid
xlabel('Frequency in Hz')
ylabel('Magnitude in dB')
0 500 1000 1500
0.1
0.2
0.3
0.4
0.5
0.6
0.7
0.8
0.9
1
Frequency in Hz
M
a
g
n
i
t
u
d
e
TextEnd
0 500 1000 1500
-20
-18
-16
-14
-12
-10
-8
-6
-4
-2
0
Frequency in Hz
M
a
g
n
i
t
u
d
e

i
n

d
B
TextEnd
(c)
n
n = 9.8696e+06
d
d = 1.0000e+00 4.4429e+03 9.8696e+06
w = j*2000*pi;
mag = abs(n/(w^2 + d(2)*w + d(3)))
mag = 2.4254e-01
SOLUTION TO 21.5. (a)
max
is that value of e that places the magnitude response curve through A
max
at =
p
. Therefore
A
max
= 10log
10
H( j
p
)
2
=10log
10
1+
max
2 p
p






2n








= 10log
10
1+
max
2
( )
Therefore
max
2
=10
0.1A
max
1 which upon a square root yields the final answer.
(b) Similarly,
min
puts the magnitude response curve through the A
min
spec. Hence
A
min
= 10log
10
H( j
s
)
2
=10log
10
1+
min
2 s
p






2n








Therefore

min
2
=
10
0.1A
min
1
s
p






2n
which is equivalent to the required formula.
SOLUTION TO 21.6. The relationship of and
c
is given by the formula:
c
=
p
()
1 n
. Further,
max
in putting the magnitude response curve through the A
max
spec produces
c min
, and
min
in putting the
magnitude response curve through the A
min
spec produces
c max
. Hence, from the solution to problem
5,
c min
=
p
(
max
)
1 n
=
p
2n
10
0.1A
max
1

c max
=
p
(
min
)
1 n
=
s
2n
10
0.1A
min
1
SOLUTION TO 21.7. (a) From above material, the second order Butterworth NLP transfer function is
H
NLP2
(s) =
1
s
2
+ 2s +1
and from tables, the third order is
H
NLP3
(s) =
1
s
3
+ 2s
2
+ 2s +1
(b)
n1 = 1; d1 = [1 sqrt(2) 1];
n2 = 1; d2 = [1 2 2 1];
w = 0:.01:5;
h1 = freqs(n1,d1,w);
h2 = freqs(n2,d2,w);
plot(w,abs(h1))
grid
xlabel('Normalized Frequency')
ylabel('Magnitude')
hold
plot(w,abs(h2),'r')
hold off
0 1 2 3 4 5
0
0.1
0.2
0.3
0.4
0.5
0.6
0.7
0.8
0.9
1
Normalized Frequency
M
a
g
n
i
t
u
d
e
TextEnd
Notice how the 3
rd
order filter has a sharper transition to zero.
(c) % The simplest way to obtain the step response is as follows:

syms s t
StepResp1 = ilaplace(1/(s^3 + sqrt(2)*s^2 + s))
StepResp1 =
1-exp(-1/2*2^(1/2)*t)*cos(1/2*2^(1/2)*t)-exp(-1/2*2^(1/2)*t)*sin(1/2*2^(1/2)*t)
StepResp2 = ilaplace(1/(s^4 + 2*s^3 + 2*s^2 + s))
StepResp2 =
1-exp(-t)-2/3*exp(-1/2*t)*3^(1/2)*sin(1/2*3^(1/2)*t)

Thus the step response of the second order Butterworth normalized LP filter is:
v(t) = u(t) e
0.70711t
cos(0.70711t) sin(0.70711t) [ ]u(t)
and that of the third order Butterworth normalized LP filter is:
v(t) = 1 e
t
( )
u(t) 1.1547e
0.5t
sin(0.86603t)u(t)
SOLUTION TO 21.8.
fp = 100; fs = 1200;Amax = 0.3; Amin = 35;
n = buttord(fp,fs,Amax,Amin,'s')
emax = sqrt(10^(0.1*Amax) - 1)
emin = sqrt(10^(0.1*Amin) - 1)/(fs/fp)^n
fcmin = fp/((10^(0.1*Amax)-1)^(1/(2*n)))
fcmax = fs/((10^(0.1*Amin)-1)^(1/(2*n)))
wcmin = 2*pi*fcmin
wcmax = 2*pi*fcmax
wc = wcmin;
fc = fcmin;
[z,p,k] = buttap(n)
% Numerators are each 1. Denominators are the polynomials
d1 = poly(p(1:2))
d2 = poly(p(3))
zplane(p)
grid
pause
znew = z*wc
pnew = p*wc
knew = k*wc^n
f = 0:fc/50:1.2*fs;
h = freqs(knew*poly(znew),poly(pnew),2*pi*f);
plot(f,abs(h))
grid
xlabel('Frequency in Hz')
ylabel('Gain magnitude')
pause
plot(f,20*log10(abs(h)))
xlabel('Frequency in Hz')
ylabel('Gain in dB')
grid
n = 3
emax = 2.6743e-01
emin = 3.2538e-02
fcmin = 1.5521e+02
fcmax = 3.1324e+02
wcmin = 9.7524e+02
wcmax = 1.9681e+03
z = []
p =
-5.0000e-01 + 8.6603e-01i
-5.0000e-01 - 8.6603e-01i
-1.0000e+00
k = 1
% Numerators are each 1. Denominators are the polynomials
d1 = 1.0000e+00 1.0000e+00 1.0000e+00
d2 = 1 1
-1 -0.5 0 0.5 1
-1
-0.8
-0.6
-0.4
-0.2
0
0.2
0.4
0.6
0.8
1
Real part
I
m
a
g
i
n
a
r
y

p
a
r
t
znew = []
pnew =
-4.8762e+02 + 8.4458e+02i
-4.8762e+02 - 8.4458e+02i
-9.7524e+02
knew = 9.2753e+08
0 500 1000 1500
0
0.1
0.2
0.3
0.4
0.5
0.6
0.7
0.8
0.9
1
Frequency in Hz
G
a
i
n

m
a
g
n
i
t
u
d
e
0 500 1000 1500
-60
-50
-40
-30
-20
-10
0
Frequency in Hz
G
a
i
n

i
n

d
B
SOLUTION TO 21.9.
fp = 100; fs = 1200;Amax = 0.3; Amin = 35;
n = buttord(fp,fs,Amax,Amin,'s');
emax = sqrt(10^(0.1*Amax) - 1);
emin = sqrt(10^(0.1*Amin) - 1)/(fs/fp)^n;
fcmin = fp/((10^(0.1*Amax)-1)^(1/(2*n)));
fcmax = fs/((10^(0.1*Amin)-1)^(1/(2*n)));
wcmin = 2*pi*fcmin;
wcmax = 2*pi*fcmax;
[z,p,k] = buttap(n);
wc = wcmax;
fc = fcmax;
znew = z*wc
pnew = p*wc
knew = k*wc^n
f = 0:fc/50:1.2*fs;
h = freqs(knew*poly(znew),poly(pnew),2*pi*f);
plot(f,abs(h))
grid
xlabel('Frequency in Hz')
ylabel('Gain magnitude')
pause
plot(f,20*log10(abs(h)))
xlabel('Frequency in Hz')
ylabel('Gain in dB')
grid
znew = []
pnew =
-9.8406e+02 + 1.7044e+03i
-9.8406e+02 - 1.7044e+03i
-1.9681e+03
knew = 7.6235e+09
0 500 1000 1500
0
0.1
0.2
0.3
0.4
0.5
0.6
0.7
0.8
0.9
1
Frequency in Hz
G
a
i
n

m
a
g
n
i
t
u
d
e
0 500 1000 1500
-40
-35
-30
-25
-20
-15
-10
-5
0
5
Frequency in Hz
G
a
i
n

i
n

d
B
SOLUTION TO 21.10.
fp = 75; fs = 450;Amax = 1; Amin = 45;
n = buttord(fp,fs,Amax,Amin,'s')
emax = sqrt(10^(0.1*Amax) - 1)
emin = sqrt(10^(0.1*Amin) - 1)/(fs/fp)^n
fcmin = fp/((10^(0.1*Amax)-1)^(1/(2*n)))
fcmax = fs/((10^(0.1*Amin)-1)^(1/(2*n)))
wcmin = 2*pi*fcmin
wcmax = 2*pi*fcmax
[z,p,k] = buttap(n)
d1 = poly(p(1:2))
d2 = poly(p(3:4))
zplane(p)
grid
pause
wc = wcmin;
fc = fcmin;
znew = z*wc
pnew = p*wc
knew = k*wc^n
W = 0:0.01:fs/fp;
h = freqs(k*poly(z),poly(p),W);
plot(W*wc/(2*pi),abs(h))
grid
xlabel('Frequency in Hz')
ylabel('Gain magnitude')
pause
plot(W*wc/(2*pi),20*log10(abs(h)))
xlabel('Frequency in Hz')
ylabel('Gain in dB')
grid
n = 4
emax = 5.0885e-01
emin = 1.3721e-01
fcmin = 8.8800e+01
fcmax = 1.2323e+02
wcmin = 5.5795e+02
wcmax = 7.7427e+02
z = []
p =
-3.8268e-01 + 9.2388e-01i
-3.8268e-01 - 9.2388e-01i
-9.2388e-01 + 3.8268e-01i
-9.2388e-01 - 3.8268e-01i
k = 1
d1 = 1.0000e+00 7.6537e-01 1.0000e+00
d2 = 1.0000e+00 1.8478e+00 1.0000e+00
znew = []
pnew =
-2.1352e+02 + 5.1548e+02i
-2.1352e+02 - 5.1548e+02i
-5.1548e+02 + 2.1352e+02i
-5.1548e+02 - 2.1352e+02i
knew = 9.6912e+10
-1 -0.5 0 0.5 1
-1
-0.8
-0.6
-0.4
-0.2
0
0.2
0.4
0.6
0.8
1
Real part
I
m
a
g
i
n
a
r
y

p
a
r
t
0 100 200 300 400 500 600
0
0.1
0.2
0.3
0.4
0.5
0.6
0.7
0.8
0.9
1
Frequency in Hz
G
a
i
n

m
a
g
n
i
t
u
d
e
0 100 200 300 400 500 600
-70
-60
-50
-40
-30
-20
-10
0
Frequency in Hz
G
a
i
n

i
n

d
B
SOLUTION TO 21.11.
fp = 75; fs = 450;Amax = 1; Amin = 45;
n = buttord(fp,fs,Amax,Amin,'s');
% The order mfile may not be available in the student edition.
emax = sqrt(10^(0.1*Amax) - 1);
emin = sqrt(10^(0.1*Amin) - 1)/(fs/fp)^n;
fcmin = fp/((10^(0.1*Amax)-1)^(1/(2*n)));
fcmax = fs/((10^(0.1*Amin)-1)^(1/(2*n)));
wcmin = 2*pi*fcmin;
wcmax = 2*pi*fcmax;
[z,p,k] = buttap(n);
wc = wcmax;
fc = fcmax
znew = z*wc
pnew = p*wc
knew = k*wc^n
W = 0:0.01:fs/fp;
h = freqs(k*poly(z),poly(p),W);
plot(W*wc/(2*pi),abs(h))
grid
xlabel('Frequency in Hz')
ylabel('Gain magnitude')
pause
plot(W*wc/(2*pi),20*log10(abs(h)))
xlabel('Frequency in Hz')
ylabel('Gain in dB')
grid
fc = 1.2323e+02
znew = []
pnew =
-2.9630e+02 + 7.1533e+02i
-2.9630e+02 - 7.1533e+02i
-7.1533e+02 + 2.9630e+02i
-7.1533e+02 - 2.9630e+02i
knew = 3.5940e+11
0 100 200 300 400 500 600 700 800
0
0.1
0.2
0.3
0.4
0.5
0.6
0.7
0.8
0.9
1
Frequency in Hz
G
a
i
n

m
a
g
n
i
t
u
d
e
0 100 200 300 400 500 600 700 800
-70
-60
-50
-40
-30
-20
-10
0
Frequency in Hz
G
a
i
n

i
n

d
B
SOLUTION TO 21.12. Here we require that
s
3
+ 2s
2
+ 2s +1= s
3
+
C
1
+ C
2
C
1
C
2
s
2
+
C
1
+ C
2
+ L
LC
1
C
2
s +
2
LC
1
C
2
Thus
LC
1
C
2
= 2 C
1
+ C
2
+ L = 4 and 2 =
C
1
+ C
2
C
1
C
2

L
L
=
LC
1
+ LC
2
2
in which case, LC
1
+ LC
2
= 4. Therefore, L C
1
+ C
2
+ L ( ) = 4L = 4 + L
2
. Equivalently
L
2
4L + 4 = (L 2)(L 2) = 0
Hence L = 2 H is the only solution. Thus C
1
C
2
= 1 C
1
=
1
C
2

1
C
2
+ C
2
+ 2 = 4 or equivalently
C
2
2
2C
2
+1 = (C
2
1)(C
2
1) = 0 which implies that C
1
= C
2
= 1 F is the only solution, as was to be
shown.
SOLUTION TO 21.13. (a) By voltage division
H(s) =
1
Cs + G
Ls +
1
Cs + G
=
1
LC
s
2
+
1
RC
s +
1
LC
(b) With R = 1 ,
H(s) =
1
LC
s
2
+
1
RC
s +
1
LC
=
1
s
2
+ 2s +1
requires that C = 1/ 2 F and L = 2 H.
(c) fc = 1000;
wc = 2*pi*fc
wc = 6.2832e+03
Kf = wc;
Km = 1000;
C = 1/sqrt(2);
L = sqrt(2);
Cnew = C/(Kf*Km)
Cnew = 1.1254e-07
Lnew = L*Km/Kf
Lnew = 2.2508e-01
(d)
Km = C/(wc*1e-6)
Km = 1.1254e+02
Kf = wc;
Rnew = Km
Rnew = 1.1254e+02
Lnew = L*Km/wc
Lnew =
2.5330e-02
SOLUTION TO 21.14. (a) By voltage division
H(s) =
1
Cs
Ls + R
s
+
1
Cs
=
1
LC
s
2
+
R
s
L
s +
1
LC
(b) With R = 1 ,
H(s) =
1
LC
s
2
+
R
s
L
s +
1
LC
=
1
s
2
+ 2s +1
requires that L = 1/ 2 F and C = 2 H.
(c)
L = 1/sqrt(2); C = 1/L;
Km = 10;
Kf = 2*pi*500;
Rnew = 10;
Cnew = C/(Km*Kf)
Cnew = 4.5016e-05
Lnew = Km*L/Kf
Lnew = 2.2508e-03
(d)
Km = C/(1e-6*Kf)
Km = 4.5016e+02
Lnew = L*Km/Kf
Lnew = 1.0132e-01
Cnew = C/(Km*Kf)
Cnew = 1.0000e-06
SOLUTION TO 21.15.
(a) Since 1 = 2/(LC), L = 2/C. Since (1/L + 1/C) = (C/2 + 1/C) = sqrt(2), we have that C is a root of
the quadratic 0.5C
2
sqrt(2)C + 1 = 0. Hence
v = [0.5 -sqrt(2) 1];
r = roots(v)
r =
1.4142e+00
1.4142e+00
C = r(1)
C = 1.4142e+00
L = 2/C
L = 1.4142e+00
(b)
Km = 1e3;
Kf = 2*pi*3500;
Cnew = C/(Km*Kf)
Cnew = 6.4308e-08
Lnew = L*Km/Kf
Lnew = 6.4308e-02
(c)
Km = C/(Kf*10e-9)
Km = 6.4308e+03
Cnew = C/(Km*Kf)
Cnew = 1.0000e-08
Lnew = L*Km/Kf
Lnew = 4.1356e-01
Rs = Km
Rs = 6.4308e+03
RL = Rs
RL = 6.4308e+03
SOLUTION TO 21.16.
(a) Let G = R
L
. Then by voltage division
H(s) =
V
out
V
in
=
1
Cs + G
Ls + R
s
+
1
Cs + G
=
1
LC
s
2
+
1
CR
L
+
R
s
L






s +
1 + R
s
R
L
LC
(b) Since 1 = 1.25/(LC), L = 1.25/C. Since (R
s
/L + 1/R
L
C) = (2C/1.25 + 1/8C) = sqrt(2), we have that
C is a root of the quadratic (16/1.25)C
2
8sqrt(2)C + 1 = 0. Hence
C = roots([16/1.25 -8*sqrt(2) 1])
C =
7.8427e-01
9.9615e-02
L = 1.25 ./C
L =
1.5938e+00
1.2548e+01
(c)
Km = 1e3;
Kf = 2*pi*5e3
Kf = 3.1416e+04
Cnew = C/(Km*Kf)
Cnew =
2.4964e-08
3.1708e-09
Lnew = L*Km/Kf
Lnew =
5.0734e-02
3.9942e-01
SOLUTION TO 21.17.
(a) Define G = 1/R
s
, execute two source transformations, and apply voltage division to obtain
H(s) =
V
out
V
in
=
1
1
Cs + G
+ Ls +1

G
Cs + G
=
1
R
s
LC
s
2
+
1
R
s
C
+
1
L






s +
1 +1 R
s
LC
(b) Since L = 1.5/C, the values of C are the roots of the quadratic, (2/1.5)C
2
2sqrt(2)C + 1 = 0.
Hence
C = roots([(2/1.5) -2*sqrt(2) 1])
C =
1.6730e+00
4.4829e-01
L = 1.5 ./C
L =
8.9658e-01
3.3461e+00
Km = 2e3;
Kf = 2*pi*5e3;
Cnew = C/(Km*Kf)
Cnew =
2.6627e-08
7.1347e-09
Lnew = Km*L/Kf
Lnew =
5.7078e-02
2.1302e-01
Rsnew = 2*Km
Rsnew = 4000
SOLUTION TO 21.18.
(a)
H( j) =
K
j

p






+1
2
=
K

p






2
+1
The 3 dB down frequency,
c
, occurs when
1

p






2
+1
=
1
2
Equivalently

c
=
p
2 1 = 0.64359
p
= 6.4359 10
4
rad/sec.
(b) h(t) = K
p
2
te

p
t
u(t) since
H(s) =
K
s

p
+1






2
=
K
p
2
s +
p ( )
2
.
Further,
H(s)
1
s
=
K
p
2
s s +
p ( )
2
=
K
s

K
s +
p

K
p
s +
p ( )
2
Hence, the step response is
K 1 e

p
t

p
te

p
t




u(t)
SOLUTION TO 21.19.
(a) Using voltage division,
H(s) =
V
C
V
in

V
out
V
C
=
1
Cs +
1
L
2
s +1
1 + L
1
s +
1
Cs +
1
L
2
s +1

1
L
2
s +1
=
1
L
1
L
2
C
s
3
+
1
L
1
+
1
L
2






s
2
+
L
1
+ L
2
+ C
L
1
L
2
C
s +
2
L
1
L
2
C
(b) Matching coefficients in
1
L
1
L
2
C
s
3
+
1
L
1
+
1
L
2






s
2
+
L
1
+ L
2
+ C
L
1
L
2
C
s +
2
L
1
L
2
C
=
1
s
3
+ 2s
2
+ 2s +1
yields C =
2
L
1
L
2
and
L
1
+ L
2
+ C
L
1
L
2
C
=
L
1
+ L
2
+
2
L
1
L
2
2
= 2; equivalently, L
1
+ L
2
+
2
L
1
L
2
= 4. Further,
1
L
1
+
1
L
2






=
L
1
+ L
2
L
1
L
2
= 2 implies that L
1
+ L
2
= 2L
1
L
2
implies 2L
1
L
2
+
2
L
1
L
2
= 4. This requires that
L
1
L
2
= 1 and from earlier equations that L
1
+ L
2
= 2L
1
L
2
= 2 which forces L
1
= L
2
= 1 H and C = 2 F.
The idea is to match the denominator coefficients and thus the dc gain is 0.5 instead of the desired 1. A
transformer or some amplifier device is needed to increase the gain to 1.
(c) Using MATLAB:
Km = 1000;
Kf = 2*pi*20e3;
Lnew = Km/Kf
Lnew =
7.9577e-03
Cnew = 2/(Km*Kf)
Cnew =
1.5915e-08

Hence, L
1new
= L
2new
= 7.96 mH and C = 15.9 nF.
(d) SPICE simulation
V
0
R
1K
L
7.96m
L0
7.96m
R0
1K
C
15.9n
IVm
MAG(V(IVM))
Frequency (Hz) Prob 21.19-Small Signal AC-1
+0.000e+000
+100.000m
+200.000m
+300.000m
+400.000m
+500.000m
+10.000k +20.000k +30.000k +40.000k
SOLUTION TO 21.20. (a) From figure P21.19a
V
in
V
1
R
s
= I
1
, V
2
= V
out
in which case
V
1
= z
11
I
1
+ z
12
0 = V
in
R
s
I
1
Also
V
2
= z
21
I
1
+ z
22
0 = V
out
This implies that V
in
= (z
11
+ R
s
)I
1
and V
out
= z
21
I
1
. Finally we conclude that
V
out
V
in
=
z
21
z
11
+ R
s
(b) Now from figure P21.20b, we have V
1
= V
in
and V
2
= V
out
= R
L
I
2
. This implies that
I
2
=
V
out
R
L
= y
21
V
1
+ y
22
V
2
Thus
V
out
V
in
=
y
21
y
22
+
1
R
L
=
y
21
y
22
+ G
L
Consider here that
V
out
V
in
=
z
21
z
11
+1
=
1
s
2
+ 2s +1
=
1
s
s +
1
s
+ 2




=
1
2s
s
2
+
1
2s






Hence z
21
=
1
2s
and z
11
=
s
2
+
1
2s





. This leads to the circuit
with L =
1
2
H and C = 2 F.
( c) Similarly,
y
21
y
22
+1
=
1
2s
s
2
+
1
2s





+1
implies y
21
=
1
2s
and y
22
=
s
2s
+
1
2s
.
This yields the same circuit as above with
L = 2 H and C =
1
2
F.
(d) Here K
m
=1000 and K
f
= 5000rad / s
(d-i) For (b),
L =
1
2

10
3
5 10
3
=
1
5 2
= 0.1414 H
and
C = 2
1
5 10
6
= 0.2828 F
(d-ii) For (c)
L = 0.2828 H and C = 0.1414 F.
SOLUTION TO 21.21.
(a) From earlier developments
H(s) =
Y
in
Y
out
=
1
R
1
Cs +
1
R
2
(b) Let C = 1 F, and R
1
= R
2
= 1 .
(c) Kf = 2*pi*3500
Kf = 2.1991e+04
Km = 1/(Kf*1e-9)
Km = 4.5473e+04
In the final design, R
1
= R
2
= 45.5 k.
SOLUTION TO 21.22.
(a) From problem 21 and voltage division,
H(s) =
1
R
1
Cs +
1
R
2

1
C
2
s
R
3
+
1
C
2
s
=
1
C
1
R
1
s +
1
C
1
R
2

1
R
3
C
2
s +
1
R
3
C
2
(b) By inspection, let C
1
= 0.1 F, R
1
= 1 , R
2
= 10 , R
3
= 10 , and C
2
= 0.1 F, in which case
H(s) =
10
s + 1

1
s +1
=
10
s +1 ( )
2
(c)
Kf = 1e5;
Km = 0.1/(Kf*1e-9)
Km = 1000
Hence, in the final design
C
1
= 1 nF, R
1
= 1 k, R
2
= 10 k, R
3
= 10 k, and C
2
= 1 nF.
(d) Cascade the circuit of figure P21.22 with another op amp section. For the first part of the design,
again set
p
= 1 and use the same values as in part (c). The extra op amp section has the same values as
the first section. As such, final values are the same as in part (c).
SOLUTION TO 21.23. The 2
nd
order NLP Butterworth transfer function is: H
NLP
(s) =
1
s
2
+ 2s +1
. The
design parameters and steps are detailed in the excel spread sheet below. An additional design called
design C is also listed. For input attenuation, the resistor R
1
is replaced by the voltage divider R
3
-R
4
combination.
w0^2 w0/ Q Num w0 Q KNLP KMA Kf=wp KmR KMB KMC
1 1.41 1.00 1.0000 0.7071 1.00 22507.86 6283.20 10000.00 22507.86 15915.46
KMS
19492.37
RA RB K C1 C2 R1 R2 alpha R3 R4
Design A 0 1 2Q 1/ (2Q) 1 1 KNLP/ K 1/ alpha 1/ (1-alpha)
Design B 1 1 2 1 1/ Q 1 Q KNLP/ K 1/ alpha 1/ (1-alpha)
Design C 1 1-1/ Q 3-1/ Q 1 1 1 1 KNLP/ K 1/ alpha 1/ (1-alpha)
Saraga RA RA/ 3 3-Apr rt(3)Q 1 1/ Q 1/ rt(3) KNLP/ K 1/ alpha 1/ (1-alpha)
RA RB K C1 C2 R1 R2 alpha R3 R4
Design A 0.0000 1.0000 1.4142 0.7071 1.0000 1.0000 1.0000 1.0000 #DIV/ 0!
Design B 1.00 1.0000 2.0000 1.0000 1.4142 1.0000 0.7071 0.5000 2.0000 2.0000
Design C 1.00 -0.4142 1.5858 1.0000 1.0000 1.0000 1.0000 0.6306 1.5858 2.7071
Saraga 3.00 1.0000 1.3333 1.2247 1.0000 1.4142 0.5774 0.7500 1.3333 5.6569
w0 scale
Design A 0.0000 1.0000 1.4142 0.7071 1.0000 1.0000 1.0000 1.0000 #DIV/ 0!
Design B 1.00 1.0000 2.0000 1.0000 1.4142 1.0000 0.7071 0.5000 2.0000 2.0000
Design C 1.00 0.1140 2.1140 1.0000 1.0000 1.0000 1.0000 0.6306 1.5858 2.7071
Saraga 3.00 1.0000 1.3333 1.2247 1.0000 1.4142 0.5774 0.7500 1.3333 5.6569
wp scale
Design A 0.000E+00 1.000E+00 2.251E-04 1.125E-04 1.000E+00 1.000E+00 1.000E+00 1.000E+00 #DIV/ 0!
Design B 1.00 1.000E+00 2.000E+00 1.592E-04 2.251E-04 1.000E+00 7.071E-01 5.000E-01 2.000E+00 2.000E+00
Design C 1.00 1.140E-01 2.114E+00 1.592E-04 1.592E-04 1.000E+00 1.000E+00 6.306E-01 1.586E+00 2.707E+00
Saraga 3.00 1.000E+00 1.333E+00 1.949E-04 1.592E-04 1.414E+00 5.774E-01 7.500E-01 1.333E+00 5.657E+00
Km scale
Design A 0.000E+00 1.000E+00 1.000E-08 5.000E-09 2.251E+04 2.251E+04 1.000E+00 2.251E+04 #DIV/ 0!
Design B 10000 1.000E+04 2.000E+00 7.0711E-09 1.000E-08 2.251E+04 1.592E+04 5.000E-01 4.502E+04 4.502E+04
Design C 10000 1.140E+03 2.114E+00 1.000E-08 1.000E-08 1.592E+04 1.592E+04 6.306E-01 2.524E+04 4.308E+04
Saraga 30000 1.000E+04 1.333E+00 1.000E-08 8.165E-09 2.757E+04 1.125E+04 7.500E-01 2.599E+04 1.103E+05
SOLUTION TO 21.24 AND 21.25. In problem 21.8, the transfer function information was computed in
MATLAB as:
% Numerators are each 1. Denominators are the polynomials
d1 = 1.0000e+00 1.0000e+00 1.0000e+00
d2 = 1 1
Further we know from MATLAB that
fcmin = 1.5521e+02
The Saraga design and Design A for d1, the second order section of each filter, are given by the excel
spread sheet below, as well as two alternate designs labeled B and C.
w0^2 w0/ Q Num w0 Q KNLP KMA Kf=wp KmR KMB KMC
1 1.00 1.00 1.0000 1.0000 1.000 41016.58 975.22 10000.00 20508.29 20508.29
KMS
35521.40
RA RB K C1 C2 R1 R2 alpha R3 R4
Design A 0 1 2Q 1/ (2Q) 1 1 KNLP/ K 1/ alpha 1/ (1-alpha)
Design B 1 1 2 1 1/ Q 1 Q KNLP/ K 1/ alpha 1/ (1-alpha)
Design C 1 1-1/ Q 3-1/ Q 1 1 1 1 KNLP/ K 1/ alpha 1/ (1-alpha)
Saraga RA RA/ 3 3-Apr rt(3)Q 1 1/ Q 1/ rt(3) KNLP/ K 1/ alpha 1/ (1-alpha)
RA RB K C1 C2 R1 R2 alpha R3 R4
Design A 0.0000 1.0000 2.0000 0.5000 1.0000 1.0000 1.0000 1.0000 #DIV/ 0!
Design B 1.00 1.0000 2.0000 1.0000 1.0000 1.0000 1.0000 0.5000 2.0000 2.0000
Design C 1.00 0.0000 2.0000 1.0000 1.0000 1.0000 1.0000 0.5000 2.0000 2.0000
Saraga 3.00 1.0000 1.3333 1.7321 1.0000 1.0000 0.5774 0.7500 1.3333 4.0000
w0 scale
Design A 0.0000 1.0000 2.0000 0.5000 1.0000 1.0000 1.0000 1.0000 #DIV/ 0!
Design B 1.00 1.0000 2.0000 1.0000 1.0000 1.0000 1.0000 0.5000 2.0000 2.0000
Design C 1.00 0.1140 2.1140 1.0000 1.0000 1.0000 1.0000 0.5000 2.0000 2.0000
Saraga 3.00 1.0000 1.3333 1.7321 1.0000 1.0000 0.5774 0.7500 1.3333 4.0000
wp scale
Design A 0.000E+00 1.000E+00 2.051E-03 5.127E-04 1.000E+00 1.000E+00 1.000E+00 1.000E+00 #DIV/ 0!
Design B 1.00 1.000E+00 2.000E+00 1.025E-03 1.025E-03 1.000E+00 1.000E+00 5.000E-01 2.000E+00 2.000E+00
Design C 1.00 1.140E-01 2.114E+00 1.025E-03 1.025E-03 1.000E+00 1.000E+00 5.000E-01 2.000E+00 2.000E+00
Saraga 3.00 1.000E+00 1.333E+00 1.776E-03 1.025E-03 1.000E+00 5.774E-01 7.500E-01 1.333E+00 4.000E+00
Km scale
Design A 0.000E+00 1.000E+00 5.000E-08 1.250E-08 4.102E+04 4.102E+04 1.000E+00 4.102E+04 #DIV/ 0!
Design B 10000 1.000E+04 2.000E+00 5.000E-08 5.000E-08 2.051E+04 2.051E+04 5.000E-01 4.102E+04 4.102E+04
Design C 10000 1.140E+03 2.114E+00 5.000E-08 5.000E-08 2.051E+04 2.051E+04 5.000E-01 4.102E+04 4.102E+04
Saraga 30000 1.000E+04 1.333E+00 5.000E-08 2.887E-08 3.552E+04 2.051E+04 7.500E-01 4.736E+04 1.421E+05
The first order (leaky integrator) section is common to both problems. This section consists of an input
resistor (conductance) R
1
(G
1
) connected to the inverting terminal with a parallel R
2
-C combination
feeding back from the output. The transfer function is: H(s) =
G
1
Cs + G
2
. For the normalized design we
set G
1
= G
2
= 1 S (R
1
= R
2
= 1 ) and C = 1 F. This design can be scaled independently of the S&K 2
nd
order section. Hence we set C
new
= 50 nF. Thus K
m
= 20,508.29. Hence R
1
= R
2
= 20,508.29 .
SOLUTION TO 21.26 AND 21.27. The relevent data from the solution of problem 21.10 is:
k = 1
d1 = 1.0000e+00 7.6537e-01 1.0000e+00
d2 = 1.0000e+00 1.8478e+00 1.0000e+00
fcmin = 8.8800e+01
wcmin = 5.5795e+02
In providing the designs, we set forth all the possible S&K designs using two excel spreadsheets, one for
each second order section.
The designs for denominator d1 with numerator equal to 1 are:
w0^2 w0/ Q Num w0 Q KNLP KMA Kf=wp KmR KMB KMC
1 0.7654 1.00000 1.0000 1.3066 1.0000 46834.37 557.95 10000.00 17922.81 17922.81
KMS
40559.76
RA RB K C1 C2 R1 R2 alpha R3 R4
Design A 0 1 2Q 1/ (2Q) 1 1 KNLP/ K 1/ alpha 1/ (1-alpha)
Design B 1 1 2 1 1/ Q 1 Q KNLP/ K 1/ alpha 1/ (1-alpha)
Design C 1 1-1/ Q 3-1/ Q 1 1 1 1 KNLP/ K 1/ alpha 1/ (1-alpha)
Saraga RA RA/ 3 3-Apr rt(3)Q 1 1/ Q 1/ rt(3) KNLP/ K 1/ alpha 1/ (1-alpha)
RA RB K C1 C2 R1 R2 alpha R3 R4
Design A 0.0000 1.0000 2.6131 0.3827 1.0000 1.0000 1.0000 1.0000 #DIV/ 0!
Design B 1.00 1.0000 2.0000 1.0000 0.7654 1.0000 1.3066 0.5000 2.0000 2.0000
Design C 1.00 0.2346 2.2346 1.0000 1.0000 1.0000 1.0000 0.4475 2.2346 1.8100
Saraga 3.00 1.0000 1.3333 2.2630 1.0000 0.7654 0.5774 0.7500 1.3333 3.0615
w0 scale
Design A 0.0000 1.0000 2.6131 0.3827 1.0000 1.0000 1.0000 1.0000 #DIV/ 0!
Design B 1.00 1.0000 2.0000 1.0000 0.7654 1.0000 1.3066 0.5000 2.0000 2.0000
Design C 1.00 0.1140 2.1140 1.0000 1.0000 1.0000 1.0000 0.4475 2.2346 1.8100
Saraga 3.00 1.0000 1.3333 2.2630 1.0000 0.7654 0.5774 0.7500 1.3333 3.0615
wp scale
Design A 0.000E+00 1.000E+00 4.683E-03 6.859E-04 1.000E+00 1.000E+00 1.000E+00 1.000E+00 #DIV/ 0!
Design B 1.00 1.000E+00 2.000E+00 1.792E-03 1.372E-03 1.000E+00 1.307E+00 5.000E-01 2.000E+00 2.000E+00
Design C 1.00 1.140E-01 2.114E+00 1.792E-03 1.792E-03 1.000E+00 1.000E+00 4.475E-01 2.235E+00 1.810E+00
Saraga 3.00 1.000E+00 1.333E+00 4.056E-03 1.792E-03 7.654E-01 5.774E-01 7.500E-01 1.333E+00 3.061E+00
Km scale
Design A 0.000E+00 1.000E+00 1.000E-07 1.464E-08 4.683E+04 4.683E+04 1.000E+00 4.683E+04 #DIV/ 0!
Design B 10000 1.000E+04 2.000E+00 1.000E-07 7.654E-08 1.792E+04 2.342E+04 5.000E-01 3.585E+04 3.585E+04
Design C 10000 1.140E+03 2.114E+00 1.000E-07 1.000E-07 1.792E+04 1.792E+04 4.475E-01 4.005E+04 3.244E+04
Saraga 30000 1.000E+04 1.333E+00 1.000E-07 4.419E-08 3.104E+04 2.342E+04 7.500E-01 5.408E+04 1.242E+05
The designs for denominator d2 with numerator equal to 1 are:
w0^2 w0/ Q Num w0 Q KNLP KMA Kf=wp KmR KMB KMC
1 1.8478 1.00 1.00 0.54 1.00 19399.08 557.95 10000.00 33117.77 17922.81
KMS
17922.81
RA RB K C1 C2 R1 R2 alpha R3 R4
Design A 0 1 2Q 1/ (2Q) 1 1 KNLP/ K 1/ alpha 1/ (1-alpha)
Design B 1 1 2 1 1/ Q 1 Q KNLP/ K 1/ alpha 1/ (1-alpha)
Design C 1 1-1/ Q 3-1/ Q 1 1 1 1 KNLP/ K 1/ alpha 1/ (1-alpha)
Saraga RA RA/ 3 3-Apr rt(3)Q 1 1/ Q 1/ rt(3) KNLP/ K 1/ alpha 1/ (1-alpha)
RA RB K C1 C2 R1 R2 alpha R3 R4
Design A 0.0000 1.0000 1.0824 0.9239 1.0000 1.0000 1.0000 1.0000 #DIV/ 0!
Design B 1.00 1.0000 2.0000 1.0000 1.8478 1.0000 0.5412 0.5000 2.0000 2.0000
Design C 1.00 -0.8478 1.1522 1.0000 1.0000 1.0000 1.0000 0.8679 1.1522 7.5703
Saraga 3.00 1.0000 1.3333 0.9374 1.0000 1.8478 0.5774 0.7500 1.3333 7.3912
w0 scale
Design A 0.0000 1.0000 1.0824 0.9239 1.0000 1.0000 1.0000 1.0000 #DIV/ 0!
Design B 1.00 1.0000 2.0000 1.0000 1.8478 1.0000 0.5412 0.5000 2.0000 2.0000
Design C 1.00 0.1140 2.1140 1.0000 1.0000 1.0000 1.0000 0.8679 1.1522 7.5703
Saraga 3.00 1.0000 1.3333 0.9374 1.0000 1.8478 0.5774 0.7500 1.3333 7.3912
wp scale
Design A 0.000E+00 1.000E+00 1.940E-03 1.656E-03 1.000E+00 1.000E+00 1.000E+00 1.000E+00 #DIV/ 0!
Design B 1.00 1.000E+00 2.000E+00 1.792E-03 3.312E-03 1.000E+00 5.412E-01 5.000E-01 2.000E+00 2.000E+00
Design C 1.00 1.140E-01 2.114E+00 1.792E-03 1.792E-03 1.000E+00 1.000E+00 8.679E-01 1.152E+00 7.570E+00
Saraga 3.00 1.000E+00 1.333E+00 1.680E-03 1.792E-03 1.848E+00 5.774E-01 7.500E-01 1.333E+00 7.391E+00
Km scale
Design A 0.000E+00 1.000E+00 1.000E-07 8.536E-08 1.940E+04 1.940E+04 1.000E+00 1.940E+04 #DIV/ 0!
Design B 10000 1.000E+04 2.000E+00 5.412E-08 1.000E-07 3.312E+04 1.792E+04 5.000E-01 6.624E+04 6.624E+04
Design C 10000 1.140E+03 2.114E+00 1.000E-07 1.000E-07 1.792E+04 1.792E+04 8.679E-01 2.065E+04 1.357E+05
Saraga 30000 1.000E+04 1.333E+00 9.374E-08 1.000E-07 3.312E+04 1.035E+04 7.500E-01 2.390E+04 1.325E+05
SOLUTION TO 21.28. For this problem we use the excel spread sheet given below. First we observe that
H(s) =

K
s
2
+
0
Q
s +
0
2
H(
0
s) =

K
0
s ( )
2
+
0
Q
0
s ( ) +
0
2
=

K
0
2
(= K
NLP
)
s
2
+
1
Q
s +1
Thus after this type of frequency scaling, the new transfer function is:
H
new
(s) =
0.7943
s
2
+
1
1.1286
s +1
The dc gain is of course 0.7943 and the modification of the circuit to achieve the correct dc gain is given
in the spread sheet below via R
3
and R
4
which constitute a voltage divider that replaces R
1
.
w0^2 w0/ Q Num w0 Q KNLP KMA Kf=wp KmR KMB KMC
0.82306 0.8038 0.65378 0.9072 1.1286 0.7943 1131.42 43982.40 10000.00 501.23 501.23
KMS
979.83
RA RB K C1 C2 R1 R2 alpha R3 R4
Design A 0 1 2Q 1/ (2Q) 1 1 KNLP/ K 1/ alpha 1/ (1-alpha)
Design B 1 1 2 1 1/ Q 1 Q KNLP/ K 1/ alpha 1/ (1-alpha)
Design C 1 1-1/ Q 3-1/ Q 1 1 1 1 KNLP/ K 1/ alpha 1/ (1-alpha)
Saraga RA RA/ 3 3-Apr rt(3)Q 1 1/ Q 1/ rt(3) KNLP/ K 1/ alpha 1/ (1-alpha)
RA RB K C1 C2 R1 R2 alpha R3 R4
Design A 0.0000 1.0000 2.2573 0.4430 1.0000 1.0000 0.7943 1.2589 4.8621
Design B 1.00 1.0000 2.0000 1.0000 0.8860 1.0000 1.1286 0.3972 2.5179 1.6588
Design C 1.00 0.1140 2.1140 1.0000 1.0000 1.0000 1.0000 0.3758 2.6613 1.6019
Saraga 3.00 1.0000 1.3333 1.9549 1.0000 0.8860 0.5774 0.5957 1.6786 2.1917
w0 scale
Design A 0.0000 1.0000 2.4881 0.4883 1.0000 1.0000 0.7943 1.2589 4.8621
Design B 1.00 1.0000 2.0000 1.1023 0.9766 1.0000 1.1286 0.3972 2.5179 1.6588
Design C 1.00 0.1140 2.1140 1.1023 1.1023 1.0000 1.0000 0.3758 2.6613 1.6019
Saraga 3.00 1.0000 1.3333 2.1548 1.1023 0.8860 0.5774 0.5957 1.6786 2.1917
wp scale
Design A 0.000E+00 1.000E+00 5.657E-05 1.110E-05 1.000E+00 1.000E+00 7.943E-01 1.259E+00 4.862E+00
Design B 1.00 1.000E+00 2.000E+00 2.506E-05 2.220E-05 1.000E+00 1.129E+00 3.972E-01 2.518E+00 1.659E+00
Design C 1.00 1.140E-01 2.114E+00 2.506E-05 2.506E-05 1.000E+00 1.000E+00 3.758E-01 2.661E+00 1.602E+00
Saraga 3.00 1.000E+00 1.333E+00 4.899E-05 2.506E-05 8.860E-01 5.774E-01 5.957E-01 1.679E+00 2.192E+00
Km scale
Design A 0.000E+00 1.000E+00 5.000E-08 9.813E-09 1.131E+03 1.131E+03 7.943E-01 1.424E+03 5.501E+03
Design B 10000 1.000E+04 2.000E+00 5.000E-08 4.430E-08 5.012E+02 5.657E+02 3.972E-01 1.262E+03 8.315E+02
Design C 10000 1.140E+03 2.114E+00 5.000E-08 5.000E-08 5.012E+02 5.012E+02 3.758E-01 1.334E+03 8.029E+02
Saraga 30000 1.000E+04 1.333E+00 5.000E-08 2.558E-08 8.682E+02 5.657E+02 5.957E-01 1.645E+03 2.148E+03
SOLUTION TO 21.29. (a) H
HP
(s) = H
NLP
c
s




=
1
c
s




2
+ 2
c
s




+1
. At s = j
p
,
H
HP
( j
p
) = H
NLP
c
j
p






=
1
c
j
p






2
+ 2
c
j
p






+1
=
1

5.5
7




2
j 2
5.5
7




+1
Thus in MATLAB,
Magfp = 1/abs(1 - (5.5/7)^2 -j*sqrt(2)*(5.5/7))
Magfp = 8.5091e-01
Attenfp = -20*log10(Magfp)
Attenfp = 1.4023e+00
Magfs = 1/abs(1 - (5.5/1)^2 -j*sqrt(2)*(5.5/1))
Magfs = 3.3040e-02
Attenfs = -20*log10(Magfs)
Attenfs = 2.9619e+01
Thus the attenuation at f
p
is 1.4023 dB and that at f
s
is 29.619 dB.
(b) From problem 21.15, the transfer function is
H
cir
(s) =
1 LC
s
2
+
1
C
+
1
L




s +
2
LC
and the values of L and C realizing the 2
nd
order Butterworth NLP transfer function can be computed
according to
1
C
+
1
L
= 2,
2
LC
=1
1
C
+
C
2
= 2 C
2
2 2C + 2 = 0
(C 2)
2
= 0 C = 2 F L = 2 H
(c) Here K
m
= 1000. Ls
L
C
s

1
1
LK
m C
s
and
1
Cs

s
C
C

K
m
C
C
s. Thus in
MATLAB,
wc = 2*pi*5.5e3
wc = 3.4558e+04
Km = 1000;
C = sqrt(2); L = sqrt(2);
Lhp = Km/(C*wc)
Lhp = 2.0462e-02
Chp = 1/(Km*wc*L)
Chp = 2.0462e-08
Therefore, the resistors take on values of 1 k, the inductor is changed to a capacitor of value of C
hp
=
20.46 nF and the capacitor is changed to an inductor of value L
hp
= 20.46 mH.
SOLUTION TO 21.30. (a)
=
p
, A
max
= 2 dB, =
max
= 10
0.12
1 = 0.76478,
c
=
1
6
10
0.12
1
= 1.0935
Wc=1/(10^0.2-1)^(1/6)
Wc = 1.0935e+00
(b)
wchp = 2*pi*5e3/Wc
wchp = 2.8730e+04
fchp = wchp/(2*pi)
fchp = 4.5725e+03
Thus
chp
=
p

c
= 28.73 krad/s.
( c ) (i) NLP HP transformation: scale by w
chp
.
C
hp
=
1
L
lp chp
=17. 404 F, L
hp
=
1
C
lp chp
= 34.807 H
(ii) Magnitude scale to obtain proper value of capacitors.
Clp = 1;
Llp = 2;
Chp = 1/(Llp*wchp)
Chp = 1.7404e-05
Lhp = 1/(Clp*wchp)
Lhp = 3.4807e-05
Cnewhp = 100e-9;
Km = Chp/Cnewhp
Km = 1.7404e+02
Lnewhp = Km*Lhp
Lnewhp = 6.0578e-03
K
m
=174.02, C
hp,new
=
C
hp
K
m
= 100 nF, L
hp,new
= K
m
L
hp
= 6.06 mH
SOLUTION TO 21.31.
The 2
nd
order NLP Butterworth transfer function is: H
NLP
(s) =
1
s
2
+ 2s +1
. Using the transformation
s to 1/s, we obtain the NHP Butterworth transfer function:
H
NHP
(s) = H
NLP
1
s




=
s
2
s
2
+ 2s +1
= K

s
2
s
2
+ d(1) s + d(2)
INPUT: d(1) d(2) K K Kf Km KmR
1.414213562 1 1.33334 1 18849.6 12247.44871 30000
NHP Params w0 Q alpha
1 0.70710678 0.74999625
NHP Crt Pars C1 = Q C2 = rt(3) R1 = 1 R2 =1/ (rt(3)Q) R R/ 3
0.707106781 1.73205081 1 0.816496581 1 0.333333333
HP Crt Params C1new=C1/ (Km*Kf) C2new R1new=R1*Km R2new=R2*Km Rnew=R*KmR Rnew/ 3
3.06293E-09 7.5026E-09 12247.44871 10000 30000 10000
A plot of the design without input attenuation is shown below. Notice that as predicted the gain is 4/3.
R1
30k
XOpAmp
Vpls
15
Vminus
15
Vin C0
7.503n
C
3.063n
R0
12.247k
R
10k
IVm
R2
10k
MAG(V(IVM))
Frequency (Hz) S&K HP-Small Signal AC-3
(V)
+0.000e+000
+500.000m
+1.000
+100.000 +316.228 +1.000k +3.162k +10.000k
Input attenuation requires that we replace C1 with a series combination of capacitors in which C1 = C3
+ C4 and (1/C3)/(1/C4 + 1/C3) = alpha. Here then, C1 = C3 + C4 and alpha = C4/(C3 + C4) = C4/C1.
Thus C4 = alpha*C1 and C3 = (1 alpha)*C1. Thus
Input
Attenuation
C3 = (1 - alpha)C1 C4 = alpha*C1
7.65744E-10 2.29719E-09
SOLUTION TO 21.32. The fourth order Butterworth NLP transfer function can be obtained from tables or
from MATLAB as follows:
[z,p,k] = buttap(4)
z =
[]
p =
-3.8268e-01 + 9.2388e-01i
-3.8268e-01 - 9.2388e-01i
-9.2388e-01 + 3.8268e-01i
-9.2388e-01 - 3.8268e-01i
k =
1
% Second Order Sections
n1 = 1;
d1 = poly([p(1),conj(p(1))])
d1 =
1.0000e+00 7.6537e-01 1.0000e+00
n2 = 1;
d2 = poly([p(3),conj(p(3))])
d2 =
1.0000e+00 1.8478e+00 1.0000e+00

Thus,
H
NLP
(s) =
1
s
2
+ 0.76537s +1

1
s
2
+1.8478s +1
having frequency response
w = 0:0.01:3.5;
h = freqs(k*poly(z),poly(p),w);
plot(w,abs(h))
grid
xlabel('Normalized Frequency')
ylabel('Magnitude 4th Order Butterworth')
0 0.5 1 1.5 2 2.5 3 3.5
0
0.1
0.2
0.3
0.4
0.5
0.6
0.7
0.8
0.9
1
Normalized Frequency
M
a
g
n
i
t
u
d
e

4
t
h

O
r
d
e
r

B
u
t
t
e
r
w
o
r
t
h
TextEnd
The Saraga design parameters are given in the following Excel tables:
INPUT: d(1) d(2) KNLP/ NHP w0LP/ HP Q
NLP H(s) 0.76537 1 1 1 1.306557613
NHP H(s) 0.76537 1 1 1 1.306557613
K Kf Km KmR alpha
1.33334 18849.6 22630.24168 60000 0.74999625
NHP Crt Pars C1 = Q C2 = rt(3) R1 = 1 R2 =1/ (rt(3)Q) R R/ 3
1.306557613 1.732050808 1 0.441886576 1 0.333333333
Scale by w0HP 1.306557613 1.732050808 1 0.441886576 1 0.333333333
HP Crt Params C1new=C1/ (Km*Kf) C2new R1new=R1*Km R2new=R2*Km Rnew=R*KmR Rnew/ 3
3.06293E-09 4.0604E-09 22630.24168 10000 60000 20000
Input Attenuation C3 = (1 - alpha)C1 C4 = alpha*C1
7.65744E-10 2.29719E-09
INPUT: d(1) d(2) KNLP/ NHP w0LP/ HP Q
NLP H(s) 1.8478 1 1 1 0.541184111
NHP H(s) 1.8478 1 1 1 0.541184111
K Kf Km KmR alpha
1.33334 18849.6 10000 30000 0.74999625
NHP Crt Pars C1 = Q C2 = rt(3) R1 = 1 R2 =1/ (rt(3)Q) R R/ 3
0.541184111 1.732050808 1 1.066827827 1 0.333333333
Scale by w0HP 0.541184111 1.732050808 1 1.066827827 1 0.333333333
HP Crt Params C1new=C1/ (Km*Kf) C2new R1new=R1*Km R2new=R2*Km Rnew=R*KmR Rnew/ 3
2.87106E-09 9.18879E-09 10000 10668.27827 30000 10000
Input Attenuation C3 = (1 - alpha)C1 C4 = alpha*C1
7.17777E-10 2.15329E-09
SOLUTION TO 21.33. Using MATLAB,
fp = 5e3; fs = 1.5e3;
wp = 2*pi*fp; ws = 2*pi*fs;
Amax = 3; Amin = 40;
n = buttord(wp,ws,Amax,Amin,'s')
n =4
[z,p,k] = buttap(n)
z =[]
p =
-3.8268e-01 + 9.2388e-01i
-3.8268e-01 - 9.2388e-01i
-9.2388e-01 + 3.8268e-01i
-9.2388e-01 - 3.8268e-01i
k = 1
d1 = real(poly([p(1),p(2)]))
d1 =
1.0000e+00 7.6537e-01 1.0000e+00
d2 = real(poly([p(3),p(4)]))
d2 =
1.0000e+00 1.8478e+00 1.0000e+00
In general,
H
NHP
(s) = H
NLP
1
s




=
1
1
s




2
+
0LP
Q
1
s




+ (
0LP
)
2
=
1
(
0LP
)
2

s
2
s
2
+
1
0LP
( )
Q
s + 1
0LP
( )
2
=
(
0HP
)
2
s
2
s
2
+
0HP
Q
s + (
0HP
)
2
The S&K Saraga design for d1 is given by the following excel spreadsheet:
INPUT: d(1) d(2) KNLP/ NHP w0LP/ HP Q
NLP H(s) 0.76537 1 1 1 1.306557613
NHP H(s) 0.76537 1 1 1 1.306557613
K Kf Km KmR alpha
1.33334 31416 45260.48336 60000 0.74999625
NHP Crt Pars C1 = Q C2 = rt(3) R1 = 1 R2 =1/ (rt(3)Q) R R/ 3
1.306557613 1.732050808 1 0.441886576 1 0.333333333
Scale by w0HP 1.306557613 1.732050808 1 0.441886576 1 0.333333333
HP Crt Params C1new=C1/ (Km*Kf) C2new R1new=R1*Km R2new=R2*Km Rnew=R*KmR Rnew/ 3
9.18879E-10 1.21812E-09 45260.48336 20000 60000 20000
Input Attenuation C3 = (1 - alpha)C1 C4 = alpha*C1
2.29723E-10 6.89156E-10
The S&K Saraga design for d2 is given by the following excel spreadsheet:
INPUT: d(1) d(2) KNLP/ NHP w0LP/ HP Q
NLP H(s) 1.8478 1 1 1 0.541184111
NHP H(s) 1.8478 1 1 1 0.541184111
K Kf Km KmR alpha
1.33334 31416 20000 60000 0.74999625
NHP Crt Pars C1 = Q C2 = rt(3) R1 = 1 R2 =1/ (rt(3)Q) R R/ 3
0.541184111 1.732050808 1 1.066827827 1 0.333333333
Scale by w0HP 0.541184111 1.732050808 1 1.066827827 1 0.333333333
HP Crt Params C1new=C1/ (Km*Kf) C2new R1new=R1*Km R2new=R2*Km Rnew=R*KmR Rnew/ 3
8.61319E-10 2.75664E-09 20000 21336.55655 60000 20000
Input Attenuation C3 = (1 - alpha)C1 C4 = alpha*C1
2.15333E-10 6.45986E-10
This completes the design.
SOLUTION TO 21.34. For the woofer,
H(s) =
8
Ls + 8
=
8
L
s +
8
L
Thus,
8
L
= 2000 2 L = 636 H.
For the tweeter,
H(s) =
8
1
Cs
+ 8
=
8Cs
8Cs +1
=
s
s +
1
8C
Thus,
1
8C
= 2000 2 C = 9.95 F.
SOLUTION TO 21.35. For the woofer,
H
NLP
(s) =
1
s
2
+ 2s +1

The transfer function of the following circuit
is
H(s) =
1
LC
s
2
+
1
C
s +
1
LC
Thus 2 =
1
C
C = 0.70711 F and since
1
LC
= 1, L = 2 = 1.4142 H. Frequency scaling the
element values by K
f
= 4000 and magnitude scaling by K
m
= 8 yields C =
0.70711
K
m
K
f
= 7.0337 F and
L =
1.4142K
m
K
f
= 0.90032 mH:
For the tweeter we first realize the NLP Butterworth transfer function as above to obtain as
above
with Thus C = 0.70711 F and L = 2 = 1.4142 H. We now apply the frequency transformation
s
C
s
to each element (capacitors become inductors and inductors become capacitors according to
figure 21.24) and we obtain the HP circuit topology
where C
new
= 7.0337 F and L
new
= 0.90032 mH.
SOLUTION TO 21.36. Consider figure (a). Let the current entering the RC network from Z
1
be dentoted
by I
fa
. Let the voltage from this point to ground be denoted V
fa
. Then
V
out,a
= V
fa
+ I
fa
Z
1
= H
a
(s)V
in
For figure (b) with a similar denotation of voltage and current, we have
V
out,b
= V
fb
+ I
fb
+
V
fb
Z
1
k 1










Z
1
k
= V
fb
+ I
fb
Z
1
k
+
k 1
k
V
fb
=
1
k
(2k 1)V
fb
+
I
fb
Z
1






If
(2k 1)V
fb
V
fa
and I
fb
I
fa
(**)
then
V
out,b

1
k
H
a
(s)V
in
=
V
out ,a
k
For gain enhancement, k < 1. However, for the (**) to be valid, we require that
Z
1
k 1
be large relative
to what it sees in the RC network. Hence, in general, k must be close to 1. Thus only small gain
enhancements are possible. For such a potentially sensitive approach to gain enhancement, it might be
better simply to add another op amp stage as op amps are comparatively inexpensive.
CHAPTER 22 PROBLEM SOLUTIONS
SOLUTION TO PROBLEM 22.1
(a) For figure P22.1a, T
0
= 2 and
0
= . Let t
0
= -1 in equation 22.5b. Then f(t) = (t) and
c
n
0.5 (t)e
jnt
dt
1
1

0.5 for all n


From equation 22.6, a
n
= 1 and b
n
= 0 for all n. Finally from equation 22.2
f (t) 0.5 + cos(nt)
n1

(b) For figure P22.1b, T


0
= 2 and
0
= . Let t
0
= 0 in equation 22.5b. Then f(t) = - (t -1) and
c
n
0.5 (t 1)e
jnt
dt
1
1

0.5e
jn
From equation 22.6, b
n
= 0 for all n, and
a
n
= -1 for n even
a
n
= 1 for n odd
Finally from equation 22.2

f (t) 0.5 + cos(t) cos(2t) + cos(3t) cos(4t) +K
SOLUTION TO PROBLEM 22.2
(a) T
0
= 1 and
0
= 2 . Let t
0
= 0 in equation 22.5b. Then

f (t) e
ln(2) ( )t
and from equation 22.5b

c
n
e
ln(2) ( )t
e
j 2nt
dt
0
1

e
ln(2)+ j 2n ( )t
dt
0
1

1
ln(2) + j2n ( )
e
ln(2)+ j 2n ( )
1
[ ]

0.5
ln(2) + j2n ( )
(b) Using the above result for c
n
c
0
=
0.5
ln2
= 0.7213,
c
1
=
0.5
ln2+ j2
= 0.7213e
-j1.4609
c
2
=
0.5
ln2+ j4
= 0.397e
-j1.516
From equation 22.6
d
0
= c
0
= 0.7213
d
1
=2c
1
= 0.158,

1
=-1.461x180/ =-83.7
o
d
2
=2c
2
= 0.0795

2
=-1.516x180/ =-86.84
o
Thus, f(t) in the form of equation 22.3 is
f(t) = 0.7213 + 0.158cos(2t - 83.7
o
) + 0.0795os(2t - 86.84
o
)
SOLUTION TO PROBLEM 22.3.
(a) T
0
= 1 and
0
= 2 . Let t
0
= 0 in equation 22.5b. Then

f (t) e
ln(2) ( )t
u(t) u(t 0.5) [ ]
and from equation 22.5b

c
n
e
ln(2) ( )t
e
j 2nt
dt
0
0.5

e
ln(2)+ j 2n ( )t
dt
0
0.5

1
ln(2) + j2n ( )
e
0.5 ln(2)+ j 2n ( )
1
[ ]

1
(1)
n
2
ln(2) + j2n ( )
(b) Using the above result for c
n
, and MATLAB to evaluate the numerical result,
n= 0;
c0= (1- (-1)^n/sqrt(2))/(log(2) + j*2*n*pi
c0
4.2256e-01
n=1;
c1= (1- (-1)^n/sqrt(2))/(log(2) + j*2*n*pi)
c1 =
2.9612e-02- 2.6843e-01i
abs(c1)
ans =
2.7006e-01
degreec1=angle(c1)*180/pi
degreec1 =
-8.3705e+01
n=2;
c2= (1- (-1)^n/sqrt(2))/(log(2) + j*2*n*pi)
c2 =
1.2817e-03- 2.3237e-02i
abs(c2)
ans =
2.3272e-02
degreec2= angle(c2)*180/pi
degreec2 =
-8.6843e+01
From equation 22.6 and equation 22.3
f(t) = 0.4226 + 0.54cos(2t - 83.7
o
) + 0.04654os(2t - 86.84
o
)
SOLUTION PROBLEM 22.4. (a) f(t) = cos(4t) sin(2t) = 0.5[ sin(6t) - sin (2t)] .
The fundamental angular frequency of f(t) is
0
= 2 rad/s. The given f(t) can be expressed as
f(t) = -0.5 sin(
0
t) + 0.5sin(3
0
t) . Observe that b
1
= -0.5, b
3
= 0.5 and all other a
i
and b
i
are zero.
From equation 22.4 , d
1
= 0.5 /-90
o
and d
3
= 0.5 /90
o.
From equations 22.6a and 22.6b.
c
1
= 0.25j and c
3
= -0.25j. All other c
n
are zero for n positive
(b) f(t) = sin
2
(4t) cos
2
(8t)= 0.5[1 - cos(8t)]x0.5 1+ cos(16t)
= 0.25 [ 1 - cos(8t) + cos(16t) - cos(8t) cos(16t)]
= 0.25 - 0.375 cos(8t) + 0.25cos(16t) - 0.125cos(24t)
The fundamental angular frequency of f(t) is
0
= 8 rad/s. The given f(t) can be expressed as
f(t) = 0.25 -0.375 cos(
0
t) + 0.25cos(2
0
t) - 0.125sicos(3
0
t). Observe that a0= 0.5, a
1
=-0.375, a
2
=
0.25 , a
3
= -0.125 and all other a
i
and b
i
are zero.
Fom equation 22.4,
d
0
= 0.25, d
1
= 0.375 /180
o
, d
2
= 0.25 /0
o
, and d
3
= 0.125 /180
o
,
From equations 22.6a and 22.6b.
c
0
= 0.25, c
1
= -0.375 and c
2
= 0.25, c3 = -0.125. All other c
n
are zero for n positive.
(c)
f(t) = 2 + 1.5 sin(500t)- 2cos(2000t)]cos(10
6
t)
=2cos(10
6
t) + 0.75sin(1000500t) + 0.75sin(999500t)
- cos(1002000t) - cos(998000t)
The fundamental angular frequency of f(t) is
0
= 500 rad/s. The given f(t) can be expressed as
f(t) =2cos( 2000
0
t) + 0.75sin(2001
0
t) + 0.75sin(1999
0
t) - cos(2004
0
t) - cos(1996
0
t)
Observe that: a
1996
= -1, b
1999
= 0.75, a
2000
= 2, b
2001
= 0.75 , a
2004
= -1, and all other a
i
and b
i
are
zero. From equation 22.4 , d
1996
= 1 /180
o
, d
1999
= 0.75 /-90
o
, d
2000
= 2 /0
o
,
d
2001
= 0.75/-90
o
, d
2004
= 1 /180
o
, and all other d
i
are zero.
From equations 22.6a and 22.6b.
c
1996
= -0.5

, c
1999
= -j0.375, c
2000
= 1 ,
c
2001
= -j0.375
o
, dc
2004
=-0.5

, and all other c
n
are zero for n positive..
SOLUTION PROBLEM 22.5
By inspesction, the derivative of f(t) is
f '( t)
A
T
A (t nT)
n

A
T
f (t)
where f

(t) is shown in figure 22/7, with its Fourier series given by equation 22.20b, i.e.
f (t)
A
T
+
2A
T
cos(n
0
t)
n1

Therefore
f '( t)
2A
T
cos(n
0
t)
n1

The dc component is the average value of f(t) and is given by 0.5A.


Other terms in the Fourier series of
f(t) are obtained by integrating the cosine terms in the above expression. The result is
f (t) 0.5A
A
n
sin(n
0
t)
n1

SOLUTION TO PROBLEM 22.6


Denote by f
5
(t) the waveform of figure P22.5, with A= 0.5 and T =1. Then by inspection
f (t) f
prob5
(t) + 0.5
Substituting the result of problem 22.5 into the above equation, we have
f (t) 0.75 +
0.5
n
sin(n
0
t)
n1

SOLUTION PROBLEM 22.7


Consider the square wave g(t) shown in figure 22.4 with its Fourier series given by equation 22.13. By
inspection, the derivative of f(t) is
f '( t)
4
T
g(t) f'(t) = -
4
T
g(t)
Substituting equation 22.13 into the above expression, we have
f '( t)
8A
T
sin(n
0
t)
n
n1,odd

The dc component is the average value of f(t) and is given by 0.5A.


Other terms in the Fourier series of
f(t) are obtained by integrating the sine terms in the above expression. The result is
f (t) 0.5A +
4A

2
cos(n
0
t)
n
2
n1,odd

SOLUTION PROBLEM 22.8


The method used below is simpler than that suggested in the hint.
We first sketch the waveform of f'(t) and observe that it may be expressed as the sum of two periodic rectangular
pulse trains:
f'(t)=
1
T
[ f
p
(t + 0.5T) +f
p
(t - 0.5T)]
where f
p
(t) is sketched in figure 22.8. Using equation 22.23, we have
f'(t)=
1
T
{ [A +
2Asin(n)
n

n=1

cos(n
0
(t+0.5T))]
-[A +
2Asin(n)
n

n=1

cos(n
0
(t-0.5T))]}
=
1
T
{
2Asin(n)
n
[

n=1

cos(n
0
(t+0.5T))-cos(n
0
(t-0.5T)]
\
=
1
T
[
- 4Asin
2
(n)
n
sin(n
0
t)]

n=1


The dc component is the average value of f(t) and is given by A.
Other terms in the Fourier series of
f(t) are obtained by integrating the sine terms in the above expression. The result is
f(t) = A +
2A

2
{ [
sin(n)
n
]
2
cos(n
0
t)}

n=1


SOLUTION TO PROBLEM 22.9
We first sketch the waveform of f'(t) and observe that it may be expressed as the sum of a periodic rectangular
pulse train and a periodic impulse train:
f'(t)=
1
T
f
p
(t + 0.5T) - f

(t )
where f
p
(t) is sketched in figure 22.8, and f

(t) in figure 22.7.


Using equations 22.23 and 22.20b, we have
f'(t)=
1
T
[A +
2Asin(n)
n

n=1

cos(n
0
(t+0.5T))
- [
A
T
+
2A
T
n=1

cos(n
0
t)]
=
2A
T
{
sin(n)
n
[
n=1

cos(n
0
(t+0.5T)) -cos(n
0
t)]}
The dc component is the average value of f(t) and is given by 0.5A.
Other terms in the Fourier series
of f(t) are obtained by integrating the sine terms in the above expression. The result is
f(t) =0.5A +
A
n
{ [
sin(n)
n

n=1

sin(n
0
(t+0.5T)) -sin(n
0
t)]}
It remains to rewrite the expression in the form of equation 22.2.
To this end, let b = sin(n)/(n) and re-write the terms within [ ] as follows:
bsin(n)cos(n
0
t) + {bcos(n) - 1}sin(n
0
t)
Hence, f(t) in the form of equation 22.2 has the coefficients, for n=1,2...
a
n
=
A

2
n
2
sin
2
(n)
b
n
=
A

2
n
2
{sin(n)cos(n) -n}
d
n
= a
n
2
+ b
n
2
=
A

2
n
2
sin
4
(n) + {sin(n)cos(n) -n}
2

=
A

2
n
2
sin
2
(n) +(n)[n - sin(2n)]

n
= tan
-1
(
-b
n
a
n
)
The result is item 6 of table 22.4
SOLUTION PROBLEM 22.10.
Observe that the present f(t) can be derived from that of problem 22.9 by (a) replacing t by -t; and (b)
replacing by . Thus the Fourier series for f(t) is:
f(t) =0.5A + a
n
cos(n
0
t) + b
n [
n=1

sin(n
0
t) ]
where
a
n
=
A

2
n
2
sin
2
(n)
b
n
=
-A

2
n
2
{sin(n)cos(n) -n}
d
n
= a
n
2
+ b
n
2
=
A

2
n
2
sin
4
(n) + {sin(n)cos(n) -n}
2

=
A

2
n
2
sin
2
(n) +(n)[n - sin(2n)]

n
= tan
-1
(
-b
n
a
n
)
SOLUTION PROBLEM 22.11. Following the hint, we have the second derivative of f(t) given by
f ''( t)
1
(1 )T
f (t + T)
1
(1 )T
f (t)
where fd(t) is given in example 22.5. Notice that we have focused on the part of the waveform over
[T, (1)T]. By making use of equation 22.20b, we obtain
f ''( t)
2A
(1 )T
2
cos(n
0
t + 2n ) cos(n
0
t) [ ]
n1

4A
(1 )T
2
sin(n
0
t + n )sin(n ) [ ]
n1

Therefore,
f (t) f (t) [ ]
ave
+
4A
(1 )T
2
sin(n )
n
0
( )
2
sin(n
0
t + n )




1
]
1
1
n1

A
2
+
Asin(n )
n
2

2
(1 )
cos n
0
t + (n 0.5) ( )




1
]
1
1
n1

Letting T = 1 and = 0.25 we obtain,


f (t)
A
2
+
16Asin n

4


_
,
3n
2

2
cos n2t + (0.25n 0.5) ( )






1
]
1
1
1
1
n1

Therefore, d
0
0.5A, and
d
n

16Asin n

4


_
,
3n
2

2
.
It follows that d
1
= 0.38211A and d
2
= 0.13509A.
SOLUTION TO PROBLEM 22.12.
Denote by f
p(
t, ) the period rectangular waveform of figure 22.5, with A=1.
Then we can express the present f(t), with T= 4, as the sum of 3 terms:
f(t) = 3f
p
(t-0.125T, 0.25) + 4 f
p
(t - 0.5T, 0.5) -2
From equation 22.14b, and equation 22.12c, for n= 1,2,...
c
n
=
3
n
sin(0.25n)e
-jn0x0.125T
+
4
n
sin(0.5n)e
-jn0x0.5T

=
3
n
sin(0.25n)e
-j0.25n
+
4
n
sin(0.5n)e
-jn

The numerical values of the first few Fourier series coefficients are:
c
0
= averge value of f(t) =
1
4
(1 + 4 - 2) = 0.75
c
1
=
3

sin(0.25)e
-j0.25
+
4

sin(0.5)e
-j
= -0.7958 -j 0.4775
c
2
=
3
2
sin(0.5)e
-j0.5
+
4
2
sin()e
-j2
=-j0.4775
Solution Problem 22.13
(a) For sinusoidal steady analysis, the transfer function is
H(j) =
Y
L

Y
L
+Y
C
+ Y
R
=
1
1 +Z
L
Y
C
+ Z
L
Y
R
=
1
(1 -
2
LC) + j
L
R
=
1
(1 - 410
-5

2
) + j10
--3

The transfer function evaluated at various input frequencies are listed below.
H(0) = 1
H(j377) = 0.2128 /-175.4
o
H(j3x377) = 0.0199/-178.7
o
H(j5x377) = 0.0071/-179.2
o
Using equation 15.7 and superposition, we obtain the steady state output voltage (in V):
v
out
(t) = 200 + 200 20.2128cos (377t -175.4
o
) +60 20.0199cos (3377t +30
o
-178.7
o
)
+80 20.0071cos (5377t +50
o
-17.2
o
)
= 200 + 242.55cos (377t -175.4
o
) + 21.196cos (3377t -148.7
o
)
+ 20.5668cos (5377t -129.2
o
)
(b) From equation derived in P11.39,
V
out,eff
= 200
2
+42.55
2
+ 1.196
2
+0.5668
2
= 204.48 V
The average power absorbed by the 10 k resistor is
P
av
=
204.48
2
10
4
= 4.1812 W
Solution Problem 22.14
One correction in the problem statement: in the angle expression, 5000 should be 10000.
(a) Using the identity cos(x)cos(y)= 0.5cos(x+y) + 0.5cos(x-y), we have
v
in
(t) = 0.1cos (998,000t) +0.2cos (999,000t) + 2cos (1,000,000t)

+ 0.2cos (1,001,000t) + 0.1cos (1,002,000t) V
(b) The transfer function has a constant magnitude of 10, and a phase shift proportional to the deviation
from
c
. at =
c
+ 2
m
, the phase shift is -9 degrees. From these facts, we can write directly
v
out
(t) = cos (998,000t +9
o
) +2cos (999,000t + 4.5
o
) + 20cos (1,000,000t)
+ 2cos (1,001,000 - 4. 5
o
) + cos (1,002,000t - 9
o
) V
(c) Using the identity
cos(x)cos(y) = 2 cos(
x+y
2
)cos(
x+y
2
)
we can group the terms in v
out
(t) and re-write it as
v
out
(t) = 2 cos( 2
m
t - 9
o
)cos(
c
t) +4cos(
m
t - 4.5
o
)cos(
c
t) + 20cos(
c
t)
= [20 + 4cos(
m
t - 4.5
o
) +2 cos(2
m
- 9
o
)]cos(
c
t) = g(t)cos(
c
t)
Thus

g(t) = 20 + 4cos(
m
t - 4.5
o
) +2cos(2
m
t - 9
0
)
With t
d
= 78.54 s, then
m
t
d
= 1000 x 78.54x10
-6
= 0.07854 rad, or 4.5
degrees. We have
10 f(t - t
d
) = 20[1 + 0.2cos(
m
(t - td) +0.1cos( 2
m
(t - td)]
= 20[1 + 0.2cos(
m
t - 4.5
o
) + 0.1 cos( 2
m
t - 9
o
)]

= g(t)
SOLUTION PROBLEM 22.15. (a) This proof is a special case of the general proof given in the
solution to Problem 22.16. See the solution to problem 22.16, below.
FOR THE REMAINING PARTS, WE USE THE FOLLOWING MATLAB CODE:
% chapter 22, problem 15.
%part (b).
Vmax= 30*pi;
Vmin=0;
T=4;R = 1; C = 1;
voutmin= Vmin+(Vmax - Vmin)/(1+ exp(0.5*T/(R*C)))
voutmax = Vmax - (Vmax -Vmin)/(1+ exp(0.5*T/(R*C)))
t1= 0: 0.05: 2;
vseg1= Vmax+(voutmin- Vmax)*exp((-t1/(R*C)));
t2= 2:0.05: 4;
vseg2= Vmin+(voutmax- Vmin)*exp(-(t2-T/2)/(R*C));
t= [ t1 t2];
v= [vseg1 vseg2];
plot(t,v)
grid
xlabel('t in seconds')
ylabel('Steady State Output Voltage in V')
%part (c)
error1= 100*(12.235- voutmin)/voutmin
error2= 100*(82.013 - voutmax)/voutmax
TO OBTAIN
(b)
voutmin =
1.1235e+01
voutmax =
8.3013e+01
(c)
error1 =
8.9045e+00
error2 =
-1.2048e+00
SOLUTION PROBLEM 22.16. CORRECTION: In the problem statement,
s
should be s.
For simplicity, let us consider the case when the transfer function is a voltage ratio, i.e., H(s) = V
out
./V
in
.
If a constant input v
in
(t) = V
con
is applied to the stable network, then the v
out
() = KV
con
,
independent of the initial conditions. This is because the zero-input response for a stable network
approaches zero as t approaches infinite. To see this observe that the zero-state response is given by
v
out
(t) L
1
K
s +1

V
con
s

'


)

L
1
KV
con
s

KV
con
s +1

'


)

KV
con
1 e
t
( )
u(t)
from which v
out
() = KV
con
as asserted.
For the remainder of our proof we make use of the fact that in steady state, v
out
(t+T) = v
out
(t)
with t = 0 in our case. Specifically, after the first order network has reached steady state, the v
out
(t)
waveform will be periodic as shown in figure P22.15c, where the time reference has been chosen so that
v
out,min
occurs at t = 0.
Recall equation 8.19
x(t) x() + x(t
0
+
) x()
[ ]
e
(t t
0
)
Applying this equation to the interval [0, T/2], we have
b
0
KV
max
+ a
0
KV
max
( )e
0.5T /
KV
max
+ a
0
KV
max
( ) (1)
Note that e
0.5T /
. Similarly, applying equation 8.19 to the interval [ T/2, T] leads to
a
0
KV
min
+ b
0
KV
mn
( ) (2)
Equations (1) and (2) can be written as a single matrix equation
1
1



1
]
1
a
0
b
0



1
]
1

(1 )KV
max
(1 )KV
min



1
]
1
(3)
Solving equation (3) by matrix inverse (or Cramer's rule) results in
a
0
b
0



1
]
1

1
2
1
1
1



1
]
1
(1 )KV
max
(1 )KV
min



1
]
1

K
+1
1
1



1
]
1
V
max
V
min



1
]
1

K
+1
V
max
+ V
min
V
max
+ V
min



1
]
1
v
out
(t) [ ]
min
a
0
K
V
max
+ V
min
+1
K
V
max
+ V
min
+ V
min
V
min
+1
KV
min
+
V
max
V
min
+1
KV
min
+ K
V
max
V
min
1+1/
KV
min
+ K
V
max
V
min
1+ e
0.5T /
and
v
out
(t) [ ]
max
b
0
K
V
max
+ V
min
+1
K
V
max
+ V
min
+ V
max
V
max
+1
KV
max
K
V
max
V
min
1 +1/
KV
max
K
V
max
V
min
1+ e
0.5T /
This complete completes the derivation of the desired formulas.
SOLUTION TO PROBLEM 22.17
There is one correction in the problem statement. 1-kHz should be 5.1mHz.
(a) The transfer function is, according to equation 4.3, and using the given element values R
1
= 10 k,
R
f
= 50 k and C = 20 mF:
H(s) =
V
out
V
in
= -
Z
f
Z
1
= -
Y
1
Y
f
= -
1
R
1
Cs +
1
R
f
= -
R
f
R
1
1 + R
f
Cs
=
-5
1 + 1000s
(b) Here we have We have K= -5, t = 1000 s, Vmax = 1 V, Vmin = -1 V and T = 1/f = 1/0.0051=
196.15 s. In using the equations derived in problem 22.16, we note that the subscripts min and max
should be switched in this case because K is negative. The following MATLAB codes perform the
needed numerical calculations.
f= 5.1e-3;
Cf=20e-3;
Rf=50e3;
Rs= 10e3;
T=1/f;
K= -Rf/Rs;
tau=Rf*Cf;
['part (b)']
Vmax=1;
Vmin=-1;
voutmax= K*( Vmin + (Vmax-Vmin)/(1+exp(0.5*T/tau)))
voutmin= K*( Vmax - (Vmax-Vmin)/(1+exp(0.5*T/tau)))
answers from MATLAB are:
voutmax =
0.2449
voutmin =
-0.2449
(c)
['part (c)']
t1= 0:0.005*T:0.5*T;
v1= 5 + (-0.245 -5).*exp(-(t1/tau)) ;
t2= 0.5*T:0.005*T:T;
v2= -5 + (0.245 +5).*exp(-(t2-0.5*T)/tau) ;
t=[t1,t2];
v=[v1,v2];
plot (t,v)
xlabel('time in seconds')
ylabel('vout in V')
grid
-0.3
-0.2
-0.1
0
0.1
0.2
0.3
0 50 100 150 200
time in seconds
v
o
u
t

i
n

V
SOLUTION PROBLEM 22.18.
v
i
(t) = cos (t) = cos(6000t)
v
o
(t) = 10v
i
+ v
i
2
+ v
i
3

= 10cos(t) + cos
2
(t) + cos
3
(t)
= 10cos(t) +0.5 +0.5cos(2t) + 0.75cos(t) + 0.25cos(3t)
= 0.5 + 10.75 cos(t) + 0.5cos(2t) 0.25cos(3t)
The effective values of various components of the output are:
dc fundamental 2nd harmonic 3rd harmonic total harmonic
0.5 10.75/2 0.5/2 0.25/2 0.559/2
The total harmonic distortion is
0.559
10.75
100% = 5.2%
and the average power at the fundamental frequency is
0.5
10.75
2
100
= 0.5778 W
SOLUTION TO PROBLEM 22.19.
v
o
(t) = V
0
+ V
1
cos(t) + V
2
cos(2t) + V
3
cos(3t)
We use equation 22.30 and the values of v
o
(t) read from the oscilloscope
to compute V
k
. The total harmonic distortion is given by
H.D. =
V
2
2
+ V
3
2
V
1
100 %
MATLAb codes:
vo0= 10; vo60= 5.2; vo120= -4.6; vo180= -9.6;
V0= (vo0+2*vo60+2*vo120 +vo180)/6
V1= (vo0+ vo60 - vo120 -vo180)/3
V2= (vo0 - vo60 - vo120 +vo180)/3
V3= (vo0-2*vo60+2*vo120 -vo180)/6
% total harmonic distortion
HD= 100*sqrt( V2^2 +V3^2)/V1
The following answers are obtained frm MATLAB output.
V0 = 0.2667 V
V1 = 9.8000 V
V2 = -0.0667
V3 = 0 V
HD = 0.6803 (percent)
SOLUTION PROBLEM 22.20.
We shall follow the solution given in example 22.12, and only indicate the needed changes below. There are two
corrections in example 22.12: (1) 20cos(T) should be 20cos(0.5T) and (2) 1.842cos(t) should be 1.842
cos(3t).
The new input is
v
i
(t) = 0.9 cos(6000t)
The positive peak of the output sine wave is clipped for T.
0.9 20cos(0.5T) = 15 = cos
-1
(15/18) = 0.1864
Equation 22.35 becomes
Equations 2.36 becomes
Using A=1, and 0.1864 in item #2 of table 22.4, equation 22.37 becomes
and equation 22.38 becomes
Equation 22.39 becomes
from which the harmonic distortions are:
third order 1.076/16.57 = 6.595%
fifth order 0.55/16.57 = 3.319%
SOLUTION PROBLEM 22.21. Corrections: (1) cos(
1
)e

1
+
RC
cos(
2
) should be
cos(
1
)e

1
+
RC



_
,

cos(
2
) and (2) on page 943, equation 22.46 ,
2
should be
2
, i.e., equation
22.46 should be cos(
1
)e

1
+
RC



_
,

.
The proof is similar to that given on page 943 for the half-wave rectifier case, except for some
minor changes described below.
For the case of a full-wave rectifier, the output voltage waveform is a modification of figure 22.17
as shown below.
The exponential decay of v
o
(t) starts with the value V
m
cos(
1
) at =
1
(instead of
1
2, as in the
half-wave rectifier case). Therefore, in equation 22.45, change T to T/2 and 2 to . In other words
v
0
t ( ) V
m
cos(
1
) ( )e

t
1

T
2



_
,

RC






_
,




V
m
cos(
1
) ( )e

t
1
( )
RC
In equation 22.46, make correction (2) above, and change 2 to , i.e.,
cos(
1
)e

1
+
RC



_
,

cos(
2
)
The desired proof is complete.
SOLUTION PROBLEM P22.22
Refer to figure 22.17. Assuming
1
= 0, we have
v
0
(t) = V
m
exp(
-t
RC
) = V
m
1 - (
t
RC
) + (
t
RC
)
2
- (
t
RC
)
3
+ ...
Assuming
2
= 0, we compute the average value of v
o(t)
over the time interval [ 0, T]. For the case
RC >> T, we can approximate v
o
(t) over this interval by keeping only the first two terms of the infinite
series. Thus
v
0
(t) V
m
1 -
t
RC
which indicates that the plot of v
o
vs. t over the interval [ 0 T] is approximately a straight line.
Therefore the average of the v
o
(t) over [ 0 T] is equal to v
o
(T/2). Thus, for the case RC >> T, or
equivalently RC >> 2,
V
dc
v
o
T
2
= V
m
1 -
T
2RC
= V
m
1 -

RC
SOLUTION PROBLEM 22.23
Given values are: C = 20e-6 F, R =100k, V
m
= 20 V; f = 60 Hz. From equation 22.48b
V
dc
(1 -

RC
)V
m
= (1 -
1
2fRC
)V
m
= (1 -
1
26010
5
2010
-6
)20 = 19.916 V
To calculate the ripple factor, we first calculate
2
from equation 22.47,
and then use the result in equation 22.50.

2
= cos
-1
(
2
RC
) = 0.1289 rad
ripple factor
1 - cos(
2
)
3 [ 1 +cos(
2
)]
=0.2406%
For the diode average and peak currents, use equations 22.49a and 22.52
I
dc
=
V
dc
R
=
19.917
100,000
= 0.199 10
-3
A
i
d,peak
V
m
Csin(
2
) = 19.410
-3
A
SOLUTION PROBLEM 22.24
Equations 22.44 - 22.52 are derived for a half-wave rectifier. For a full-wave rectifier, some of these equations will
be modified slightly as given below. The difference arises from changing T to T/2.
The new equation for
2
is derived in problem 22.21, and repeated below.
The given values are: C = 20e-6 F, R =100k, V
m
= 20 V; f = 60 Hz.
A modification of equation 22.48b gives
V
dc
= (1 -

2RC
)V
m
= (1 -
T
4RC
)V
m
= (1 -
1
46010
5
2010
-6
)20 = 19.96 V
To calculate the ripple factor, we first calculate
2
from equation derived in problem 22.21, and then use the result
in equation 22.50.

2
cos
-1
(e
-

RC
) =5.23 degrees
ripple factor
1 - cos(
2
)
3 [ 1 +cos(
2
)]
=0.12%
For the diode average and peak currents, use equations 22.49a and 22.52
I
dc
=
V
dc
R
=
19.96
100,000
= 0.1996 10
-3
A
i
d,peak
V
m
Csin(
2
) = 13.7510
-3
A
_
SOLUTION PROBLEM 22.25. CORRECTIONS: (1) 195 should be 1950 . (2) 100 F should
be 10 F.
Since this problem only requires an estimate of the answer, we can use reasonable approximations to simply
the solution. Let H(s) V
o
(s) I(s) be the transfer function for the linear circuit to the right of the diodes. Then,
H(s)
V
in
(s)
I(s)

V
o
(s)
V
in
(s)
Z
in
(s)
Z
par
(s)
R + Z
par
(s)
where Z
par
(s) is the impedance of the parallel C-R
L
.
The first step is to find the magnitude H( j ) . As long as 1/R
L
<< C and 1/(C) << R, the
following approximations are valid:
Z
par
( j )
1
j C +
1
R
L

1
C
which means that the parallel impedance is essentially that of the capacitor, and
Z
par
( j )
R + Z
par
( j )

Z
par
( j )
R

1
RC
and again since 1/(C) << R,
Z
in
( j )
1
C
Using these approximations,
H( j ) Z
in
( j )
Z
par
( j )
R + Z
par
( j )

1
C

1
RC

R
RC ( )
2
It is given that I
dc
= 0.01 A. From the short pulse property, the input current i(t) consists of very short pulses at
120 Hz and all ac components of i(t) have peak magnitudes approximately equal to twice the average dc value.
Hence the peak magnitudes are 0.02 A. Therefore the magnitude of 120-Hz component of the output voltage is
H( j2 120) 0.02
R
RC ( )
2
0.02
1950
2 120 1950 10 10
6
( )
2
0.02 0.18042 V
Hence, the effective value is 0.18042 2 0.12757 V and the ripple factor is
0.12757
30
100 0.42524 %
SOLUTION PROBLEM 22.26
Since this problem only requires an estimate of the answer, we can use reasonable approximations
to simply the solution. Let H(s) V
o
(s) I(s) be the transfer function for the linear circuit to the right of
the diodes. Then
H(s) =
V
o
I
=
V
in
I

V
o
V
in

Under the condition 1/(C) << R and 1/(C) << R
L
, we have
Z
in
(s) Z
C
(s) =
1
Cs
and
V
o
V
in

1
RC
2
Using these approximations,
H(j) Z
in

V
out
V
in
=
1
C
(
1
RC
)
2
= R(
1
RC
)
3

For a full-wave rectifier, the fundamental frequency is 120 Hz, and = 240 rad/s. It is given that I
dc
= 0.01 A. From the short pulse property, the input current i(t) consists of very short pulses at 120 Hz
and all ac components of i(t) have peak magnitudes approximately equal to twice the average dc value.
Hence the peak magnitudes are 0.02 A. Therefore the peak magnitude of 120-Hz component of the
output voltage is
H(j120) x0.02 = R(
1
RC
)
3
0.02= 975
(
1
1209751610
-6
)
3
0.02 = 0.012 V
Hence, the effective value is 0.012/ 2 =0.0848 V and the ripple factor is

0.0848
30
100% = 0.2827 %
SOLUTION PROBLEM 22.27
Using H(j) of figure P22.27 and the given formula for c
n
, we have
c
n
=
1

s
H(j) e
-jnT0
d =
1

s
e
-jnT0
d =
1
-jnT
0

s
[ e
-jnT0
]-0.25 s
0.25s
-0.25 s
0.25s

-0.5 s
0.5s
=
1
n
sin (n

2
), n = 1,2,...
For n = 0, we have c
0
= d
0
= a
0
/2 = [Average value of H(j) ]= 0.5 .
For n = 1,2,..., from equation 22.6
b
n
= 0,
d
n
=a
n
=
2
n
sin (n

2
),

The Fourier series representation of H(j) is
H(j) = 0.5 +
2

cos(T
0
) - -
1
3
cos(3T
0
) + -
1
3
cos(5T
0
) - -
1
7
cos(7T
0
+ ...
A plot of H(j) vs. curve using the first 11 terms (n=0,1,...11) of the Fourier
series is given below together with the MATLAB codes.
-0.5
0
0.5
1
1.5
0 20 40 60 80 100
w in radians
H
(
j
w
)
%chapter 2, problem 27.
ws= 100;
T0=2*pi/ws;
w= 0: ws/200:ws;
d0w = 0.5;
d1w = (2/pi)*cos(T0*w);
d3w= -(2/pi/3)*cos(3*T0*w);
d5w= (2/pi/5)*cos(5*T0*w);
d7w= -(2/pi/7)*cos(7*T0*w);
d9w= +(2/pi/9)*cos(9*T0*w);
d11w= -(2/pi/10)*cos(11*T0*w);
H =d0w +d1w + d3w +d5w +d7w + d9w +d11w;
plot(w,H)
xlabel(' w in radians')
ylabel('H(jw)')
grid

You might also like